Download as pdf or txt
Download as pdf or txt
You are on page 1of 644

Jan 2020

Floride toxicity & stannus floride Toxicity of fluride is 8mg/kg. Wt of pt is 22lb. If (1


0.4% stannous fluride is used how much minimum Dose required to cause
toxicity? (4.1mg of stannous flouride contains 1mg floride)
Ans. The wt is 2.2 Lb = 10 kg Toxicity of = 8* 10= 80 mg of fluoride If 4.1 mg of
stannous fluoride contains 1 mg fluoride Then we need to calculate how many of
stannous Florida contains 80 mg ( toxicity) = 4.1 x 80 = 328 mg of stannous
fluoride will cause toxicity How much from the product we use? 0.4 gm = 400
mg of stannous Florida in 100 ml How many ml for 328? (328 x 100) / 400 = 82
ml

87 year old women metastatic breast cancer did mastectomy and started (2
chemo and radiation, eventhough she took her antiemetics as prescribed she is
havinh N&V 6-7 times / day and she came to the pharmacist saying im so tired
id rather die then go through this again:

Q1. What is the appropriate question to ask her?


a) are u sure u took ur antiemetic as prescribed?
b) what adverese effect were u expecting from ur chemo treatment?
C) do u know how ur chemotherapy works?
D)

Q2) what is the next thing to tell her?


a) explain how her chemo works and why she is experiencing this effect

B) tell her its her right to refuse chemo


C) tell her this is only in the first time it will stop

MOA of Omalizumab (3
Ans . inhibits the binding of IgE
Rituximab (4
Ans . acts on CD20

Rituximab targets CD20, a transmembrane protein present on virtually all B cells from the stage at which they
become committed to B-cell development until it is downregulated when they differentiate into
antibody-secreting plasma cells. Rituximab works by targeting the CD20 antigen on normal and malignant
B-cells. Then the body's natural immune defenses are recruited to attack and kill the marked B-cells. Stem cells
(young cells in the bone marrow that will develop into the various types of cells) do not have the CD20 antigen.
Abatacept acts by binding CD80/86 on antigen-presenting cells.

Infliximab acts as TNF alpha inhibitor

MOA of empagliflozin (5
Ans . SGLT2 inhibitors
A patient came with rash on their skin upon diagnosis it is found to be lupus, (6
what is the appropriate treatment? Hydroxychloroquine
Corticosteroides
Tacrolimus as Immunosupressor

Metformin 2 structures relationship (7


tautomerism

30 year old Chinese women pregnant for the first time she developed (8
preeclampsia she is taking methyldopa and prenatal vitamins

Q1) why did she get preeclampsia?

Age (A
Ethnicity (B
First time pregnant (C
Because of prenatal vitamins (D

Q2) what to counsel her about methyldopa?


It will cause photosensitivity (A
It might cause hypotension and dizziness (B

Q3) what other than methyldopa can she take?


Labetalol (a
Hydrochlorothiazide (b

Atmoxetine mechanism of action: (9


Selective norepinehphrine reuptake inhibitor

Galantamine mechanism of action: (10


Competitive, reversible acetylcholinesterase inhibitor.
Naltrexone is used for alcohol craving what neurotransmitter it reduces in the (11
brain:
Dopamine (a
Seretonin (b
Acetylcholine (c
Norepinephrine (d

Mechanism of action of infliximab (12


TNF alpha inhibitor

Tallman letters question which one is correct? (13


BUSPirone / BUPROPrione (a
DOPAmine/ DOBUTamine (b
What part of the DNA does the RNA polymerase enzyme in transcription bind (14
to?
Ans. Promoter
The gene that is not mutated what is it called? (15
Website where patients and providers can obtain information on
Ans. Wild gene drug safety alerts, advisories, warnings, and recalls: can be readily
accessed by patients and providers. Canada Vigilance Program is
Question about Health Canada medeffect part of MedEffect. (16
which prescription narcotic is written correctly? (17

what happened when you added ethinyl group to estrogen? (18


higher biovailability and increased resistance to metabolism,
RR AND NNT case rendering it more suitable for oral administration. (19
Enhances effectiveness
loading dose calculation (20
calculating phenytoin dose from tabs to liquid but he was on phenytoin sodium (21
and the liquid form is a phenytoin free form and he mentioned that the phenytoin
sodium carries 92% of free phenytoin.
calculating dose from subcutaneous morphine 5mg qid to oral morphine, (22
calculate the oral dose needed for one day if you know that the bioavailabily of
the sc form is twice that of the oral form.
there were two graphs regarding the Iv to oral comparison (23
what should we do to enhance the nasal spray bioavailability? (24
a)double the dose

b) micronized particles

question about pertussis vaccine (25


Pertussis vaccine is a vaccine that protects against whooping cough. Whooping cough is a
respiratory disease caused by Bordetella pertussis bacteria.
DPT 2 months, 4 months, 6 months, 15-18 months, 4-6 years, 11-12 years
2 structures that are mirror image to each other, which type of isomerism? (26

Non super imposable mirror images are enantiomers..

A pregnant women with asymptomatic bacteruria and when she takes penicillin (27
it causes her hives and severe rash, the bacteria culture was sensitive to
nitrofurantoin, cloxacillin, cephalosporin.
Q1) What is the appropriate treatment?
Don’t give her any meds (a
Nitrofurantoin for 7 days (b
Cotrimoxazole for 3 days (c
A kind of cephalosporin 7 days (d
Q2)she came later complaining to u of recurrent vaginal candidiasis what to give
her?
Fluconazole once dose (A
Clotrimazole 1% (B
Miconazole 2% (C

Betacarotene to vitamin A (28


Ans . oxidative cleavage

Sumatriptan is structurally related to? (29


Ans . tryptamine
Acute otitis media for 18 mon child reccurent after 3 weeks (30
Q1) causative organism?
H.influenza (a
N. meningitis (b
S.pneumonia (c
Stap. Aereus (d
Q2)what to give him?
Amoxicillin 10 days (a
Amox/clav 10 days (b
Other options (c

Mab are absorbed via? (31


i chose lymphatic and blood
selectively decrease T helper 2 and mast cell activity. Target skin inflammation without causing
Tacrolimus acts on? immunosuppression. Tacrolimus bonds to an immunophilin, FK506 binding(32
protein (FKBP). This
complex inhibits calcineurin phosphatase. The drug inhibits calcium-dependent events, such as
il2 interleukin-2 gene transcription, nitric oxide synthase activation, cell degranulation, and
apoptosis.
Vegan people deficient in? (33
Vitamin B12
Peg is added to filgrastim to? (34
increase t half Conjugation with PEG results in reduced renal clearance and a longer
half-life
The most abundant in adaptive immunity? (35
Antibodies (a
Natural killer (b
Esinophils (c
Peg is added to supp to? (36
increase wettability
Ethynyl gp in ethynyl estradiol increase oral bioavailbilty (37
Idarcuzimab is antidote for dabigatran (38
The anti deprressant with highest QT prolongation (39
Citalopram SSRI (a
Sertraline (b
Venlafaxine (c
Tardive dyskinesia is caused by antipsychotic drugs (40
Volume of distribution is (41
is the ratio between dose and cp (A

a patient was taking salbutamol 2 puffs 2 times a day for 1 month it was (42
improving her asthma, but not anymore this is called?
Anaphylaxis (a
Tachyphylaxis (b
Intolerance (c
The drug that cause respiratory acidosis (43
Morphine

Which of the following is an obligate intracellular bacteria making it resistant to (44


cell wall active antibiotics?
Ans. Chlamydophila pneumonia

Doc for smoking cessation with schizophrenia? (45


psychosis : NRT
Ans . bupropion cardiovascular : Bupropion

Drolozamide decrease aqueous humor production (46


Test to be done after 1 month of isotretinoin? (47
Liver function (a
Kidney function (b
Other options (c
A patient came to ur pharmacy asking to test for sore throat strep, after testing (48
he does not have it. He asks u to check again because he thinks he has it, he is
suspecting what?

False positive (a
False negative (b

When changing from paroxetine to venlafaxine (49


No need to wash out when changing from SSRI TO SNRI (a
Taper parox for 7 days then start venlafaxine (b
Wash out paroxetine for 5 days then start (c
Was out for 2 weeks (d

Sphincter that transfer chyme from stomach to duodenum (50


Ans .pyloric sphincter

A 67 year old patient with diabetes, angina and hypertension not controlled for (51
diabetes and hypertension he is taking metformin hba1c (8%) and taking ACE
inhibitor for hypertension
Q1) what to add to metformin?
Sitagliptin (a
Acarbose (b
Empagliflozin (c
Q2)what to give him to control his hypertension?
Losartan (a
Hydrochlorothiazide (b
Nifedipine (c
A beta blocker (d

A patient is drinking grapefruit juice everyday with breakfast which medication should (52
get your attention
Pravastatin HS (a
Donepezil (b
Nifedipine in the morning (c

Patient taking nitrates what should he not take (53


The options had tadalafil PRN

A women just had a baby 4 weeks ago and came to you saying she is crying all day and (54
tired all the time, what should be your response?
All moms are like that (a
Its only like that in the beginning once your baby gets some sleep it will be better (b
Try to get some sleep when your baby sleeps because this is symptoms of you just (c
being tired.
Check with your doctor because it might be postpartum depression (d

Pyrogen in the parentral preparation (55


Is a Bacterial metabolism (a
Changes the physical appearance of the ampoule (b

If you are a patient and went to a doctor to prescribe morphine according to the law (56
you should inform the doctor that u had a prescription before from another doctor for
morphine for how many days?
30 days (A
60 days (B
10 days (C
15 days (D

The motor cortex is in which part of the brain (57


Frontal lobe (a
Parietal lobe (b
Occipital (c
A patient is taking insulin and gets hypoglycemia when he exercises what to tell him (58
Take a snack before the exercise (a
Take a bar of chocolate when u feel hypoglycemia (b

What is the systemic review the employer does to compare the expectations that was (59
agreed on
Ans . Appraisal

Testosterone requires : (60


Written prescription with no refill (a
Written or verbal with no refill (b
Written or verbal and refill should be with specific interval (c
Written or verbal and refill is allowed (d

Mechanism of action of Omeprazole? (61


Ans . H/K ATPase pump inhibitor

Site for fast insulin absorption? (62


Abdomen (a
Thigh (b
Upper arm (c

A patient just started taking an antidepressant and she is complaining I cant sleep and (63
asks you when will she start sleeping?
1-2 weeks (a
2-4 weeks (b
6-8 weeks (c
Q2) when should you evaluate this antidepressant?
1-2 weeks (a
2-4 weeks (b
6-8 weeks (c
A patient with COPD taking salbutamol and tiotropium is not feeling better, what to (64
add?
Formerterol (a
Oral corticosteroid (b
Formeterol+ ICS (c

In Which should the whole house be treated? (65


Lice (a
Pinworm (b

Which hormone increases with age? (66


Thyroxin (a
Parathyroid (b

Which gland shrinks with age? (67


Thymus (a

What to do to protect from cataract (68


Ans .Wear eye protection
A chancre is a painless genital ulcer most commonly formed during the
primary stage of syphilis. This infectious lesion forms approximately 21 days
Chancre is caused by: after the initial exposure to Treponema pallidum, the gram-negative (69
spirochaete bacterium yielding syphilis.
Ans. treponema pallidum

Statins decrease LDL and? (70


HDL (a
Cholesterol (b
Vascular inflammation (c

Some body was planting his garden and had itching that made him not sleeping last (71
night
hydrocortisone cream (a
dimenhydranate (b
loratidine (c

Difficulty in swallowing (72


Ans. Dysphagia

Acute gout case had CV disease doc (73


indomethacin (a
prednisone (b
colchicine (c
febuxostat (d

Doc for stroke prevention in patient 70 years old and has atrial fibrillation (74
Ans. Dabigatran

In order of importance the cyp450 enzymes (75


Cyp3a4>2d6>2c9>1a2

Ganisetron inhibits which receptor (76


Serotonin (a
Dopamine (b

A patient came to your pharmacy wants to measure his blood pressure what should u (77
tell him
Rest for 30 min (a
Lie down for 10 min (b
Don’t talk while taking the measurements (c

A patient with angina and hypertension which of these will increase the lifetime and (78
has negative inotropic effect:
Dobutamine (a
Lisinopril (b
Metoprolol (c

If an employer dismisses a pharmacist because a breach in contract what is this called: (79
a) employers right b ) constructive dismissal C) habeas corpus

A horome increase with PH (80

Ans .Gastrin

BPH sign (81


Ans Hesitating

sign and symptoms of hypothyrodism (82


ans. Dry skin

bupropione for smoking cessation When to use? (83


Before stopping date with one to two weeks (a
skin protection should be from UltravioletA snd B (84

heart control rate with atrial fibrillation (85

acebutolol Acebutelol has ISA so can Increase heart rate in this case (a
digoxin (b
amiodarone (c
diltiazem (d
S. Verdan transfer to heart by (86
Ans .Mouth
In the evolution of prokaryotes to eukaryote the most difference is in: (87
Nucleus membrane (a
Cant remember other options (b
mixed case between diarrhea and constipation (88
Ans. IBS

main risk or cause peptic ulcer (89


ans. Smoking

preeclampsia characteristic (90


proteinuria (a
hyperuricemia (b
glucosuria (c

Diaper rash should be referred to the doctor if (91


Blusters and pus
Refrence book for fungal feet treatment (92
CTMA for minor element Medeffect website
Canada Vigilance Program
medEffect site for reporting drug SE Marketed Health Products Directorate (93
Health Canada
Refugee are treated under which plan (94
a)provincial
b)not insured CADTH: Independent not-for-profit organization funded by Federal and P/T
governments of Canada that
c)Federal and procedures to support healthcare professionals to make informed
clinical decisions.
-reviews drugs and provides recommendations on whether the drug should
be added to the formulary
CADTH ROLE for recommendations and evaluatiion of drug after approval (95
Drug after recieved NOC Next step will be in market (96
Vomiting metabolic alkalosis hypokalemia
vomiting need monitor for K Diarrhea metabolic acidosis (excessive loss of bicarbonate)
(97
tyrosine kinase is (98
transferase Tyrosine kinase belongs to the enzyme that can transfer a phosphate group (a
from ATP to a protein in a cell.
hydrolase (b
atrial fibrillation risk factor for stroke (99
above 65 years old (a
history of AF (b
uneducated patient cannot sign the consent form before operation , the most (100
appropriate is to
draw a diagram for him (a
summarize whats written in the consent form (b
explain the risk and benefits of the operation (c
a women with seizure smokes 2 cig/ day for smoking cessation: (101
non pharmacological treatment (a
buproprion (b
varnecilline (c
nicotine replacement therapy (d

Liability: Employer’s/employee’s action leading to harm & negative consequences


liability is: Liabilities in economy are what you owe other parties. (102

IN A normal ECG highest peak is: (103


Ans. QRS
most credible study: (104
meta analysis of case study (a
randomized clinical trial (b
case report (c

a 16 year old girl is asking for emergency contraception when to refer to doctor (105
irregular menses (a
vomiting (b
her age (c

natural health product (106


botanical (a
vitamins and minerals (b
Iron deficiency anemia - Induced by long-term ingestion of omeprazole. Omeprazole is known to
cause prolonged suppression of gastric acid secretion. Since acid plays an important role in the
absorption of food iron (nonheme iron), this drug may lead to an iron deficiency state.
PPIs are known to inhibit gastric acid secretion. Nonheme iron requires gastric acid for
conversion to the ferrous form for absorption.

in a diabetes study what is the surrogate end point (107


amputation (a
Hba1c (b
Case about patient taking PPI reason for anemia? (108
Women was taking levothyroxine and became pregnant what should happen to the (109
dose?
Increase the dose Sodium is the major cation of the extracellular fluid.
Potassium is the major intracellular cation.
Chloride is the predominant extracellular anion.
Bicarbonate. Bicarbonate is the second most abundant
polyethylene glycol is a surfactant for Diazepam anion in the blood. (110
Most abundant electrolyte intracellularly VS interstitial fluid? K (111
Antipseudomonal: Ciprofloxacin, Aminoglycosides, Piperacillin/Tazobactam , Ticracellin,
anti pseudomonas AB Meriopenems , Carbepenams, Cefipime, Ceftazedime (112
Not used: Gram positive selective antibiotics like first generation cephalosporins & vancomycin
Most peptide hormon is Glucagon (113
itracanzol councel on empty stomach (114
linoliec acid essential acid get from outside (115
A & B drugs in pharmacoeconomics study. “Concluded” that A is superior to B. In the (116
answers there were choices like “is A better in lowering BP and cheaper OR is the
study conclusive

Independent pharmacy that you own , but you bought the name and there is central (117
purchase
Banner Wrong (a
Banner : no central purchase
Franchise (b
Franchise isn’t indépendant
Sole (c

Which of these needs new drug submission from health canada (118
Bioequivalent (a
Biosimilar (b
Generic (c
Interchangeable (d
A patient taking a lot of meds and tells you I don’t know how will I take all these (119
meds which response shows empathy
don’t worry we will help you take you medication as easy as you can (a
‫ ه‬we will help you (b
U must be overwhelmed from all these meds (c
Most stimulant for respiration: (120
O2 (a
Co2 (b
Hco3 (c

Pharmacodynamics example effect (121


Positive symptoms for psychosis is hallucinations (122
Pharmacist presenting marketing material to a doctor comparing 2 drugs saying that (123
drug A is better than drug B , what on the material will make the doctor say its correct
Confidence interval (a
Relative risk (b
Absolute risk (c
NNT (d
When should a pharmacist report adverse event from a vaccine (124
Only the one he gives (a
Any event even if he is not the one who gave it ( or in another pharmacy) cant (b
remember

A patient has Alzheimer and recently had agitation the doctor prescribed (125
Resperidone,
His son came to u to ask u about his prescription
Pantoprazol
Atorvastatin
Other meds
Q1) what to tell him about resperidone
Elderly patients with dementia-related psychosis treated with
It causes insomnia and day drowsiness antipsychotic drugs are at an increased risk of death.(a
RisperiDONE is not
approved for the treatment of patients with dementia-related psychosis.
It increases risk of mortatilty and stroke with alzh patients (b
I cant remember the rest (c
Q2) what to tell him
Don’t worry the nurses will take care of him (a
Tell the doctor to change it (b
Q3) what else in the prescription should get ur attention
The metabolism of Atorvastatin can be decreased when combined with
Risperidone.
126) patient came complaining of pain in left calf when u ask what could be the
reason
a) long flight 1 week ago
B) atheroscloratic check on his right knee 2 weeks ago
c) taking prednisone 50 mg for 5 days for COPD attack 2 days ago
D)taking thiazides for 2 days

◦ calculation about potassium and phosp. in inection the numbers was in mmole
◦ calculation toxic dose aluminium
Voluntary and involuntary Cerebrum - frontal lobe
◦ congnitive motor which lobe Balance and body movements Cerebellum
◦ pharmaccist check the employee work and compare with something not sure
For pregnant we can use methyldopa, Nifedipine
◦ hypertension for pregnancy other than methyldopa XL , hydralazine or labetalol ( avoid thiazides,
atenolol and ACEI)
◦ pregnancy UTI and there culture resistance mentioned was antiobtic culture result
was menty Competitive, reversible acetylcholinesterase inhibitor. Galantamine
blocks the breakdown of acetylcholine in the synaptic cleft, thereby
◦ galantamine mechanism of action increasing acetylcholine neurotransmission. It also acts as an
allosteric modulator of the nicotinic receptor, giving its dual
TNF-alpha inhibitor ◦ adalimumab mechanisam of action mechanism of action clinical significance.

◦ was last question about COPD or UTI patient on salbutamol


◦ case cancer with sever vomiting
◦ was something related to franchise or banner
◦ Perutsis patient call ask about the dis fever, redness and soreness or swelling where the injection was
given, nausea, headache, tiredness and aching muscles.
◦ sturtucture tryptamine
◦ pregnancy after antibiotic with vaginal discharge vulvovaginitis one question what
the dischage look and other what to give her.
◦ patient start insomnia (I am not sure) was to treat insomnia or he was start SSRi but
when to he start see the effect and other question when dr decide to change the drug
• on drug want to stop and use citalopram what to do washout or tapper
• question about how to do the antibiotics culuture (the multiple choice was B-
lactame) the most popular antibiotics used for cell culture are gentamicin and
PenStrep, a cocktail of both penicillin and streptomycin.
• pharmacy customer asking about his father(father not pharmacy customer) using
doneprzil and he was fine on it but recently they notice that he..... so doctor add
other medication some medication mentioned the question was what you should
inform the patient son first (‫اﺳﺋﻠﮫ‬٣‫ )ﻛﺎن ﻋﻠﯾﮭﺎ‬See question 125
• patient take grape fruit on morning only and he use (nifidipine morning,See question 52
pravastation HS and some more medication what you should inform him
• preeclampsia See question 90
• patient pastpartum come to pharmacy she’s very tired and not sure if mentioned
crying or not the question was this sign for what See question 54
When converting between
medication formulations, it
may be important to consider
the salt form of the active
ingredient. In this case,
Dilantin® capsules are
formulated as the sodium
salt, while Dilantin® Oral
Suspension is manufactured
as the free acid. This
variation in salt forms may
result in different doses of
the parent compound. In this
case, phenytoin sodium
capsules, 100 mg, contain
92 mg (92%) phenytoin. The
Capsules = 3*100*92% = 276 mg
free acid form of phenytoin in
125 mg ————> 5ml
the suspension yields 125
276 mg ————> x
mg of free phenytoin per 5
X= 276*5/125 = 11 ml
ml (100% phenytoin).
Idarucizumab is the antidote of Although the extended- release capsule can be given once daily, it is
appropriate to provide the oral suspension in 2-3 equally divided doses.
Fentanyl 5.5 ml twice daily

Dabigatran #
Me! As 100 mg Phenytoin Na in Cap. equivalent to = 92 mg
Phenytoin base
Lorazebam So, the patient dose = 100 X 3 TID = 300 mg cap. Which is
equivalent to 276 mg Phenytoin base
As 125 mg in 5 ml; So 276 mg in X, X = 11.04 ml; which is 5.5
ml to be given as BID.
pharmacy business long term planning finance
a) strategic planning

b) i don’t remember the choices 🙈


Kan dawa oral
Dose w iv dose

High clearance
Low clearance
Polyexponential kinetics
Haga kaman

(-wDrug A dominat over Drug B


Drug A more effective and cost effective

)-Risk factor for osteoporosis


Long term use of Corticosteroids
JULY 2019
1. A 28 years old pregnant patient in her third trimester having uncomplicated urinary
tract infection. The best choice of antibiotics to treat her UTI may include:
a. Nitrofurantoin
b. Trimethoprim + Sulfamethoxazole.
c. Norfloxacin.
d. Gentamicin.
e. Methanamide
Ans: A
Treat asymptomatic bacteriuria and symptomatic cystitis in pregnant women with a 3 - 7
days course of
amoxicillin (if the organism is known to be susceptible), amoxicillin/clavulanate,
cephalexin or nitrofurantoin with appropriate follow-up.
Nitrofurantoin is usually avoided near term because of the risk of inducing hemolytic
anemia in fetus or newborn, especially in those with G6PD deficiency; however, this
toxicity is rare.

2. Deet 30 application duration and before or after sun screen?


Concurrent application of sunscreen with the insect repellent N, N diethyl-
metatoluamide (DEET) may lower
SPF but efficacy of DEET appears to be
maintained. If application of DEET is delayed after the sunscreen
has been applied, the sunscreen product maintains more of its original SPF.

3. Which drug used in treatment of ADHD patient with hisory of cocaine abuse?
a- Amphetamine
b- Methylphenidate
c- Atomoxetine
d- Dexamphetamine
Ans: C
Although Treating ADHD with stimulants can decrease the risk of substance abuse, they
are contraindicated when there is a history of drug abuse.
CTC - Contraindications to stimulants include history of hypersensitivity to
sympathomimetic amines, symptomatic cardiovascular disease (including moderate to
severe hypertension, advanced atheriosclerosis), uncontrolled hyperthyroidism, history of
drug abuse and concurrent use with an MAOI.

4. The most side effects of donepezil are on:


a- Respiratory system
b- Liver
c- Kidney
d- Heart
e- Gastrointestinal tract
Ans: E
S.E: Cholinesterase inhibitors: theoretically, these agents may lower seizure threshold,
increase the risk of GI ulceration or bleeding, or exacerbate COPD or asthma.
Donepezil: >10%: headache, nausea, diarrhea. <10%: vomiting, anorexia, fatigue, sleep
disturbance, syncope, muscle cramps, urinary frequency. Bradycardia (uncommon), heart
block (uncommon), rhabdomyolysis (uncommon), neuroleptic malignant syndrome
(uncommon).

5. Risk factor for DVT?


a) Diabetes
b) HTN
c) Obesity
d) Hernia repair surgery
Ans: C

6. HTN patient untreated for long period. This can


cause which one of the following?
a) Aortic stenosis
b) Alzheimer
c) Left ventricular hypertrophy
Ans: C

7. Most common opportunistic viral infection in HIV patients


is? Cytomegalovirus virus
8. Which of the following virus commonly cause retinitis in HIV patient?
a) Cytomegalovirus virus
b) Syncytial virus
c) Varicella virus
d) Epsestein barr virus
Ans: A
First choice is valganciclovir po or iv. Iv ganciclivir, iv foscarnet.
For maintenance: po valganciclovir, iv foscarnet, or iv cidofovir.

9. Structure of benzodiazepine?
The ring B in benzodiazepine is "diazepine"

10. A 54-year-old patient recently diagnosed BPH. He is having the following medication
captopril, sildenafil for his erectile dysfunction and Hydrochlorothiazide. He has no allergy
but has postural hypotension but not high. What drug you should recommend for this
patient?
A. Finasteride
B. Tamsulosin
C. Dutasteride
D. Doxazosin
E. Prazosin
Ans: B

11. Which blot is used to separate protein?


a) Northern blot
b) Southern blot
c) Eastern blot
d) Western blot
Ans: D

12. Dorsal cavity contains?


a) Lungs
b) Teeth
c) Spinal cord
d) Bladder
Ans: C – contains brain & spinal cord

13. A prescription says to take 15


mg of morphine po q4h. Below is a
chart with a recommended initial
duragesic (fentanyl) dose based
upon oral morphine dose, how
many 25 mcg/hr duragesic patch
are required to fill a 10-day
supply?
a. 3
b. 9
c. 12
d. 10
Ans: A
firstly, put in your mind that
duragesic dose is 1 patch every 3 days (72hr).
the dose of morphine in table based on total daily dose.
So, 15 mg q4hr  6 doses  90mg morphine daily.
This means that we need 1 patch every 3 days as the dose of morphine falls in the range
60 -134.
Total number of duragesic patches for 10 days is 3 patches.

14. In hepatic transplant patients, most common infections?


viral
 Cytomegalovirus
 Epstein-Barr Virus
 Herpes Simplex Virus and Varicella Zoster Virus
 Human Herpesvirus 6 and 7
 Adenovirus
Fungal
 Candidiais is the most common fungal infection in liver transplantation.
 Aspergillosis is the second most common fungal infection after liver transplantation

15. Cause of Neural tube defect?


a) Alcohol
b) Valporic acid
c) Peptic ulcer
Ans: B
Moderate risk women of neural tube defect NTD includes:
Maternal or paternal personal or family history of other folate-sensitive congenital
anomalies (limited to specific anomalies for cardiac, limb, cleft palate, urinary tract,
congenital hydrocephaly).
Maternal or paternal family history of NTD in a first- or second-degree relative.
Maternal diabetes; maternal kidney dialysis.
Maternal use of folate-inhibiting medications (carbamazepine, cholestyramine, metformin,
methotrexate, phenobarbital, phenytoin, primidone, sulfasalazine, triamterene,
trimethoprim, valproic acid).
Maternal GI malabsorption conditions (e.g., Crohn disease, active celiac disease, gastric
bypass surgery).
High risk includes: Maternal or paternal personal NTD history or a previous NTD
pregnancy

16. Counseling patient with showing empathy?


a) I know how you feel
b) It feels like you are frustrated because this medication is not working
c) Don’t worry, will find a solution
Ans: A
what is meant by empathy listening:
a- Feeling what the talker feels
b- A knowledge with what the other person says
c- Listening with open mind
d- Understand what the other say
Ans: A

17. Optic nerve cell types


a) Cones
b) Rods
c) Ganglion
Ans: C

18. Quetiapine act on


a) 5HT1A
b) 5HT2A
c) 5HT1A + D2
d) 5HT2A + D2
Ans: D ‐ Effectively binds 5HT2a, H1, alpha1. alpha2 in brain and has low affinity to D2
receptors.

19. Eye drops that can cause cataract


a) Alpha blockers
b) Beta blockers
c) Corticosteroids
Ans: C

20. Glucose to Pyruvate resulting ATP what is the name of process:


a) Glycolysis
b) Glycogenysis
c) Gluconeogenesis
d) Glycogenolysis
Ans: A
Under anaerobic conditions (absence of O2): Glycolysis involves the
conversion of glucose to lactate (lactic acid). Q. This can occur in cells without
mitochondria.
Under aerobic conditions (presence of O2): Glycolysis involves the conversion
of glucose to pyruvate (pyruvic acid), this occurs in mitochondria.

21. In DNA, the bond between the deoxyribose sugar and the phosphate is which
of the following?
(A) A polar bond
(B) An ionic bond
(C) A hydrogen bond
(D) A covalent bond
(E) A van der Waals bond
Ans: D
Hydrogen bond exists between nitrogen bases of two strands of the DNA.
There are covalent bonds among base, sugar and phosphate of nucleotide.

22. Adalimumab MOA: Binds to TNF‐α, interfering with endogenous TNF‐α activity by
blocking its interaction with cell surface receptors.

23. All these microorganisms classified as nocosomial bacteria, except:


a) Enterobacteria
b) Legionella
c) Pseudomonas
d) Moraxella catarrhalis
Ans: D

24. Non pharmacological method to help in smoking


cessation are all except?
a) Acupunctue
b) Cognitive
c) Massage therapy
d) Laser therapy
e) Hypnotherapy
Ans: C

25. breathing Center in brain?


a) Brain stem
b) Thalamus
c) Hippocampus
d) Cereberum
Ans: A
Brain stem: the posterior part of the brain consisting of pons and medulla oblongata and
mid brain.
Medulla oblongata helps regulate breathing, heart, blood vessel function, digestion,
swallowing and sneezing.

26. Canadian health act includes:


a. Accessibility, comprehensiveness, affordability
b. Portability, universality, affordability, accessibility
c. Universality, public administration, accessibility
d. proftibility, Universality, public administration
e. public administration, accessibility, sustainability
Ans: C
Universality, public administration, accessibility, comprehensiveness & portability

27. mechanism of action of sitagliptin?


a) Dipeptidyl Peptidase-4 DPP - 4 Inhibitors
b) Glucagon-Like Peptide-1 (GLP-1) Agonists
c) Sodium-Glucose Cotransporter 2 Inhibitor
Ans: A

28. When a party negotiates terms with a union, this is called:


a) collective bargaining
b) petition
c) mediation
d) haggling
e) being cheap
Ans: A

29. Denosumab what receptor work on?


a) Interluken 1
b) Interluken 6
c) RANK Ligand Inhibitors
d) TNF Inhibitors
Ans: C

30. Head bone that contains nasal sinus?


a) Parietal
b) Frontal
c) Occipital
d) Temporal
Ans: B

31. scaphoid bone located in?


The scaphoid bone is one of the carpal bones of the wrist. It is
situated between the hand and forearm on the thumb side of the
wrist (also called the lateral or radial side). It forms the radial
border of the carpal tunnel.

32. CDR under whichorganisation


CDR is a branch of CADTH. CDR provides drug
reimbursement recommendations for listing (or for not
listing) and advice to federal, and P/T public drug plans.
33. Protease is under which enzyme class?
a) Lyase
b) Hydrolase
c) Esterase
Ans: B

34. Cranial Nerve responsible for emesis?


a) vagus nerve
b) glossopharyngeal nerve
c) olfactory nerve
d) abducent nerve
Ans: A

35. The presence of the albumin with insulin is?


a. Buffer
b. Preservative
c. Solvent
d. Nutrient
e. Antiadsorbant
Ans: E, and to increase the duration of activity.

36. Lady wants to stop smoking and suffered from


schizophrenia
a) Bupropion
b) Nicotine patch
c) Varenicline
Ans: B
Bupropion inhibits the reuptake of norepinephrine and dopamine, which are involved in
the pathogenesis of affective, cognitive, and psychomotor impairment in schizophrenia.
Because of reports on bupropion-associated psychoses, it is reluctantly used in
schizophrenic patients.
Varniciline has Neuropsychiatric side effects such as suicidal/homicidal ideation have
been reported; monitor closely for changes in mood/behaviour.

37. Codeine and paroxetine interaction


a) Serotonin syndrome
b) Codeine won’t work
c) Paroxetine won’t work
d) Nothing will happen
Ans: B
The serotonin syndrome may occur if the SSRI, MAOi, SSRI, TCA, are combined with
Tramadol, Meperidine, Amphetamine, Dopamine, Cocaine, Methyldopa,
Dextromethorphan, Caffeine, Lithium, SSRI, MAO Inhibitors, TCAs, St. John wort.
The analgesic actions of codeine are derived from its conversion to morphine by the
CYP450 2D6 enzyme system.
Inhibitors of CYP2D6 (e.g., amiodarone, cimetidine, fluoxetine, moclobemide, paroxetine,
quinidine) may antagonize codeine's effect.

38. A case of mother has achild 2 years old and they want her to remember what
happened in her pregnancy. What type of bias in the study?
a) Recall bias
b) Selection bias
c) Lead time bias
d) Publication bias
Ans: A

39. Apatient came with prescription with his name but upon checking the pharmacist
discoverd these drugs don't belong to this patient. He asked the patient and found that
drugs for his brother that doesnt have insurance. If you accept this prescription you
violate?
a) Veracity
b) Autonomy
c) Professional competency
Ans: C

40. Which one is the most complicated factor in assessing the UTI in women with CVD?
a) sex
b) Pregnancy
c) Diabetes
Ans: C

41. Vaccine given to 1-year children and has booster dose at 4 to 6


years? MMR

42. Patient takes Dilinuim (X sodium) 200 mg po tid, because of swallowing


difficulties his prescriber will change to liquid form, available Dilinium suspension
as 125 mg/5ml pure X, Dilinium tables contain 92% pure X, how many Dilinium
suspension should he takes daily?
The idea is tablet contains 92% of drug and solution contains 100%
so, 200 x 3 x 92% = 552mg of tablets.
125 mg ----- 5ml
552 mg ----- x
X = 21.6 ml = 7.2ml for every dose

43. Advantage of use of Taq pcr? Thermostable


the enzyme is thermostable polymerase hence it can even work at a higher temperature.
Efficiency: It is highly efficient. ... The half-life of thermostable polymerase is higher than
any other commercially available polymerase.

44. Counseling for cephalexin suspension:


Shake well – refrigerate – take on an empty stomach - expect mild diarrhea
Cephalexin, Amoxicillin, Amoxicillin/clav, Metronidazole, Erythromycin
All these 2‐8 refrigerators

45. Which one you have to refer to dr immedietly


a. inflammation of the eye with purulent discharge
(after 48hr)
b. foot tinea fungal infection
c. pain in the ear with drain for less than 24 hrs
d. child 9-month-old with fever
Ans: D A since D less than 72 hrs

Children < 6 months of age with a fever should be assessed by health-care Practitioners
(refer).
Children < 2 Y REFER IF FEVER PERSIST FOR 72
hrs

47. who is insured by non-insured health benefits NIHB?


a) Prison
b) Inmate
c) Police
d) Refugee
e) Natives
Ans: E
48. Thrombosis first choice for cancer patients is?
a) Dabigatran
b) warfarin
c) fondaparinux.
Ans: B C
Vitamin K antagonists are used with caution as cancer patients are susceptible to wide
fluctuations in INR.
Malignancy and pregnancy LMWH
Low molecular weight heparins Renal failure heparin or warfarin

preferred for long-term


anticoagulation.
49. Nosocomial infection example is? Ventilator acquired pneumonia
 Hospital-acquired pneumonia: Ventilator-associated pneumonia
 Urinary tract infection
 Gastroenteritis
 Puerperal fever
 Central line-associated blood stream infection

50. Gemcitabine mechanism of action?


a) Antimetabolite
b) Alkylating agent
c) Microtubule inhibitor
d) Monoclonal antibody
Ans: A

51. The most lipophilic narcotic so used in spinal analgesia


a) Meperdine
b) Sufentanil
c) Hydromorphone
Ans: B. Fentanyl and sufentanil are the lipophilic opioids most widely used intrathecally
in the context of postoperative pain. Both drugs present a quick onset (10-15 min) with a
short clinical duration (2-5 h).
52. which is suitable method for measuring temperature in 2 years old child?
a) Oral
b) Rectal
c) Axilary
d) Ear
Ans: B
In children ≤ 5 years of age, rectal thermometry is the gold standard in Canada for
definitive measurement of temperature.

2 question case 9 months with 38°c fever and mother worried from febrile seizures.
53. How to manage the fever
a) Acetaminophen
b) Ibuprofen
c) Frequent Cold Bathe
d) Wait and see if no improvement, refer
e) Refer to physician now
Ans: A

54. What to tell mother about febrile seizures


1. 20 % of children get febrile seizures on age 3 to 5 years
2. only children with neurological problems get febrile seizures
3. wait and see
Ans: C
A febrile seizure, also known as a fever fit or febrile convulsion, is a seizure associated
with a high body temperature but without any serious underlying health issue or
neurological problems. They most commonly occur in children between the ages of 6
months and 5 years.
If the seizure lasts longer than 5 minutes, call an ambulance. The child should be taken
immediately to the nearest medical facility for diagnosis and treatment.

55. cholesterol found in? plasma membrane


The principal components of the plasma membrane are lipids (phospholipids
and cholesterol), proteins, and carbohydrate groups that are attached to some
of the lipids and proteins. A phospholipid is a lipid made of glycerol, two fatty
acid tails, and a phosphate-linked head group.

56. Most common Gram *Positive* *G+ve* organism in bacterial meningitis?


a) Streptococcus Pneumonia
b) N. meningitidis
c) E. coli
d) Influenzae type b
Ans: A

57. you are a manager in a community pharmacy and you decided to give service with
price which only covers your cost and expenses, you decide to make this service with 10
cases per month, therefore you will have:
a- Increase in net profit
b- Increase in the revenue
c- Increase in the inventory
d- decease in net profit
Ans: B
You are making more money (net revenue) because you sold more items, however you
sold these items at their cost price so you did not make more net profit.

58. How to overcome the issue of confounding factor in statstics?


Strategies to reduce confounding are:
 randomization (aim is random distribution of confounders between study groups)
 restriction (restrict entry to study of individuals with confounding factors - risks bias in
itself)
 matching (of individuals or groups, aim for equal distribution of confounders)
 stratification (confounders are distributed evenly within each stratum)
 adjustment (usually distorted by choice of standard)
 multivariate analysis (only works if you can identify and measure the confounders)

59. High uric acid untreated for long time cause:


A. osteoarthritis
B. tophi
C. osteoporosis
Ans: B tophi may develop during the intercritical period (stage III)

60. The doctor is suspecting that his patient is suffering of hyperthyroidism, which assay
can be used to confirm his diagnostic?
A. Serum TSH
B. T4 in urine
C. Sensitive TSH
D. TSH in urine
E. T3 counts
Ans: C

61. Seborrhea ttt?


a) Triclosan
b) sulphur
c) ketoconazole
Ans: C. Ketoconazole is a First-line treatment of dandruff and seborrheic dermatitis.

62. Cystic fibrosis pathophysiology?


a) Chloride
b) calcium
c) sodium
d) bicarbonate
Ans: A
Cystic fibrosis is an autosomal recessive disease caused by defects in the
CFTR gene, which encodes for a protein that functions as a chloride channel,
& also regulates flow of other ions across the apical surface of epithelial cells.

63. Clozapine structure is?


a) Benzodiazepine
b) butyrphenones
c) hydantoin
Ans: A. Clozapine is a benzodiazepine that is 5H-dibenzo [1,4]
diazepine substituted by a chloro group at position 8 and a 4-
methylpiperazin-1-yl group at position 11. ... It is a benzodiazepine, a N-methylpiperazine,
a N-arylpiperazine and an organochlorine compound.

64. If crcl < 15 which anticoagulants can


be used?
a) Warfarin
b) Apixapan
c) Fondaparinux
Ans: A

65. Apixapan mech? Direct Factor Xa Inhibitors


66. vitamin K is secreted from:
A- Lung B- Kidney C- Colon D- Spleen E- Liver
Ans: C

67. What is a problem of emulsion?

68. Patient taking allopurinol after acute gout attach


and not responding what is the alternative
uricosuric action:
a) Colchicine
b) Febuxostat
c) Prednisolone
Ans: B

69. What is the role of Canadian Pharmacists Association (CPhA)?


Largest organization of pharmacists in Canada.
Advocates on behalf of pharmacists to governments, and the public (example: they
have a campaign to increase the scope of practice of pharmacists)
Leads a number of national projects, publications, conferences
Lobbies governments and publishes recommended government policies
Publishes references like CPS, CTC, CTMA
& Offers CE (continuing education)
programs.

70. a study where all pharmacists took a


questionnaire to see their opinion about
something
a. Case study
b. Cross sectional
c. Delphi analysis
d. Lingitudinal study
Ans: C
The Delphi method is a forecasting
process framework based on the results of
several rounds of questionnaires sent to a
panel of experts. Several rounds of
questionnaires are sent out, and the
anonymous responses are aggregated and
shared with the group after each round.

71. Extracellular & intracellular ions:

72. On what receptor does rasagline works?


a) Monoamine Oxidase B
b) Monoamine Oxidase A
c) Monoamine Oxidase A & B
Ans: A

73. What cells line the blood brain barrier???


a) capillary endothelial cells
b) schwann cells
Ans: A schwann cells produce mylein sheath around neuronal axon

74. which is responsible for emesis in brain?


There are two medullary centres of vomiting in the brain known as the sensory
“chemoreceptor trigger zone (CTZ- area postrema)” and the integrative vomiting centre.

75. Side effects of phenytoin are:


A) gingival hyperplasia, ataxia, nystagmus
B) Steven johnson syndrome
C) hirsutism and acne
D) All of the above
E) A and B only
Ans: E

76. What decrease mortality in CHF?


a) Digoxin
b) Atorvastatin
c) Amlodipine
d) Captopril
Ans: D. ACEIs & BBs are Recommended in all patients because they improve symptoms
and reduce the risk of hospitalization, MI and death in patients with HfrEF.

77. What we use for resistant schizophrenia?


a) Clozapine
b) Asenapine
c) Flupentixol
d) Olanzapine
Ans: A. Clozapine ↓D2 ↓5HT2, ↓H1, ↓M, Alpha1-2. the Only antipsychotic with proven
efficacy in treatment resistant schizophrenia.

78. Bupropion works on? Serotonin (very little), dopamine & norepinephrine
reuptake inhibitor (sNDRI).

79. Long term treatment of PPIs cause decrease of absorption to what?


a) vit c
b) thiamine
c) folic acid
d) cyanocobalamine
Ans: D
S.E of all PPIs except dexlansoprazole: Abdominal pain, diarrhea, flatulence,
hypomagnesemia, ↓ serum vitamin B12 level. Long-term use of PPIs has come under
scrutiny due to the association of PPI therapy with a number of serious complications,
including CV events, C. difficile infections, community-acquired pneumonia, dementia,
kidney disease and osteoporotic fracture.
80. What the problem of tertiary refrence reletive to primary refrence?
Time to update (long lag time)

81. Glutin sensitivity occurs when


the person eats?
a) Rye
b) Milk
c) Rice
Ans: A

Case: diabetic pateint age 70 y, on


insulin therapy has HbA1c 8 mmol,
Glucose morning 10 mmol, Evning
10 mmol, he has dementia and
taking donepzil, has GIT problems
GERD and taking CaCo3, and his
blood sugar is high, has blood pressure and take BBs, he comes to the pharmacy with
dizziness and BP 85/55, and bradycardia?

82. when should this patient should check his HbA1c? 3 months
HbA1c measurements every 3 months for patients who have not achieved target
values; testing every 6 months may be acceptable in stable patients who
consistently meet glycemic targets.

83. what should this patient take to decrease his HbA1c?


the suitable choice was liraglutide

84. is the HbA1c level is suitable for this pateint?


YES. A higher HbA1c of up to 8.5% may be more appropriate if risk of hypoglycemia
outweighs the benefits of tight control, e.g., in frail elderly patients, those with
limited life expectancy or patients with a history of recurrent severe hypoglycemia

85. what is the cause of his dizziness and bradycardia?


The interaction between BBs and donepzil
Cholinesterase inhibitors: theoretical concern regarding antagonistic effect of combined
therapy with cholinesterase inhibitors and drugs with anticholinergic activity, or additive
bradycardic effects when combined with BBs or calcium channel blockers; few reports of
actual interactions
86. what is disease/drug interaction in his case?
Donepzil and GERD  increase the risk of GI ulceration or bleeding.

87. what should be used to treat his GERD? PPIs


PPIs are superior to H2RAs for reduction of symptoms of GERD and healing of esophagitis
in patients with moderate to severe GERD.

88. Teething pain in infants what treatment after nonpharmalogical measures?


a) Clove oil
b) acetaminophen
c) ibubrufen
d) benzocaine gel
Ans: B

89. What is recommended to store in fridge?


a) claritromycin
b) ketoconazol sus
c) azithromycin
Ans: B

90. which is used as Humectant +preservative +solubilizing agent?


a) Glycerine
b) ethyl alcohol
c) petrolatum
d) urea
Ans: A

91. The filler used in the hard gelatin capsule


a. Starch
b. Cellulose
c. Di basic phosphate
Ans: A

92. A woman has a menopausal stage and facing hot flashes, she had DVT 5 months ago
and using warfarin, all of the following can be used to treat her hot flashes except?
a) Cognitive behavioural therapy
b) estorgen hormone therapy
c) using fan
d) avoiding triggers (caffeine, alcohol, stress).
e) Clonidine
Ans: B  Increased risk of VTE, CVD, breast cancer

93. Asthma patient in emergency, all can be used to control his asthma except?
a) Fluticasone
b) Ipratropium
c) Formetrol
d) Oxygen
e) Salbutamol
Ans: C

94. Protease is under which enzyme class?


a) Lyase
b) Hydrolase
c) Esterase
Ans: B
A protease (also called a peptidase or proteinase) is an enzyme that catalyzes (increases
the rate of) proteolysis, the breakdown of proteins into smaller polypeptides or single
amino acids. They do this by cleaving the peptide bonds within proteins by hydrolysis, a
reaction where water breaks bonds.

95. chemotherapy induced diarrhea, which one is appropriate to treat


this side effect?
96. the most abundant solute in plasma is?
a) Albumin
b) Prothrombin
c) Chlorine
d) Potassium
Ans: A
Plasma is about 92% water, with plasma proteins as the most abundant solutes. The
main plasma protein groups are albumins, globulins, and fibrinogens.
97. The highest iron absorption in the body will be obtained if taken:
a) On empty stomach not to irritate
b) With food not to chelate
c) With orange juice
Ans: C
Iron salts have greater absorption when given on an empty stomach, but may cause
irritation. Also, Vitamin C enhances absorption.

98. Achlorhydric condition can lead to dificiencyof?


a) Iron
b) Vit. B12
c) Folic acid
d) Niacin
Ans: B Elderly are achlorhydric and commonly deficient in Vitamin B12.

99. What is the drug of choice for absence seizures in a child < 2 years of age?
A. Phenytoin
B. Phenobarbital
C. Ethosuximide
D. Valproic acid
E. Primidone
Ans: C
Reduces propagation of abnormal electrical activity in the brain, most likely by inhibiting
T-type calcium channels. It is only effective in treating absence seizures.

100. Causative organism responsible for impetigo? S. aureus


Impetigo: Superficial infection of skin primarily caused by Staphylococcus aureus and
most often affecting young children.
Two clinical forms include:
Crusted or nonbullous (S. aureus and/or Streptococcus pyogenes – found in Face, arms or
legs)  treated by topical antibacterial as fusidic acid, mupirocin, ozenoxacin for 5 days.
Bullous (S. aureus-mediated toxin - found in Moist, intertriginous areas (axillae, neck
folds, diaper area)  treated by oral antibacterial as cephalexin for 7 days.

101. In research cost $ 100,000 cause increase in patient life by 0.5 year, so what is the
true to express the Cost-Effectiveness analysis of that research?
a) $ 100,000 / QALY
b) $ 200,000 / LGY (Life Gained per Year)
c) $ 50,000 / QALY
d) $50,000 / LGY
Ans: B
Quality added life year is a cost utility analysis, it means how much i will spend to add to
the patient life one year lived in full health. If it is costing me 100,000 to add half year
lived in full health (100% quality of life) then it will cost the double to add one full year

102. What type of hypersensitivity reaction of Contact dermatitis? Type IV

103. Least drug interaction with Lithium?


a) Iron
b) Hydrochlorothiazide
c) Ibubrofen
d) Captopril
Ans: A
Toxic levels may result when adding NSAIDs, ACEIs, ARBs and especially thiazide
diuretics. Reduce lithium dose and check serum level if long-term treatment with these
agents is required.
Stop lithium temporarily during acute intermittent illnesses causing fluid and electrolyte
losses.
Patients taking lithium need to maintain their usual salt and caffeine intake and monitor
fluid intake and output, making adjustments in the event of unexpected losses due to
vomiting or diarrhea.
104. Which HMG-CoA reductase inhibitor is the lowest to be metabolized by Cyt P450:
a) Simvastatin
b) Atorvastatin
c) Pravastatin
d) rosuvastatin
Ans: C

105. what is the side effect


of intiation of perindopril
therapy
a) flushing
b) angioedema
c) headache
d) diarrhea
Ans: B

106. Total sales is = $530,000; Cost of goods sold = $350,000 What % of gross margin?
A. 50%
B. 34%
C. 20%
D. 100%
Ans: B  [(530,000 – 350,000) / (530,000)] x 100 = 33.96%

107. Patient started chemotherapy, experiences diarrhea, nausea and cramps what does
really concern?
1. Blood in stool
2. More than 4 stools daily
3. Nausea more than 4 hours
4. Abdominal cramping for more than 10 days
Ans: A

108. Patient started taking NSAID and experienced daily heart burn, what do you
recommend?
a) Ranitidine
b) PPI once daily
c) Antacids
Ans: B.
109. Endocarditis is caused by
a) staphylococcus aureus
b) Streptococcus viridans
c) staphylococci pyogenes
d) bacillus substilus
e) bacillus mirabilus
f) E coli
Ans: A. CTC. The Gram-positive bacteria staphylococci 40%, streptococci 20% and
enterococci 10% account for the majority of pathogens causing IE.

110. Rheumatoid arthritis is? Symmetric, more than 45min in the


a) Symmetric, less than 45 minutes pain, low grade fever. morning, there is fever and inflammation
and swelling
b) Happen only in weight bearing joint. Happens in both weight bearing and none
bearing
c) Obesity is not a risk factor Obesity is a risk factor
Ans: A.

111. Lady is taking hydromorphine for cancer pain. Now she is in hospital and having
another pain and took morphine but no response, this can be due to?
A potentiation
B addiction
C tolerance
D resistance
E toxic effect
Ans: C

112. Adult female 42 years has urinary incontinence. She said it has been very bothersome
lately and that she could not go out without worrying about if she does not find a toilet.
She also added that there is leakage when she coughs or exercises & she has been stressed
for the past few days, what kind of urinary incontinous?
A. involuntary incontinence
B. urge incontinence
C. Stress incontinence
D. overflow incontinence
Ans: C

113. The ethical law whereby pharmacist can refuse to fill a prescription for moral or
religious reasons is called:
a) personal clause
b) notwithstanding clause
c) conscience clause
d) freedom of faith clause
e) mutual respect clause
Ans: C
Conscience clauses are legal clauses attached to laws which permit pharmacists,
physicians, and/or other providers of health care not to provide certain medical services
for reasons of religion or conscience. In many cases, the clauses also permit health care
providers to refuse to refer patients to unopposed providers. Those who choose not to
refer or provide services may not be disciplined or discriminated against. The provision is
most frequently enacted in connection with issues relating to reproduction, such as
abortion, sterilization, contraception, and stem cell-based treatments, but may include any
phase of patient care.

114. Ketoacidosis means all of the following except?


a) Hypoglycemia
b) Volume depletion (depleted in Sodium, Potassium, Chloride and water)
c) Acidosis
d) Depressed levels of consciousness
e) Detectable ketones in the urine or blood
Ans: A

115. A membrane between two different concentration (solute and solvent) what happen?
a) Solute move from less con to more con
b) solute move from more con to less con
c) solvent move from less to more
d) solvent move from more to less
Ans: C

116. pt suffering from COPD, he is taking erythromycin, what antibiotics can be added to
him?
a) ciprofloxacin
b) levofloxacin
c) clindamycin
Ans: B

117. you are a pharmacy manager and have two pharmacists working at the pharmacy,
you figured that there is a shortage of narcotic stock, you setup surveillence camera and
figured the one who took this narcotic, what is the appropriate action?
A) Call the pharmacist who was involved in the incidence
B) As a pharmacy manager, use your authority and fire him.
C) Call the regulatory college and report the case
D) Call the Royal Canadian Mounted Police RCMP
E) Report the incidence to the Office of Controlled Substances
Ans: D
Reporting wrong behavior of healthcare professional protect public and reputation of
healthcare system.
Each regulatory body have set different requirement for reporting by the members of the
profession.
If the behaviour of the professional misconduct from your colleague, directly report the
incident to the college of pharmacy not your manager.
If the behaviour of the professional misconduct from a pharmacist working in a nearby
pharmacy and you knew that he is incompetent and unable to do his duties, report the
incident to his manager and keep in touch with him to know the results.
In case of prescription forgery or stolen narcotics from staff or other person, call the
police directly Royal Canadian Mounted Police RCMP and after that call the college in
10 days.

118. This tablet should NEVER be crushed, because once it is


crushed, it’s action completely is lost:
a) Sugar coated tablets
b) Chewable tablets
c) Lozenges
d) Osmotic tablets
e) Effervescent tablets
Ans: D

119. drug of choice for emergency glucoma?


a) Pilocarpine
b) Acetazolamide
c) Dorzolamide
d) Timolol
Ans: B. Acetazolamide and methazolamide lower IOP by decreasing the production of
aqueous humor. Their use is normally reserved for emergencies because of significant
side effects.

120. What cause QTc prolongation?


a) Citalopram
b) Amiodarone
c) Nitrofurantoin
d) Amoxycillin
Ans: A. Causes dose-dependent QTc prolongation.

121. Most important risk factor for Peptic ulcer?


a) Smoking
b) high fat
c) spicy food
Ans: A

122. What cause priapism?


a) Trazodone
b) Imipramine
c) Phenelzine
Ans: A

123. Drug that cause prerenal failure


1. Insulin
2. Furosemide
Ans: B

124. Fast allergic reaction is?


a) Contact dermatitis
b) Atopic dermatitis
Ans: B

125. Digoxin mechanism of action?


a) Increases intracellular Calcium
b) Inhibit beta receptor on heart
Ans: A
Digoxin: Cardiac glycoside increases myocardial contractility and efficiency by ↑
intracellular calcium which causes heart muscle fibers. Half life T1/2 = 26 hr to 45 hr.
+ve inotropic: Inhibit membrane bound Na+/K+ activated ATPase. ↑ intracellular sodium
concentration and reduce calcium transport form cell thus facilitate calcium entry via
voltage gated membrane channel
–ve chronotropic: Increase vagal tone of sinoatrial (SA) node. Reduced CNS sympathetic
out flow. Systemic arteriolar and venous constriction
Vagomimetic effect.

126. Arylpropionic acid related compounds such as ibuprofen show which of the
following?
a) Streoisomerism
b) homologs
c) conformational
d) structural

Ans: A. A structural isomer, or constitutional isomer, is a type of isomer in which


molecules with the same molecular formula have different bonding patterns and their
atomic organisations, as opposed to stereo isomers, in which molecular bonds are
always in the same order and only spatial arrangement differs.
Ibuprofen has a stereocenter in the α-position of the propionate moiety. There are
two possible enantiomers of ibuprofen and each of these has different biological
effects and metabolism within the body. The mirror images or setreoisomers are non-
superimposable, much like the left and the right hand.
The two optical isomers of ibuprofen are identified by the prefixes R- and S+.
The stereoisomers are similar in properties such as melting point, boiling point and
solubility. The S+ form is the more pharmacologically active form and the R- form has
no anti-inflammatory effect.
127. Which drug shows more norepinephrine than serotonergic?
a) Amitriptyline
b) Fluxetine
c) Nortriptyline
Ans: C

128. graph has 2 axis, X axis indicates numbers of physician visits in the last 3 months,
Y axis represents patient taking fluticasone in asthma, what does this means?
Means the pt who use fluticasone in large conc decrease no visit to dr

129. which proportions should be used from 2 ointments containing 0.15% and 0.25%
dexamethasone, to make 150 mg dexamethasone of 0.2% of dexamethasone? We will use
allegation method. Same idea of this one

130. Apatient came with prescription with his name but upon checking the pharmacist
discoverd
these drugs don't belong to this patient. He asked the patient and found that drugs for his
brother that doesn’t have insurance. If you accept this prescription you violate.
a) Veracity
b) Autonomy
c) Professional competency
Ans: A C

131. In comparison between drug A and drug B in the treatment of smoking cessation,
drug A reduces the side effect in 44 patients of 500 patients, drug B reduces the side effect
of 56 patients of 500 patients,
What is the odd ratio?
a) 0.7
b) 1.3
c) 2.6
d) 4.5 The odds ratio is calculated by dividing the odds of the first group by the
odds in the second group.
Ans: A A/C / B/D

132. Which structure is related to Macrolides?

Amphetamine Macrolides Cephalosporin Morphine

133. Aliphatic ring turned into cyclic  by


lactonization (lactone ring) organic reaction
that forms a ring (the lactone) by the addition
of an oxygen across a carbon-carbon double
bond.

134. In a clinical trial of a drug to prevent migraines, 2 of 100 people taking the drug
experience a migraine (2%), compared with 4 of 100 people taking a placebo (4%).
calculate absolute risk reduction, Relative risk reduction & number needed to treat
ARR = CER - EER = 2 – 4 =2 4-2=2

RRR = CER - EER / CER = 2 - 4 / 2 = 1 4-2 / 4 = 0.5


NNT = 1/ARR = 1 / 2 = 0.5 which equal 50% so we need to treat 50 persons to prevent one
bad outcome

135. Drug X given oral by 250 mg q 8 hrs, bioavailaibility =1, clearance = 2.8 ml/hr
what is the steady state conc?
Css = F×D / cl×t where, F bioavailability D dose Cl clearance T interval
Css = 1 x 250 / 2.8 x 8 = 11.16 mg/ml
136. cost utility & cost effectiveness.

137. A patient is taking oral contraceptives and


she always starts her medication on Sunday,
she forgot to take two doses in the second week,
what should you told her to do:
a- Discard the strip and begin with
another one
b‐ Take the daily tablet until the next
Sunday and start a new strip
c- Take two tablets today, then two
tablets tomorrow and then continue
the strip
d- Safely continue the strip without
warring
e- Talk to the doctor
138. isomersim & tautomerism 3questions (know the main idea)
JANUARY 2019

1- Which of the following increase with age?


A‐ Parathyroid hormone
B- Total body water
C- Bone mass
D- Hair growth
E- Clearance
SERUM PARATHYROID HORMONE (PTH) rises with age in both men and women (1–10),
and this has generally been attributed to an age-related decline in renal function (1, 5).
The glomerular filtration rate (GFR) is known to decline with age, and serum PTH is
elevated in patients with renal insufficiency

2- Suitable corticosteroids for diaper rash?


a) 0.025 % hydrocortisone
b) 0.5% hydrocortisone
c) 0,05 % betamethasone
d) Acetamide 0.05%
Hydrocortisone 0.5%, 1%

3- Dorzolamide MOA for glaucoma?


A‐ decrease production of aqueous humor
B- decrease out flow from ciliary body
Carbonic Anhydrase Inhibitor, Decrease IOP by inhibiting an enzyme involved
in the formation of aqueous humor.

4- Last layer of the eye?


A- cornea
B- retina
C- sclera
D- Pupil
If they mean the outer layer it should be sclera, but if they mean the inner
layer, it will be the retina.

5- Isoniazid side effect?


A- Skin patches
B- Steven Johnson's syndrome
C‐ Peripheral neuropathy
Consider giving pyridoxine 25 mg/day for children to prevent peripheral
neuropathy.
S.E: Asymptomatic increase in hepatic aminotransferases and bilirubin (10– 20%),
clinical hepatitis (symptoms may occur within weeks to months), peripheral
neuropathy (dose related). Hematologic effects. GI upset. Gynecomastia, seizures,
drowsiness, drug induced lupus, toxic encephalopathy, pancreatitis, fever, skin rash,
mood changes, lymphadenopathy.

6- Woman taking combined OCP, which element shall be monitored after 4 weeks?
1- CA
2‐ K
3- Na
4- CL
5- Mg
OCP containing drosepirenone: Risk of hyperkalemia in patients prone to increased K+,
e.g., renal disease, concomitant ACEI, ARB, potassium-sparing diuretics, NSAID. Check K+
after 1st cycle.

7- Pharmacist assistant can do which of the following?


A‐ enter a new prescription into software
B- Check the prescription is authentic and right
C- Take oral prescriptions
D- Advice about OTC

8- A woman wants to stop smoking; she had a jaw surgery recently and she has anorexia
nervosa?
1- nicotine gum
2‐ nicotine patches
3- nicotine lozenges
4- bupropion
5- Varenicline

9. Which of the following cause acute hepatitis?


A‐ hepatitis A
B- hepatitis B
C- hepatitis C

10- Source of calcium for muscle contraction?


A‐ Extracellular
B- Blood
There are two sources:
1) calcium sequestered in the sarcoplasmic reticulum S.R. of the smooth muscle cell.
2) extracellular calcium that can enter the smooth muscle cell via calcium channels on
the membrane of the smooth muscle cell

11- Patient taking metformin and Amiodarone (I think) his hemoglobin is decreased and
has high MCV? What's the cause of anemia?
A- Hemolytic anemia
B‐ Vit B. 12

12- Intrinsic factor is needed for absorption of? Vit B 12

13- What in a study will lead to bias? difference in sample size

14- Should be concurrent administered with Isoniazide treatment:


a) Thiamine
b) Niacin
c) Pyridoxine
Vit B6 pyridoxine 50 mg/day PO (to minimize the peripheral neuropathy)

15- Case of BPH, already taking Ramipril, his last blood pressure reading was 148/90,
Terazosin and Finasteride were prescribed
First question, what do u think about the prescription?
A- Polypharmacy
B‐ Suitable management
C- Duplicate therapy C
Second question, What's contraindicated with Finasteride?
A- Tadalafil
B‐ Testosterone

16- Case of a girl with pain and discharge from ear, she is in swimming course, what's the
most likely to be the cause?
1- acute otitis media
2- chronic otitis media
3‐ otitis externa
17- Least causing hypoglycemia?
A- Gliclazide
B- Glimepiride
C- Glyburide
D‐ Sitagliptin

18- Bacterial vaginosis?


A- Itching, white, thick discharge
B‐ Inflammation with mild pain and malodour discharge
C- Itching with purulent discharge

19- In the body, the N of NO is taken from?


A‐ L‐ Arginine
B- Alanine
C- Leucine

20 - You have a patient who converted from male to female (transgender). On his first vist
to pharmacy What you will instruct your staff to do?
1. Ask him whether he'd like to be called as a male or female
2. Use of gender‐neutral language if the patient doesn't want to discuss the matter
3. Just call him according to what you have in the system
4. don’t discuss the topic until he does

21- a patient is not willing to take her prescribed Erythromycin anymore because of its
GIT side effects, what's the best assertive and appropriate way to recommend changing
the prescription while talking to her physician? 3
1‐ I am sorry to disturb you, do you have time to discuss Mrs. FH case?
2- Are you aware that Erythromycin that u prescribed caused GIT side effects to Mrs. FH?
3- I am calling about MRS FH., She claims she can't take her medication because she can't
tolerate the GIT SEs, how about considering Azithromycin?

22- Patient prescribed Codeine and she is taking more tablets than required with no effect,
what's the cause?
A- Hepatic impairment
B- Renal impairment
C‐ She is a poor Codeine metabolizer

23- Most basic amino acid?


A- Alanine B‐ Arginine C- Phenylalanine D- Methionine E- Leucine
Arginine is the most basic amino acid. There are three amino acids that have basic side
chains at neutral pH. These are arginine (Arg), lysine (Lys), and histidine (His). Their side
chains contain nitrogen and resemble ammonia, which is a base. Their pKa's are high
enough that they tend to bind protons, gaining a positive charge in the process.
Two amino acids have acidic side chains at neutral pH. These are aspartic acid or
aspartate (Asp) and glutamic acid or glutamate (Glu). Their side chains have carboxylic
acid groups whose pKa's are low enough to lose protons, becoming negatively charged in
the process.

24 - A case of insomnia and the patient was prescribed Sertraline how long it takes to see
the effect?
a) 1 - 2 weeks
b) 2 ‐ 4 weeks

25- Same case after how long to assess the therapy?


a) 2-4 weeks
b) 6‐8 weeks.
c) 8-10 weeks
d) 10-12 weeks

26 - How to protect light sensitive ingredient? Film coating

27- Rhabdomyolysis symptoms are all except:


A) kidney failure
B) muscle cramp & pain
C) muscle weakness
D) dark urine
E) odor urine

28- Case of atrial fibrillation, what to use to decrease heart rate?


A‐ Digoxin B- Amlodipine C- Amiodarone D- Acebutolol

29- Used for migraine prophylaxis?


A- Tramadol B- Naratriptan C‐ Topiramate D- Carbamazepine E- Nefidipine

30- Gingival hyperplasia affects?


A‐ Gum
B- Lips
Gingival hyperplasia is an overgrowth of gum tissue around the teeth. There are a
number of causes for this condition, but it’s often a symptom of poor oral hygiene or a
side effect of using certain medications such as cyclosporin, tacrolimus and phenytoin.

31 - Montelukast MOA? Leukotriene receptor antagonist

32 - Most common cause of Otitis media for preschool children?


Acute otitis media  Streptococcus pneumoniae, H. influenzae and Moraxella catarrhalis.
Acute otitis externa  Pseudomonas aeruginosa (20–60%) and Staphylococcus aureus
(10–70%).

33- Which of the following doesn't cause CAP?


A‐ STAPHYLOCOCCUS AUREUS
B- Chlamydophila pneumonia
C- Streptococcus pneumoniae
D- Mycoplasma Pneumonia
Strep. Pneumoniae (accounts for half of cases requiring hospital admission), Mycoplasma
pneumoniae, H. influenzae, Chlamydophila pneumoniae, M. catarrhalis, Legionella spp.,
Mycobacterium tuberculosis (uncommon).
Gram-negative bacilli, e.g., E. coli, Klebsiella spp., Enterobacter spp., Serratia spp., P.
aeruginosa. Respiratory viruses, viral + bacterial Fungi

34- What can cause syphilis (Chancre or chancroids on genitals):


a- Neisseria gonorrhea
b- Clostridium difficile
c‐ Treponema pallidum

35- Most teratogenic?


A- Herpes simplex
B- Herpes zoster
C‐ Rubella
D- Human papillomavirus
E - Neisseria gonorrhea
Viruses are known to be teratogenic in humans: cytomegalovirus, rubella (most
teratogenic), herpes simplex, Venezuelan equine encephalitis, Treponema pallidum and
varicella viruses.
36 - Carbonic anhydrase produced by?
a) Reticulocytes
b) Myocytes
c) Pneumocytes
d) Astrocytes
e) Erythrocyte
f) Osteocyte
Carbonic anhydrase, enzyme found in red blood cells, gastric mucosa, pancreatic cells,
and renal tubules that catalyzes the interconversion of carbon dioxide (CO2) and carbonic
acid (H2CO3).
Carbonic anhydrase plays an important role in respiration by influencing CO2 transport in
the blood. The enzyme also functions in the formation of hydrochloric acid by the
stomach.

37 - Contraception after delivery, don’t keep tablet schedule?


a) Should take Progesterone injection
b) Progesterone pills after 4 weeks
c) IUD after 4 weeks
d) Combined OCPs
e) Give OC after 1 week
f) Avoid medroxyprogesterone

38 - Manufacturers what to do after obtaining NOC (Phase 3)?


a) Patency
b) NDN
c) Sell in market
d) Clinical trials

39 - Impetigo pathogen in preschool children


a) S. aureus B) S. pyogenes

40 - Acute renal failure have all except?


a) Pedal edema
b) Swelling Answer is C) Decreased blood perfusion to kidney will lead to compensation
of the kidney and increase BP so here the answer is hematuria
c) Hematuria
d) Hypertension
Hypertension is both an important cause and consequence of chronic kidney disease
41- Plasmid function? Plasmids are often used for DNA recombination and cloning.
CHIMERIC (25% mouse): XIMAB
42- Rituximab? HUMAN (100%) : UMAB
HUMANIZED (10% mouse 90% human): ZUMAB
A) Rat MURINE (100%): MOMAB
-Mab (monoclonal antibodies) -
B) Human Cept (soluble receptors)
C) Chimeric Mib for proteases
Nib for inhibitors (tyrosine kinase inhibitors)
D) Mice

43- What to give for a 6-month-old complaining of constipation?


a. Milk of magnesia
b. PEG  glycerin supp is DOC
c. Docusate

44- what the role of neutophil in primary immune respone:


a‐ engulfing parasite (phagocytosis) Phagocytosis by activated macrophages
b- opposing antigen
c-inflammatory lipid mediators

45- Isotretinoin side effect? Teratogenicity.


Common: mucocutaneous dryness, myalgia, arthralgia, headache, photosensitivity.
Uncommon–Rare: hypertriglyceridemia, mood disorder, possibly suicide ideation,
pseudotumor cerebri.
Rare (possibly related): erythema multiforme, Stevens‐Johnson syndrome, toxic
epidermal necrolysis.

46- Oral contraception during breastfeeding:


Progestin-only methods have no adverse breastfeeding outcomes or negative outcomes
with regards to growth, health and development of infants.
Avoid COC use < 6 weeks postpartum, COC may have a negative effect on milk supply.

47- Type of vaccine we give at 1 Year old and a booster preschool age 4 to 5 years?
a) MMR vaccine
b) Polio vaccine
c) Hepatitis B vaccine
d) Pneumonia (Pneumococcal)
e) Meningococcal

48 - How to store breast milk?


a) Could be refrigerated for 48 hours
b) Freezer for 1 year
c) Room temperature for 12 hours
d) The rest after baby feed could kept in the fridge for 24 hours
Fresh expressed breast milk can be stored for:
 6 to 8 hours at room temperature (no warmer than 25°C [77°F]),
 up to 5 days in the refrigerator (at a temperature of below 4°C [<39°F]),
 2 weeks in your refrigerator freezer (not in the door),
 3-6 months in the freezer compartment of a refrigerator with separate doors, or
 6-12 months in a separate chest-type freezer (at a temperature below -20°C [-
4°F]).
Never mix fresh breast milk with chilled or frozen breast milk because it can cause
bacteria to grow and lead to food poisoning.

49 - SLE management first line?


a) Tacrolimus
b) NSAIDs
c) immunosupressive
d) Hydroxychloroquine sulfate

50 - Linezolid poisoning
Headache, diarrhea, cytopenias (anemia, thrombocytopenia, leukopenia), optic
neuropathy.
Caution with medications that increase serotonin level as there is increased risk of
serotonin syndrome. Contraindicated if within 2 wk of an MAOI.

51 - Pantohenic acid Vit B5 as a coenzyme required for the activity of which of the
following?
Synthesize coenzyme‐A (CoA), as well as to synthesize and metabolize proteins,
carbohydrates, and fats.

52- β amyloid plaques found in:


a) Parkinson’s disease
b) Schizophrenia
c) Alzheimer’s disease
53- Cox 2 function
Produce prostaglandins 
inflammation, pain & fever

54 - Why pharmacists ask open ended question to patient?


Use open‐ended Use closed‐ended Avoid:
questions: questions:
When you need to gather When you want a yes, no Using only open-ended
symptom information that or number response. questions (will make the
is uppermost in the When you want to gather discussion much longer and
patient’s mind. specific, focused unfocused).
When you need to begin a information about the Using only closed-ended
line of questioning. nature of symptoms and questions (will prevent
When you want to treatment. discussion from moving to
determine the patient’s When you need to gather details of which you are
understanding, level of information quickly. unaware, may create
sophistication of When you need to keep a passivity). Asking several
assessment of the problem talkative patient focused. questions in succession
and level of vocabulary that Examples: without patient input. For
would be appropriate to Do you... (have any example, asking “Is the pain
use. discharge?) Have you... mostly in your stomach or is it
To convey a willingness to (ever had this before?) more like heartburn?” “What
listen. To promote rapport Will you... (be able to take does it feel like?” “Do you
and trust. Examples: this every 4-hours?) usually get it after you eat?”
Describe... (the feeling to How many times... (do you one after another without
me) Explain... (how the pain wake up during the night? giving the patient a chance to
feels) answer
Tell me about... (the Asking leading or biased
heartburn) How... (does the questions. For example,
headache feel?) What... “There isn’t any blood in your
(brings you in today?) stool, is there?”
Asking questions that start
with “why” (can seem
judgmental and cause
defensiveness)
Using slang or medical
terminology

55 - when Levofloxacin will be first choice in pneumonia? If the cause is Legionella sp.
An alternative to betalactam/macrolide combination for patients on hospital wards.
For hospitalized patients, the dose of levofloxacin is 750 mg once daily for 5 days

56 - Influenza shot at busy pharmacy

57 - Where to put vaccine in the fridge shelf and temperature?


1‐ A thermometer shall be placed in the middle shelf of the refrigerator
2- Shall be refrigerated from 2-12° c
3- Can be refrigerated in the same refrigerator of food but in a separate part
4- You have to check and document the degree of the refrigerator daily  twice daily

58- A kid was successfully treated with Amoxicillin for otitis media and he got infected
again after 3 weeks, what do you recommend?
A. Amoxicillin 5 days
Recurrence less than 3 months : amox/clav for 10
B. Amoxi/clav 10 days days
Recurrence more than 3 months : amox 10 days
C. Clotrimazole Recurrence after 10 months : Amox 10 days
D. Clindamycin
Amoxicillin clav 10 days  Use high-dose regimen after failure of high-dose amoxicillin.
Clindamycin  Reserve for cases of true anaphylactic-type beta-lactam allergy and/or
treatment failure.

59- Who is responsible for hospitals monitoring and quality in Canada? Hospital
Administrators and Hospital Boards

60 - Plant / Herbal medicine in Canada is regulated?


Natural and Non‐prescription Health Products Directorate (NNHPD)
61 - Special drug access in Canada?

62 - Calculate how many tablets of Calcium needed given meq and molecular weight

63- Drug release form osmotic tablet depends on?


The release of drug from osmotic system depends upon various formulation factors
such as solubility, osmotic pressure of the core components, size of
the delivery orifice and nature of the rate controlling membrane.

64 - Who is not covered by public health insurance?


A- Natives
B- RCMP
C‐ Ontario police
D- Military
E- Refugees sponsored by the country
F- Inmates

65 – Where does RNA polymerase attach on DNA to for replication?


Transcription of two genes.
(a) RNA polymerase moves from the 3′ end of the template strand, creating an RNA
strand that grows in a 5′ → 3′ direction (because it must be antiparallel to the
template strand). Note that some genes are transcribed from one strand of
the DNA double helix; other genes use the other strand as the template.
(b) A uracil is being added to the 3′ end of the transcript for gene  Growth is thus
5′ → 3′.
Initiation of transcription.
(a) RNA polymerase searches for a promoter site.
(b) It recognizes a promoter site and binds tightly, forming a closed complex.
(c) The holoenzyme unwinds a short stretch of DNA, forming an open complex.

66 - short term treatment of insomnia?


a. Flurazepam
b. Zopiclone
c. Lorazepam
d. triazolam
Because these pharmacokinetic properties may confer a higher risk of abuse and
dependence, a shorter treatment course (5–7 days) is recommended for triazolam,
always starting at the lowest dose (0.125 mg).
67 - Verapamil side effect?
a) Constipation
b) Postural hypotension
c) Diarrhea
d) Tachycardia

68 - Vitamin B6 is? Pyridoxine

69- About pharmacist evaluation: appraisal form

70- Profitability means: revenue cover COGS, expenses and more is left to reward the
owner

71 - L- dopa side effect?


N & V, orthostatic hypotension, dyskinesias, hallucinations, confusion.

72 – Pertussis outbreak and patient concern:


a) Tell patient that this is a vaccine
preventable disease and you can update
your vaccine record
b) Isolate patients for 5 days and treat with
Macrolide antibiotics
c) Only one dose, no booster doses.
d) All are true

73- Why you should be change Insulin site of injection? To avoid Lipodystrophy

74 - Primary literature?
a) PubMed
b) Clinical study
c) Compendium of Pharmaceuticals & Specialties (CPS)

75 - Limited authorization of the Pharmacist to prescribe under what?


a) Accessibility
b) Affordability
c) Sustainability
d) Portability
76- A cat scratched a boy what is the best choice?
A) Clindamycin
B) Amoxicillin
C) Amoxi‐Clav
D) Cotrimoxazole
E) Doxycycline

77 - Which inhaler requires priming?


a) Spiriva Respimat
b) Breezhaler
c) Turbuhalers
d) Proventil
Prior to first use, the SPIRIVA RESPIMAT cartridge is inserted into the SPIRIVA RESPIMAT
inhaler and the unit is primed. When using the unit for the first time, patients are to
actuate the inhaler toward the ground until an aerosol cloud is visible and then repeat the
process three more times. The unit is then considered primed and ready for use. If not
used for more than 3 days, patients are to actuate the inhaler once to prepare the inhaler
for use. If not used for more than 21 days, patients are to actuate the inhaler until an
aerosol cloud is visible and then repeat the process three more times to prepare the
inhaler for use

78 - Most common cause of Peptic ulcer in US? H. pylori and Braids

79 - What are marijuana side effects?


a) Weight loss
b) Red eye

80 - What is the part of brain responsible for fine motor movement and coordination?
a) Cerebellum

81 - Where is the location of breathing/ Respiratory center?


a) Medulla oblongata
b) Cerebellum

82 - Structure then it says the Warfarin has which ring?


A. Coumarin (correct) ring-closed hemiketal tautomeric ...

Coumadin
83- Patient had psoriasis and went away now coming with psoriasis again, u give?
A. Pimecrolimus
B. Calcitonin

84- After 3 weeks it getting worse what do u give for moderate to severe psoriasis
A. Acitretin
B. Apremilast

85 - A way to measure efficiency of inventory keeping


A) Turnover
B) Rolling

86 - Sciatic nerve innervates?


a) Leg
b) Face
c) Neck

87 - Indapamide will cause?


a- Hypocalcemia
b. Hypoglycemia
c ‐Hyperuricemia

88- Case about a female with fatigue and tingling in the hands, her lab analysis reveals low
level of hemoglobin and high MCV, what is cause of her anemia?
a) Iron deficiency
b) Hemolysis
c) Vit B12

89 - Which vitamin will be needed for vegetarians? B12


90- All are true about vitamin A except?
a) Antioxidant
b) Safe for pregnancy

91 - Where nucleosidases (promotes the hydrolysis of a nucleoside) are secreted from?


a) Colon
b) Pancreas

92 - What virus causes fetal defect and deafness?


a) Rubella
b) Varicella zoster
c) HSV
d) HPV

93 - What is the components of glutathione?


A tripeptide composed of three amino acids (cysteine, glutamic acid, and glycine)

94 - What is the reaction a structure removal of CH3-CH2-C=O from NH2 to form a


structure with NH2?
a. aliphatic hydroxylation
b. oxidation
c. reduction
d. hydrolysis
e. dehydrogenase

95 - How to remove pyrogen?


a) Filtration
b) Distillation
c) Chemical reactions

96 - Finasteride is? 5 alpha reductase inhibitors

97 - Best to treat Tinea pedis?


a) Miconazole
b) Nystatin
c) Mupirocin
d) Clotrimazole

98 - Rituximab act on?


A- TNF
B- IL
C‐ B‐ LYMPHOCYTES
A chimeric monoclonal antibody that effectively removes memory B cells, but not
plasma cells, from the circulation (B cell depletor)

99 – Needs a prescription and sales record?


a) Tramadol
b) Testosterone
c) Naloxone
d) Clozapine
e) Nalbuphine
f) Oxycodone

100 - Glycoprotein is protein and? Carbohydrate

101 - Urine discoloration? Nitrofurantoin

102 - In depolarization phase?


a) Na voltage gated channel
b) Voltage gated channel
c) Ca channel

103 - Risk of QT prolongation? Torsade’s de pointe

104 - Ointment base for ophthalmic ointments?


a) Glycerin
b) Methylparaben
c) White petroleum

105 - Treatment for pulmonary hypertension - Endothelin receptor blocker is? Bosentan

106 - Resistant schizophrenia? Clozapine

107 - A glidant is a substance that is added to a powder to improve its flowability.


a) Potato starch
b) Talc
c) Lactose
d) Corn Starch

108 - Role of Surfactant in suppository  improve drug dispersion into hard fatty
excipients, to increase the spreading of the melted suppository on the rectal mucosa
leading to a greater contact surface, to reduce the viscosity of the molten mass and to
reduce the pathway of drug particles to the interface.

109 - Naltrexone used in?


a) Chronic alcoholism
b) Acute alcoholism
c) BZD withdrawal

110 - Case of a child with tonsillitis 3 months ago and successfully cured by Amoxicillin for
5 days
, now he has recurrency, what best option?
1) Amoxicillin for 5 days
2) Amoxicillin for 10 days
3) Amoxicillin + calvulinic acid for 5 days
4) Amoxicillin + calvulinic acid for 10 days
5) Cotrimoxazole

111 – Structure of Phenytoin and


Fosphenytoin?
a) Fosphenytoin is a prodrug of
phenytoin

112 - structure of fatty acid with one of these?


a) Omega 3
b) Polyunsaturated fatty acid
c) Trans fatty acid

113 - Fecal occult blood test used for screening of?


A- Chron's disease
B‐ Colorectal cancer

114 - Side effect of leflunomide?


1-Reanl failure
2‐Hepatic failure (Toxicity)
3-Skin pigmentation
S.E: Nausea, diarrhea, anorexia, alopecia, headache, hypertension, rash, hepatic toxicity,
cytopenias, anorexia, weight loss, pulmonary fibrosis, interstitial lung disease.
Monitoring: Baseline CBC, LFTs, hepatitis B and C serology; CBC, LFTs, creatinine monthly
× 3 months, then Q1–3 months, LFTs monthly if also on MTX. Alcohol restriction may
minimize hepatotoxicity.

115 - Flu vaccine season in North America from and to?


a) Mid of September to Mid of October
b) Mid of October to mid of December
c) Mid of October to Mid of April  this is flu season

116 - Which antibiotic taken on an empty stomach?


a) Mincocycline
b) Azithromycin
c) Ciprofloxacin
d) Cloxacillin  1 to 2 hrs brefore food to ↑ absorption
Azithromycin tablets and liquid can be taken with or without food but the capsules should
be taken an hour before food or two hours after food.

117 – Candesartan, Irbesartan MOA? ARB


Binds to AT1 receptors and prevent formation of angiotensin II, a naturally
occurring substance that causes blood vessels to narrow.

118 - Gout (uric acid) affect joint and ... kidney

119. Categorical variable ... eye color

120 - Liver Cirrhosis? Problem that causes a lack of coagulation factors


Cirrhosis is the severe scarring and poor function of the liver caused by long-term
exposure to toxins such as alcohol or viral infections.
Acute or chronic hepatocellular diseases may display decreases in the vitamin K-
dependent factors (prothrombin; factors VII, IX, and X; proteins C and S), whereas other
parameters remain normal.

121 - Why to give Carbidopa with Levodopa: to decrease peripheral conversion of


levodopa

122- Most of the bacteria in the intestine (Colon)?


a) Facultative anaerobic
b) Anaerobic bacteria
c) Aerobic bacteria
d) Anaerobic bacteria

123 - DDI of St John's wart with Tamoxifen:


St John's wart is a CYP450 enzyme inducer  ↓ tamoxifene ef icacy.
Tamoxifen is a prodrug, and most of the therapeutic effect in treating breast cancer stems
from its metabolite, endoxifen. Since cytochrome P450 (CYP) 2D6 is the most important
enzyme in the production of endoxifen, drugs that inhibit CYP2D6 would be expected to
reduce tamoxifen efficacy. In addition to drug–drug interactions (DDI) involving CYP2D6,
there is growing evidence that enzyme inducers can substantially alter the disposition of
endoxifen, reducing tamoxifen efficacy.

124 - Biopharmaceuticals manufacturing order?


A- Downstream, production, upstream
B- Downstream, upstream, production
C‐ Upstream, production, downstream
Bioprocessing or biotechnology is used in the production of pharmaceuticals, foods,
flavours, fuels and chemicals with the aid of a biocatalyst such as an
enzyme, microorganisms, plant cell, or animal cell in a bioreactor.
Downstream processing is required to remove impurities, bulk-volume reduction and
simultaneous concentration of the desired product from the bioreactor.

125 - Treatment of constipation with IBS?


a) Loperamide
b) Linaclotide
c) Fluoxetine

126 - study that found that there is relation between coffee intake, smoking and cancer
this is?
a. Direct association
b. Indirect association
c. Chance

127 - A drug concentration 0.25 mg/ml Doctor give a dose of 15 mcg/kg for a child wt 18
Ibs Calc the volume should be given to the child
Wt of child = 18 / 2.2 = 8.18 kg
total dose = 15mcg x 8.18 = 122.7mcg = 0.1227 mg
0.25 mg x 1ml = 0.1227 x X X = 2ML 0.25 / 1 ml
0.1227 / X ml
X = 0.1227/0.25= 0.4908
Ans. Is 0.5 ml
128 - What is true about cold chain?
a) Minimum ‐ maximum thermometer kept in the middle shelf
b) Domestic fridge could be used if food kept separate
c) Temperature should be monitored and reordered daily  twice daily

129 – Main Risk factor of osteoarthritis?


a) Obesity
b) Excessive Alcohol intake
c) Caffeine everyday
d) Smoking

130 - First hormone secreted from GIT when PH increase?


a) Gastrin
b) Secretin
c) Pepsin
131 - Hormone digest nucleic acid? Pancreatic nuclease enzymes digest nucleic
acids (DNA and RNA) to nucleotides in the duodenum

132 – Emulsion problem, except:


A- Creaming
B- Coalescence
C‐ Precipitation
D- Sedimentation
133 - What affect o/w emulsion
A‐ Particle size
Website where patients and providers can obtain
B- Solubility information on drug safety alerts, advisories,
warnings, and recalls: can be readily accessed by
patients and providers. Canada Vigilance
134 - Role of MedEfect? Program is part of MedEffect.
Medeffect it is a centralized source for drug
related side effects

135- Role of CADTH?


CADTH provides report on cost
effectiveness, safety, clinical effectiveness of
CADTH: Independent not-for-profit organization funded by Federal and P/T governments of Canada that
new drugs provides recommendations and evidence regarding correct use of drugs, diagnostic tests & medical, dental, and
surgical devices and procedures to support healthcare professionals to make informed clinical decisions.
-reviews drugs and provides recommendations on whether the drug should be added to the formulary
136 - Child 12 years old with temperature 40 what is not suitable?
a) Ibuprofen
b) Acetaminophen
c) Water compress
d) Alcohol compress
e) Naproxen

137 - Patient has AF what is a risk factor of stroke?


a) History of AF
b) Age above 65
c) Smoking
d) Family history B according to chads2 score
e) Weight
Risk factors for stroke are hypertension, smoking, DM, dyslipidemia, excessive
alcohol intake, high body mass index, low exercise, family history of vascular
disease or hemostatic disorders

138 - Prednisone tapering 4 gm qid for 7 days 4 gm tid for 7 days 4 gm bid for 7 days 4 gm
once for 7 days 2 gm bid fot 7 days ‫ … ﻣﺶ ﻓﺎﻛﺮﺓ ﺍﻟﺒﺎﻗﻲ‬and you have tablets of 4 gm and 0.5 gm
77 tablets of 4 gm and 40 tablets of 0.5 gm 77 tablets of 4 gm and 39 tablets of 0.5 gm....

139 - Patient's mother has pneumonia and she got her treatment for 10 days she thinks
she is ok now how to do u confirm her pneumonia is resolved?
a) Sputum Culture  also for acute bronchitis
b) X-ray
c) Blood test (Hematocrit)
d) Antibody tetr
e) Pleural effusion

140 - SSRI need How much to see improvement? Minimum 2 – 4 weeks


When to change to another drug? 6 – 8 weeks

141 - A patient is said to have weak/interrupted stream, dribbling, hesitancy, straining,


nocturia, frequency, urgency & significantly elevated PSA level. this could be indicative of?
a) thyroid carcinoma
b) lymphocytic leukemia
c) BPH
d) chronic renal failure
e) cholestatic hepatitis

142 - What is true about preservative?


a) Multidose injection should have preservative
b) TPN not have preservative
Multiple-dose vial (MDV) describes a vial in which antibacterial preservatives are present
and may be used more than once based on the manufacturer’s recommendations. Single-
dose vial (SDV) is intended to be used only once

143 – Case about a patient complaining about heat and redness, severe, swollen right leg
and calf pain, 2 days ago he was in a long flight 12 hours, what might be the reason behind
his complain? It was a long question with other information.
1) Due to his arrhythmia
2) Orthoscopy on left leg one week ago
3) immobility

144 - Fentanyl receptor


a) Mu
b) Mu and delta
c) Kappa

145 - How much time Terazosin need?


a) 2 weeks
b) 4 weeks
c) 6 weeks
Patient should notice an effect on particular symptoms in 2 to 4 weeks of starting to take
the medication
a minimum of 4–6 weeks may be required to assess whether a beneficial response has
been achieved.

146 – Telangiestacia?
Vasodilatation of coetaneous blood vessels

147- Which drug can be used with grapefruit?


a) Lovastatin
b) Atrovastatin
c) Simvastatin
d) Pravastatin

148 - AF what take to protect


against stroke?
If vulvular (any valves ) —-> warfarin
a) ASA Non vulvular —> DOAG dabigatran
b) Warfarin So answer is D
c) Dalteparin
d) Dabigatran  Useful
alternatives to
warfarin for stroke
prevention

149 -Mechanism of action of PGI2?


a) Increase in platelet aggregation
b) Vasodilatation
Epoprostenol (PGI2) is a member of the family of prostaglandins that is derived
from arachidonic acid. The major pharmacological actions of epoprostenol is
ultimately inhibition of platelet aggregation. Prostacycline (PGI2) from endothelial
cells activate G protein‐coupled receptors on platelets and endothelial cells.

150- Pharmacy personally owned and operated, a taken name and centralized drugs and
marketing?
a) Banner
b) Franchise
c) Mass merchandise
Type Comment Advantages Disadvantages
Franchise Grants the rights • Access to business’ • Fee to be part of a
(license) to operate proprietary knowledge franchise (IDA
under an established & operating methods pharmacies, Medicine
business model • Use the above to run Shop)
the business • Not a form of ownership
successfully but an ownership style
• Easy to start • No capital investment
• Fixed and secure (do not own physical
salary assets).
Banner Own locations and • No central purchase • Capital investment
pay franchises fee or required. require because you own
commission to • Franchising company physical assets.
corporations. provide name and • No fixed salary.
marketing.

151 - Patient came to the pharmacy with diarrhea, what can be the cause of diarrhea?
a) Cholestyramine  constipation (>10%)
b) Misoprostol
c) Phenytoin

152 - Viral Pharyngitis?


a) Not required treatment
b) Treatment with antibiotic
There is no specific treatment for viral pharyngitis. You can relieve symptoms by gargling
with warm salt water several times a day (use one half teaspoon or 3 grams of salt in a
glass of warm water).
Taking anti-inflammatory medicine, such as acetaminophen, can control fever.

153 - Blocker of Adenosine receptor?


An adenosine receptor antagonist is a drug which acts as an antagonist of one or
more of the adenosine receptors. Examples include caffeine (non‐selective),
theophylline, and theobromine.
The use of adenosine receptor antagonists, such as caffeine, and agonists has been
shown to protect against neurological diseases such as spinal cord injury, stroke,
Alzheimer's and Parkinson's diseases

154- Choose a suitable sun block depends on?


A. SPF 8 and above
B. Easy to apply
C. Water resistance
D. UVA and UVB coverage

155 – starch basic unit is


a) Glucose
b) Fructose
c) Maltose
d) Galactose

156 -Teriparatide is used for treatment of?


A. Preeclampsia
B. Osteoporosis
Teriparatide is also a first-line for corticosteroid-treated individual with fractures & low
BMD.
Dose: 20 mcg S.C daily for 24 months (lifetime exposure).

157 - The blood vessel that transfers blood directly to peripheral circulation?
The peripheral vascular system is the part of the circulatory system that consists of
the veins and arteries not in the chest or abdomen (i.e. in the arms, hands, legs and feet).
Blood is carried through the body via blood vessels. The peripheral arteries
supply oxygenated blood to the body.
An artery is a blood vessel that carries blood away from the heart, where it branches into
ever-smaller vessels.
Eventually, the smallest arteries, vessels called arterioles, further branch into tiny
capillaries, where nutrients and wastes are exchanged.
Capillaries come together to form venules, small blood vessels that carry blood to the
peripheral veins, a larger blood vessel that returns blood to the heart.
158 - Canagliflozin mechanism of action? Sodium‐Glucose Co‐ transporter 2 (SGLT2)
Inhibitor
They↓ glucose reabsorption, ↑ urinary glucose excretion, ↓ blood glucose.
SGLT2 also decreases reabsorption of sodium and causes osmotic diuresis.

159 -Itraconazole dose in toenail infection (onychomychosis)?


Oral therapy
 Itraconazole duration: Fingernails: 6 wk. Toenails: 12 wk
 Floconazole duration: Fingernails: 12–16 wk Toenails: 18– 26 wk
 Terbinafine duration: Fingernails: 6–12 wk. Toenails: 12– 24 wk.
Topical therapy
 Ciclopirox olamine 8% lacquer monotherapy, a pyridone antifungal, daily for 48
weeks.
 Efinaconazole 10% topical solution, triazole antifungal, applied to affected nail once
daily for 48 w.

160 – how to increase protein diffusion through skin?


a) Use Protease
b) Iontophoresis

Transdermal Delivery of Proteins


 Proteins are complex molecules with large molecular weights, acid–base side chains
and are polar in nature.
 Stability issues, along with their complex nature, make proteins difficult drug
candidates for delivery.
 Currently, proteins are being predominantly administered by the parenteral route.
However, since most proteins have short half-lives, this route has the disadvantage
of the requirement for repeated administrations and low patient compliance.
 The skin, the most accessible organ of the body with a large surface area, offers an
appealing alternative for delivering proteins into the systemic circulation. However,
stratum corneum, the outermost barrier of skin which is made up of dead
keratinocytes, acts as a rate limiting barrier.
 To overcome the stratum corneum barrier, several enhancement techniques have
been investigated, some of the major ones being chemical enhancers,
iontophoresis, microneedles, sonophoresis, laser ablation, thermal ablation,
radiofrequency ablation, jet injectors and electroporation.
 Iontophoresis and electroporation are electrically assisted enhancement techniques
where current is applied to increase drug delivery through the skin.
 On the other hand, microneedle technology is a physical enhancement method
which is a bridge between conventional hypodermic needles and passive
transdermal patches. Micron-sized needles porate skin in a minimally invasive
manner to enable delivery of proteins.
 Similarly, all the enhancement methods enable delivery of drugs in different ways
and bioavailabilites of ≥50% may be possible

161 – used to treat hypercalcemia?


A- Calcitriol
B- Thiazide
C‐ Pamidronate

162- DNA translation order?


A- DNA  mRNA  PROTIEN  this is transcription process (DNA  mRNA)
B- DNA  tRNA  PROTIEN
C‐ mRNA  tRNA  PROTIEN
D‐ mRNA  DNA  PROTEIN ↔ revers transcriptase process (mRNA  DNA)
Translation is a process by which the genetic code contained within an mRNA molecule is
decoded to produce the specific sequence of amino acids in a polypeptide chain.
It occurs in the cytoplasm following transcription and, like transcription, has three stages:
initiation, elongation and termination.
 Initiation ("beginning"): in this stage, the ribosome gets together with the mRNA
and the first tRNA so translation can begin.
 Elongation ("middle"): in this stage, amino acids are brought to the ribosome by
tRNAs and linked together to form a chain.
 Termination ("end"): in the last stage, the finished polypeptide is released to go
and do its job in the cell.

163 - Which of the following can be taken in the morning?


A‐ Atorvastatin  Can be taken anytime of the day, because it has long half‐life
B- Simvastatin  with evening meal (also, Lovastatin, bezafibrate)
C- Fenofibrate  with largest meal of the day
D- Pravastatin  at bed time

164 - Symptoms of irritable bowel syndrome?


1) GI bleeding
2) Gastric upset
3) Abdominal discomfort

165 - Osmotic Pump: zero order kinetics doesn't depend on concentration


Osmotic pumps consist of an inner core containing drug and osmogens, coated with
a semipermeable membrane. As the core absorbs water, it expands in volume,
which pushes the drug solution out through the delivery ports

166 - Ampicillin in 5% dextrose degrades by first order kinetic, at rate constant of 0.026 h.
What is the shelf life of ampicillin?
a) 4 hr  T90 = 0.105 / k = 0.105 / 0.026 = 4hrs
b) 8hr
c) 2hr
d) 16hr
e) 12hr

167 - Look at women from 1972 and check for cardiovascular and other issues this is?
a) Case series
b) Case control  LOOK FOR 2 GROUPS AND COMPARE HISTORIES
c) Case report  LOOK FOR SINGLE MEDICAL OCCURRENCE

168 - Which of the following medications covered by health Canada?


A- Asthma medication for kids
B‐ Antihypertensive for hospitalized patients
C- Senior medication
D- Chemotherapy for outpatients
E- Papilloma Vaccine

169 - Calculation of AR, RRR

170 – HMG CoA is derived from?


a) Acetoacetyl CoA
b) Mevalonate
c) Heme A

171 -Major source of energy in the body:


Adenosine triphosphate (ATP)

172 - Question about filter size for


sterilization: the size 0.22 u

173- In biotechnology, issue of the host is


A. Ligand
B. Metal
C. pathogen interaction (cells can be
infected with viruses)

174 - What correct about Aerochamber


A. Wash with isopropyl alcohol
B. Breath 2‐ 3 times slow & deep
C. Hold breath for 30 seconds

175 - Asthmatic patient (child) not on any treatment what do u start as a first choice?
A. Terbutaline or Salbutamol

176 - He is still waking up at night? Formoterol and corticosteroid

177 - Patient still waking up although currently using low dose of the upper med u chose.
a) Keep the regimen as it is
b) Give oral corticosteroids
c) Refer to emergency
d) Increase dose of correcting agents  after that add oral corticosteroid

175- Vancomycin dose was given at 10:00 and 22:00. Css is reached and trough level was
found to be 22 units at 11:00 so what to do:
a. Increase dose
b. Decrease dose
c. Keep the same
d. Measure at later time
e. Extend the interval
Vancomycin Sampling time Normal values: Dosage should be stable
IV Peak: 30 min–2 h after 1 h Peak: 25–40 for 20–30 h.
infusion. mg/L. Trough: Half-life about 6 h in
Trough: ≤ 5 min prior to 15–20 mg/L. patients with normal renal
next dose. function. Usually obtain
trough levels.
179 - Makes the PH 7.4 what is responsible? Arginine  the most basic amino acid

180 - Which organ has most Gram-positive bacteria colonization:


a. GI b. skin c. pharynges

181 - What is the common factor that warfarin and heparin inactivate?
A. X B. VII C. II

182 - Protamine when given for heparin bleeding? It totally inactivates the heparin
Intravenous protamine sulphate can rapidly reverse the anticoagulant effects of heparin. Protamine sulphate is a
basic protein derived from fish sperm that binds to heparin to form a stable salt.
183 - Confidence interval 95% and 99% what is true
A. 95 is wider than 99 CI B. They have different standard of error

184- If u want to counsel a patient on hypoglycemia symptoms what do counsel him on?
A. Fruity smell breath
B. Sweating
C. Polyurea
D. Thirst
 Mild to moderate hypoglycemia has autonomic symptoms: sweating, tremors,
tachycardia, hunger, nausea and a general sensation of weakness. Easily treated with
an oral source of sugar “RULE OF 15”. Fifteen grams of glucose will raise the blood
glucose approximately 2 mmol/L within 20 min.
 Severe hypoglycemia requires assistance in its recognition and/or treatment.
Neuroglycopenic symptoms such as confusion, altered behaviour, difficulty speaking
and disorientation can progress to seizures and coma that prevent the patient from
appropriately treating the hypoglycemic episode. In unconscious patients with no IV
access, 1 mg of glucagon IM or SC temporarily increases blood glucose, allowing for the
intake of oral carbohydrate.

185 - A drug cloned from an available drug in the market, so the drug is? Biosimilar
 A biosimilar is a new, highly similar version of a biologic drug that comes to the
Canadian market after the patent for original product has expired.
 Biosimilars were previously called subsequent entry biologics (SEBs).

186. A question why long acting antibiotics like Azithromycin (Macrolides) (short course)
have high VD would be resistant due to: little exposure for a longer period of time
187 - Patient has Psoriasis and Newly diagnosed with SLE, which one can be used?
a) Infliximab  FOR SEVER PSORIASIS
b) Mycophenolate mofetil  FOR SEVER SLE
c) Anakinra  NOT USED IN BOTH DISEASES, it is Interleukin‐1 (IL‐1) Inhibitors for
RA
d) Hydroxychloroquine  INITIAL THERAPY FOR SLE

188 - Water for injection produced by?


I. Ultrafiltration
II. Double distillation
III. Reverse osmosis
a. I only b. III only c. II and III d. I and II e. I, II, and III

189 -Lyophilization is? Lyophilization or freeze drying is a process in which water is


removed from a product after it is frozen and placed under a vacuum, allowing the
ice to change directly from solid to vapor without passing through a liquid phase.

190 - 0.3 % Nitroglycerin in hydrophilic petrolatum mitt 120 gm, you have 30gm which is
true?
a) Use 3 gm Nitroglycerin
b) Use surfactant
c) Use glycerin as levigating agent

191 - Which drug work on ppar alpha and ppar gama?


a) Sitagliptin  DPP ‐ 4 Inhibitors
b) Fenofibrate  PPAR α
a) Pioglitazone  Dual PPAR α / γ agonist (also Lobeglitazone)
b) Rosiglitazone  PPAR γ

192 - Given a structure and the question is how many enantiomers. The structure had 3
chiral centers so 2 to the power of 3 is equal to 8

193 - Given 2 structures and NH2 is removed from the reaction what is this reaction?
Deamination

194- Choose the suitable enzyme for the reaction?


Delaminases

Oxidative deamination is stereospecific and is catalyzed by L- or D-amino


acid oxidase.
195- Determine type of isomerism

196 - Which part is responsible for the


activity? pharmacophore

197 - Mechanism of Niacin?


a) Increase hydrolysis of TG
b) Decrease synthesis of free faty acids
c) Increase utilization of triglycrides
Niacin, greatest HDL raising effect 15 to 35%, TG reduction 20 to 50% (higher
dosage). Strongly inhibits lipolysis in adipose tissue, thereby reducing production of
free fatty acids.

198 -Vit K has which nucleus?


Vitamin K and its derivatives contain a 2‐methyl‐1,
4, naphthoquinone nucleus with a lipophilic side
chain. The structure is similar to warfarin and other
coumarin-like anticoagulants, which function
as vitamin K antagonists.

199 - Dystonia cause by antipsychotic how to treat:


benztropine

200 - A man came to measure his blood pressure, what is the most appropriate response?
a) Lay down before measurement
b) Wait for 30 mins before measuring
c) Take an average of 10 measurements
d) Don’t talk while you are measuring the blood pressure
e) Should measure both arms

201 - The hormone secreted from Anterior Pituitary?


a) Luteinizing hormone
b) Thyroxine
c) ADH

202 - After instilling eye drops why we press on the eyes?


a) To reduce redness
b) To avoid drug leakage
c) To decrease systemic absorption from nasolacrimal duct

203 - Primary cause of embolism or thrombus formation


a) Dyslipidemia
b) Malignancy Cancer is a primary cause for deep vein thrombosis

204 - Which antidepressant will increase weight and sedation?


a) Mirtazapine
b) Bupropion
c) Citalopram
d) Venlafaxine

205 - 26 years old pregnant women with diabetes, what can be used? Insulin

206 - Anti histamine Ophthalmic eye drop:


A. Olopatadine
B. Naphazoline
C. Cromolyn sodium
D- Prostaglandin analogue

207 - A statistical study and how to do randomization?


a. by cards and envelopes
b. by date of entry to hospital
c. by availability of intervention

208 - Governance, funding, and delivery of service?


a. federal government
b. provincial and territory
c. health Canada

209 - What is pharmacotherapy?


a. systematic analysis of drug related problem
b. written plan
c. treatment using drugs rather than radiation & surgery.
Pharmacotherapy is therapy using pharmaceutical drugs, as distinguished from therapy
using surgery (surgical therapy), radiation (radiation therapy), movement (physical
therapy), or other modes.
210 - Role of a health care professional: work as legally permitted
Alzheimer’s and Parkinson’s tremors are similar ... unilateral inc
with rest improve by movement.
211 - Alzheimer tremor characteristics: but essential tremor is bilateral and increases with movements
a. bilateral. decreases with rest

B. mostly at rest and improves with movement

212 - Naproxen given one structure and asked


that it exists in a mixture of 2 structures what
are they?
a. optical isomers
b. geometric
c. racemic
Naproxen is a chiral compound, meaning that
it can exist in one of two isomeric forms.
A racemic mixture is one that contains both of
the isomeric forms in which a compound can
occur.
The two parts of the racemic mixture, the R and S forms, then had to be separated from
each other.

213 - Dabigatran:
a. given once daily for AF  for venous thromboembolism
b. has less interactions than warfarin
c. no GI bleeding  can cause bleeding

214 - A case of a patient complaining of blood in stool and decrease in weight: refer to
doctor

215 - Given 5 drugs with instructions which is unaccepted abbreviation according to ISMP

216 - A.S: left ear

217 - Intrinsic and extrinsic pathway of blood clotting: both activate factor 10 (X)

218 - Calculate NNT 220 - Calculate risk ratio

219 – Calculate cost effectiveness 100000/0.5=200000


220 - Which drug decrease oral
contraceptives effect: oxcarbazepine

222- Drug A cost and effect, is more than


drug B so:
a‐ implement drug A because it is
more cost effect than B.
b- don’t implement drug A (can’t remember the rest)

223 - Cause of pneumonia in children: S. Pneumonia

224 - What to do at high season of vaccine:


assign technician to prepare patient for the pharmacist review

225 - Diabetes Insipidus caused by the deficiency or resistance of:


A. Insulin
B. Glucagon
C. ADH or vasopressin
D. estrogen
E. Progesterone

226 - A case where the patient is given Clarithromycin and she is on Citalopram what is
the concern.
Both drugs can prolong QT wave

227 - Duchenne syndrome what is deficient: Dystrophin


Duchenne muscular dystrophy (DMD) is a genetic disorder characterized by
progressive muscle degeneration and weakness due to the alterations of a protein
called dystrophin that helps keep muscle cells intact. There is no known cure
for Duchenne muscular dystrophy.
Treatment aims to control symptoms to improve quality of life. Steroid drugs can
slow the loss of muscle strength. They may be started when the child is diagnosed or
when muscle strength begins to decline
228 - Natural immune mechanism in female against UTI?
a) Length of Urethra
b) Urine Acidic PH

229 - Which medicine is converted to drug of abuse?


a. Amphetamine prolonged release
b. Dextroamphetamine regular release

230 - Linear statin structure converted to cyclic what is the reaction: Lactonization

231 - Untreated hyperthyroidism what will lead to Atrial fibrillation

232 - Complementary therapy covered by health Canada: homeopathy

233 - Psoriasis treatment mild: topical hydrocortisone

234 - Psoriasis treatment severe covering 35% of body treatment: etanercept

235 - Mechanism of action of Bisphosphonate?


a) Inhibits Osteoblast
b) Inhibits Osteoclast
c) Acts on Ca receptors on bone

236 – BPH patient what to educate about the treatment:


a) Take St johns wart  it is saw palmetto that used in BPH
b) No abrupt cessation
237- Nasal route: avoid FPE Avoid first pass metabolism

238 - Scopolamine? Blocks muscarinic receptors

239 - In pharmacy what is it called splitting the work and checking each from different
aspects:
workflow analysis
Workflow Analysis is the process of breaking down the performance of a workflow and
examines trends for improvement. By looking at a workflow at a granular task level,
business users can tweak processes for optimal efficiency and workplace productivity.

240 - Raloxifene action on Selective Estrogen Receptor Modulators. Raloxifene acts as


a partial agonist of the ERα and as a pure antagonist of the ERβ.

241 - Cloning involves which cells: B cells


Clone (B-cell biology) The process of immunological B-
cell maturation involves transformation from an undifferentiated B cell to one that
secretes antibodies with particular specificity.

242 - Calculate K

243 - Calculate percent of cortisone 0.1% given ration as 1:3


244 - Calculate mmole of potassium

245 - Patient given TMP/SMX and ACEI what to monitor: potassium level

246 - Stable angina: triggered by vigorous exercise and causes chest pain

247 - Enteric coated drug release depends on: pH of GI

248 - What is in CPS: has information about manufacturing companies

249 - Vitamin to be given to patient on INH?


a) Thiamine
b) Riboflavin
c) Pyridoxin
250 - Medication causing nephrotoxicity?
a. warfarin
b. cisplatin
Also, Carmustine, ifosfamide, methotrexate, mitomycin, streptozocin

251 - COPD case the patient is taking PRN salbutamol and Glycopyrroinum (long acting
antimuscarinic) now his case is worsening what to do?
a. change salbutamol to regular
b. change to salmeterol
c. change glyocpyrronium to combination of glycopyrroinum and indacaterol
d. adds inhaled cortisone

252 - Mechanism of action of Gemfibrozil


Activate Peroxysome profilator activating receptors alpha (PPAR alpha)

253 - Absorption of medicine through alimentary tract through which layer: mucosa
The mucous membrane‐lined tube of the digestive system that extends from the
mouth to the anus and through which food passes, digestion takes place, and wastes
are eliminated; it includes the pharynx, esophagus, stomach, and intestines.
alimentary tract digestive tract

254 - Innate immunity is humoral plus what? cell mediated

255 - Febuxostat mechanism: xanthine oxidase inhibitor

256 - Current assets includes: Account Receivable

257 - Patient with Alzheimer what to do regarding counselling:


talk little and give written Instructions

258 - Benzodiazepine acts on GABA receptor


Enhance the effect of the neurotransmitter gamma-aminobutyric acid (GABA) at the
GABAA receptor, resulting in sedative, hypnotic (sleep-inducing), anxiolytic (anti-anxiety),
anticonvulsant, and muscle relaxant properties.

259 - Aerobic bacteria found in which part of the body?


a) colon
b) skin
260 - Selective Cox2 inhibitor?
a) Naproxen
b) Meloxicam
c) Celecoxib

261 - Avoid what in kidney stone: calcium

262 - Calculate infusion rate

263 - With which medicine we have to monitor eye: hydroxychloroquine

264 - Schizophrenia diagnosis based on: hallucinations and delusion

265 - Choose the right combination: ferrous fumarate has highest elemental iron

266 - Tyrosine kinase belongs to which enzyme group: transferase


Tyrosine kinase can transfer a phosphate group from ATP to a protein in a cell.

267 - What will decrease drug absorption from MDI: large particle size

268 - Advantage of glass over plastic:


a. glass more durable
b. glass more heat resistant

269 - If patient is illiterate what to do with consent:


a. explains to relative
b. give summary of what is in the consent
c. explains the risks and benefits of procedure

270 - What happens in zero order: hepatic enzyme saturation

271 - Cause of Osteoporosis: long term use of corticosteroids

272 - What happen when drug with low hepatic clearance dissociate from tissue & plasma
protein?
a- Decrease in half life
b- Increase in volume of distribution
c‐ Increase in renal clearance
273 - Calculate hepatic clearance given total clearance as 2.8l/hr the drug is given IV. Total
dose was 150mg and the drug found in urine was 60mg. answer was 1.7l/hr

274 - Which parenteral give prolonged effect: depot

275 - Drug follows first order kinetics given as IV bolus, one compartment, a plot of
concentration against time on semilog scale will show
a) Straight line
b) Curved line

276- A case of patient with symptoms of hyperthyroidism what lab test to diagnose: TSH
The DOC for paroxysmal atrial
277 - What is the treatment for paroxysmal atrial fibrillation: fibrillation is catheter ablation
otherwise beta blockers are first
a. amiodarone b. digoxin c. diltiazem d. amlodipine choice unless contraindicated . Use
Non DHP- CCB

278 - Absorption of monoclonal antibodies through: vascular and lymphatic

279 - Calculate dose of Morphine from solution to tablet

280 - A man worried about his mom she has delirium in long term home. She started to be
aggressive so start taking Risperidone. He is worried about this medicine because?
a. Risperidone is only for psychotic symptoms of delirium
b. Risperidone is associated with increased risk of falls

281- same case above; how to reassure patient:


a. Tell him to refer to doctor
b. Tell him that long term facility will take care of his mom
c. Tell him the she should be kept in a room alone
d. Tell him we can do nothing for her because she is not our patient

282 - same case above; the best place for the patient concern is to ask?
a. Doctor
b. Pharmacist
c. Nurse

283 - what not to give with Rizatriptan because it also acts on serotonin? Meperidine
JULY 2018
1. Overdose of ascorbic acid causes?
a) Kidney stones
b) Gastric bleeding

2. Citalopram and Escitalopram chemical structures are? Optical


isomers ‐ enantiomers

3. Tamsulosin’s is metabolized by? CYP450 3A4, 2D6

4. Where are proteins synthesized?


a) As per translation and transcription
b) In nucleus
c) In cytoplasm C
In ribosome which is in cytoplasm outside the nucleus

5. The enzyme that is in Acetyl-CoA reaction? (something like that)


a) Thioester
b) Carbamate

6. Varenicline side effect?


a) Headache
b) Seizures
c) muscle cramps
d) Mood Change
S.E: Nausea (30%); may be mitigated by taking on a full stomach, increasing water
intake or reducing dose. May cause insomnia; take second daily dose at suppertime.
Neuropsychiatric side effects such as suicidal/homicidal ideation have been
reported; monitor closely for changes in mood/behaviour.

7. Which part of the body is located in the dorsal part of the body?
Spinal cord
8. If someone is allergic to eggs, how will you advice to him to take the flu vaccine?
a) Take it in the emergency department or where there are doctors around
b) Do skin test before taking New guidelines : taken in the pharmacy
c) Be prepared to wait 2 hours

9. What substance is released in tobacco smoking? Nicotine Acrolein

10. What are the most common bacteria that causes gastroenteritis?
Campylobacter jejuni is usually the most common cause of
community‐acquired inflammatory enteritis and food poisoning

11. Which of the following is correct in regards to asthma?


a) Pneumococcal vaccine helps in preventing acute exacerbations of asthma
b) Family history plays a role in asthma
c) Smoking is the major risk factor in asthma
d) Viral infections precipitate acute exacerbations of asthma

12. A case of an old woman who takes salbutamol and tiotropium and
hydrochlorothiazide is suffering of productive cough and runny nose for 2 days.
What med would you give her?
a) Echinacea to treat the common cold
b) Give her dextromethorphan and guaifenesin to take all day long
c) Refer her to the doctor to determine the reason for her worsening COPD
symptoms
d) Take diphenhydramine to sleep because no need for drug therapy.
e) Another cough medication

13. The daughter of a cancer patient comes to the pharmacy with a prescription of
hydromorphone and lactulose for her sick mother who is already taking morphine
and (some other laxative). What is your concern as a pharmacist?
a) Forged prescription Overdose
b) Taking two laxatives at once
c) wrong pain managment

14. A common name for Otitis externa? Swimmer’s ear

15. In ECG, Ventricular depolarization is?


a) QRS
b) PR
c) ST

16. The highest hormone during ovulation in the menstrual cycle is?
a) LH
b) Estrogen

17. Deficiency in testosterone results in 2ry hypogonadism in males, it is produced by?


a) Adrenal gland
b) Posterior pituitary
c) Anterior pituitary
d) Hypothalamus
Testosterone is produced by the gonads (by the Leydig cells in testes in men and by the
ovaries in women), although small quantities are also produced by the adrenal glands in
both sexes.

18. What hormone raises the basal body temperature of women during the
menstrual cycle? Progestrone
thiamin (B1)
riboflavin (B2)
19. Thiamin is the name of which vitamin? niacin (B3)
pantothenic acid (B5)
a) Vit B3 pyridoxine (B6)
b) Vit B1 biotin (B7)
folate or 'folic acid' when included in supplements (B9)
c) Vit B2 cyanocobalamin (B12).

20. A case about a lady who wants to get pregnant and is taking valproic acid and
phenytoin for epilepsy, what do you suggest?
a) Stop valproic acid
b) Stop phenytoin
c) polytherapy is better than monotherapy in this case
d) Stop anticonvulsants, no need for them during pregnancy
Enzyme‐inducing AEDs (e.g., carbamazepine, phenytoin, phenobarbital) can
potentially increase the degradation of vitamin K in the fetus. Vitamin K is
routinely given to all newborns at delivery to prevent hemorrhagic disease,
which is likely sufficient to overcome any AED‐induced vitamin K breakdown.
21. Same above case, how much folic acid should the patient be taking?
a) 1mg a day  FOR PATEINTS WHO ARE USING METHOTREXATE
b) 3 mg a day
c) 5 mg daily
d) 0.4 mg daily  FOR PREGNANT WOMEN

22. What is the best prevention for cataract?


a) Taking multivitamins
b) wearing eye protection
c) administer artificial tears
d) Vit B complex
e) Steroids eye drops
Excessive exposure to sunlight is a risk factor for
cataract

23. Which of the following causes ototoxicity? Amikacin

24. First line therapy for a Post MI patient with heart failure and blood pressure of
160/90 and ventricular enjection fraction of 35 %?
a) ACEi + BB
b) ACEi + HTCZ
c) BB+ Digoxin
d) ACEi + Digoxin
e) ARB +BB

25. Antipsychotic med given for +ve and –ve symptoms?


Aripiprazole

26. patient taking Allopurinol but is not suffering of renal failure, what med will you
change him to? Febuxostat
It is an option for patients with severe renal insufficiency because very small
quantities are excreted renally compared with allopurinol

27. what neurotransmitter released form stimulation of Vagal nerve? Ach

28. To protect from ulcers?


a) Misoprostol
b) Omeprazole

29. Women comes to pharmacy complaining of a rash on both her arms that she got
while gardening, it’s very itchy and keeping her up at night, what do you give her?
a) Hydrocortisone 1% ointment
b) Diphenhydramine 0.5% ointment
c) Refer to physician
short-term intermittent use of first-generation antihistamines (e.g., diphenhydramine,
hydroxyzine) for sleep disturbance in AD, due to their sedating properties, but not as
substitute for better disease control.
30. When do you refer a child with diaper dermatitis to a physician?
a) When the rash is red with white scaly patches
b) Rash has blisters and pus

31. Acne med? Apply on acne spots only

32. Which of the following cause epigastric pain and worsens GERD?
a) Nifedipine
b) Naproxen
c) Acetaminophen
33. What cells line the blood brain barrier?
a) Astrocytes
b) Satellite cells
c) Schwann cells  principal ganglia of PNS
d) Neurons

34. Med causes venous pooling? Nitroglycerin

35. Which is considered a food additive?


a) Antioxidants
b) Probiotics
c) Neutraceuticals
Nutraceuticals (Vitamin C, Carotenoids,
Resveratrol)
They are important natural bioactive compounds that confer health‐promoting and
medical benefits to humans.

36. What is not a symptom of diabetic foot?


a) Neuropathic pain
b) Opening of the skin
c) Erythema
d) Foul smell
e) Purulent discharge

37. Patient taking salbutamol but his asthma symptoms are not controlled, what
med do you add? Long acting B agonists  then Inhaled corticosteroids

38. Radial nerve damage causes inability of? Extension of the elbow

39. Sitagliptin mechanism? DPP‐4 inhibitors

40. Ondansetron is? 5‐HT3 receptor antagonist

41. which of the following enzymes is not a Phase 2 metabolic enzyme?


Aldehyde dehydrogenase
42. Conjunctivitis in newborns caused by which bacteria?
N. gonorrhea
Bacterial: (hyperacute, acute or chronic)
 Neonates: Chlamydia trachomatis, Staphylococcus aureus, Haemophilus influenzae,
Streptococcus pneumoniae, Neisseria gonorrhoeae, Neisseria meningitidis
 Children: H. influenzae, S. pneumoniae, S. aureus
 Adults: S. aureus, coagulase-negative Staphylococcus organisms, H. influenzae, S.
pneumoniae, N. gonorrhoeae, N. meningitidis
Viral: Adenovirus and Herpes simplex virus

43. Extended spectrum B-lactamase ESBL resistance?


ESBLs are the enzymes that have the ability to hydrolyze and cause resistance to various
types of newer β-lactam antibiotics, including the expanded-spectrum (or third
generation) cephalosporins (eg. cefotaxime, ceftriaxone, ceftazidime) and monobactams
(eg. aztreonam), but not the cephamycins (eg. cefoxitin and cefotetan) and
carbapenems (eg. imipenem, meropenem and etrapenem).
These enzymes are sensitive to β‐lactamase inhibitors (sulbactam, clavulanic acid,
and tazobactam)

44. A drug that causes health event? (a very weird question, can’t remember
words)
When it’s eliminated there will be no risk for health event
45. All are true about warfarin are true except?
Causes renal failure

46. Cause of malaria? Plasmodium species

47. Metabolic acidosis is cause by? ASA overdose


Treatment: Sodium bicarbonate (NaHCO3)

48. Low dose drug with mucositis? Methotrexate


CHIMERIC (25% mouse): XIMAB
HUMAN (100%) : UMAB
49. Rituximab is a? Chimeric HUMANIZED (10% mouse 90% human):
ZUMAB
MURINE (100%): MOMAB
-Mab (monoclonal antibodies) -
50. What is Blood pressure? Cept (soluble receptors)
cardiac output * peripheral resistance Mib for proteases
Nib for inhibitors (tyrosine kinase inhibitors)

51. A drug that causes parkinsons? Metoclopramide

52. A structure that has a triangle and Cl and positive


charge? Anticancer

53. COPD is caused by? Oxidative stress in airways


Airway oxidative stress is broadly defined as an imbalance between prooxidative and
antioxidative processes in the airway.
Airway oxidative stress has been associated with worse disease severity,
reduced lung function, and epigenetic changes that can diminish response to steroids.
Chronic obstructive pulmonary disease (COPD) is a pathological lung condition directly
related to cigarette smoke. There is now a large body of evidence that an increase in
oxidative stress plays an important role in the pathogenesis of COPD.
Markers of oxidative stress have been demonstrated in the airways, breath condensate,
sputum, blood, and urine of smokers and patients with COPD. There is also evidence that
endogenous antioxidants such as reduced glutathione (GSH) and lung GSH biosynthesizing
enzymes are significantly decreased in these patients

54. Women that has a history of breast cancer and is now


suffering of hot flushes, what med to give her?
a) Venlafaxine
b) Estrogen

55. Raloxifene?
Selective Estrogen Receptor Modulator

56. Rivastigmine is the drug of choice for?


Lewy body dementia and alzheimer disease

57. Nonlinear kinetics is for which of the following drugs?


Phenytoin

58. DOC for myoclonic seizures? Valproic acid

59. Codeine is a? Prodrug of morphine

60. Drug for hyperthyroidism? Methimazole

61. Semen fluid is produced from? Seminal vesicle

62. Patient has A blood antigens and Rh. What is his blood
type?
a) A positive
b) A negative

63. heme structure contains? Tetrapyrrole ring


A heme is an organic, ring-shaped molecule. Due to its
special structure, a heme is capable of holding, or “hosting” an
iron molecule. A heme is made from 4 pyrroles, which are
small pentagon-shaped molecules made from 4 carbons and 1
nitrogen. Four pyrroles together form a tetrapyrrole.

64. therapeutic goal in the treatment of a child with croup?


a) To decrease hospitalization
b) to drop his fever in 24 hrs
c) To prevent antibiotic use
65. A traveler from Europe ran out of his foreign med. His med is not available in
Canada. Which reference do you use to look for its equivalent? Martindale

66. Where to look up information about a new marketed drug?


Drug Product database DPD

67. Where to look up pediatric dose of prednisolone?


a) e‐CPS
b) Compondium of Therapeutic for minor ailments

68. Something about the funding for health Canada?


a) Primarily government funded
b) Entirely government funded

69. Structure of Zidovudine, Zidovudine is a


prodrug activated by what reaction?
a) Acetylation
b) Phosphorylation
c) Sulphonation
ZDV is a prodrug and must be activated by phosphorylation in lymphocytes to exert its
antiviral action.

70. Where is the DNA synthesized?


a) Nucleus
b) Mitochondria
c) Ribosomes
DNA replication occurs in the cytoplasm of prokaryotes and in the nucleus of
eukaryotes. Regardless of where DNA replication occurs, the basic process is the
same.
71. When is the drug submission form submitted? After phase 3

72. A 19-year-old girl wants to use plan B, what makes you refer her to doctor?
a) because she is only 19
b) because she has had multiple partners in the past 2 months
c) she has an irregular menstrual cycle.
d) Used 2 times before
Also, can’t use it if she has abnormal vaginal bleeding
73. What is the duty of a pharmacy manager in a pharmacy?
 Pharmacy manager is responsible for the actual management and operation of
the pharmacy.
 Supervising staff to ensure that they are compliant with practice standards,
 Develop, implement and maintain policies and procedures to comply with the
legislative requirements of operating a licenced pharmacy.
 Responsible for reporting changes to the pharmacy’s operational information,
such as hours of operation, pharmacy staff roster and types of pharmacy services
provided to the College.

74. Same as the previous question in the exact wording and the exact choices but is
asking about the job of an accounting manager in a pharmacy?
a) Responsible for the equity capital
b) Responsible to calculate the federal and provincial taxes
https://resources.workable.com/accounting-manager-job-description

75. Institutionalized elderly usually has deficiency of what of the following?


a) Vitamin B1
b) Vitamin D

76. Who should go to the house of an elderly to check that there is nothing
dangerous there that could cause falls? Nurse
https://www.registerednursern.com/private-duty-nursing-private-duty-nurse-job-
description/

77. Side effects of Amiodarone


a) Skin pigmentation
b) Neuropathy
c) Pancreatitis

78. What drug metabolism is affected by valproic acid?


a) Lamotrigine
b) Leviteracetam
c) Vigabatrin
d) Bromocriptine
e) Lorazepam
Metabolism markedly inhibited by valproic acid/divalproex sodium and
increased by enzyme inducing AEDs (carbamazepine, phenytoin, phenobarbital,
primidone).

79. Person with otitis media and is allergic to pencillins and erythromycin gave him
an upset stomach before, what med do you give him?
a) Azithromycin
b) Cotrimoxazole
c) cephalexin

80. A case of an old woman who fell and fractured her hip, why wouldn’t you test
her bone mineral density?
a) She already fractured her hip
b) She is old
c) She already did a surgery
d) She doesn’t have a history of fracture
e) She is a female

81. What supplement can be added to reduce fall and improve bones?
a) Calcium
b) Vitamin D
c) Vitamin B12

82. Bisphosponate, what should we consider to make decision to give Bisphosphonate?


a) Based on how she responded to physiotherapy
b) Site of fracture
c) Side effects and long duration of therapy
Clinical challenges in using bisphosphonates to treat osteoporosis include
appropriate selection of patients for initiating therapy, choosing which
bisphosphonate to use, monitoring therapy to assure that medication is taken
correctly and the desired effect is achieved, determining when drug discontinuation
should be considered, and managing side effects, possible side effects, and fear of
side effects. Strategies for treating patients with bisphosphonates should consider
each of these issues.

83. A case of an obese diabetic patient taking metformin + Glyburide, and has
hypoglycemia, Hb A1c 7.8. she is a widow who resigned 3months ago and became
recently became more physically active (walking, swimming, gardening) and depends
on restaurant food in her diet, symptoms of hypoglycemia?
a) Sweating
b) Thirst
c) Nocturia
d) Fatigue

84. Same above case, lately she had 3 hypoglycemic attacks, what is causing her
hypoglycemia?
a) High dose of metformin
b) High dose of glyburide
c) Became more physically active
d) Something about Food related

85. Same above case, who do you refer the patient to?
a) Dietitian
b) Occupational therapist
c) physiotherapist
d) Life coach
e) Fitness trainer

86. Calculate the T90 of a warfarin dosage (given) following zero order
kinetics. Ans. Formula found on formula sheet provided in the exam booklet

87. What antiobiotic is given for uncomplicated cystitis for 3 days for a patient
allergic to sulfa drugs?
a) Cephalexin
b) Nitrofurantoin
c) Moxifloxacin D
Cephalexin 7 days
d) Levofloxacin Levo 3 days

88. In what disease does all the household need to be treated?


a) Pinworms  scabies also
b) Head lice Pets do not transmit human lice and should not be treated, but should be treated in scabies and pinworm.
No need to treat all family members in lice but in Scabies & pinworm all should be treated even if
c) Impetigo asymptomatic

89. A case about head lice, what medicine is given that does not cause neurotoxicity
as father wants a non-neurotoxic drug?
a) Permerthrin
b) Dimethicon
c) Ivermectin

90. Same above case, who needs to be treated?


a) Only people with head lice or nits
b) All house hold contacts should be treated
c) Other Children in house who donot have lice should be treated

91. which of the following is a job a technician can do?


a) Receive a verbal prescription of alprazolam
b) Show a patient how to use a diabetes monitor

92. A case about a hypertensive patient with BPH, received a prescription of


finasteride and terazosin, regarding the prescription?
a) Polypharmacy
B or C according to the details
b) Duplication of therapy
c) Suitable treatment for BPH
Finasteride improves long-term urinary symptoms versus placebo, but is less effective
than doxazosin. Long-term combination therapy with alpha blockers
(doxazosin, terazosin) improves symptoms significantly better
than finasteride monotherapy.

93. same case above, why was the patient not given tamsulosin instead of terazosin?
a) Age
b) Pain
c) Hypertension

94. same case above, what drug is contraindicated in such case? Pseudoephedrine

95. What is an absolute contraindication with OCPs?


a) Migraine with aura
b) Atrial fibrillation
c) Endometriosis

96. During the evolution of prokaryotes to eukaryotes, what is the difference?


a) Golgi bodies
b) DNA
c) nuclear membrane
d) Mitochondria
The nuclear envelope, also known as the nuclear membrane, is made up of two lipid
bilayer membranes which in eukaryotic cells surrounds the nucleus, which encases
the genetic material. The nuclear envelope consists of two lipid bilayer membranes, an
inner nuclear membrane, and an outer nuclear membrane.
The nuclear envelope, an outer membrane, surrounds the nucleus of a eukaryotic cell.
The nuclear envelope is a double membrane, consisting of two lipid layers (similar to
the plasma membrane). ... Although prokaryotic cells have no nucleus, they do have
DNA. The DNA exists freely in the cytoplasm as a closed loop.

97. Put the following in the order from smallest to the largest?
Amino acids  proteins  ribosomes  cytoplasm  endoplasmic
reticulum Amino Acids —> proteins —> ribosomes —> E.R. —> cytoplasm

98. Iontophoresis mechanism:


Applying local electric current to help in transdermal drug delivery

99. Function of glycerin in ointments?


a) Stabilizer
b) Levitating agent
c) Emulsifier
d) Antimicrobial
e) Solubilizing agent

100. Side effect of pilocarpine? Headache


ORAL: Asthenia, chills, conjunctivitis, dizziness, headache, hyperhidrosis, myalgia, nausea,
pruritus, rash, rhinitis, skin flushing, tachycardia, taste perversion, tremor, vomiting.
OPHTHALMIC: Reduced vision in patients with cataracts, blurred vision due to refractive
shift, brow ache, GI upset (rare), retinal tear / detachment (rare).

101. Why happen when you take parenteral preparations NOT isotonic?
a) Hemolysis of RBCs
b) Drug instability
c) pH problems

102. Cold chain means, upon receiving stock of medication, what will u do first?
a) Put insulin packages in the fridge
b) unpack vaccines and Put in the frigde
c) Put Epipens in the fridge
d) keep the stock room in which you receive the meds in at controlled room temp.

103. Which of the following meds cause angioedema?


a) ACEi
b) ARBs

104. What to monitor when using metformin?


a) HbA1C and creatinine clearance
b) Random blood glucose and creatinine clearance

105. What antipsychotic drug causes hyperglycemia?


a) Olanazapine
b) Quetiapine
c) Haloperidol

106. Paliperidone is a prodrug of? Resperidone

107. BPH pt intended to reduce BP. Which is suitable?


a) Terazosin Prazocin only HTN
Dexazocin, Terazocin Both BPH & HTN
b) Alfuzosin Tamsulosin and Alfuzosin only BPH

108. Which drug should not be taken at night as it causes insomnia?


a) Mirtazipine
b) Citalopram
c) Venlafaxine
d) Bupropion

109. Which is ADH antagonist?


a) posterior pituitary hormone
b) demeclocycline
c) Tolvaptan
d) desmopressin
e) lypressin
110. A pharmacist left his lunch break to council a patient, what does he follows?
a) Fidelity
b) Veracity
c) Autonomy

111. Head lice treatment for pregnant woman


a) Permethrin
b) Dimethicon
c) Ivermectin

112. When can we consider acne therapy failed?


Parameter Timeframe/Degree of change Actions
Short‐term Effectiveness Endpoints (Acne resolution/control)
Lesion count Decrease by 10–25% within 4–8 wk, with If endpoints not
control, or more than a 50% decrease achieved,
within 2–4 months. consider further
therapy.
Comedones Resolve by 3–4 months.
Inflammatory Resolve within a few wk.
lesions
Anxiety or Achieve control or improvement within
depression 2–4 months.

113. Which drugs improves the mortaility in Hear failure and has negative inotropic
effect?
a) Dobutamine
b) Carvedilol
c) Nifidipine

114. Telangiectasiasis is common in


a) Rosacea
b) Psoriasis
c) Acne
d) Foot infection

115. Which is an interval variable?


a) Eye color, Clinical diagnosis, Sex
b) Celcious temperature, Blood pressure
c) BMI category, Pain score
d) Number of visits to hospital in a year

116. Drug at therapeutic dose caused Tremor and weight gain and polyuria:
a) Lithium
b) Antidepressant med
c) Anti diabetic med

117. Which of the following medications doesn’t need a wriiten prescription and
Verbal prescription can be used? Testosterone

118. Triple therapy (Amoxicillin, Clarithromycin, Metronidazole) used for 


peptic ulcer
b-ordinal
119. Gynecomastia, hyperkalemia, and menstrual cycle Data has order like any scale or
disease stages
irregularities are all common side effects of: ‫عددهم قليل و مترتبني‬
a) amiloride 2- continuous
‫و دى نوعني‬
b) triamterene a- Rati0
c) spironolactone zero ‫و دى زى ما انت شايف كده قيم فيها‬
b- interval
d) furosemide zero ‫و دى قيم ما فيهاش‬
e) indapamide

120. Methyl phenidate prescription? No transfere

121. which doesn’t cause peptic ulcer?


a) Acetaminophen
b) Celecoxib Acetaminophen is the least

c) Tramadol
d) cyclobenzaprine
e) Morphine

122. Which part of the brain control vision?


a) Occipital lobe
b) Temporal
c) Frontal
123. What causes pre-renal failure?
a) Diabetes
b) Dehydration
c) Glomerulonephritis
d) BPH

124. What is added to liposomes to increase e their residence time in circulation?


a) PEG
b) Glycerin
c) Antibodies
d) Glyceraldehyde

125. Which cell secretes mucus?


a) Ciliary body  aqueous humor production
b) Goblet CELLS
c) G cells  secret gastrin hormone
d) Chief cells  secret pepsinogen and gastric lipase
Main role of goblet cells is to secrete mucus in order to protect mucous membranes where
they found. They accomplish this by secreting mucins, large glycoproteins formed mostly
by carbohydrates.

126. Biological factors affecting intestinal


absorption:
a) pH
b) Membrane permeability
c) Amount of Food

127. You take 10-12 hours off when you use


Nitrates so that which of the following can
be accumulated again?
a) Epinephrine
b) VEGF
c) Adenosine
d) cGMP

128. Why should we give metformin with food, to prevent?


a) Lactic acidosis
b) Vit B12 deficiency
c) Hypoglycemia
d) Constipation
e) nausea

129. Drug cause osteoporosis?


a) Clozapine
b) Anastrazole

130. Not metabolized by Glucuronidation


a) Nitrosamine
b) Alkene

33 years old pregnant lady for preeclampsia (Abnormal state of pregnancy characterized
by hypertension and fluid retention and albuminuria; can lead to eclampsia if untreated)
131. Drug of choice for
a) lisinopril
b) Aspirin
c) Methyldopa

132. If the above patient doesn’t get relief, what would be the alternative drug
a) Labetalol
b) Tylenol with codeine
c) mefenamic acid

133. What causes chlamydia? Chlamydia trachomatis.

134. Side effect of Sildenafil


a) Blurred vision
b) Blue color pigment
c) Flash lights
d) Blepharasma

135. Pregnant woman should not touch cat to prevent infection with?
Toxoplasmosis

136. Intermediate formed during heme metabolism? biliverdin


137. Most common pathogen in CAP
a) Hemophilus influenza
b) Streptococcus pneumonia
c) Chlamydia

138. Who can prescribe narcotics?


a) Nurse Practitioner
b) Rigestered nurse
c) Chiropractor

139. Calculate ARR, placebo 2%, Experimental 2.5 %?


ARR = EER ‐CER = 0.025 – 0.002 = 0.023 2.5%-2%=0.5% OR 0.025-0.02=0.005

140. Combine low dose ASA and Ibuprfen everyday?


a) Ibuprofen decrease antiplatelet effect of Asprine
b) Aspirin dec anti-inflammatory of ibuprofen
c) Increase risk of duodenal Ulcer
141. Which type of parenteral can be given IV?
a) Oil in water emulsion
b) Water in oil emulsion
c) Injectable Suspension
d) Depot injection

142. what increase the amount of warfarin?


a) Cephalexin
b) Ampicillin
c) Amoxicillin

143. Which antipsychotic not used in Diabetes?


a) Olanzapine
b) Aripiprazole
c) Quetiapine

144. A 30 years old person healthy patient got seizure for the first time. Cause of seizure
a) Brain tumor
b) Migrane
c) Stroke
d) Pneumonia

145. Refugees who has just about one year in Canada. Which drug plan will he be under?
a) Interim health plan
b) Correctional heath plan
c) Public health plan
d) Canadian forces hearth plan

146. First line for panic disorder


a) Venlafaxine
b) Phenelzine
c) Mirtazepine
d) Bupropion
e) Benzodiazepine

147. What gland formed after puberty and


diminished in elderly? Thymus gland
148. What is secreted from the anterior pituitary?
a) Estrogen
b) Oxybutin
c) FSH
d) ADH

149. Shift between 2 opioids to prevent


a) Tolerance
b) Cross dependence

150. Tamoxifen structure, Which drug won’t


reduce its effect?
a) Amiodarone
b) Benzodiazepine
c) Terbinafine
d) Paroxetine
e) Diphenydramine
Tamoxifen is a prodrug, and most of the therapeutic effect in treating breast cancer
stems from its metabolite, endoxifen.
Since cytochrome P450 (CYP) 2D6 is the most important enzyme in the production of
endoxifen, drugs that inhibit CYP2D6 would be expected to reduce tamoxifen efficacy.
In addition to drug–drug interactions (DDI) involving CYP2D6, there is growing
evidence that enzyme inducers can substantially alter the disposition of endoxifen,
reducing tamoxifen efficacy.

151. Tacrolimus mechanism of action


a) TNF alpha
b) Inhibit IL 2

152. What is the cause of DVT?


a) Progesterone
b) Active Tumor/ cancer
c) Levithyroxine

153. Autism spectrum, what does the term spectrum means?


a) Different severity
b) Range of symptoms
Autism spectrum disorder is
a serious condition related
to brain development that
impairs the ability to
communicate and interact
with others. The term
"spectrum" in autism
spectrum disorder refers to
the wide range of symptoms
and severity.

154. 68-year-old male who is depressed and on venlafaxine 150 mg. his physician
decided to switch him to citalopram. When we change venlafaxine with another drug,
we should:
a) Reduce dose of venlafaxine to 75 for 5 days and then start citalopram
b) Taper down venlafaxine while starting citalopram at low dose then increase
dose
c) Wash out
d) Stop venlafaxine for 5 days and then start citalopram
155. Baby had croup, dr ordered dexamethasone oral?
a) Bed time
b) Take with Food

156. Chronic pain treated by:


a) Mirtazepine I don’t think this question is right as none of these choices has proven
efficacy in pain management.
b) Venlafaxine Among SNRI, duloxetine is the best studies and most used one so if we
c) Lorazepam suppose that these choices are right I can pick Venlafaxine

157. Protein synthesis takes place in the


a) Nucleus
b) Cytoplasm
c) Golgi body

158. 92 years old female with Alzheimer falls


in the heath care unit. What might bw the
reason?
a) Rivastigmine
b) Zopiclone
c) Metformin

159. First line therapy for Stress incontinence?


a) Estrogen cream
b) Oral Oxybutynin
c) Pelvic floor muscle exercise  1st line for urge incontinence also
d) Solifenacin Cross-sectional study design is a type of observational study design. In a
cross-sectional study, the investigator measures the outcome and the exposures in the
study participants at the same time. ... They may be conducted either before planning
a cohort study or a baseline in a cohort study.
160. Cross sectional study is: For example, a cross-sectional study might be used to determine if exposure to
a) Observational study specific risk factors might correlate with particular outcomes. A researcher might collect
cross-sectional data on past smoking habits and current diagnoses of lung cancer.
b) Detect spread of disease in a specific period.
ODR OBSERVATIONAL, DESCRIPTIVE, RETROSPECTIVE

161. Coadministration of dextromethorphan and venlafaxine cause:


a) Increase effect on serotonin receptors  ↑ risk of serotonin syndrome
b) Increase duration of venlafaxine
162. Acetyl co A makes covalent bonds
a) Acetylation
b) Thioester with carboxylic acids

163. Which is written correctly according to ISMP?


a) OD
b) QHS
c) IU
d) Nightly PRN

164. What type of enzyme in unmutated and normal?


a) Major type
b) Recessive type
c) Wild type
d) Active type
e) Inactive type

165. What is the Median of 20, 50, 60, 70, 80? 60

166. Parkinsonism medication


a) Pramipexole
b) Donepezil Dementia
c) Amitriptyline

167. Methotrexate mechanism?


a) Antimetabolite
b) Alkylating agent

168. Sleep Apnea puts person at risk of? Stroke

169. Infliximab  Risk of infection

170. Which kind of vaccine is chicken pox?


a) Varicella Vaccine
b) Killed vaccine
c) Toxoid
d) live attenuated
171. Clinical trials Phase 2
a) Large scale trial in diseased
b) Small scale in heathy
c) Small scale in diseased

172. Lab test for Myocardial infarction


a) CK The gold standard for diagnosing myocardial infarction has been the World
Health Organization definition, which requires any 2 of 3 criteria: ischemic
b) Creatinine symptoms, electrocardiographic changes, and elevated creatine kinase-MB
c) cholesterol levels.

173. Medication should not be taken with antacid?


a) Moxifloxacin
b) Polythiouracil

174. Patient diagnosed with endocarditis due to S. Viridans, the most probable
sourse of infection is:
a) Mouth
b) Lung

175. Side effects of Dry mouth and blurred vision?


a) Belladonna
b) Foxglove

176. Medication for diabetes mellitus that increase Post prandial secretion of
insulin from pancrease?
a) Repaglinide
b) SGLT 2
c) Insulin

177. Trichomonas vaginalis treatment? Metronidazole

178. Carboxylic Acids metabolism? Glucuronidation

179. Which should be taken on empty stomach? Norfloxacin

180. Which medication has Post antibiotic effect? Amikacin

181. Most abundant White blood cells?


a) Monocytes
b) Neutrophils
c) Eosinophils
d) Lymphocytes

182. Patient worry about side effects of mediactions. What should the pharmacist do?
a) Tell her she will be fine
b) Tell her other patients worry too, so it is not a big deal
c) Ask about her concerns and explain

183. what will guarantee therapeutic relationship between patient and pharmacist
a) Make decision
b) Give advice
c) Show empathy
d) Create dialogue

184. Patient used to administer pink pills for metoprolol 40. Now he is given blue
pills? What is appropriate for the pharmacist to do?
a) Sincere apology and take to private room
b) Advice generic can differ in color and shape

185. Patient with history of heparin induced thrombocytopenia. What should be avoided?
a) Enoxaparin
b) Fondaparinaux

186. What should be given to reduce infection in patient at risk of neutropenia?


a) Infliximab
b) Pegfilgrastim

187. what will affect the bioavailability of drug from a patch.


a) Diffusivity of drug from patch
b) Solubility of drug in patch
c) Shape of patch
d) Size of patch

188. What is not a part of Best Possible Medication History?


a) Allergies
b) Discontinued medication
c) Herbals
d) Non pharmacological treatments

189. Which circulatory system absorbs fats?


a) Pulmonary
b) Spleen
c) Retroperitoneal
d) Renal
e) Lymphatic

190. which drug is used in Psoriasis?


a) Nystatin tazarotene topical are used to treat acne vulgaris in adults and adolescents
who are at least 12 years old. Tazorac is also used to treat plaque psoriasis
b) Tazarotene gel (raised, silvery flaking of the skin) in adults.

191. What is not related to Vit A?


a) Deficiency causes vision problems and night blindness
b) Skin problems
c) Recommended for new borns

192. Graph of conc vs time on semilog what shape will you get? First order kinetics

a) Straight line
b) Curved

193. Is it first or Zero order? first order

194. Doctor wants 0.5 ml of epinephrine, which comes as 1:100 but pharmacy has
1:1000. What concentration will pharmacist measure to give the same drug
concentration as in 0.5 ml? 5ml

195. Test shows tha drug is resistant to amoxicillin and oxacillin, But sensitive to
erythromycin, vancomycin, and some other pencillin. What kind of resistance is
suspected? Extended spectrum B‐lactamase ESBL resistance

196. What is the cost to implement a new diagnostic test? Cost effective

197. What should NOT be on the Label of non-sterile compounding preparation?


a) Use by date
b) Name of ingredients
c) Strength of ingredients
d) Expiry date of each ingredient

198. Formula for total clearance:


a) Rate of drug excreted in urine/time
b) Product of elimination constant and volume of distribution

199. Role of public health agency?


a) Distributing vaccine
b) Manufacturing Vaccine
c) Control and prevent infectious diseases

200. Whichh genetic polymorphism affects enalapril


a) CYP2c9
b) Angiotensin converting enzyme

201. The ability to make decision for oneself?


a) Capacity
b) Conscientious

202. 2 brands contain different ingredients with same therapeutic efficiency. This is an
example of what?
a) Therapeutic equivalent
b) Therapeutic alternative

203. What is Not an ophthalmic side effect of Isotretinoin?


a) Dry eyes
b) Keratitis
c) Glucoma
d) Itching

204. Not innervating management levels:


a) Flat
b) Heirarchical

205. Patient took a drug tablet crushed and supplied in a feeding tube to the Jujunum,
which is not appropriate? omeprazole Formulated as capsules or enteric
coated tablets.. Becomes deactivated
by gastric acid ..
A perishable good is any product in which quality deteriorates due to
environmental conditions through time.

206. Management to take care of Perishable medication? Contingency management.


It is like an urgency plan to deal with different issues in the pharmacy or other
businesses.

207. Cap with MRSA. what is the drug of choice?


a) Vancomycin
b) Ciprofloxacin
c) Meropenam

208. Thromboembolic event can be seen in which?


Thrombocytopenia

209. Non profit organization responsible for collecting information?


a) Health Canada
b) CIHI

210. Which drug has an effect on NE and serotonin and act on 5HT3 and 5HT2?
a) Amitryptilene
b) Bupropion
c) Quetiapine

211. NAPRA
a) Advice of drug scheduling to provinces
b) Put extended scope of practice for pharmacist and pharmacy technician
c) Give certification for pharmacist and technician

212. Question of contemplation and smoking  Intention to treat

213. which is true about warfarin?


Safe in renal failure as it is metabolized in liver.

214. pharmacy technician can do all except?


a) Check rx appropriateness
b) Enter computer data
c) Receive rx
d) refill previous medication
e) train patient how to use glucose monitoring device
215. When to submit New drug submission? After clinical trials

216. Hematocrit? Ratio of volume of RBCs to total blood volume.

217. Pharmacist sell the medication below acquisition cost is called?


Loss leader.

218. Mechanism of action of Memantine? NMDA antagonist

219. Misprofessional behavior? give narcotics to assistant


acetaminophen 300 mg and codeine
phosphate 60 mg.
220. Tylenol 4? written prescription, part refill allowed Straight narcotic: written only , part fills
allowed

221. First order calculation, given intercept on Y axis and dose, calculate the Vd?

222. Glucagon is? Polypeptide hormone

223. Natural killer cells function?


Natural killer cells, or NK cells, are a type of cytotoxic lymphocyte critical to
the innate immune system. The role NK cells play is analogous to that of cytotoxic T
cells in the vertebrate adaptive immune response. NK cells provide rapid
responses to virus‐infected cells, acting at around 3 days after infection, and
respond to tumor formation.
They have the ability to recognize stressed cells in the absence of antibodies and
MHC, allowing for a much faster immune reaction. They were named "natural
killers" because of the initial notion that they do not require activation to kill cells
that are missing "self" markers of MHC class 1. This role is especially important
because harmful cells that are missing MHC I markers cannot be detected and
destroyed by other immune cells, such as T lymphocyte cells.
There are three main classes of biopolymers, classified according to the
monomeric units used and the structure of the biopolymer formed:
224. Not a biopolymer? Glucose polynucleotides (RNA and DNA), which are long polymers composed of 13
or more nucleotide monomers; polypeptides, which are short polymers of
amino acids; and polysaccharides.
225. Microcytic anemia? Iron deficiency

226. Bariatric surgery for? Obese patient

227. Sustained release is best for? short half‐life drugs


228. IV protein? no first pass effect

229. Acetyl choline and succinylcholine similarities?

230. Varnecline mechanism of action


Act on nicotinic receptors. It is partial agonist that binds selectively to alpha4,
beta2, nicotinic acetylcholine receptors with a greater affinity than nicotine.

231. Cross sectional=prevalence


Case control = no prevalence no incidence no odd ratio
Cohort = incidence

232. Capecitabine = antimetabolite drug used for cancer

233. What is the formula for inventory ratio/ Turnover rate?


Total sales / average inventory Sales -COGS / inventory average

234. Oligonucleotide Probe?


They are short stretches of single‐stranded DNA or RNA used to detect the presence
of complementary nucleic acid sequences (target sequences) by hybridization.

235. Drug with high clearance ratio is affected by? Hepatic blood flow

236. calculation for conversion of Morphine SC to PO?

237. drug of choice for acute gouty arthritis? Colchicine

238. Alcohol is contraindicated with simvastatin because it may cause?


a) Dark urine
b) Rhabdomyosis
Square root of [H (cm) * W (kg) / 3600]
239. Calculate Body surface area

240. Relative Bioavailability F = (AUC oral/dose oral) / (AUC IV / dose IV)

241. Which should heathy patient have least value of.


CTC .. Short courses (2–4 weeks) of metronidazole and/or
242. Fistula treatment = Metronidazole ciprofloxacin are useful for the treatment of
patients with CD and perianal fistulae or isolated colonic CD

243. Osteoarthritis = 30 minutes stiffness

244. What to administer with NSAIDs induced ulcer? Omeprazole

245. HIV = Plasm viral load

246. Improve adherence for old


patients = Blister pack

247. Interferon side effect =


flu like symptoms

248. Arachidonic acid 


Leukotrienes

249. Allergic rhinitis symptoms?


Nasal congestion

250. The most common cause of diabetic ketoacidosis and coma in the diagnosed and
treated diabetic patient is?
A- insulin over dosage
B‐ failure of the patient to use insulin properly
C- electrolyte depletion
D- use of wrong type of insulin
E- excessive physical activity

251. Warfarin + Cotrimoxazole interaction:


cotrimoxazole, inhibit the metabolism of warfarin
and possibly increase the risk of hemorrhage
252. B blocker side effect? sexual dysfunction

253. Blood flow to Left atrium? Oxygenated blood From lungs through pulmonary veins
Left atrium: The upper right chamber of the heart.
The left atrium receives oxygenated blood from the lungs and pumps it down into
the left ventricle which delivers it to the body.

254. DNA replication  S phase

255. chemotherapeutic drug that act on M phase?


a) Methotrexate  S phase
b) Prednisone  G1 phase
c) Bleomycin  G2 phase
d) Vincristine

256. Eye preparation? single vial

257. Extensive skin allegory that is bothersome at night and day? Refer to doctor
Bradykinin is a potent endothelium-dependent vasodilator and mild
diuretic, which may cause a lowering of the blood pressure. It also
258. Bradykinin? pro inflammatory causes contraction of non-vascular smooth muscle in the bronchus and
gut, increases vascular permeability and is also involved in the
mechanism of pain.
259. about debride wound Debridement is the removal of unhealthy
tissue from a wound. It will improve wound
healing. There are different ways to do
260. ibuprofen isomerism? The two optical isomers debridement. Debridement involves the
removal of necrotic tissue to promote wound
of ibuprofen are identified by the prefixes R‐ and S+. healing. During wound healing, the affected
area can become overrun with necrotic – or
The stereoisomers are similar in properties such as dead – tissue. This can be harmful to the
melting point, boiling point and solubility. The S+ form isbody's ability to recover and develop new
skin, so debridement may be necessary to
the more pharmacologically active form and the R‐ form remove that dead material.
has no anti‐inflammatory effect.
Jan 2018
1. Which drug does NOT interact with OCP
a. Phenytoin
b. Amoxicillin
c. CBZ
d. Celebrex
e. rifampin

2. Who is responsible for the pricing of newly patent medication


a. Review board chosen by the parliament
b. Manufacturer
Patented Medicine Prices Review Board PMPRB

3. A patient case with COPD with diabetes taking the following


Venlafaxine qam
Formeterol/ budesonide
Salbutamol 100 mg
Metformin, Glyburide
Tiotropium 18 mcg
What should the pharmacist be concerned about?
a. Venlafaxine in the morning
b. Interaction between metformin and glyburide
c. Taking Formeterol and salbutamol  risk of side effects tremors and tachycardia
d. Dose of salbutamol
e. Dose of tiotropium

4. Patient has acne and will start taking isotretinoin


a. Should start OCP before by 1 week
b. Continue OCP 1 month after stopping isotretinoin
c. Check with a pregnancy test every three months
d. Use adequate contraception method

5. What drug DON’T cause Steven syndrome and epidermal necrosis


a. hydralazine
b. quinidine All don’t cause except D

c. warfarin
d. sulfamethixazole
6. Which drug shows a high alert medication and dangerous abbreviation according to
ISMP?
a. Insulin 10 units
b. Methotrexate 5mg po daily
c. Warfarin 50mg po tid
d. Levothyroxine 0.008 mg po bid

7. Which vitamin causes renal calculi?


a. Vit C
b. Vit A
c. Vit K

8. If a woman is taking OCP and she is experiencing spotting in the 10-21 days, what
should the pharmacist advise her
a. Increase estrogen
b. Increase progesterone
c. Stop the OCP

9. A taxi driver came to you with a prescription of sumatriptan for his migraine and you
know that it may cause dizziness, what is the most appropriate action to be done?
a) Call the physician to explain to him your fears from side effect of the medication
b) Refuse to dispense the prescription
c) Dispense the medication and give the patient a verbal counsel about the side effect of
the medication For me it is C because written counseling without verbal is insufficient .. the
patient might throw the paper away as soon as he gets out of your pharmacy
d) Dispense the medication and give the patient a printed copy of the side effect of
the medication

10.Nurse patient 35 years old, coming in with a prescription of Zoster vaccine. He never
had chicken box before, he never took varicella vaccine before, didn’t do any test to check
if he has the Antibodies. (he is in a province where pharmacist can order lab tests)
a. Dispense the vaccine normally and give it to him
b. Give him the varicella vaccine before the zoster vaccine
c. Oder a test to see if he is immune to the disease
d. Tell him he is young to the vaccine

11.Question about a study where all pharmacists took a questionnaire to see their opinion
about something?
a. Case study
b. Cross sectional
c. Delphi analysis
d. Longitudinal study

12.People in a gathering all got food poisoning, what is the bacteria that caused it
a. C. perfinges
b. Fragilis
c. (No salmonella, no e. coli in choices)

13. UTI with pseudomonas aureginosa treatment


a. Ciprofloxacin

14.Which have no activity against Pseudomonas aureginosa:


Antipseudomonal list
a. Cefazolin Ciprofloxacin
b. Cefipim Aminoglycosides
Piperacillin/Tazobactam
c. Ceftazidem Ticracellin
Meriopenems
d. Ciprofloxacin Carbepenams
e. Pipracillin / tazobactam Cefipime
Ceftazedime

15.Patient who came to the ER with acute exacerbated severe COPD give what
a. Prednisone 5 days
b. Doxycycline 500mg 1 day and 250 mg for 6 day
c. tiotropium
d. (No SABA nor LABA bronchodilator in choices)

16.An obese women (BMI 39) with type 2 DM and overweight, she tried diet and exercise
but not effective and wanted to lose weight give her
a. Metformin
b. Liraglutide
c. Methylphenidate
d. Bupropion

17.Normal patient (with no disease or complication and younger than 60 Y\O) blood
pressure should be
a. <140/90
b. <130/80
c. <120/80

18.Now the patient has diabetes blood pressure should be


a. <140/90
b. <130/80
c. <120/80

19. Where is the anticodone


a. tRNA
b. mRNA
c. DNA
An anticodon is a trinucleotide sequence complementary to that of a corresponding codon
in a messenger RNA (mRNA) sequence. An anticodon is found at one end of a transfer
RNA (tRNA) molecule.

20.Patient is travelling, what advice NOT to give him


a. If travelling by plane, don’t drink a lot of water
b. Don’t drink from tap water
c. Don’t eat vegetables and green salad
21. A patient with scalp seborrhea, what pharmacological treatment in a shampoo
a. Salicylic acid
b. Selenium sulfide
c. Coal tar
d. triclosan
Treatment of Dandruff and Seborrheic Dermatitis of the Scalp

22.Patient Taking metformin now pregnant, she did not see physician yet to switch
insulin, what pharmacist do?
a. Stop immediately and go to the doctor as soon as you can to change to insulin
b. Continue metformin until you go to the doctor to get insulin
c. Call the doctor to tell him that she can’t take metformin
In women (excluding those with T1DM) who cannot take insulin, it may be reasonable to
consider glyburide or metformin, recognizing that use of either agent is off-label.

23.Structure of omega 3 fatty acid

24.Structure of allopurinol?
xanthine oxidase inhibitor

25.Structure of bisphosphonate, related to


a. Pyrophosphate
b. collagen
26.If you have creatine and became creatine phosphate, CK utilizes what
a. ATP
b. ADP

27.What is true about protecting from skin cancer and sun block
a. SPF 15 or greater
b. It protects you from UVB and UVA Both r right

28.In acne, why would you put clindamycin and benzyl alcohol together
a. For sensitive skin
b. For bacterial resistance

29.Patient case he is over 60 yrs. and he wants to improve his sexual life, he has erectile
dysfunction and angina and he take Nitroglycerin (but hasn’t taken it for 4 month) which
drug causes his ED:
a. ACE inhibitors
b. Metoprolol
c. alpha blockers
d. statins

30.What should be a concern for the pharmacist if he takes? tadalafil & nitrates

31.Why would tadalafil be better than other agents in this class


a. It will cause spontaneous erection
b. It could be taken with grapefruit
c. It enables him multiple partner
d. no delay in absorption after food

32.What is rate determining in the erodible matrix tablet


a. Polymer
b. Something about Solubility

33.Mechanism of action of Quetipine


a. 5HT 1a and less dopamine
b. 5HT 2a and less dopamine
c. Dopamine only

34. Zoplicon Case! how long can a patient take Zoplicon


a. 10 days
b. 30 days
c. 90 days
d. 60 days
e. 120 days
Does not accumulate; tolerance to zopiclone’s hypnotic effect may be delayed and rebound
insomnia may be reduced.
Treatment with zopiclone should be as short as possible. Generally, the duration of
treatment varies from a few days to two weeks with a maximum, including the tapering
off, of four weeks.

35.Someone just initiated treatment for depression how long should he stay on it
a. 1 year
b. 6 months

36.Women with migraine taking ibuprofen and now its not working what you should give
her
a. Naproxen
b. Triptan
c. Acetaminophen

37.What is inverse antagonist Inverse agonist


a. Bind to a different part of the receptor
b. Block the antagonist of binding
c. Show action opposite to agonist

38.Sealed ampoule sterilization


a. Steam sterilization
b. UV sterilization
c. Ethelyn oxide sterilization

39.What causes hyperuricemia? thiazides

40.Structure of benzodiazepine ---------------- -----

41.How many cis bonds in structure


42.Patient with acute attack of asthma, what you don’t use
a. Oxygen
b. Salbutamol
c. Prednisone
d. Ipratropium
e. Tiotropium

43.ARB mechanism of action


a. Angiotensin type II receptor blockers
b. Decrease the aldosterone

44.What about a spacer is true (long question)


a. To be used with turbohaler  use with MDIs
b. Increases the drug in the lungs
c. You don’t clean it with water
Spacers have two advantages: they often improve coordination between delivery of
the medication from the inhaler and breathing it onto the bronchial tubes; and they
reduce the amount of medication that settles in your mouth and throat.

45.Question about the flexible insulin, patient taking aspart and detemir, insulin taken
depends on
a. Carbohydrates in meal
b. Pre-prandial physical exercise

46.Diastolic blood pressure is which variable


a. Discrete
b. Nominal
c. Continuous Continuous - interval
d. Ordinal

47.Cells for regeneration of myelin sheath


a. Schwann cells
b. Myoglia
c. Dendritic cells
d. satellite cells
Schwann cell, also called neurilemma cell, any of the cells in the peripheral nervous
system that produce the myelin sheath around neuronal axons.
Dendritic cells (DCs) are antigen-presenting cells (also known as accessory cells) of the
mammalian immune system. Their main function is to process antigen material and
present it on the cell surface to the T cells of the immune system. They act as messengers
between the innate and the adaptive immune systems.
Myosatellite cells, also known as satellite cells or muscle stem cells, are small
multipotent cells with very little cytoplasm found in mature muscle. Satellite cells are
precursors to skeletal muscle cells, able to give rise to satellite cells or differentiated
skeletal muscle cells.

48.Sodium dodecyl sulfate SDS (anionic surfactant) role in gel electrophoresis


a. To keep the nature of protein (don’t remember other options)
SDS‐PAGE is an electrophoresis method that allows protein separation by mass. The
medium (also referred to as ′matrix′) is a polyacrylamide‐based discontinuous gel.
It uses sodium dodecyl sulfate (SDS) molecules to help identify and isolate protein
molecules.

49.What is the complex of Protein and DNA


a. Chromatin
b. Nucleosome
c. histone
Chromatin is a mass of genetic material composed of DNA and proteins that condense to
form chromosomes during eukaryotic cell division. Chromatin is located in the nucleus of
our cells.
The primary function of chromatin is to compress the DNA into a compact unit that will
be less voluminous and can fit within the
nucleus.
Nucleosomes are an example
of chromatin structure, where you've got DNA
wrapped around a core of histone
proteins. And genes are regions of DNA that are
transcribed into RNA by RNA polymerases. They
serve as a scaffold for formation of higher order
chromatin structure as well as for a layer of regulatory control of gene expression.
A histone octamer is the eight-protein complex found at the center of a nucleosome core
particle. It consists of two copies of each of the four core histone proteins (H2A, H2B, H3
and H4). The octamer assembles when a tetramer, containing two copies of both H3 and
H4, complexes with two H2A/H2B dimers. They are the chief protein components of
chromatin, acting as spools around which DNA winds, and playing a role in gene
regulation.

50.What causes otitis media


a. H influenza (no s. pneumonia option)

51.Meningitis treated by? Ceftriaxone

52.The most abundant solute in the plasma


a. Albumin
b. Prothrombin
c. Chlorine
d. Potassium

53.Artificial Passive immunity


a. By taking from mother immunity in breast milk
b. Injecting AB directly (different wording)

54.Rickettsia causes? rocky mountain fever

55.Drug infliximab, origin is


a. Human
b. Human and mouse
c. Rabbit
56.Which follows nonlinear kinetics Phenytoin

57.Which can’t be taken in late pregnancy ASA

58.The filler used in the hard gelatin capsule


a. Starch
b. Cellulose (or derivative of it I don’t remember its name)
c. Dibasic phosphate

59.Problem in emulsion? creaming

60.The mass and ion charge ratio used in?


mass spectrometry

61.The cranial nerve responsible for mastication


a. Trigeminal
b. Glossopharyngeal

63.Person has pancreatitis, lab test shows?


lipase

64.If there is a problem with sciatic nerve?


will not flex the knees

64.Case of a patient with an absence in the P wave,


(arterial fibrillation) … long question, he should take
what
a. Antiplatelet to reduce stroke
b. Antiarrhythmic to reduce stroke

65.Patient that has long term uncontrolled


hypertension, what is the long effect
a. Aortic valve relapse
b. Left ventricular hypertrophy
c. Aortic stenosis

66.Someone with alopecia, what is the main goal of treatment


a. Increase hair density
b. Reversal of hair loss
c. graying of hair
d. generating hair follicles

67.A drug that causes lengthening of eye lashes and pigmentation? bimatoprost

68.A type of ownership that your personal assets won’t be affected by the creditors
a. Liability
b. Corporation
c. Sole

69. you are a manager in a community pharmacy and you decide to give service with price
which only covers your cost and expenses, you decide to make this service with 10 cases
per month, therefore you will have:
a- Increase in net profit
b‐ Increase in the revenue
c- Increase in the inventory

70. Example of surrogate end point?


a. Hb1Ac
b. Mortality
Surrogate (marker) endpoint: https://www.youtube.com/watch?v=UyMLH9CfVcI
Measure of effect of a specific treatment that may correlate with a real
clinical endpoint but does not necessarily have a guaranteed relationship
Examples:
1) Hypertension: arterial blood pressure  surrogate for CVA, MI & HF
2) Hypercholestrolemia: cholesterol levels  surrogate for atherosclerotic disease
3) HIV: CD4 count or viral load  surrogate for complications of HIV
4) Glucoma: intraocular pressure  surrogate for loss of vision
5) Diabetes mellitus: blood glucose / hemoglobin A1c  surrogate for complications

71.Kaplan-Meire equation? survival analysis

72. In an experiment want to know the number of newly prescribed oxycodone, outcome
drug side effects and refill prescription was undergoing under 4 physicians what is the
name of the used test?
(this was really tricky question because the outcome contains both parametric and non-
parametric data)
Agreed on student T test
A.chi square
B.wilcoxon
Here, we are comparing one sample drug “oxycodone”.
If it is more than 2 drugs, we would choose A.

73.who’s responsible for initiating Recall of the medication by


a. Manufacture
b. ISMP
c. Canada vigilance
d. CADTH
https://www.canada.ca/en/health-canada/services/drugs-health-products/compliance-
enforcement/information-health-product/drugs/recall-policy-0016/policy.html
Health Canada expects responsible parties to voluntarily recall a health product when:
 It contravenes the Act or the Regulations, or
 It presents a risk to the health of Canadians.
Responsible Party - The person responsible for initiating and conducting a recall.
Without limiting the generality of the foregoing, responsible parties may include
manufacturers, distributors, importers, persons in Canada responsible for the sale of the
product and wholesalers for drugs; sponsors of clinical trials; manufacturers, importers
and distributors for medical devices; manufacturers, importers, distributors, and product
licence holders for natural health products; blood establishments for blood; source
establishments for cells, tissues and organs; and processors for semen.

74.Not true about what is affected in elderly


a. Increase in GIT motility
b. Increase PH

75.Difference between acne and rosacea is that rosacea has:


a) Pustules
b) Nodules
c) Comedones
d) Telangiectasia
(care that we answered this question before as comedones because he was asking about
acne but here asks about the characteristic feature of rosacea)
76. Which of the following medication is most commonly
associated with urine discoloration?
A: Benztropine
B: Carbidopa/levodopa
C: Entacapone
D: Pramipexole
E: Rasagiline

77.What you should put “protect from sun exposure” Sulphonamide


Drug‐induced photosensitivity:
Tetracyclines, Fluoroquinolones e.g. ciprofloxacin, Sulfonamides, doxycycline, nalidixic
acid, Ibuprofen, Naproxen, Ketoprofen, Celecoxib, Salicylates, piroxicam,
Frusemide, Bumetanide, Hydrochlorothiazide, Enalapril, Diltiazem
Isotretinoin, Acitretin, Benzophenones, Para-aminobenzoic acid (PABA), Cinnamates
Amiodarone, Quinine, Quinidine, Hydroxychloroquine, Dapsone
voriconazole, chlorpromazine and thioridazine
Drugs to protect from sunlight:
78.Patient with hypertension and can’t sleep and tired in the morning and has allergy
that’s been there for a while, give him what
a. Diphenhydramine
b. Cetirizine
c. Xylometazoline
d. Pseudoephedrine

79.Question what is not true? Mg antacids causes constipation


Al & Ca antacids that cause constipation, Mg cause diarrhea.

80.PT is taking verapamil, got constipation, which drug exaggerate it


a. Ca carbonate
b. Bismuth
c. mg

81.A patient has abdominal pain and bloating. He had


hard bullet like stool followed by diarrhea. He has
a. Crohn disease
b. Ulcerative colitis
c. IBS
d. Constipation

82. Mechanism of action of Zafirlukast?


Cysteinyl leukotriene receptor blocker

83.Which AA causes dimerization


a. cysteine
b. Serine  can go under phosphorylation
c. glycine  one of the monomers that forms proteins

84.Women with C-section needs analgesics and she is


breast feeding, what analgesic to avoid (not sure about these choices)
a. Codeine & Oxycodone
b. Morphine
c. Acetaminophen
d. Ibuprofen
e. Hydromorphone
Codeine & Oxycodone  both medications were associated with a high rate of
depression in the neonate.

85.After what age codeine is used as antitussive.


a. 16 years b.12 years Codiene can be used after 12 years old unless this child had
adenoidectomy or tonsillectomy then it will be 18

86.Natural treatment of osteoarthritis


a. Hyaluronic acid b. Feverfew  migraine prophylaxis

87.What you need to check the eyes for? hydroxychloroquine & amiodarone

88.postmenopausal Women is treated for osteoporosis by a drug that causes hot flashes
a. Raloxifene b. Teriparatide c. Estrogen

89.initial Colchicine toxicity side effect


a. Diarrhea
b. Hepatotoxicity
c. Nephropathy
d. Seizure
S.E: Common at higher doses: abdominal pain and cramps, diarrhea, nausea and vomiting.
Rare: neuropathy, myopathy, bone marrow suppression.
Chronic use can produce hemorrhagic gastroenteritis, hematuria, alopecia,
agranulocytosis and peripheral neuropathy

90.Resistant schizophrenia? clozapine

91.which one Causes compulsive behavior?


a) Amantadine
b) Pramipexole
c) Benztropine
d) Rasagline

92.Acetyl cholinesterase inhibitor used for Alzheimer


a. Neostigmine  physostigmine not neostigmine is the one used for alzheimer
b. Galantamine
Selective, competitive, reversible acetylcholinesterase inhibitor and also enhances
the action of acetylcholine on nicotinic receptors.
93.What causes weight loss? topiramate

94.Which one is antimetabolite?


methotrexate

95. 5 fluorouracil structure undergoes a


metabolism reaction by
a. CYP 450
b. Xanthine oxidase
c. Dihydrofolate reductase
d. Aldehyde dehydrogenase
e. dihydropyrimidine dehydrogenase

96.Most common side effect of varenicline:


a) Nausea
b) Constipation
c) Diarrhea
d) Lacrimation

97.What is true about


rheumatoid arthritis:
a. Primarily affect weight
bearing joints
b. Happen after 50 year
c. Can have systemic side
effects
d. Morning stiffens less than 45
minutes

98.Angioedema side effect of:


a. ACEI
b. Ccb blocker

99.Which drug is given by


iontophoresis in tennis elbow?
Corticosteroid injection

100. What is true about clopidogrel?


a. require neutropenia monitoring
b. ADP irreversible blocker
Irreversibly block the P2Y12 platelet receptor but Ticagrelor binds to the P2Y12 ADP
receptor in a reversible manner

101. Cold sore triggered by


a. UV light
b. Acidic food
Common stressors that can precipitate recurrences include:
Emotional stress, dental extraction, fatigue, fever, hormonal factors, hyperthermia,
menstruation, physical trauma or surgery, sun exposure (UV light) and upper respiratory
infection.

102. Related to LMWH monitoring? LMWH doesn’t require monitoring

103. Which hypolipidimic drug reduce arrhythmia


a. Statin b. Fibrate c. Resin

104. Empathy listening:


a) Feeling what other feels
b) Understand what other say

105. A drug is completely metabolized in liver and the patient has kidney failure and very
sensitive to the drug due to:
Accumulated active metabolites

106. Dose of finasteride\ Initial treatment of Alopecia


a. 1mg
b. 5mg

107. Patient on isotretinoin, what drug to avoid before using it?


a. Doxycycline
b. Ciprofloxacin
c. Tetracycline
Tetracyclines: rare cases of benign intracranial hypertension (pseudotumor cerebri):
allow 7 days washout after stopping tetracyclines before starting isotretinoin.
108. Which causes celiac disease?
Celiac disease is an immune disease in which people can't eat gluten because it will
damage their small intestine. If you have celiac disease and eat foods with gluten,
your immune system responds by damaging the small intestine.

109. A women has menopausal stage and facing hot flushes and had deep vein thrombosis
5 months ago and taking warfarin and some other medicines what is not recommended
a. Like reducing caffeine
b. Reducing smoking
c. Use of fan for hot flushes
d. Use of estrogen replacement therapy.
e. clonidine therapy

110. A kid with dried and


peeled skin in his elbow and
knee. What is possible
diagnosis?
a. eczema
b. Seborrhea
c. Psoriasis
111. Prednisone prescription begin with 40 mg and decrease by 5 mg on week interval,
you have tablets 5 mg, how many tablets?
It was about 255 tablets 252 tablets

112. For how long you should keep records of narcotics?


a. One year
b. 2 years
c. 5 year

113. Patient on warfarin INR 4 with no bleeding signs what is your recommendation?
a) Omit 2 doses and reinitiate two days later
b) Refer to doctor to vitamin k oral
c) Refer to emergency for iv vitamin k
d) Recommend food with high
vitamin k content

114. What is present in dorsal cavity?


a) Teeth
b) Spine
c) Knee

115. Rapid acting insulin


a) NPH
b) Aspart
c) Glargin

116. For Which drug can have its prescription transferred only once?
a. Lorazepam
b. Methylphenidate

117. What is the body innate immunity against pneumonia?


a. Low PH of sputum
b. Secretion of thick sputum.

118. Dabigatran action:


c. Acts on Antithrombin
d. Inhibit platelet aggregation
119. Common causative organism of community acquired pneumonia? S. pneumonia

120. What common causative organism for hospital acquired pneumonia? Pseudomonas
aureginosa

121. Hydrocortisone 10 gm 2.5% with 2 gram 5 % with 14 gm base ointment, what is the
concentration of hydrocortisone in the final preparation?

122. What drug must be included in treatment of acute pneumonia? Macrolides


123. What is proteomics?
a) It’s the study of proteolytic enzymes
b) it’s the study of codons something like that
c) It’s the study of interaction of proteins

124. Cause of hand warts? Human Papilloma Virus

125. Rx: lorazepam, Alcohol 10%, PEG 40%. what is the role of PEG?
a) Buffer
b) Surfactant Cosolvent
c) solubilizer
126. Skin functions all except:
a) Temperature regulation
b) Locomotion
c) Secretion
d) Sweating

127. Epidural injected in


a. spine,
b. skin,
c. joint,
d. lung

128. Folic acid to prevent? spinabifida

129. Nystagmus involve all except?


a. involuntary movement,
b. oscillation
c. rapid eye movement
d. Repeating
e. promising eye

130. What is true about Tardive dyskinesia?


a. Caused by neuroleptic drugs
b. TX by anticholinergic drug.

131. why doctors perceive pharmacist is not collaborating


a) The pharmacist lack training
b) The pharmacist doesn’t follow up the patient’s lab results

132. Cruciate ligament present in,


a. knee b. hip c. shoulder d. ankle

133. disadvantage of Tertiary source literature


a. Increase lag time till publish
b. No credible reference
c. limited because of needs fees for subscription
134. In a hospital there’s a mistake of dispensing a medication but was caught before the
patient takes the medication. This is
a. Malpractice
b. Medication error
c. Near miss
d. Prescription error

135. What opioid is highly fat soluble and used for anesthesia? Sufentanil

136. Pharmacotherapy ultimate goal based on?


A. Pharmacists decision
B. Patient decision
c. Physician decision

137. Lack of Anti diuretic hormone cause


a) Hyperuremia
b) Urcemia
c) Hyperlipidemia
d) Diabetes insipidus

138. TSH release from,


a. hypothalamus
b. anterior pituitary
c. posterior Pituitary

139. What is true about dissolution?


a. Arrhenius equation used to calculate it  for the temperature dependence of reaction
rates
b. PH of the liquid determine dissolution if the solute is weak acid
c. Noyes‐Whitney determines "rate of dissolution".

140. Which carry blood from intestine/gut to liver,


a. Portal Hepatic Vein From systemic to liver : hepatic artery + portal vein ( portal vein consists of
splenic & mesentric veins)
b. portal artery
From liver to systemic : hepatic vein to inferior venacava to heart

141. Long chain of carbon structure, in u shape, how many cis carbon present
a. 0,
b. 2,
c. 4,
d. 6

142. Who can prescribe narcotics


a. nurse practitioner
b. chiropractor
PRESCRIBING AUTHORITY
Who is permitted to prescribe narcotics after New Classes of Practitioners Regulations?
✓ Doctors of medicine ✓ Dentists ✓ Doctors of veterinary medicine
✓ Midwives ✓ Podiatrists or chiropody ✓ Nurse Practitioners
Only as permitted by provincial regulations! Meaning they must be registered and able to
practise as a doctor, dentist, midwife, etc. under the laws of each province
*A chiropractor is not a physician so he can not prescribe narcotics

143. When shifting from a narcotic to another why we give less than the equivalent dose?
avoid tolerance

144. A question on angle of repose used to measure what?


a. Flowability

145. What is the rate of breathing?


It depends on the age and the health ststus of the
person

146. Patient is taking ibuprofen why ASA low dose


won’t be effective?
a. ASA won’t perform the full action as antiplatelet
b. ASA won’t perform the as anti-inflammatory

147. A patient taking narcotic for 6 months and you doubt he is abusing it he told you he
talked to the doctor and the doctor said no issue. What is the proper action?
Talking to the doctor

148. What is true about transdermal patch? Used for drugs with short half life

149. question about the AUC and therapeutic dose of a drug that gets metabolized by
cyp3a4
150. What is correct about croup? It resolves in 2‐3 days
Croup (laryngo tracheobronchitis) is a common cause of upper airway obstruction in
children. It is most prevalent in the late fall to early winter months.
Viruses are the most common cause, particularly parainfluenza virus types 1 and 3.
Influenza A and B, adenovirus, respiratory syncytial virus (RSV), metapneumovirus,
coronavirus & mycoplasma have also isolated.
Symptoms of croup resolve in most children within two days but can persist up to one
week.

151.What is the duration of time required to set levothyroxine level on a new patient?
4 ‐ 6 weeks

152. What test is required for a patient taking valproic acid? Liver function
Conduct liver function tests prior to initiation of therapy and periodically,
especially in the first 6 months of therapy and if symptoms of hepatic dysfunction
occur

153.What is the mechanism of action of carbamazepine and what is the pharmacokinetics


profile? Fast sodium channel blockers

154.Which of these drugs is a PPAR gamma inhibitor?


a) Thiazolidinedione
b) Fibrates  PPAR alpha inhibitor

155.How many chromosomes does a child inherit from each parent? 23

156.What is name of the process of synthesis of MRNA from DNA?


a) Translation
b) Transcription

157. Which of these viral drugs is cardiotoxic?


CTC ‐ Cardiotoxic Drugs: Alcohol, Anthracyclines, Cocaine, Cyclophosphamide,
Imatinib, Trastuzumab.

158. What drug is used to delay NV in cancer patient?


a) Metoclopramide
b) Phenothiazine
c) 5HT3 antagonist
159.What is the name of the insulin receptor that is used in phosphorylation?
Insulin is the major hormone controlling critical energy functions such as glucose and lipid
metabolism. Insulin activates the insulin receptor tyrosine kinase (IR), which
phosphorylates and recruits different substrate adaptors such as the IRS family of
proteins.

160.What drug is used for NV in travelling? dimenhydrinate

161. Diazepam Injection U.S.P


Diazepam 5 mg/ml
Ethanol 10%, Propylene glycol 40%, Benzyl alcohol 1.5%, Water for Injection qs 100%. In
the formulation given above, which of the following ingredients function as cosolvents?
a. Ethanol
b. Propylene glycol
c. Benzyl alcohol

162.A question about PAE (post antibiotic effect):


Killing the microorganisms after the antibiotic is less than the MIC

163. Counseling for itraconazole tab?


a) Used on Empty stomach
b) Should be for 6 months
c) Causes Taste disturbance

164. Which of these diseases that does not cause autoimmune disease?
165. Which disease state would most likely be exacerbated by pseudoephedrine?
A: Psoriasis
B: BPH worsen BPH symptoms because they tighten muscles in the prostate and
C: Depression bladder neck. When these muscles tighten, urine can't easily leave the
bladder.
D: Hypothyroidism
E: All of Above

166.Which of these drugs relaxed skeletal muscle?


Many choices and one of them was diazepam and it is the answer

167. ocular delivery? Ointments have More bioavailability than drops

168. Incremental 9000$ QALY, I can’t remember well

169. Morphine dependence withdrawal caused by? meperidine / nalbuphine


170. Sumatriptan? 5HT1B, 1D RECEPTOR AGONIST

171. What is true about centrosomes?


A. Centrosomes contains acid
hydrolyzing enzyme
B. Centrosomes are main organizing
microtubule center
C. Centrosomes contain protein
(rough) and lipid (smooth) synthesis
D. Centrosomes are primary site for
biological protein synthesis.
E. Centrosomes are enzymes that
produces and decomposes hydrogen
peroxides.
Tips: Centrosomes are organizing
microtubule center Lysosome contains
acid hydrolyzing enzyme. Endoplasmic
reticulum contains protein (rough) and
lipid (smooth) synthesis. Ribosomes
are primary site for biological protein
synthesis. Peroxisomes contain
enzymes that produces and
decomposes hydrogen peroxides.
Peroxisomes breakdown long chain
fatty acids.

172. Which have most effective action


against B. frigillis? Metronidazole
Bacteroides fragilis is an obligately
anaerobic, Gram-negative, rod-
shaped bacterium. It is part of the
normal microbiota of the human colon and is generally commensal, but can cause
infection if displaced into the bloodstream or surrounding tissue following surgery,
disease, or trauma.

173. Function of public health agency Canada.


• Prevent and control chronic diseases and injuries.
• Prevent and control infectious diseases.
• Prepare for and respond to public health emergencies.
• Serve as a central point for sharing Canada’s expertise with the rest of the world.
• Apply international research and development to Canada’s public health programs and
strengthen intergovernmental collaboration on public health and facilitate national
approaches to public health policy and planning.

174.Why bacterial can’t


produce erythropoietin?

175. Alpha lipoic acid used


for what?

176. Which organism


cause endocarditis valve?
Staphylococcus aureus

177. Prodrug of acyclovir


has which aminoacid?
valine

178. What causative factor


lead to hair loses in male?
Dihydrotestosterone

179. Cons are responsible for? color vision

180. Mechanism of imatinib? Tyrosin kinase inhibitor

181. Amitriptyline adverse effect? Anticholinergic (dry mouth, blurred vision,


constipation, urinary hesitancy, tachycardia, delirium), antihistaminergic (sedation,
weight gain), orthostatic hypotension, lowered seizure threshold; sexual
dysfunction.

182. Levothyroxine counseling: Taken in the morning to avoid insomnia

183. Dysphonia definition or cause? ICS


the air sacs in the lungs (alveoli) are damaged. Over time, the inner walls of
the air sacs weaken and rupture — creating larger air spaces instead of
many small ones.
18. A question about emphysema

185.Patient with drug box and his prescription


mentioned furosemide 2 tablets once at
morning
Furosemide tab morning evening

186. function of cholecystokinin promote what?


Pancreas secretion

187. Drug causes dry mouth? Tiotropium

188. Drug causes Parkinsonism?


Haloperidol

189. which does not interact with


cyclosporine,

Cyclosporine A calcineurin inhibitor which binds to a cyclophilin to form complex that


2–4 mg/kg binds to calcineurin. This complex cannot perform the dephosphorylation of
daily IV NFATc (cytosolic Nuclear Factor of Activated T cells), so NFATc cannot enter
the nucleus to promote reactions that are required for the synthesis of
cytokines, including IL-2. Resulting in a decrease in IL-2, which is the
primary chemical stimulus for increasing the number of T lymphocytes.
S.E: Renal toxicity (monitor renal function), hypertension, hypertrichosis,
cytopenia, gum hyperplasia, electrolyte imbalances, nausea, diarrhea,
seizures, opportunistic infections.
Metabolized by CYP450: many possible drug interactions (e.g.,
erythromycin, ketoconazole, rifampin, St. John’s wort).
May help avoid colectomy in patients with severe disease not responding to
other therapies.
Cyclosporine is not usually used for treatment of IBD.
Do not use in pregnancy or during breastfeeding.
190. Spirometer use

191. Question about ibuprofen isomers? Stereoisomers

192. When shaking a suspension system, it decreased viscosity? thixotropic


193. A question about cold chain of vaccines?
Cold chain storage for vaccines - 2oC and 8oC for vaccines, with alarmed thermometer

194. What is Natural killer work?


NK cells are activated in response
to interferons or macrophage-
derived cytokines.
They serve to contain viral
infections while the adaptive
immune response generates
antigen-specific cytotoxic
T cells that can clear the
infection. NK cells work to control
viral infections by secreting IFNγ
and TNFα.
195. In hypovolemia all are true except?
a. Increased urine output
b. thirst
c. Oligurea
d. Loss consciousness
e. postural lightheadedness

196. The aim of medication reconciliation?

197. type of business with limited liability?


a‐ corporate
b- franchise
c- banner
Limited liability is usually for a company or corporation and it means that the company is
responsible for all the liabilities or loans etc
So if the business becomes bankrupt the owners will not be taken to jail. the assets will be
sold to cover the loan. This is different to a sole proprietorship where you as the owner
will be fully responsible for all long term and short-term liabilities. If you don't pay people
back you could be taken to jail...

198. T wave in ECG indicates?


Ventricle repolarization
JULY 2017

1. Pt asking for something for Diarrhea, upon check his med. Hx, which from the following
may be responsible for Loose Stool:
a) Phenytoin
b) Misoprostol
c) Gabapentin
S.E of misoprostol: Diarrhea (dose related),
abdominal cramps, flatulence.

2. Theoretical shelf life prediction


a) 4* C 90 & Active
b) Ing 25* C 90%
c) 25* C 95%

3.Opthalmic ppt that should be refriegirated? Latanoprost

4. Which is a live vaccine


a) MMR
b) Diphteria
c) Trivalent
d) Influenza

5. Opportunistic infection with someone taking immunosuppressive


a) CMV
b) HIV

6. 68 years old hs past CHF, has Ventricular fraction 32% and on ACE and B Blocker but
still shows some symptoms. What can be added to decrease mortality and morbidity?
a) HCT
b) Digoxin
c) ARB
d) Aliskerin
e) spironolactone
ACEIs, BBs & SPIRONOLACTONE are the only agents known to reduce mortality in
HF.

7. You have 250 ml of D5W. You have in stock vials 5 ml containing 15 mmoles of K and
22 moles of phosphate. how much mmoles are there in 250 ml D%W parental
preparation for this patient weighting 132 Ibs? If phosphate dose is 0.24 mmol/kg
Ans. 21.12 mmoles of K

8. Emergency contraception. Most appropriate counceling?


No need for follow up after the dose.

9. Pt was post MI and arthritis, hyperlipidemia, hypertension and smoke 2 pack,


how long should he stay on ASA + Clopidogrel
a) 1 month
b) 6 Months
c) 1 Year
d) 2 Weeks

10. Dilution Calculation

11. Plasma conc 15 was given and Elem. Rate const. What will be the concentraion after
8 hours (K= 0.15 hr-1 and Cp= 15)
a) 4.5 mg
b) 10mg
c) 1.5 mg

12. Denosumab MOA? RANK Ligand inhibitor


The 1st biologic agent approved for the treatment of osteoporosis, is a human
monoclonal antibody that binds receptor activator of nuclear factor kappa‐B ligand
(RANKL), thus preventing interaction with its receptor (RANK) on the surface of
osteoclast precursors and osteoclasts. Inhibition of RANKL results in diminished
osteoclast formation, function and survival, and the marked reduction in bone
resorption increases cortical and trabecular bone mass and strength

13. If a patient has an INR > 20 and active bleeding that is clinically significant (i.e.,
hematuria), the pharmacist should?
a) INR above 4.5 inc risk of major bleeding
b) Protamine sulphate completely resolve LMWH effect
c) hold the drug therapy, administer vitamin K & fresh frozen plasma
d) INR within range so no bleeding

14. Start from the medline of your teeth and


count four teeth to lateral. What is it called?
a) Incisors
b) Pre molar
c) Molar
d) Wisdom

15. Dental prophylaxis for patient with


mitral valve with antibiotic from which?
a) Strept Viridins
b) Strept aureus

16. Which is the most common organism


in Otitis Media?
The primary bacterial pathogens involved in
acute otitis media are Streptococcus
pneumoniae, Haemophilus influenzae and
Moraxella catarrhalis.

17. Reaction given of conjugation


of Salicylic acid.
What type of conjugation?
a) Glycine
b) Glutathione
c) Acetylation

18. Patient is ok with SABA then


she started toexperience less
response. What is it called?
a) Chemotaxis
b) Tachyphylaxis
c) Anaphylaxis
Tachyphylaxis is a medical term describing an acute, sudden decrease in response
to a drug after its administration; i.e. a rapid and short‐term onset of drug
tolerance. It can occur after an initial dose or after a series of small doses.
Increasing the dose of the drug may be able to restore the original response

19. Vit A Which is incorrest? Given with Pregnancy

20. QT Prolongation may cause:


a) Ventricullar fibrillation
b) Atrial Fibrillation

21. Given P value = 0.25 what will be ur observation and the drug has risk reduction 10%
a) Not significant and not clinically important
b) No significal difference
c) There is a sig, difference

22. Patient has prescriped 500 mg of a tablet but he needs suspension


a) Order new prescription
b) Give him the equivalent dose of suspension
c) Refuse to fill
d) Fill the tablet

23. Cystic fibrosis is ttt by?


a) pancreatic enzymes
b) potassium
Cystic fibrosis is an inherited disorder that causes severe damage to the lungs, digestive
system and other organs in the body.
The treatment options may include:
 Antibiotics to treat and prevent lung infections
 Anti-inflammatory medications to lessen swelling in the airways in your lungs
 Mucus-thinning drugs to help you cough up the mucus, which can improve lung
function
 Inhaled medications called bronchodilators that can help keep your airways open by
relaxing the muscles around your bronchial tubes
 Oral pancreatic enzymes to help your digestive tract absorb nutrients
24. Which is the name of Heroine?
Diacetyl morphine

25. All are common pathogen in CAP EXCEPT?


Enterococci
CAP Pathogens:
Strep. Pneumoniae (accounts for half of cases
requiring hospital admission), Mycoplasma
pneumoniae, H. influenzae, Chlamydophila pneumoniae, M. catarrhalis, Legionella spp.,
M. tuberculosis (uncommon). Gram-negative bacilli, e.g., E. coli, Klebsiella spp.,
Enterobacter spp., Serratia spp., P. aeruginosa

26. Drug for Resistant schizophrenia


a) Quetipine
b) Reserpine
c) Clozapine

27. Why in elderly there is a decrese in serum creatinine?


decrease production of creatinine because lean body mass (which is the source of
creatinine) decreases with age.

28. 1ry antibiotic for otitis media?


a) Amoxicillin
b) Amox/clav
c) others

29. Patient with uncontrolled gout what will happen? Tophi

30. Ethics scenario your waiver co payment fees for patient, you violate
a) Justice
b) Illegal

31. Patient in ER with asthma O2 level 90%. He takes Fluticasone and SABA. What will
be the added therapy?
a) Keep the same
b) increase the dose of fluticasone
c) Make SABA every 20 min.
d) Add Formeterol
32. Which drug work on ppar alpha and ppar gama?
c) Sitagliptin  DPP ‐ 4 Inhibitors
d) Fenofibrate  PPAR α
c) Pioglitazone  Dual PPAR α / γ agonist (also Lobeglitazone)
d) Rosiglitazone  PPAR γ

33 Fibrates work by
a) IL 6
b) Increase TG excretion
c) other unrelated options

34. which test is used for colorectal cancer screening:


A- pap smear test
B‐ fecal occult blood test
C- prostate specific antigen
D- colon specific antigen

35. What is the nominal vriable from the following


a) Height
b) Weight
c) Eye color

36. What is expressed by dollars?


a) Cost Benefit analysis
b) Cost Effectiveness analysis
c) QALY

37. Breast feeding women want a reliable contraception


a) Medroxyprogesterone
b) Combined inj
c) combined oral
d) Progesteron only tab

38. What is the thing that you can ask if you want to hire someone? Interview question
a) Are you married?
b) What is your native language?
c) Have you covicted any crime?
39. Vegeterian lack in
a) Vit B1
b) Folic Acid
c) Vit B12  MCV is high in vit B12 anemia
d) Vit B2

40. Manifestation of Ulcerative colitis?


a) Blood in stool
b) Constipation
c) Weight gain
Symptoms: diarrhea, abdominal pain, rectal bleeding, weight loss, extraintestinal
manifestations (aphthous ulcers, arthritis, erythema nodosum, iritis, perianal disease,
fever).

41. What is true regarding Crohn's disease?


a) Occurs from mouth to rectum
b) Confinement of disease to the colon and rectum
c) No fistula formation

42. Isotrietinon most common side effect


a) Mucousitis Most common dure to mucocutaneous dryness
b) Follulitis
c) Skin pigmentation
d) Toxic epidermal necrolysis Rare
Side effects of isotretenoin: Teratogenicity. Common: mucocutaneous dryness, myalgia,
arthralgia, headache, photosensitivity. Uncommon–Rare: hypertriglyceridemia (monitor
for dyslipidemia), mood disorder, possibly suicide ideation, pseudotumor cerebri,
erythema multiforme, Stevens-Johnson syndrome, toxic epidermal necrolysis.
Tetracyclines: rare cases of benign intracranial hypertension (pseudotumor cerebri):
allow 7 days washout after stopping tetracyclines before starting isotretinoin.

43. What is true regarding uric acid? It is an end product of Purine metabolism

44. Lobe that contains the primary motor area (motor cortex) that enables voluntary
control of skeletal muscle movements:
A) frontal lobe
B) occipital lobe
C) parietal lobe
D) diencephalon
E) temporal lobe

45. Demylinated (destruction of cell myelin sheeth) disease


a. Cardiomegaly
b. Hemorrhagic cystitis
c. Myasenthia gravis
d. Multiple sclerosis

46. Which drug has positive ionotropic action


a) Amlodipine
b) Enalapril
c) Dobutamine
d) Furosemid

47. Drug cause Gingival hyperplasia


a) Phenytoin
b) Verapamil
c) Gabapentin
d) Lithium
S.E of phenytoin: Gingival hyperplasia; rash 5–10%, which rarely can be very serious;
increased liver enzymes; blood dyscrasias; dose-related encephalopathy; coarse facial
features. Idiosyncratic: Skin rash and Steven Johnsons syndrome. Birth defects like cleft
palate. Headache, nausea, vomiting, constipation, dizziness, feeling of
spinning, drowsiness, trouble sleeping, or nervousness may occur. Hirsutism and acne.

48. Which is released from the hypothalamus


a) TSH
b) Oxytocin
c) Testosterone
The hormones produced in the hypothalamus are corticotrophin‐releasing hormone,
dopamine, growth hormone-releasing hormone, somatostatin, gonadotrophin-releasing
hormone and thyrotrophin-releasing hormone.
Oxytocin (Oxt) is a peptide hormone and neuropeptide. It is normally produced in
the hypothalamus and released by the posterior pituitary. It plays a role in social
bonding, sexual reproduction, childbirth, and the period after childbirth.

49. Teriparatide used in? Osteoporosis


50. Exenatide is
a) DPP4
b) Meglitinide
c) Aromatase inhibitor
d) GLP‐1

51. Tadalafil is prefereed over sildenafil for the following reasons


a) After a fat heavy meal
b) For older person

52. Pt admitted with hyperthyroidism symptoms. What is the lab test you can order?
a) B12
b) Serum TSH
c) Plasma thyroglobulin

53. untreated hypothyroidism symptoms


a) Dry skin
b) weight loss
c) diarhea
d) tachycardia

54. How many days for the next ovulation in a woman in her 1st day of mensturation
a) 28
b) 14
c) 7

55. Drug induced kidney injury includes are all of the following except:
a) Sulfonylurea
b) vancomycin
c) ACEIs
d) Cephalosporins
e) Spironolactone

56. Acute Acne treatment in senstive skin


a) Benzoyl peroxide 5% in alcohol
b) Benzoyl Peroxide 2.5% in alcohol
c) Benzoyl peroxide 5% in acetone
d) Benzoyl peroxide 5% in water
e) Benzoyl peroxide 2.5% in water
Lower strengths of benzoyl peroxide may provide similar efficacy with less
irritation.

57. Which of the following about Metabolic Acidosis is NOT TRUE?


a) It generally occurs due to loss of bicarbonate from the body.
b) Hypokalemia is generally a result of metabolic acidosis.
c) Administration of Arginine HCl generally resolves metabolic acidosis
d) It stimulates the respiratory center to increase excretion of CO2 from the body.

58. IgE exhibit its action by activating:


a) Macrophages
b) T-cells
c) Mast cells
d) B cells
e) natural killer cells

59. Protamine sulphate can be used as an anitdote for overdose for?


a) Dalteparin
b) Dabigatran
c) Rivaroxiban
d) Apicixiban
e) Warfarin

60. OTC and OATP are


a) Transporter
b) cytokines
c) Interleukins
Ornithine transcarbamylase (OTC) (also called ornithine carbamoyltransferase) is
an enzyme that catalyzes the reaction between carbamoyl phosphate (CP)
and ornithine (Orn) to form citrulline (Cit) and phosphate (Pi). There are two classes
of OTC anabolic & catabolic.
An organic‐anion‐transporting polypeptide (OATP) is a membrane transport
protein or 'transporter' that mediates the transport of mainly organic anions across
the cell membrane. Therefore, OATPs are present in the lipid bilayer of the cell
membrane, acting as the cell's gatekeepers. OATPs belong to the Solute Carrier
Family (SLC), more specifically the Solute Carrier Organic Anion (SLCO) gene
subfamily
61. What is true about anticoagulants and bleeding?
a) INR above 4.5 - increase risk of major bleeds
b) Protamine sulphate completely resolves affects of LMWH 3.
c) INR within therapeutic range - no bleeding
d) All are true

62. What is the lab test you should order to confirm appendicitis
a) WBC
b) ESR
Appendicitis is an inflammation of the appendix. It's a medical emergency that almost
always requires surgery as soon as possible to remove the appendix. Luckily, patient
can live just fine without it.
There's no blood test to identify appendicitis. A blood sample can show an increase
in your white blood cell count, which points to an infection.
Tests can help diagnose appendicitis: Examination of abdomen to look for
inflammation, Urine (pee) test to rule out a urinary tract infection, Rectal exam, CT
scans & Ultrasound.

63. Drugs cross placenta by


a) Active transport
b) Passive diffusion
c) Endocytosis

64. Pharmacy manager decided to sell product with less than its price, that called?
a) Competitive price
b) Loss leader
A loss leader (also leader) is a pricing strategy where a product is sold at a price
below its market cost to stimulate other sales of more profitable goods or services.
With this sales promotion/marketing strategy, a "leader" is used as a related term
and can mean any popular article, i.e., sold at a normal price.

65. Responsible for funding


a) Federal government Mainly federal but provincial too through taxes
b) Provencial government

66. Drug with non linear kinetic. What will happen upon increasing dose?
a) saturation of elimination
b) obey first order  it obeys zero order
67. MHC class I-histochemical complex
a) activate AB
b) activate cytokines
c) agglutination
d) cytotoxic T cells
The major histocompatibility complex (MHC) is a set of genes that code for cell surface
proteins essential for the acquired immune system to recognize foreign molecules
in vertebrates, which in turn determines histocompatibility.
The main function of MHC molecules is to bind to antigens derived from pathogens
and display them on the cell surface for recognition by the appropriate T-cells. MHC
molecules mediate interactions of leukocytes, also called white blood cells (WBCs), which
are immune cells, with other leukocytes or with body cells. The MHC determines
compatibility of donors for organ transplant, as well as one's susceptibility to
an autoimmune disease via crossreacting immunization.
The MHC gene family is divided into three subgroups: MHC class I, MHC class II, and MHC
class III. Class I MHC molecules have β2 microglobulin subunit which can only be
recognised by CD8 co-receptors. Class II MHC molecules have β1 and β2 subunits and can
be recognised by CD4 co-receptors. In this way MHC molecules chaperone, which type of
lymphocytes may bind to the given antigen with high affinity, since different lymphocytes
express different T-Cell Receptor (TCR) co-receptors.

68. Cisplatin MOA? Platinum likes with DNA


Cisplatin is believed to kill cancer cells by binding to DNA and interfering with its
repair mechanism, eventually leading to cell death

69. Patients complaints of GERD, when should u refer to physician?


a) painful swallowing
b) regurgitation
c) excess salivation
d) heart burn at night

70. Chronic plaque psoriasis appear on?


a) Abdomin
b) Face
c) neck
d) trunk
e) Elbow  also, neck, scalp & back & knee
70. Fisrt line for SLE?
a) Mycophenolate
b) Tacrolimus
c) Hydroxychloroquine

72. Folic acid in pregnancy need to be increased with:


a) Phenytoin
b) Lamotregine

73. Primary literature


a) Clinical trials
b) pubmed
c) CPS
d) eTC

74. Scopolamine work as? anticholinergic


Tertiary amine plant alkaloid, produces peripheral effects similar to those of atropine.
Acts by interfering with the transmission of nerve impulses by acetylcholine in the
parasympathetic nervous system (specifically the vomiting center).
75. Essential Amino acid?
Lysine

76. Fentanyl works on?


a) MU
b) Mu & Kappa
c) Delta
Fentanyl binds to opioid receptors,
especially the mu opioid receptor, which
are coupled to G‐proteins

77. Histones related to? DNA


Histones are highly alkaline proteins found in eukaryotic cell nuclei that package
and order the DNA into structural units called nucleosomes.
They are the chief protein components of chromatin, acting as spools around which
DNA winds, and playing a role in gene regulation

78. Major cause of sinusitis? S. pneumonia


Causes may be allergic, viral, bacterial or fungal, but acute rhinosinusitis is most
often secondary to viral respiratory infections.
The most common bacterial pathogens in acute rhinosinusitis are Streptococcus
pneumoniae and unencapsulated strains of Haemophilus influenzae. Moraxella
catarrhalis is common in children. S. aureus are more common in chronic sinusitis

79. Separation of organelles


a) Mass spectrometer
b) Gel electrophoresis
c) Centrifugation
Centrifugation is a technique which involves the application of centrifugal force to
separate particles from a solution according to their size, shape, density, viscosity of
the medium and rotor speed. Not only is this process used to separate two miscible
substances, but also to analyze the hydrodynamic properties of macromolecules.

80. What scenario will put the cofidentiality of patients in danger


When you place the computer screen in a way that anyone can see it.
81. Hydralazine metabolized by?
Acetyl gp incorporation (structures given)

82. Mechanism of action of Nystatin


a) cholesterol binding
b) Ergosterol binding

83. TTT of Onychomycosis


Oral therapy
 Itraconazole duration: Fingernails: 6 wk. Toenails: 12 wk
 Floconazole duration: Fingernails: 12–16 wk Toenails: 18– 26 wk
 Terbinafine duration: Fingernails: 6–12 wk. Toenails: 12– 24 wk.
Topical therapy
 Ciclopirox olamine 8% lacquer monotherapy, a pyridone antifungal, daily for 48
weeks.
 Efinaconazole 10% topical solution, triazole antifungal, applied to affected nail once
daily for 48 w.

84. Baby with two impetigo lesions under his nose


a) Amoxicliin
b) Topical Mupirocine
c) Erythromycin
Impetigo: Superficial infection of skin primarily caused by Staphylococcus aureus and
most often affecting young children.
Two clinical forms include:
Crusted or nonbullous (S. aureus and/or Streptococcus pyogenes – found in Face, arms or
legs)  treated by topical antibacterial as fusidic acid, mupirocin, ozenoxacin for 5 days.
Bullous (S. aureus-mediated toxin - found in Moist, intertriginous areas (axillae, neck
folds, diaper area)  treated by oral antibacterial as cephalexin for 7 days.

85. Turbuhaler councelling


a) Breath Forcebuly
b) Breath slowly, deeply
c) Breath fast, deeply

86. Drug that cause flushing


a) Anastrozole
b) Nifidipine
87. Extent of absorption is partially called?
a) Bioavailability
b) T1/2
c) Vd

88. DOC of panic Attack? Venlafaxine

89. Iontophoresis used to:


Iontophoresis, also known as Ionization, is physical process in which ions flow diffusively
in a medium driven by the use of an electric current. Iontophoresis is to be distinguished
from the carriage of uncharged molecules by diffusive fluxes of other molecules, especially
of solvent molecules, for example by electro-osmosis, that is to say by flux of uncharged
solvent molecules carried as a cross-effect of iontophoresis.

90. Tertiary structure of protein is determined by


a) Aminoacid sequence
b) Covalent bonds between native structures
Protein tertiary structure is the three dimensional shape of a protein. The tertiary
structure will have a single polypeptide chain "backbone" with one or more protein
secondary structures, the protein domains.
Amino acid side chains may interact and bond in a number of ways.
The interactions and bonds of side chains within a particular protein determine its
tertiary structure. The protein tertiary structure is defined by its atomic
coordinates. These coordinates may refer either to a protein domain or to the entire
tertiary structure.

91. Methadone is
a) Narcotic preparation
b) Straight Narcotic
c) Exempted

92. Amorphous form differs from crystalline form as it is:


a) Amorphous Is generally less soluble than amorphous
b) Amorphous is more soluble than Crystalline
c) Amorphous is more susceptible to polymorphism
d) Amorphous differs in the chemical composition
93. Natalizumab is used for multiple sclerosis to:
a) Prevent relapse
b) Decrease symptoms
c) Cure the disease

94. Optic nerve cells


a) Rods
b) Cones
c) Horizontal (this controls rhods and cones)
d) Ganglionic
The optic nerve is composed of retinal ganglion cell axons and glial cells. Each
human optic nerve contains between 770,000 and 1.7 million nerve fibers, which
are axons of the retinal ganglion cells of one retina.

95. In Type 1 diabetes and pt takes flexible insulin, what should be done to prevent
hypoglycemia?
a) Decrease target glucose level
b) Avoid snacks midday
c) Adjust dose before exercise

96. Needs healthy cells that replicates (which virus will undergo division)
a) Adenovirus
b) Retrovirus
c) Plasmid

97. What is true from the following in controlling cough


a) Codiene is given to a child 6years old.
b) Dextromethorphane can be given to child 6 yo to control productive cough.
c) Guaifenesin can be given to 6 yo to control productive cough.

98. Used as 1st line in LYME disease


a) Aminoglycocide
b) Cephalosporin
c) Ciprofloxacine
d) Doxycycline

99. Who writes policies and procedures for community pharmacy practice
a) Federal gov
b) provential
c) NAPRA

100. What does not undergo Glucuronidation?


a) Alkene
b) Nitrosamine
c) Phenol

101. What can be transferred to another pharmacy


a) Alprazolam
b) Testosterone
c) Methadone

102. Pharmacy manager attributed to


a) Minister of Health
b) NAPRA
c) Director of pharmacy and board (different wording)

103. Mass spectroscopy works by? Separation according to wt and charge

104. 1st line treatment of osteoarthritis?


a) Acetaminophen
b) ASA

105. All of the following can be abused EXCEPT


a) Diphenhydramine
b) Codiene 8 mg
c) Dextromethorphane

106. Side effect with immunosuppressant? infection

107. Atamoxiten used to treat? ADHD

108. Reducing particle size with morter and pestle?


a) Trituration
b) Levigation
c) Mixing
109. Registered pharmacy tech will do all of the following EXCEPT?
a) Review prescription for authenticity
b) Review RX for therapeutic compatibility
110. Question about Relative risk reduction calculation

111. A new medication advertise by a manufacture, the representative showed you the
brochure which says that it has 10% less CVD compared to placebo. What will
be most likely present in the brochure?
a) Relative risk reduction
b) Absolute risk reduction
c) Number needed to treat

112. Process of conversion of cholesterol to cholate (bile salt)?


a) Hydroxylase
b) Acetylation
c) oxidation

113. Blood enter the right atrium through? Superior vena cava

114. which is true about ferrous? ferrous fumerate has high elemental ferrous

115. what regulation is responsible for final formulary for a new on the basis of cost
effectivness and clinical trial:
a) canada vigilance
b) canada association for drug and technology in health
c) heath patented medicine proce review board canada
d) medication reporting system

116. Dysphonia associated with? Fluticasone

117. Syphilis is caused by? Treponema Pallidum

118. Enterococcus is resistant to


a) Piperacillin/tazobactam
b) Cefazolin
119. prepare 4 % minoxidil lotion from 2 % minoxidil lotion and minoxidil powder
(100% minoxidl). Allegation problem
120. Gemfibrozil can cause? Cholelithiasis
S.E: Upper GI disturbances, myalgias. Increased bile lithogenicity (cholelithiasis),
increased CK and creatinine (not representative of renal function deterioration).

121. Secondry prevention of stroke in atrial fibrillation


a) Asprin
b) warfarin
c) clopidogrel
d) asprin and dipyridamol

122. question related to genetic mutation

123. Drugs cause QT prolongation

124. micronization of powders improve tablets


a) Dissolution
b) Disintegration
c) Decrease surface area
125. Least useful drug in patient with acute coronary syndrome
a) ACEIs
b) BBs
c) ASA
d) Furosemide
ACS is due to stenosis of coronary arteries and reduced blood flow. Furosemide
doesn't have an effect on coronary arteries

126. Man cannot sleep, which drug used to treat insomnia?


a) Zopiclone
b) Citalopram
c) Bupropion
Zopiclone, is a nonbenzodiazepine used to treat trouble sleeping. Zopiclone is
molecularly distinct from benzodiazepine drugs and is classed as a cyclopyrrolone.

127. Suitable analgesic for patient taking ASA and clopidogrel


a) Celecoxib Both choices are bad for the heart, but ASA is already a risk for GI
ulcers-irritation, so adding a non selective NSAID will increase this risk,
b) Ibuprufen unless we use selective cox2 NSAIDS. Also there is an interaction between
ASA and ibuprofen, the latter decrease the antiplatelet action of ASA

128. Losartan block


a) angiotensin I receptor
b) angiotension 2 receptor
Binds to AT1 receptors and prevent formation of angiotensin II, a naturally
occurring substance that causes blood vessels to narrow.

129. Polysorbate 80? Solubilizing agent

130. Erectile dysfunction


a) opioids For me A
b) alpha blockers

131. Bed sores on sacrum of an old man? debride daily

132. UDP Uridine diphosphate, abbreviated UDP, is a nucleotide diphosphate. It is


an ester of pyrophosphoric acid with the nucleoside uridine. UDP consists of
the pyrophosphate group, the pentose sugar ribose, and the nucleobase uracil.
UDP is an important factor in glycogenesis. Before glucose can be stored
as glycogen in the liver and muscles, the enzyme UDP‐glucose
pyrophosphorylase forms a UDP‐glucose unit by combining glucose 1‐phosphate
with uridine triphosphate, cleaving a pyrophosphate ion in the process. Then, the
enzyme glycogen synthase combines UDP‐glucose units to form a glycogen chain.
The UDP molecule is cleaved from the glucose ring during this process and can be
reused by UDP‐glucose pyrophosphorylase

133. pharmacy with many registered pharmacy technicians, what is the main role of
pharmacists? Theraputic soundness

134. child 6 yrs, physician prescribed 20mg/kg dose at 0.5mg/ kg / min rate. If drug is
present in 1000 ml dextrose bottle. What is rate of infusion
a) 40 ml/hr
b) 10ml/hr Missing info ; weight of child to calculate full dose

c) 80 ml/hr
d) 150 ml/hr
e) 180 ml/ hr

135. Patient taking enteric coated asprin, cautious with


a) Topical salicylate
b) tramadol
c) ibuprofen
d) acetaminophen
e) diclophenac gel

136. amount/dose of drug in the blood per concentration of drug in plasma will give?
a) bioavailabity
b) volume of distribution
c) elimination rate
d) half life
e) plasma concentration

137. urea 10%, glycerin 5%, salicylic acid 30 % mittee 60 ml

138. 10 grams of 15% cream. How much drug is the to make total formulation?
a) 1500 g
b) 150 g Unclear question
c) 15 g
139. Best clinical evidence?
a) Randomizd controlled studies
b) Meta analysis of observational study
c) Cohort study
d) Case controlled study

140. Gel Electrophoresis


a) Acrylamide
b) agarose Both used but Polyacrylamide is better . It detects both protein and DNA .
Agarose only one of them
c) dextran

141. Effective councelling should be using?


a) Open ended questions
b) Closed ended questions

142. Question related to separtation of protein


a) mwt of protein
b) charge of protein Molecular size .. p. 145
c) size of protein

143. In alimentary canal, absorption takes place in


a) Mucosa
b) Submucose
c) Serosa muscles
d) adventia

144. Drug cause headache? felodipine

145. Dose needed for the patient is 0.24 mmol/kg. The patient weight 132 Ib. To prepare
250 of D5W for this patient? See question 7

146. Difference between neucleoside and neucleotide is? Phosphate gp


A nucleoside consists of a nitrogenous base covalently attached to a sugar (ribose or deoxyribose) but without
the phosphate group. A nucleotide consists of a nitrogenous base, a sugar (ribose or deoxyribose) and one to
three phosphate groups.
147. Single-nucleotide polymorphism SNP is a:
a) Single Non polymorphism
b) Single change in one base in DNA
148. Pt takes Clarithromycin, Citalopram and lisinopril, what will happen?
a) Risk of torsa de point
b) Increase plasma conc of citalopram
c) Increase clarithromycin plasma conc

149. pt has incontinence while excecise, coughing and sneezing, What type of
incontinence he has
a) Mixed
b) Stress
c) Urgent
d) Overflow

150. Role of surfactants


a) Form micelles to dissolve lipids
b) Increase surface tension
c) Decrease surface tension
d) increase surface area

151. Nephrolithiases, what is contraindicated


a) Phosphates
b) Magnesium
c) Calcium

152. Respiratory centre is present in


a) Brain stem (Medulla oblongata)
b) Midbrain
c) Hyppocampus

153. All the symptoms of Otitis media except


a) Fever
b) Ringing in ear
c) Wheezing

154. Which one is market niche rather than market segment


a) Elderly
b) Children with asthma
c) Female pt
d) High income earner
e) Parents of young children
https://www.thebalancesmb.com/how-to-find-and-master-a-niche-market-2948380

155. You have a patient who converted from male to female (transgender) On his first
visit to the pharmacy, what you will instruct your staff to do?
a) Ask him whether he'd like to be called as a male or female
b) Use of gender-neutral language if the patient doesn't want to discuss the matter
c) Just call him according to what you have in the system
d) don’t discuss the topic until he does
Tips for interacting  If it is appropriate and not embrassing, you can ask the pateint politely
with a transgender and privately what they would like to be adressed, for example; hi, I’m
client: osama, what would you like me to address you?
 Don’t assume, clarify. for example, I noticed you have a different name
in the prescription than that you used to have.
 Avoid using gender pronouns, but try to use neutral language i.e; use you
instead of sir or madam.

156. Ambulatory services are provided in


a) Outpatient clinics
b) Long term care facilities
c) Family care
d) Emergency dpartment
e) patients who can walk independantly
Ambulatory care or outpatient care is medical care provided on an outpatient basis,
including diagnosis, observation, consultation, treatment, intervention, and
rehabilitation services. This care can include advanced medical technology and
procedures even when provided outside of hospitals.

157. Most people get 50% of Vit K from:


a) Supplements
b) Food
c) Bacteria in colon

158. Regularity authority for efficiency and safety of drug before market?
a) Common Drug Review (After marketing for side effects)
b) CADTH (I think this after marketing)
c) Health Canada Therapeutic Directorate Health Canada Therapeutic Product Directorate TPD
159. What is the recent pharmacists issue these days in Canada
a) Collaborative pt care
b) Increase in physician assistants affecting pharmacist scope
c) End of life scenario (euthanasia related) and pharmacist's role
d) Recent international trade involving Canadian pharmaceuticals

160. Patient is taking lots of medications and he is annoying, what is you reflect that
shows empathy?
a) You seem to be overwhelmed
b) We will help you sort things out to fit your daily life
c) Lots of patients has the same situation

161. Long term use of PPI Will cause


a) Decrease Vit C
b) Bone fraction

162. Which is not cause by virus


a) Vericella
b) Rubella
c) Impetigo
d) Hand-foot-mouth diseases

163. What is the drug that is not for immediate action


a) ASA
b) Alteplase
c) B Blocker
d) Clopidogril

164. P value was 0.08, Alpha 1 error 5 %


a) Accept null hypothesis as there is significant difference
b) Reject null hyp as there is a sig difference
c) Fail to reject null hypothesis and there is no significant difference

165. Which one of the following if missed 5 days will not cause severe health problems
a) Propranolol
b) Clopidogril
c) Levothyroxin
d) Phenytoin
166. Acetaminophen is metabolised to toxic metabolite by
a) Acetylation
b) Methylation
CYP450 2E1 oxidation
c) Hydroxylation
d) Oxidation
e) Reduction

167. In busy pharmacy, what is appropriate to do regarding immunization in high season


a) Inform patients that no rx filling during immunization season
b) Delegate a pharmacy technician to collect patient's therapeutic data for
pharmacist review
c) Prepare shots and refrigerate it

168. 80 Patiet with Dementia and takes Rivastigmine oral tablet but his caregiver
complaints that she gets annoying GI side effects, what do u suggest?
a) Change to Transdermal
b) Recommend lower dose
c) Give her Diphenhydramine, but that may worsen her case
d) Memantine patches

169. What causes Alopecia? Cyclophosphamide

170. patient has diarrhea for 24 hours and his last stool has blood Not taking any
medication but allergic to ASA. What will pharmacist advice
a) recommend loperamide
b) no need to take anything
c) refer to physician
d) councel on consequences

171. Release of the active ingrediant from the trandermal patch INVITRO depends on
a) Adhesive
b) Color
c) Polymer

172. In compounding drugs, you should document all of the following EXCEPT
a) Formulation time
b) expiry dates of the raw materials
c) compounding instructions
173. Question about drug that mask hypoglycemic effect? B Blockers

174. ADH stimulated by? Osmolarity increase in blood

175. Laxative in infant


a) Besacodyl
b) Glycerine supp

176. dry powder formulation that is Senstive to light, Senstive to Moisture, Poor
aqeuous solubility and absorption, can be formulated as?
a) Film coated tablet
b) Sublingual
c) Nasal spray

177. STD that doesn't need the treatment for another partner
a) Trichomoniases
b) Candida
c) Chlamydia

178. K= 0.005/day, find the time for drug to reach 10% of its original strength
a) 21 days
b) 461 days
c) 390 days

179. Methylphenidate side effect? wt loss


Common, usually transient: anorexia, insomnia, weight loss, irritability, dizziness,
weepiness, headache, abdominal pain. (monitor weight & appetite every 6 months)

180. Bevacizumab act on: Vascular Endothelial Growth Factor Inhibitors (VEGF)

181. What is not given with Celiac disease


a) Rice
b) Peanut
c) Barley

182. which one causes contraction?


a) Inhibitin
b) Mucin
183. Highest cation in plasma?
a) Na
b) Hco3
c) Cl

184. Highest iron absorption


a) Empty stomach
b) with food
c) with orange juice

185. Not a cause of UTI


a) Smoking
b) Casterization
c) Neurological disorders
d) Sex
e) Pregnancy
JANUARY 2017

1. Which is not a fiber?


a) Starch
b) pectin
c) lignin

2. Capsule lock name was given. Why is this


technology employed?
To prevent dose tampering
Tamper – resistant tablets:
Prescription sedatives are usually taken in pill form; however, some are available
as suppositories or prepared as a solution for injection.
Some people tamper with the medication for non‐medical use for the drug’s
euphoric effects. Tampering involves changing the form of the medication or the
route by which it is taken or both.
There are 3 main approaches to deter tampering with opioids, each based on a
presumption that abusers will tamper with intact tablets. These include agents with
physical barriers to crushing, chewing, and extraction; agents with sequestered
aversive agents; and agents with sequestered opioid antagonists.

3. Smoking Increase what? polycyclic aromatic hydrocarbons


4. Statin and Erythromycin interaction cause?
a) Increased level of statins
b) Decrease levels of statins
c) No effect will happen
Erythromycin is a CYP3A4 inhibitor which decrease metabolism of statins

5. Which of the following is an example of public funded and privately delivered? Options
were several cases  study this algorithm
6. Which of this is not a part of pharmacovigilance process?
a) Drug surveillance
b) Medication reconciliation

7. Giving influenza vaccines at community


pharmacy is?
a) Primary prevention
b) Secondary prevention
c) Tertiary prevention

8. You buy a well know named pharmacy


corporation and pay fees for using the name of
this company. What is this called?
a) Franchise
b) Banner
c) Independent

9. If patient does not fill the prescription what


is it called?
a) Non adherence
b) Non compliance
Non‐adherent: failing to fill or take a new
prescription
Non‐Compliance: The act of not taking medication as the doctor intended (Correct
medication, Correct amount of medication, Correct time of day, Correct length of
time)
10. The federal government responsible for all except
a) health promotion program
b) health research
c) funds the provincial and territorial health care services
d) funds for hospitals
e) Import and export of pharmaceuticals in Canada

11. What is true for medicated powder?


a) It is called Chartulae
b) It is given as one-time dose
c) It is used for very potent drugs
d) It is given in wrapped paper
Chartula (plural chartulae) (medicine, obsolete) A small piece of paper for the
storage of one dose of a medicinal powder.

12. For long term effective gene therapy, gene needs to be:
a) Gene needs to be Integrated into nuclear membrane
b) Gene needs to be Fully integrated into chromosome

13. What is most prevalent endocarditis infection with the drug abusers who uses
syringes regulary?
a) Staph aureus
b) HIV
c) HCV

14. Drug micronization in tablet helps for?


a) Increases the dissolution of tablets
b) Increases the disintegration of tablets
c) Decrease surface area

15. Antiperspirant for hyperhidrosis includes:


a) Triclosan- antibacterial
b) aluminum chloride
c) sodium benzoate
d) zinc oxide
e) parabens
All antiperspirants have an aluminum‐based compound as their main ingredient.
16. A patient has migraine 3 to 4 times a month. Currently also diagnosed with benign
prostatic hyperplasia. Doctor has prescribed prazosin. Medical profile also includes
constipation. What drug is recommendation are suitable for migraine prophylaxis?
a) Amitriptyline
b) Verapamil
c) Propranolol
d) Sumatriptan

17. Drug therapy with Carbamazepine requires? White blood cells monitoring
S.E of carbamazepine: Rash 5–10% (Steven Johnson Syndrome), which rarely
serious; ↑ liver enzymes; transient neutropenia (monitor WBCs); aplastic anemia
(extremely rare); hyponatremia.

18. Methylphenidate comes under which schedule? CDSA controlled drugs Part 1

19. What is recommended for infant with teeth pain?


a) Ibuprofen solution massage on gum
b) Acetaminophen syrup
c) Benzocaine 10% gel
d) Teething biscuits
For teething pain in infants that is not relieved by nonpharmacologic measures, oral
analgesics, such as acetaminophen or ibuprofen, can be used at usual analgesic doses.
Systemic analgesics should never be rubbed on the gum.
Teething biscuits are not recommended because of their sugar content.
Topical anesthetic agents (benzocaine 7.5–10% in a gel formulation) use is controversial

20. What should be of least concern to pharmacist while in situation of pandemic?


making more profit out of his sales

21. Which of this diagnosis requires further testing


for prostate cancer?
increased levels of PSA in blood

22. Protein C and its cofactors S?


a) Inhibits clotting process
b) Thrombolytic
c) Localize clot
d) Dissolve the clot
23. Which of these needs to be avoided in GI bleeding risk patient?
a) Naproxen
b) Celecoxib

24. Sumatriptan (Structure A) is formed from


structure B, what is structure B?
a) Tryptamine
b) Serotonin
c) Histamine

25. Liver cirrhosis is a type of end stage chronic liver disease? What clinical laboratory
tests are elevated?
A) ALT
B) AST
C) Bilirubin or urobilinogen
D) Albumin
E) Proteins
AST and ALT are increased in acute liver damage. But with the passage of time or in
chronic disease sometime they also present in normal range.
Bilirubin level increases when 50 liver has lost its function so in chronic or end
stage liver failure bilirubin level increases more than Aminotransferases.

26. Pt. was taking lispro tid after meals and glargine bid pt was experiencing
hypoglycemia in night. What should pharmacist recommend?
a) Reduce all lispro doses
b) Add abedtime snack Usually go for bedtime snack before adjusting the dose

c) Increase calorie intake


d) Reduce glargine doses

27. What is true about Specificity of an instrument?


a) Negative results about truely negative process
b) Positive results about truely positive process  sensitivity

28. How does glyburide work?


It is a 2nd generation sulphonyl ureas that blocks ATP sensitive K+ channels, result
in depolarization, Ca2+ influx, and insulin exocytosis. Reduce hepatic glucose
production & increase peripheral insulin sensitivity.
29. What is Incidence rate? new incidences per unit time period

30. Which of this is not a physiological barrier for


Innate immunity?
a) inflammation
b) tears
c) gastric acids
d) skin
e) mucus secretions

31. What happens when drug gets displaced from plasma binding proteins?
Apparent volume of distribution increases

32. Photodegradation of drugs in tablets can be prevented by? Film coating

33. O.U. means? both eyes

34. Dose of folic acid in girl of child bearing age? 5 mg 0.4 mg

Case study of person with BPH and hypertension. Doctor recommended terazosin and
Finasteride. Urinary residual volume is 335 ml.

35. What do you call this condition?


a) Urinary retention
b) Urinary incontinence
c) Cystitis
36. What do you think about his therapy?
a) Appropriate therapy
b) Polypharmacy
c) Duplication of therapy

37. When will terazocin show effect?


a) 1 - 2 weeks
b) 4 ‐ 6 weeks
c) 6 months  this is for finasteride

38. What is the main reference for drug overdose?


a) CPS
b) Clinical drug information
c) CTMA

39. A lot compaction pressure in tablet may cause? capping

40. Achlorhydric condition leads to? Vit B12 deficiency

41. Deficiency of instrinsic factor may cause? Vit B12 deficiency

42. Which of this a flu immunization schedule for North America? mid Oct to mid Dec

43. In adults, what is the main organism causing meningitis?


a) S. pneumonia
b) S. aureus
c) M. tuberculosis
The most common pathogens are the same pathogens that colonize the mucosal surface of the
respiratory tract and include Streptococcus pneumoniae, Neisseria meningitidis and
Haemophilus influenzae. Most cases of meningitis occur during periods of bacteremia, there may
be direct spread of bacteria (especially with S. pneumoniae) from preexisting otitis media or
sinusitis, more rarely, from defects in the base of the skull.
44. What ion is formed in intermediate of nicotine
metabolism?
Iminium ion
Cotinine is an active metabolite of nicotine that
remains in the blood with a half‐life of 18–20
hours, making it easier to analyze.

45. What is true for dry skin treatment counselling?


Apply emollients immediate after bath

46. A knockout mice has Apolipoprotein gene removed. What will it have as feature?
a) high cholesterol
b) increased weight
A knockout mouse, or knock‐out mouse, is a genetically modified mouse (Mus musculus) in which
researchers have inactivated, or "knocked out", an existing gene by replacing it or disrupting it
with an artificial piece of DNA.
The APOE gene provides instructions for making a protein called apolipoprotein E. This protein
combines with fats (lipids) in the body to form molecules called lipoproteins. Lipoproteins are
responsible for packaging cholesterol and other fats and carrying them through the bloodstream.
Maintaining normal levels of cholesterol is essential for the prevention of disorders that affect the
heart and blood vessels (cardiovascular diseases), including heart attack and stroke.

47. EER is 74% and CER is 64%. What is NNT? 10


NNT = 1 / ARR ARR = EER – CER = 74% ‐ 64% = 10% NNT = 1 / 10% = 10

48. EER is 20% and CER is 16%. Value of X is 4%. What is X? Absolute risk reduction

49. What is the highest expenditure for private insurance in Canada?


a) Pharmaceutical drugs
b) Hospitalization
c) Doctors salary

50. Which of this antipsychotic drug is wrong choice for depression and diabetic patient?
a) Quetiapine
b) Aripiprazole
c) Brexpiprazole
d) Olanzapine

51. What has additional benefit in COPD patient? Inhaled Tiotropium


52. What is important for pharmacy manager?
Customer loyality so they come again and buy from his pharmacy

53. What is included in total CBC report? Platelet count

54. What is the side effect of Amiodarone? Skin pigmentation


S.E: "4Ps" Pulmonary damage (irreversible), Peripheral T4‐T3, Hypo or
Hyperthyroidism, Pigmentation: deposits in skin cause bluish tinge skin color.
Phototoxicities. Corneal deposits cause blurred vision, hepatic toxicity, optic
neuritis, erectile dysfunction, and photophobia.

55. Which of this requires shake well before use indication? fluticasone inhaler

56. What is correct sequence for salbutamol and fluticasone


spray together? Salbutamol, Fluticasone and rinse

57. According to Oxford university clinical trials which is


most valid study?
a) Systematic review of observational study
b) Randomized controlled trials
c) Cohort study

58. Montelukast works on which receptors?


Cysteinyl leukotrienes receptors

59. How is two chains of insulin linked? Disulfide Bonds


Levothyroxine and ferrous gluconate
should not be taken orally at the same
60. Levothryroxine overdose cause? weight loss time. Products that contain iron may
interfere with the absorption of
levothyroxine and reduce its
effectiveness. You should separate the
61. Levothyroxin is contraindicated with? ferrous gluconate dosing of these medications by at least 2
to 4 hours if possible.

62. A women is taking Ramipril for her hypertension now becomes pregnant. What will
you recommend? change to methyldopa

63. What is drug of choice for obese diabetic patient? Metformin

64. In observation studies, what is true for bias?


65. How does antibody look like?
Y shaped heavy and light chain linked by
disulfide bonds

66. Which of following cause dysphonia?


Fluticasone

67. Salt and Sugar are added to


pharmaceutical products to?
decrease osmolarity or increase
hypertonicity

68. What is not a component of pcr?


Endonucleases
The basic components of a PCR reaction include
a DNA template, primers, nucleotides, DNA
polymerase, and a buffer.

69. Oxycodone prescription rule? refills not allowed

70. Which of this is a SNRI? Venlaflaxine

71. What is true about vapor pressure? Vapor pressure increase with temperature

72. Difference in symptoms between Osteoarthritis and Rheumatoid arthritis?


Morning stiffnesss less than 30 mins

73. What is true about Clonazepam prescription transfer?


check for previous transfer as only one is allowed

74. Adalimumab is used in? Rheumatoid arthritis

75. What is monitored while giving Prednisolone? glucose levels

76. What is most critical for cold chain technique in community pharmacy?
Temp controlled stock room in delivery area of community pharmacy

77. Chancre and alopecia is symptom of which STD disease? Syphilis


78. Naltrexone used in withdrawl of? Alcohol
MOA: competitive antagonist at μ, κ, and δ receptors, with a 10‐fold higher affinity
for μ than for κ receptors. Has longer duration of action than naloxone.

79. Mechanism of Dorzolamide in glaucoma? decrease production of aqueous humor

80. Risk of jaundice is associated with? Isoniazid


S.E: GI upset, hepatotoxicity (fatigue, flu like, anorexia and nausea & vomiting),
peripheral neuropathy, Skin rash, jaundice, Blood dyscariasis (agranulocytosis,
aplastic anemia, hemolytic anemia & thrombocytopenia) thereby monitor CBC
routinely. Give vit B6 pyridoxine 50 mg/day PO (to minimize the peripheral neuropathy)

81. Enalapril metabolism is


given in structure form. Ask
what is this chemical reaction?
hydrolysis by esterases

82. Which drug shows a high


alert medication and dangerous abbreviation according to ISMP?
a) Insulin 10 units
b) Methotrexate 5mg po daily
c) Warfarin 50mg po tid

83. A nurse make mistake in dose calculation and administer to patient. You find this error
what will you do?
inform the concerned in charge person and documents as per customary process

84. Oral vancomycin use? Diarrhea caused by P. Colitis

85. Impetigo is caused by? Staph aureus

86. Drug of choice in cat bite? Amox/Clav

87. Drug of choice in Absence seizures? Ethosuccimide

88. Unit of first order reaction is? Min‐1


89. Most basic amino acid?
A- Alanine B‐ Arginine C- Phenylalanine D- Methionine E- Leucine
Arginine is the most basic amino acid. There are three amino acids that have basic side
chains at neutral pH. These are arginine (Arg), lysine (Lys), and histidine (His). Their side
chains contain nitrogen and resemble ammonia, which is a base. Their pKa's are high
enough that they tend to bind protons, gaining a positive charge in the process.
Two amino acids have acidic side chains at neutral pH. These are aspartic acid or
aspartate (Asp) and glutamic acid or glutamate (Glu). Their side chains have carboxylic
acid groups whose pKa's are low enough to lose protons, becoming negatively charged in
the process.

90. Systemic lupus erythmatousus is side effect of? Hydralazine


SLE: Chronic, multisystemic autoimmune disease in which body's immune system mistakenly attacks
healthy tissue in many parts of the body.
Symptoms vary between people and may be mild to severe. Common symptoms include painful and
swollen joints, fever, chest pain, hair loss, mouth ulcers, swollen lymph nodes, feeling tired, and a
red rash which is most commonly on the face. Often there are periods of illness, called flares, and periods
of remission during which there are few symptoms.
A wide variety of drugs are capable of inducing a drug-induced lupus (DIL – HIP MCQ TT). Commonly
associated drugs include hydralazine, isoniazid, procainamide, minocycline, chlorpromazine, quinidine,
terbinafine and tumor necrosis factor inhibitors. The condition can develop weeks to months after
starting the drug and resolves rapidly upon discontinuation.

91. Pka of acid is 3.6 and pka of base is 10.4 what pH will you see zwitter ion formation?
Ans is (Pka acid + PKb base) / 2 = (10.4 + 3.6) / 2 = 7

92. A patient with Hba1c and hypoglycemia case was


given. Find what is subjective assessment?
HbA1c test:
 In all patients who have been started on antihyperglycemic
agents, aim to reach the desired HbA1c target within 3–6
months through dosage titration or addition of other agents
 For patients on insulin: check every 3-4 months
 For patients on oral antihyperglycemic or nutritional therapy:
check every 6 months.
 A higher HbA1c of up to 8.5% may be more appropriate if the
risk of hypoglycemia outweighs the benefits of tight control,
e.g., in frail elderly patients, those with limited life
expectancy or patients with a history of recurrent severe hypoglycemia
 HbA1c measurements every 3 months for patients who have not achieved target values; testing every 6
months may be acceptable in stable patients who consistently meet glycemic targets.
93. Warfarin is affected by polymorphism of which enzyme? CyP450

94. Testosterone formed in? Testicles


Testosterone is produced by the gonads (by the Leydig cells in testes in men and by the ovaries in
women), although small quantities are also produced by the adrenal glands in both sexes.

95. Which of the following is not a cause of UTI?


a) Catheterisation
b) Smoking
c) Neurological disorders
d) Sex
e) Pregnancy

96. In Canada, patient gets most nonverbal clues from what?


a) facial expression
b) posture
c) hand gestures
d) space between individuals
e) tone of voice

97. What is the drug to be used in stoke patient within certain duration of symptom?
a) Alteplase
b) ASA
c) Clopidogrel
d) Nimodipine
Treatment criteria for Alteplase in Acute Ischemic Stroke:
 Ischemic stroke causing measurable neurologic deficit in a patient ≥18 y
 Stroke onset >1 h and ≤4.5 h before alteplase administration

98. Which is prominent side effect of PDE 5 inhibitors? hypotension

99. What is associated with


increased age?
a) Hyperparathyroidism
b) Gastric acid secretion
c) Lean body mass
100. absolute contraindication of combined oral contraceptives? Migraine with aura
Absolute Contraindications to Combined Oral Hormonal Contraception:
 Breast cancer or hormone-dependent cancer
 Cerebrovascular disease, history of cerebrovascular accident
 Complicated valvular heart disease
 History of VT, PE, known thrombogenic mutations, MI or vascular disease.
 Diabetes with microvascular complications
 < 6 weeks postpartum if breastfeeding
 Migraines with aura at any age
 Hypertension (SBP ≥160 mm Hg or DBP ≥100 mm Hg)
 Severe cirrhosis or liver tumor
 Smoker >35 years of age (≥15 cigarettes/day)

101. Which of this is associated with risk of osteoporosis? Excessive alcohol


Risk Factors Prompting Assessment for Osteoporosis (Measurement of BMD):
Older Adults (≥50 y) Younger Adults (<50 y)
Age ≥65 y Fragility fracture, Prolonged use of
Clinical risk factors for fracture (men age 50–64 y, corticosteroids, use of other high-risk
menopausal women): vertebral compression, fracture fragility, medications, e.g., aromatase inhibitors,
fracture after age 40, prolonged use of corticosteroids, use of androgen deprivation therapy, Hypogonadism
other high-risk medications, e.g., aromatase inhibitors, or premature menopause (<45 y),
androgen deprivation therapy, parent with hip fracture, Malabsorption syndrome, Primary
osteopenia identified on x-ray, current smoking, high alcohol hyperparathyroidism, Other disorders strongly
intake, low body weight (<60 kg) or major weight loss (>10% associated with rapid bone loss or fracture.
of weight since age 25), rheumatoid arthritis. Race (asian & caucasian).

102. All cause denaturation of protein EXCEPT:


a) Increasing ph more than 11
b) Adding organic solvent
c) Adding saline
d) Freezing
e) Increasing temperature

103. Hypokalemia causes increased digoxin toxicity by mechanism at Na/K ATP pump.
How do you treat this?
a) Magnesium
b) Potassium
c) Sodium
104. What has an opposite effect to insulin?
a) Epinephrine
b) NMDA
c) GABA
d) Serotonin
Epinephrine and glucagon are equally effective in preventing the fall in glucose output induced by
insulin.
Epinephrine is more effective in preventing insulin‐induced hypoglycemia by virtue of its direct
inhibitory action on insulin‐stimulated glucose utilization.

105. Which of the following is a known antidote? Digitalis antibody


Digoxin Immune Fab is a sheep antibody (26-10) FAB fragment from sheep immunized with the digoxin derivative
Digoxindicarboxymethylamine. It is used as an antidote for overdose of digoxin.
106. Granisetron is related to? serotonin
Selectively block 5‐HT3 receptors in the periphery (visceral vagal afferent fibers) and in brain
(CTZ) which used for the prevention of acute chemotherapy induced N&V.

107. If the ratio of total body water increases in infants what will be the effect on
pharmacokinetics?
volume of distribution will increase for water soluble drugs

108. Federally covered service that includes? Cataract at eye clinic

109. All of the following is used in acute gout attack EXCEPT:


a) Allopurinol
b) Colchicine
c) Probenecid
d) Celecoxib

110. A patient is having renal and heart failure. What is the correct drug for bowel
evacuation? PEG 3350

111. Liposomes circulation in blood is increased by which method? PEGylation


PEGylation (often styled pegylation) is the process of both covalent and non‐covalent attachment
or amalgamation of polyethylene glycol (PEG, in pharmacy called macrogol) polymer chains to
molecules and macrostructures, such as a drug, therapeutic protein or vesicle, which is then
described as PEGylated (pegylated).
PEGylated liposome (PEGLip) technology is a new approach to improving the pharmacodynamic
properties of therapeutic proteins. Instead of encapsulating the drug, PEGylated liposomes are
used as carriers with the protein bound non‐covalently but with high specificity to the outer
surface.
112. Drug of choice for Cold sores? Docosonal (Abreva)

113. What is useful in hepatic encephalopathy? Lactulose

114. Calculate the ratio of solution of benzalkolium chloride with 0.02% strength.
Ans is 1:5000

115. The middle nasal conchae are part of the:


A) sphenoid bone
B) vomer bone
C) ethmoid bone
D) maxillae

116. Osteoarthritis people need intially to be treated by? Acetaminophen

117. Two structures were given. Hint was to check double


bond ‘O’ getting converted to hydroxyl group.
Keto‐Enol tautomers
keto–enol tautomerism refers to a chemical equilibrium between
a keto form (a ketone or an aldehyde) and an enol (an alcohol).
The keto and enol forms are said to be tautomers of each other.
The keto form predominates at equilibrium for most ketones.

118. Which of the following causes hyperuricemia?


hydrochlorthiazide

119. Which is good choice for patient with heart failure and pulmonary edema?
Loop diuretics Furosemide

120. Which of the following drug has shown mortality reduction? Ramipril

121. Cholesterol synthesis structure given with benzene and other side benzene with OH
was there? Aliphatic hydroxylation

122. Grape fruit interaction with simvastatin? level of Simvastatin increases

123. Lutein is a derivative of? Carotenoids


Lutein and its isomer zeaxanthin belong to the group of oxygenated derivatives of carotenoids
called xanthophylls.
124. All of drugs is used in breast cancer EXCEPT?
a) Bicalutamide
b) doxorubicin
c) Docetaxel
d) Gemcitabine
Bicalutamide is used to treat prostate cancer that has spread to other areas of the body. It is used
in combination with hormone treatment. This medication works by blocking the action of
male hormones in the prostate, slowing growth of the tumors. This medication should not be used
in women or children.

125. When hospital pharmacy manager is hiring pharmacy tech what should he look for?
Recognized degree from accredited Canadian college

126. When pharmacy manager is hiring pharmacy assistant in community pharmacy, the
least concern point is? Ethnicity/ Cultural background

127. all of the following cause dry mouth except:


a) Cholestyramine
b) Tiotropium
c) Belladonna
d) Disopyramide

128. When girl child is


having symptoms like itchy
ear, leaking fluid and
already waited for 48 hrs
inoculation period. She
works in school swimming
team. What do you think of
diagnosis?
a) Otitis media
b) Otitis externa

129. When patient is visits your pharmacy with stroke symptoms, with speaking ability
but unable to recall the medicines names of recent medicines and you want to know what
he was on medication from community pharmacy? What you have to do to?
Ans is verbal consent is ok or written consent is required.
130. Which organism is involved in peritonitis infection with dialysis patients?
Ans is Staphylocus aureus
Development of a bacterial infection in the peritoneum, despite the absence of an obvious source for the
infection. It is specifically an infection of the ascitic fluid – an increased volume of peritoneal fluid.
More than 90% of cases of SBP are caused by a monomicrobial infection.
The most common pathogens include gram-negative organisms
(eg, E. coli [40%], K pneumoniae [7%], Pseudomonas species, Proteus species, other gram-negative
species [20%]) and gram-positive organisms (eg, Streptococcus pneumoniae [15%],
other Streptococcus species [15%], and Staphylococcus species [3%]).

131. Warts is caused by? Human Papilloma virus

132. Shigella vaccine true statement?


given after appearing symptoms of single rash on skin then initiate vaccine
Shigella infection (shigellosis) is an intestinal disease caused by a family of bacteria known
as shigella. The main sign of shigella infection is diarrhea, which often is bloody. Shigella can be
passed through direct contact with the bacteria in the stool.

133. In which analytical method, sample is vaporised by exposure to high flame?


Ans is Atomic absorption spectroscopy

134. A new abnormal cell growth in the body is? Neoplasm

135. Which of the following is mostly used for opthalmic ointment base?
Ans is White petrolatum

136. Pharmacist decided to waive the prescription fees for the first 10 prescriptions in the
month without any extra increase in expenses. What is the immediate effect on the income
statement? Ans is decrease in net profit

137. Which of the following drug is taken on empty stomach? Ans is Norfloxacin

138. Person weight is 71.7 kg and infusion rate of IV is 5.5 mg/kg/hr and the steady state
concentration is 17 mcg/ml. What is the total clearance? Css = R / CL
CL = R (5.5 mg/kg/hr x 71.7kg) / Css 17 mcg/ml = 23.2 mg/l = 23.2 mcg/ml

139. What is Mycophenolate mofetil?


Immunosuppressant medication used to prevent rejection following organ transplantation and to
treat Crohn's disease. Specifically, it is used following kidney, heart, and liver transplantation.
It can be given by mouth or by injection into a vein.
140. A registered nurse prescribes ceftriaxone IM injection. As a pharmacist, when you
receive this prescription what will you do?
Ans is Go ahead and look for normal prescription checks and give the prescription
as you do normally Only nurse practitioner can prescribe ..
Ans. Get a prescription from doctor

141. A case was given with confidence interval 0.67-1.04 and P value is 0.65? What do you
interpret? Ans is not significant

142. Which statins need not be given at bedtime? Atorvastatin or Pravastatin


Atorvastatin & Rosuvastatin
Pravastatin should be taken at bedtime
143. What could help in diagnostic lab test of preeclampsia? Proteinuria
Pre‐eclampsia is one of the most serious conditions affecting pregnant women. It is
diagnosed when a woman develops high blood pressure (hypertension), protein in
her urine (proteinuria), and/or swelling of the hands, feet and/or face during
pregnancy.

144. What does pharmacist use for final checking of drug label before dispensing? DIN

145. Which drug is used to prevent GI damage with Naproxen?


a) Omeprazole
b) Misoprostol

146. BisPhosphonate structures was given, what is


the mechanism of action?
Ans is bone resorption inhibitors

147. Parkinson mechanism not found till now?


Ans is dopa precursor loading

148. Nitrate free period used with nitroglycerin patch leads to increase? Ans is CGMP

149. Hospitalisation costs is covered by? Ans is federal tax only

150. Case study is given where data from several studies for a particular outcome is
studied together to get results for a same outcome. Ans is Meta analysis
151. Patient consent is not considered when patient is having lack of? cognition

152. Insurance calls pharmacy store informing that they lost the claim data after
mechanical problem with their computers and there is no evidence of data breach. They
want all the claims data for the last one year back. Ans is Give them back

153. Which nitrogenous base is present in DNA but not in RNA?


a) Thymine
b) Adenin
c) Guanine
d) Cytosine

154. DNA antisense strand is


a) Single strand has ribonucleic acid and complementary to mRNA single strands
b) Single strand has deoxy ribonucleic acid and complimentary to mRNA double strand
c) Single strand has deoxy ribonucleic acid and complimentary to cDNA
d) Single strand has deoxy ribonucleic acid complimentary to mRNA single
strand
e) Double strand has deoxy ribonucleic acid and complimentary to sDNA

155. all can be used for prophylaxis of Travellers diarrhea except?


a) Azithromycin  loperamide also
b) Ciprofloxacin
c) Bismuth subsalicylate
d) Norfloxacin
e) Rifaximin

156. Where does cholesterol synthesis occur in animals?


a) plasma membrane
b) liver
c) kidney
d) mitochondria
About 80% of total daily cholesterol production occurs in the liver and the intestines; other sites
of higher synthesis rates include adrenal glands, and reproductive organs. Synthesis within the
body starts with the mevalonate pathway where two molecules of acetyl CoA condense to form
acetoacetyl‐CoA.

157. What is important feature of innate mechanism? Memory cell production by B cells
158. Neutrophils’ first immune reaction:
a) phagocytosis by activated macrophage
b) presenting antigen
c) engulfing pathogen

159. Spirometry measure all the following except


A) Tidal volume
b) Tidal volume capacity
c) Expiratory volume capacity
d) Inspiratory volume capacity
e) Functional residual capacity

160. Patient with a problem in valve of the heart can be diagnosed by:
a) Echocardiogram
b) Electrocardiogram
c) Blood pressure measure
d) Exercise Electrocardiogram

161. A patient has lot of medications to take and he tell you “I am so frustrated how am I
suppose to take these many medications”. What is an appropriate empathy reply?
a) I know how you feel
b) Many patients take lot of medication and they are fine
c) You seem to overwhelmed by the number of medications you have to take
d) Don’t worry, we will manage this situation in a way to fit your case

162. What is best to do for 2nd treatment if head lice? 7‐10 days after 1st treatment

163. Which disease need whole family treatment as a part of preventive care?
a) Scabies  pinworms also
b) Head lice
c) Impetigo
d) Psoriasis

164. If a drug gets rapidly distributed into various tissues and organs what kind of
modelling do you use?
a) Single compartment model
b) Two compartment model
c) Three compartment model
165. High extraction ratio indicates? increased first pass metabolism

166. CSF, all are true except


a) covers brain and spinal cord
b) passes through vertebrae
c) turbid fluid
d) produced by choroid plexus

167. Which of the following in not involved in knee jerk reflex?


a) Calceneus bone
b) Patella
c) Quadriceps
d) Tendons
Knee‐jerk reflex, also called patellar reflex, sudden kicking movement of the lower leg
in response to a sharp tap on the patellar tendon, which lies just below the kneecap. The sharp tap
on the tendon slightly stretches the quadriceps, the complex of muscles at the front of the upper
leg.
The calcaneus, also called the heel bone, is a large bone that forms the foundation of the rear part
of the foot. The calcaneus connects with the talus and cuboid bones. The connection between the
talus and calcaneus forms the subtalar joint.

168. Saquinavir and St john wart interaction?


St John wart will DECREASE level of Saquinavir by altering drug metabolism

169. Angioedema is main side effect of which drug? ACE inhibitors

170. What is wrongly written in following prescription?


mg/kg/d is wrongly interpreted as D for day or dose

171. Clozapine belongs to which class of drugs?


Dibenzodiazepines
↓D2 ↓5HT2, ↓H1, ↓M, Alpha1‐2. The Only antipsychotic with
proven efficacy in treatment resistant schizophrenia.

172. What is the drug of choice for nocosomial infections


caused by P aeruginosa? Antipseudomonal list
Ciprofloxacin
Antipseudomonal beta‐lactam Imipenem/cilastatin plus ciprofloxacin orAminoglycosides
Piperacillin/Tazobactam
aminoglycoside; Ticracellin
Alternative: aminoglycoside plus ciprofloxacin Meriopenems
Carbepenams
Cefipime
Ceftazedime
173. What is the side effect with ciprofloxacin? Watery or bloody diarrhea
S.E: GI upset, headache, dizziness, photosensitivity, hepatitis. Cartilage toxicity: avoid in children.
Rare: tendonitis, tendon rupture, tendinopathy, peripheral neuropathy and neuropsychiatric
effects; retinal detachment

174. Which of these preparations is used for long term release of drugs? Depo injection

175. In cost effective analysis if you have better additional diagnostic parameter in new
study, what do you call that additional parameter? Ans is Variable

176. Which of the following is function of peroxisome? Oxidation of organic compounds


Peroxisomes are membrane‐bound packets of oxidative enzymes.
In plant cells, peroxisomes play a variety of roles including converting fatty acids to sugar and
assisting chloroplasts in photorespiration.
In animal cells, peroxisomes protect the cell from its own production of toxic hydrogen peroxide.

177. Tapering of 50 mg of Metoprolol was asked.


25 mg bid for two weeks, followed by 12.5 mg bid for two weeks

178. Tapering dose of prednisolone. Start with 50 mg once a day for two days and then
taper by 5 mg every 2 days till finished. Tablets available in 5 mg dose. How many tablets
needs to be dispensed? Ans is 90 tablets 110 tablets

179. Why is it recommended for a pharmacist to introduce himself before counselling?


Ans is to make patient feel relaxed and comfortable

180. Case of patient with past history of angina and hypertension comes to you with chest
pain symptoms. What is correct question to ask except?
Ans is Is your chest pain relieved after taking nitrates?

181. What is function of voluntary pharmacy association in the province?


VOLUNTARY PROFESSIONAL ASSOCIATIONS AND ADVOCACY
• National level: Canadian Pharmacists Association (CPhA)
• Provincial level: Ontario Pharmacists Association (OPA) etc.
• Practice-specific: Canadian Society of Hospital Pharmacist (CSHP)
Roles: Advocating for, and representing the views of pharmacists and pharmacy practice
Negotiating fees and agreements with governments
Providing continuing education and professional development
Publishing journals and providing drug information, for example: CPhA publishes CTC (RxTx), CPS
182. Doctor call your pharmacy for Testosterone cyproate rx for requirements of
prescription? Ans is Requires: verbal written and refills allowed

183. Which of the following drug is known to interact with Warfarin? Cotrimoxazole

184. Drug concentration is 10% in 250 mg tablet. How much diluent needs to be added to
this to make final 2% concentration of drug? Ans is 1000 mg diluent
Q1C1 = Q2C2
10% X 250 = 25 X x = 1250
1250 – 250 (that we have already) = 1000 mg that we need to add

185. HC cream rx was given. You need to make use of 0.75% HC and 1.5 % Glycerin.
Ans is 0.75 and 1.5 g

186. Ventricular repolarization is seen in ECG by? QRS wave

187. Women with Chlamydia trachomatis infection. What is the treatment?


Antibiotics for the Treatment of Uncomplicated Chlamydia Infection
Azithromycin Adults: 1g PO single dose, Children 1m -18 y: 12–15 mg/kg (maximum 1g) PO single dose
Preferred May be useful in situations when poor adherence is expected.
treatment. Repeat dose if vomiting <1 h post administration. May increase digoxin levels.
Can be used in pregnant and breastfeeding women.
Erythromycin Adults: 500 mg QID PO × 7 days If tolerance is a concern: 250 mg QID PO × 14 days
Alternative Children: 1 month–18 y: erythromycin base 40 mg/kg/day PO in divided doses (maximum
treatment. 500 mg QID × 7 days or 250 mg QID × 14 days).
High incidence of GI side effects and QID dosing may discourage adherence.
Can be used in pregnant and breastfeeding women but do not use erythromycin estolate
during pregnancy due to reports of cholestatic hepatitis.
Conjunctivitis: use systemic therapy since topical treatment not adequate.
Reports of infantile hypertrophic pyloric stenosis with erythromycin use in infants <6 wk.
Amoxicillin Pregnant and breastfeeding women: 500 mg TID PO × 7 days
Tetracyclines may decrease effectiveness of amoxicillin.
Main advantage is safety in pregnancy and breastfeeding; not recommended as an
alternative treatment otherwise.
Doxycycline Adults: 100 mg BID PO × 7 days
Preferred Children 9–18 y: 5 mg/kg/day PO in divided doses (maximum 100 mg BID) × 7 days
treatment. S.E: GI upset, rash, candidal vaginitis, photosensitivity.
Decreased levels with antacids, iron, barbiturates, bismuth, carbamazepine, phenytoin.
Decreases effectiveness of penicillins. Similar high cure rates to azithromycin.
Contraindicated in pregnant, breastfeeding women and children ≤8 y.
188. Applied therapeutics… The clinical use of drugs provides what information?
Ans is real clinical case studies and their therapeutic evidences

189. What is used in the treatment of Hepatitis C? Nonstructural protein

Standard therapies for HCV are now all‐oral and combine various HCV NS5B, NS5A
and protease inhibitors.
Interforn is used for chronic HBV.

190. Drug X is given along with Ritonavir. Ritonavir is CYP inhibitor. What is the
interaction? Ans is Drug X level will increase

191. Antibiotics used in high levels at site of pus formation due to which effect?
Ans is Inoculum effect
The inoculum effect (IE) is a laboratory phenomenon that is described as a significant increase in
the minimal inhibitory concentration of an antibiotic when the number of organisms inoculated is
increased. The IE generally occurs with beta‐lactam antibiotics in relation to beta‐lactamase‐
producing bacteria.

192. Erectile dysfunction is caused by?


a) Adrenergic blockers
b) Opioid analgesics
https://www.webmd.com/erectile-dysfunction/guide/drugs-linked-erectile-dysfunction

193. What is true for the vaccinations in Canada?


dTap booster dose is available

194. Which of this will provide additional benefit for short term exacerbation of COPD?
Triotropium

195. What is the doc for emphysema? Ipratropium

196. All are used in treatment of stable Stable angina except?


a) Propranolol
b) Acebutalol
c) Carvedilol
d) Nitrates
e) Clopidogrel
197. What is responsible for chronic kidney failure?
Hemorrhagic Shock

198. Which TCAs of the following is used in Obsessive compulsive disorder?


a) Imipramine  clomiramine especially is used
b) Citalopram
c) Mirtazapine
d) Topiramate

199. What will happen if dose increased from 50mg to 100mg?


Vd increase and clearance per unit time will also increase

200. For IV prep emulsions, which of the following is not right?


w/o is used for both oil soluble and water‐soluble drugs

201. A patient comes with Gabapentin prescription for early refill. He has insurance for
the drugs. What should you do for his early refill?
Ask him why he needs early refills

202. What is true about the website RxTx (online version of CTC)?
Paid website that has subscription‐based services for the health care professionals

203. What does several provinces allow Pharmacy tech to do?


Final check of prescription before dispensing

204. bacterial vaginosis characteristics?


Common Causes of Vaginitis
Cause Common Symptoms Comments
Vulvovaginal Severe pruritus of vulva & vaginal It is not sexually transmitted disease thus partner does
candidiasis area not require treatment.
Stinging/burning The 2nd most common cause of vaginitis.
Thin, white, milky or “Cottage The drug of choice Clotrimazole 1-10%, miconazole,
cheese” discharge nystatin and fluconazole
Atrophy Vaginal discharge, Spotting, Drug of choice is metronidazole
Soreness, burning
Bacterial “Fishy” odour The most common cause of vaginitis. The drug of
vaginosis Creamy discharge (yellow/grey) choice PO/pv Clindamycin or Po/pv metronidazole
Trichomoniasis Frothy, wet discharge It is a sexually transmitted disease thus partner require
Pruritus possible treatment. Drug of choice is metronidazole PO or PV
205. Toxicity of flouride is 8 mg/kg. Body weight of person is 22 lbs. if 0.4 % stannous
fluoride mouthgel is used how much is minimum dose required to cause toxicity (4.1 mg
of stannous fluoride contains 1 mg of fluoride)? Ans is 82 gm
22lb / 2.2 = 10 kg  10 kg x 8 mg = 80 mg (total dose of flouride that can cause
toxicity)
4.1 mg of stannous flouride x 80 mg of flouride = 328 mg = 0.328 gm
To get the total amout of stannous flouride
0.4 in 100 gm = 0.328 in x gm  so, x = 82gm

206. first order reaction was given. Dose was given 160 mg. Blood plasma conc after 2 hrs
and 4hr were 4 mg/L and 2 mg/L. We needed to find volume of distribution. 20mg/L
Vd = initial dose / initial concentration
In first order the drug is decreased by half concentration after each half life;
therefore, if the concentration after 2hr is 4mg/L, the initial conc will be 8mg/L
Vd = 160mg / 8mg/L = 20mg/L

207. Net profit question. Pharmacy manager saves 36000 $ by cutting down expenses.
Last year net profit was 180000 $. How much increase in net percent compared to last
year?
Ans is 20%

208. Trimethoprim sulfomethoxazole treatment needs to be given. Trimethoprim stock


was 40 mg/5ml and sulfomethoxazole was 20 mg powder. We needed to find volume for
required dose. Wt of person was given in lbs. dose needed was given…
Ans is 14.4 ml of trimethoprim stock needed to be used.
JULY 2016
1- Pioglitazone monitoring?
a) hematocrit
b) pedal edema
c) creatinine
Also, liver function (ALT) when indicated

2- A patient is taking losartan and hydrochlorothiazide for her hypertension but it is not
controlled so doctor add lisinopril as well, what should be monitored?
a) K, Mg
b) K, creatinine
c) Na

3- Mechanism of sitagliptin?
a) Thiazolidinedione
b) DPP‐4 inhibitor

4- Diagnostic of diabetes:
a) blood glucose during day < 11
b) Glycosides hemoglobin > 8.5
c) FBS < 5

5- Which one can help the most for hypertension? (There is table in Tc)
a) Reduce Na intake 1 gram
b) Reduce 2 k in 3 months
c) Exercise 30 minutes every day

6- Apatient has severe CHF and pulmonary edema what will be indicated:
a) digoxin
b) furosemide

7- Which one is true about Alzheimer?


a) number of potien pieces plaques in the brain
b) Doing independent functions will be worse in end stage of Alzheimer
Plaques form when protein pieces called beta‐amyloid. it is chemically "sticky"
and gradually builds up into plaques. The most damaging form of beta‐amyloid
may be groups of a few pieces rather than the plaques themselves
8- Which one cause compulsive behavior?
a) Amantadine
b) pramipexole
c) Benztropine
Because of the link between pramipexole and the body’s dopamine system, the drug
appears to have an impact on behavior. Patients who take the drug often report
compulsive shopping, gambling, eating and sexual activity, which can
understandably create many problems for the individual.

9- A patient is taking Varenicline what is the common side effect?


a) Chest Pain
b) Behavioral change – seriuos side effect
c) Nausea and vomitting – common
It is a prescription medication used to treat nicotine addiction. Varenicline is
a nicotinic receptor partial agonist. it stimulates nicotine receptors more weakly
than nicotine itself does. In this respect it is similar to cytisine and different from
the nicotinic antagonist.

10- What is the best recommendation for estradiol patch?


a) Red skin is a common side effect
b) you should change the patch every 3 weeks and wait for 1 week then use another
one
c) put one a patch on a place which in contact to air like arm
d) use some cream beneath of it

11- A patient has osteoarthritis using naproxen and he is using atorvastatin as well
recently she has infection and using clarithromycin. She is suffering from leg pain. Which
one is correct
a) The dose of naproxen is not enough
b) There is a medication interaction
Clarithromycin can cause drug interactions because of its ability to inhibit the
cytochrome P450 (CYP) 3A4 enzyme. Clarithromycin inhibits CYP3A4 activity by an
irreversible mechanism‐based inhibition which occurs when it is metabolized by
CYP3A4 to form reactive a nitrosoalkane via N‐demethylation. This metabolite
intermediate then covalently interacts with CYP3A4 to form metabolite
intermediate complex rendering it inactive until replaced with new enzyme.
12- In the case above what is best action for pharmacist?
a) talk to the physician directly in this regard
b) ask patient what he would prefer to do
(It was confusing)

13- A patient is abusing a medication which was prescribed 6 month ago for his pain.
when pharmacist ask her about it, she said the doctor know and there is no a big deal.
What you do?
a) ask patient to take her prescription to another pharmacy
b) talk to the prescriber directly and talk about your concern

14- In a 2-compartment system, K12 Means:


a) The rate constant of drug transfer from compartment 1 to 2
b) The rate constant of drug transfer from compartment 2 to 1
c) The half life of drug is 12 hrs
d) The elimination rate is 12 mg/h
Reaction Rate Constant from 1 (Substrate) To 2 (Yield)

15- Who control GMP for medication production?


a) Health Canada
b) Q.A
c) Q.C
d) Pharmacy Technical board

16- Which one cannot be found in minor ailments book?


a) Oral Candidiasis
b) Diaper rash
c) Immunization schedule

17- What is the highest expenditure in health care? Hospitalization

18- What is just the excessive cost for health care if patients taking advice from:
a) Family Physician
b) refer to er without particular reason
c) Pharmacist
19- What is ph of 0.02 M of HCL?
HCl is a strong acid so [H+] = 0.02 M pH = ‐ log 0.02 = 1.7

20- A patient should take prednisolone 35 mg for 2 days and reduce 5 mg every 2 days.
You should dispense for how many days?
35+35+30+30+25+25+20+20+15+15+10+10+5+5 = 280 tablet in 14 days

21-what is the final concentration of boric acid (w/v)?


Boric acid ……..X gram (can’t remember)
90%etanol………15
Water…………..30

22-Calculate the VD (L) when C0=0.325 mcg/ml and the dose is ….

23-which statement is true about Drug clearance is:


a) Elimination & metabolism
b) Elimination & absorption
c) Absorption & metabolism
Eliminatin is metabolism and excretion not absorption and excretion

24- What is mixed research in shargeld?


a) we reduce sample size to increase precise
b) we can use qualitative and quantitive researches
Mixed methods research is a methodology for conducting research that involves
collecting, analysing and integrating quantitative (e.g., experiments, surveys)
and qualitative (e.g., focus groups, interviews) research. This approach to research
is used when this integration provides a better understanding of the research
problem than either of each alone.

25- What is correct about P-value?


a) p value estimated how much the experiment results are achieved by chance
b) the smaller the p value the clinical significance is higher

26- The risk for A=0.2%, the risk for B=0.1% what is true?
a) The absolute risk reduction is 5%
b) The relative risk reduction is 1%
c) The total number need to treat is 1000
d) Ratio is 1
Relative Risk (RR) = EER/CER= (a/a+b)/(c/c+d)
Relative Risk Reduction (RRR) = CER-EER/CER
Absolute Risk Reduction (ARR) = CER-EER
Number Needed to Treat (NNT) = 1/ARR

27- A patient taking 40 units daily, every vial is 10 ml, with a concentration of 100 unit /
ml. How many ml this patient should be taken?
1 ml contain 100 units, so 40 units in 0.4ml

28- For most of drug, their elimination is respected to the amount of drug in the body,
there are
a) Zero order
b) First order
c) Two order

29- When a medication has a long half-life for reaching to steady state we use a loading
dose, for calculation of this kinetic we use?
a) concentration of the drug in plasma and the distribution
b) concentration of drug in tissues and distribution

30- Protease is under which enzyme class?


a) hydrolase
b) transferase
c) lyase
A protease (also called a peptidase or proteinase) is any enzyme that
performs proteolysis, that is, begins protein catabolism by hydrolysis of the peptide
bonds that link amino acids together in a polypeptide chain.

31- which is true for simple syrup


a) support microbial growth
b) contain alcohol 10%v/v
c) contain sucrose 85%w/v
d) form turbid solution

32- The role of PEG in ointment base?


a) Humectant
b) Solubilizer
c) Stabilizer
d) Preservative

33- A substance which shifts receptor from active form to deactive form called:
a) Neutral antagonist
Answer is A
b) Inverse agonist
c) True agonist
d) Physiological antagonist
‐Inverse agonist is an agent that binds to the same receptor as an agonist but induces
a pharmacological response opposite to that agonist.
‐A neutral antagonist has no activity in the absence of an agonist or inverse agonist
but can block the activity of either
‐Physiological antagonist describes the behavior of a substance that produces
effects counteracting those of another substance

34- What is true about


glycoprotein?
a) it is a transfer of ATP
b) is cytokine
Glycoproteins are proteins that
contain oligosaccharide chains
(glycans) covalently attached
to polypeptide side‐chains.
The carbohydrate is attached to
the protein in
a cotranslational or post
translational modification. This process is known as glycosylation. Secreted
extracellular proteins are often glycosylated.

35- For calculation of the dose of antibiotic what is important?


a) Weight of patient
b) Site of infection
c) Severity of infection
d) History of allergy

36- Patient allergic to eggs, and wants to take flue shot, what pharmacist can do?
a) Send him to emergency to take Updated .. ans is E
b) Give him divalent and trivalent vaccine
c) Make sensetivity test then give him half the dose of the vaccine
d) Give him intranasal flue shot
e) Give flu shot and tell the patient to stay there for some time too see if there is
any allergic reaction or not

37- Someone who use oxygen therapy which one is better for his nose?
a) Petrolatum gel
b) Vaseline pomade
c) water based (also saline gels)
d) hydrocortisone
‐A common complication of oxygen therapy is dryness in the nose and throat.
‐The problem with using petroleum based products is it can break down due to the
o2 that you are using and any spark can ignite it.

38- Drug shows concentration 16mg\L with a dose of 75mg but 54mg/L when a dose of
150 mg is given...this may be due to
a) saturation of hepatic enzymes
b) saturation of renal clearance
c) saturation of plasma protein binding
d) saturation of intestinal absorption
e) saturation of drug dissolution

39- Structure of toxic metabolite of


acetaminophen was given and asked for the
parent drug

40- This is the structure of methadone which one


of this has a similar structure
a) dextromethorphan
b) meperidin
c) morphine
d) amphetamine
41- What we can give as OTC for cold sore?
a) Doconasol
b) Zinc oxide
c) Pramoxine
d) Acyclovir

42- What is called bleached substance or pre med in dandruff shampoo?


a) Ketoconazole
b) Selenium sulfide
c) Zinc pyrithione
d) Coal tar

43- (Progressive discipline) is the actions that a pharmacy manager takes towards a
pharmacist who commits a dispensing error that would be:
a) warning
b) telling the responsible authorities to take actions.
c) Applying some sort of punishment now then increase punishment
If the pharmacist recommits the error another time.
d) Let a staff do some mistake then do the discipline.
e) Let the staff complete task and then appraise them.
Progressive discipline is a system where the penalties increase upon repeat
occurrences. The typical stages of progressive discipline in a workplace are:
1. Counselling or a verbal warning;
2. A written warning;
3. Suspension or demotion; and Termination.

44- Which one has effect on negative and positive behavior?


a) Haloperidol
b) trifluoperazine
c) Aripiprazole
d) Duloxetine

45- Which one can cause weight gain, tremor and thirst even in normal dose? Lithium

46- Structure of fluoxetine it is an antidepressant of the selective


serotonin reuptake inhibitor (SSRI) class. It is used for the
treatment of major depressive disorder, obsessive–compulsive
disorder (OCD), bulimia nervosa, panic disorder, and premenstrual dysphoric disorder
47- In elderly the risk of UTI is higher because of all except:
a) Inadequate nutrition
b) In complete voiding
c) Use of anticholinergic
d) Bacteria such as Enterococci and Staphylococci

48- The pharmacokinetic of infants are different because:


a) High water content
b) The ratio of fat is higher
c) A higher degree of protein binding
d) Movement of git is slow
Also incompletely developed enzyme systems

49- To achieve the least amount of bias, what study design should be used?
a) Open label
b) Single blind
c) Double blind (randomised)
d) Case-control study
e) Cohort study

50- Which one is not STD?


a) Herpes zoster (shingles)
b) Herpes simplex
c) Chlamydia
d) Gonorrhea
e) Syphilis

51- Which one is related to Treponema pallidum? Canker sores (syphilis)

52- Depolarization of ventricles is related to?


a) QRS wave
b) QT wave
c) ST wave
d) P wave
P wave associated with right and left atrial depolarization.
QRS complex associated with right and left ventricular depolarization.
ST segment reflects the current flow associated with phase 2 of ventricular
repolarization.
PR interval measures the time from the initial depolarization of the atria to the
initial depolarization of the ventricles and reflects a physiological delay in AV
conduction imposed by the AV node.
QT interval is measured the time in which the ventricles depolarize and repolarize
and is a measure of ventricular action potential (AP) duration.

53- When put a stent for a patient how long he should take ASA and clopidogrel?
a) 3months
b) 6months
c) 1 year

54- Knee replacement surgery, which can be used to prevent DVT or stroke?
a) ASA
b) Clopidogrel
c) Fondaparinaux (used in case of HIT)
d) Dalteparin

55- A patient using Ibuprofen for migraine but not working anymore what we should
suggest?
a) Naproxen
b) Acetaminophen codeine and coffee
c) Zolmitriptan
Pharmacologic choices:
1) Simple analgesics or NSAIDs: used for less than 15 days / month.
2) Combination of NSAIDs: used for less than 10 days / month
3) Opioid analgesics: used for less than 10 days / month
4) Ergot (Controlled medication): used for less than 10 days / month.
5) Triptan: Almotriptan, Eletriptan, Frovatriptan, Naratriptan, Rizatriptan,
Sumatriptan

56- Which one is not used in COPD?


a) Salbutamol (SABA)
b) Salmeterol (LABA)
c) Fenoterol (SABA)
d) Fluticasone
Pharmacologic choices:
1‐ Short acting bronchodilators (SABD): SABA & short‐acting anticholinergic
Ipratropium bromide.
2‐ Long‐acting bronchodilators, long‐acting muscarinic
antagonists/anticholinergic (LAMA) and long acting beta2‐agonists (LABA).
3‐ Glycopyrronium bromide is the first rapid‐onset LAMA DPI having a faster
onset of action than Tiotropium.
4‐ The LAMA Aclidinium bromide is dosed twice daily and has effects similar to
those of Tiotropium and Glycopyrronium on lung function
5‐ Indacaterol is a once‐daily rapid‐acting ultra‐long‐acting beta2‐adrenergic
agonist.
6‐ Oral cortisone improves lung condition and decrease hospital stay, used 10‐14
days and tapper at least within 2 weeks

57- Someone has COPD, All are common pathogens except?


a) Haemophilus influenzae
b) Streptococcus pneumoniae
c) Moraxella catarrhalis
d) Pseudomonas aeruginosa
e) Staphylococcus aerius & B Fragilus

58- Mechanism of Gemcitabine?


a) Antimetabolite
b) Platin (Cisplatin interferes with DNA replication)
c) Taxans ( mitotic inhibitors eg:paclitaxel)
d) Antitumor
Gemcitabine is a nucleoside analog in which the hydrogen atoms on the 2' carbon
of deoxycytidine are replaced by fluorineatoms.
As with fluorouracil and other analogues of pyrimidines, the triphosphate analogue of
gemcitabine replaces one of the building blocks of nucleic acids, in this case cytidine,
during DNA replication. The process arrests tumor growth, as only one additional
nucleoside can be attached to the "faulty" nucleoside, resulting in apoptosis.

59- Which of the following statement describe hemorrhagic cystitis?


a) It is caused by excretion of tumor cell breakdown products.
b) It is associated with ifosfamide or cyclophosphamide administration.
c) It is caused by the administration of mesna.
d) It can be prevented or treated with acrolein.
e) It can be treated with granulocyte colonystimulating factor (G-CSF)
Haemorrhagic cystitis is defined by lower urinary tract symptoms that
include dysuria, hematuria, and hemorrhage
Unfortunately, mesna is ineffective as a treatment once hemorrhagic cystitis has
developed
The first step in the treatment of HC should be directed toward clot evacuation.
Bladder outlet obstruction from clots can lead to urosepsis, bladder rupture, and
renal failure. Clot evacuation can be performed by placing a wide‐lumen bladder
catheter at bedside. The bladder can be irrigated with water or sodium chloride
solution. The use of water is preferable because water can help with clot lysis. Care
must be taken to not overdistend the bladder and cause a perforation.

60- Allopurinol causes toxicity with?


a) Azathioprine
b) Alendronate
Allopurinol interferes with the metabolism of azathioprine, increasing plasma
levels of 6‐mercaptopurine which may result in potentially fatal blood dyscrasias

61- What we use for osteoporosis prophylaxy except:


a) Calcitonin
b) Etidronate
c) Zolindronate
d) Alendronate
e) Raloxifen

61- Structure of leukotriene was


given and it was said: it is a
cytokine which has important role
in:
a) Asthma
b) Hypertension

62- Which one of these can be use every year?


a) Pamidronate
b) Zolidronate (zolidronic acid)
The majority of patients receive zoledronic acid 5mg once yearly for three years course

63- All the following drugs decrease the effects of oral contraceptives, Except :
a) Rifampicin (Cyp 3A4 inducer)
b) Carbamazepine (Cyp 3A4 inducer)
c) Tetracycline
d) Phenytoin
e) Ibuprofen

64- The INR has been decrease because warfarin is used with?
a) St. John's wart
b) Chronic alcohol
c) Metronidazole

63- We should advice for eye checkup when patient use which drug?
a. Aspirin
b. Hydroxychloroquine (amiodarone, phenytoin, quinidine)
c. Indomethacin
d. Cyclophosphamide
e. Auranofin

64- Blue-gray pigmentation on skin is because of: Amiodarone


4Ps of Amiodarone: Pulmonary (pneumonitis, respiratory, uscle impairment)
Phototoxicity, Pigmentation (blue skin color)
Peripeherial conversion of T4 to T3 inhibition (Hypothyroidism)

65- Interaction of Amiodarone and atorvastatin can cause: Pain and muscle weakness
Amiodarone may increase the blood levels of atorvastatin. This can increase the
risk of side effects such as liver damage and a rare but serious condition called
rhabdomyolysis that involves the breakdown of skeletal muscle tissue.

66- A patient come and the plasma concentration of phenytoin is 2 times more than
therapeutic concentration what could be sign of it:
a) unsteady gait
b) skin rash
c) nystagmus
The therapeutic range is 10‐20 mcg/mL. Total phenytoin levels (mcg/mL) and
typical corresponding signs and symptoms are as follows:
 Lower than 10 ‐ Rare
 Between 10 and 20 ‐ Occasional mild nystagmus
 Between 20 and 30 ‐ Nystagmus
 Between 30 and 40 ‐ Ataxia, slurred speech, nausea, and vomiting
 Between 40 and 50 ‐ Lethargy and confusion
 Higher than 50 ‐ Coma and seizures
67- The ototoxicity of gentamycin can be worse by: Furosemide
Either increases toxicity of the other by Mechanism: additive drug effects. Increased
risk of hearing damage (ototoxicity) and kidney problems (nephrotoxicity).

68- Once daily dosing of aminoglycosides is effective due to:


a) prolonged residence of the antibiotic in the body.
b) Post‐antimicrobial effect. (Post dose phenomena)
c) enhanced tissue accumulation.
d) Reduced renal clearance.
e) Higher peak-trough differences

69- An aminoglycoside is given for 1000 hr and next dose 2100 Regarding the though level
very very long question and long answer
He took the sample just one hr post injection, you cannot measure the trough level
accurately in this case, he should resample, and actually he should take sample just
before the next dose!!

70- We can find vitreous humor:


a) ear
b) eye
c) nose
The vitreous humor comprises a large portion of the eyeball. It is a clear gel‐like
substance that occupies the space behind the lens and in front of the retina at the
back of the eye.

71- Which one these are interval data?


a) Eye color
b) Gender
c) Height
d) Temperature

72- Which one is surrogate outcome?


a) Mortality
b) Quality of life
c) Amputation
d) Level of glucose HbA1c
Surrogate outcome usually alaboratory measurement
73- Cost / heart attack event what is that?
a) cost effective analysis
b) cost utility analysis

74-which one is correct about filtration membrane:


a) It is for liquid and solid
b) It is for parental
c) Its membrane in 0.22 micron
d) It can eliminate and destroy pyrogens

75- Polycythemia means:


a) Increase in normal RBC
b) Increase in total blood cell
c) Increase in abnormal RBC
It can be due to an increase in the number of red blood cells ("absolute
polycythemia") or to a decrease in the volume of plasma ("relative polycythemia").
Polycythemia is sometimes called erythrocytosis

76- The class Ia Antiarrhythmic agents act on


a) block Na channels
b) block k channels
c) block Ca channels

77- Which medication if used with fluoxetine can cause serotonin syndrome?
a) Dextromethorphan
b) Amphetamine (rare to occur)

78- Which one is not right about furosemide?


a) Hypervolemia is a common side effect
b) It can be used in management of glaucoma

79- In biotechnology what is really important for checking the HOST CELL?
a) Infection of cells with viruses
b) Denatured protein

80- Which one is not once daily?


a) Captopril
b) Losartan
81- A 30 healthy man experience seizure for the first time the reason is:
a) Stroke (risk factor)
b) Brain tumor
c) Pneumonia

82- Which one is not related to warfarin?


a) Polymorphism of CYP2D6
b) Vitamin k in daily intake
c) does not need renal adjustment

83- Which one is not good when giving Counselling?


a) How long does it take to be better
b) Open end and closed end questions
c) Frame time for interaction
d) How to manage of common side effect

84- Which one of these statements is judging comment rather than empathy?
a) You seem overwhelmed by the amount of medication
b) Don’t worry lots of patiensts are like you
c) Don’t worry I understand we can make it easy for you

85- What counseling in regard of clarithromycin is not correct?


a) Talk about potential interaction with statin
b) What we should do in terms of GI side effect
c) Tell about possible photosensitivity

86- which one of these is not used in uncomplicated UTI:


a) Cloxacilin
b) Norfloxacin
c) Cotrimoxazole
d) Trimethoprim

87- Cause of yellow fever: The Flavivirus causes yellow fever, and it’s transmitted when an
infected mosquito bite.

88- A is a dominant heritage. If a woman is homozygote of dominant and the man is


heterozygote of dominant the percentage of being homozygote dominant is:
a) 50% b) 25% c) 75%
89- Gingival hyperplasia in young adult caused by: Phenytoin

90- Hyperthropy means: Hypertrophy


a) Increase in size of normal cells
b) Increase in number of normal cell

91- How we can diagnosis of pancreatitis? Serum amylase and lipase

92- Amino acid which does not have a chiral carbon:


a) Glycine
b) Valine
c) Lysin
The only amino acid that does not have a chiral center is glycine. Its structure is:
H2NCH2COOH. It is attached to acetyl coA

93- The role of mRNA


a) Amplification of genetic
b) Duplication and transfer of genetic
c) Connecting the amino acid together
d) position on protein synthesis in ribosomes

94- The COMT inhibitor is inhibited of synthesis of which product: Eg . Tolcapone , entacapone

There are some structures


COMT transforms dopamine to 3 methoxy derivative deactivating it! So the
inhibitor of COMT will prevent the formation of the inactive structure with –och3

95-for the treatment of depression as first time how long the therapy should continue:
a) 1 year
b) 6 months

96- Infliximab is: TNF alpha blocker


A chimeric monoclonal antibody biologic drug that works against tumor necrosis
factor alpha (TNF‐α) and is used to treat autoimmune diseases
CHIMERIC (25% mouse): XIMAB
97- Trastuzumab, the mab suffix here means: HUMAN (100%) : UMAB
HUMANIZED (10% mouse 90% human):
a) Monoclonal antibody ZUMAB
b) Chimeric antibody MURINE (100%): MOMAB
-Mab (monoclonal antibodies) -
c) Human antibody Cept (soluble receptors)
Mib for proteases
Nib for inhibitors (tyrosine kinase inhibitors)
98- Which one is not correct?
a) Antibodies are protein vertebrate
b) For any antigen there are circulating antibodies in the plasma
c) The longer chain of antibodies determines the kind of it
Incorrect and Illogic Because Why We Get Vaccinated Then

99- What is the mechanism of tacrolimus: interfere with interleukin II


Tacrolimus is a macrolide calcineurin inhibitor. It is an immunosuppressive drug used
mainly after allogeneic organ transplant to lower the risk of organ rejection. It achieves
this by inhibiting the production of interleukin-2, a molecule that promotes the
development and proliferation of T cells

100- A patient with RA is taking methotrexate 7.5 mg OD (as initial treatment) and
Leucovorin once a week which one is true?
a) Methotrexate is not a good treatment
b) Methotrexate should be once weekly
c) Leucovorin should be every day
d) Change leucovorin to folic acid

101- Side effect of leflunomide?


a) Jaundice
b) Renal failure
c) Diarrhea

102- Which one is not a risk factor of hypothyroidism?


a) Male gender
b) T3-T4 normal but TSH is high
c) Treatment of hyperthyroidism
d) Iron deficiency

103- What recommendation of nausea and vomiting is not correct?


a) Not take fluid
b) Take cooled food
c) Not lie immediately after eating

104- Which is not involved in conductivity?


a) Purkinje fiber
b) SA node
c) AV node
d) Bundles Of His
e) Myosin fiber

105- What about diazepam is not correct:


a) It cann’t be used in alcohol withdrawal
b) Can cause falling in elderly
c) Used for insomnia
d) Target Substance Act

106- According to ISMP the Tall man letter should be like:


a) LAMOtrigin LAMOvudin
b) lamoTRIGIN lamoVUDIN From ISMP website ; lamoTRIgine lamiVUDine

c) LAMOTRIGIN LAMOVUDIN
d) lamoTRigin lamoVOdin

107- A board of pharmacy wants to hire a chief executive office they should look for?
a) has university education
b) has a good background as pharmacist
c) Has competence and good at taking responsibility
d) Has extensive experience in similar position
e) Leadership skills and market exposure

108- A pharmacy tech can do?


a) Councelling about OTC
b) Councelling about prescription medication
c) Get order from doctor for refill
d) Receive a new prescription from a regular patient

109- For checking a refill which one is not necessary?


a) Checking the original prescription
b) Strength of medication
c) Checking the name of medication on labels
d) The time of first prescription

110- A patient called and he informed you about a possible mistake in dispensing
medication
a) Refer him to the pharmacy manager
Which of the following is an example of interprofessional collaboration in the
pharmacy environment?
a. A pharmacist counselling on an OTC Vitamin D
b) Refer him to a doctor b. A pharmacist and a physician discussing the result of last nights Toronto
Maple Leafs' game.
c) Acknowledge his anger, ask and verify what is error
d) Apologies & calm down patient c. A registered pharmacy technician contacting another pharmacy about a
prescription Transfer.
d. A pharmacist helping a physician with some drug interaction information.

111- To have interprofessional collaborative what we should do? It was very long
question I couldn’t understand
.. inter is patient oriented ..
a) Give a written paper to patient Intra between Health care
professionals
b) Doing medication review

112- If a pharmacy manager feels that the pharmacy stuff does not care about
multicultural what he cannot do?
a) Put picture and stuff regarding different culture
b) Hire multicultural staff
c) Speak in different language
d) Bring and sell some approval herbal medications which are popular by the
local community

113- When lipid is absorbed and it initially goes to what circulation?


a) splenic
b) lymphatic
Lipids, or fats, are digested and absorbed in the small intestine. Fats first enter
lymphatic capillaries, called lacteals, before entering your blood circulation.
The lacteals represent another unique way fats are absorbed because lipids pass
through the lymphatic system before they make their way back to your
bloodstream.
This differs from protein and carbohydrate digestion because amino acids and
monosaccharides do not enter the lymphatic system but travel directly to blood
capillaries then to portal vein that leads to liver.

114- A disaccharide is formed by a bond between 2 monomers the bond is called:


a) Hydrogenic
b) Glycosidic
A disaccharide (also called a double sugar or biose is the sugar formed when
two monosaccharides (simple sugars) are joined by glycosidic linkage. Like
monosaccharides, disaccharides are soluble in water. Three common examples
are sucrose, lactose, and maltose.
115- Radial nerve damage causes?
a) Inability to bend elbow
b) Inability to bend knee
c) Inability to bend toe
d) Inability move shoulder (Damage of auxiliary nerve)
e) inability to breathe

116- ACL ligament present in:


a) Knee
b) Hip
c) Thigh
d) Shoulder
The anterior cruciate ligament (ACL) is one of a pair of cruciate ligaments in the
human knee.
117) According to the Narcotic Control Regulations of the Controlled Drugs and
Substances Act, which of the following practitioners may prescribe narcotic drugs?
a) Pharmacist
b) Physiotherapist
c) Chiropractor
d) Nurse practitioner
e) Physician assistant

118- Which one is not sense nerve?


a) Sciatic nerve
b) glosso pharyngeal nerve
c) cranial nerves

119- Difference between pain fiber and touch fiber, the touch fiber:
a) High Conduction Velocity
b) Unmyelinated Pain fiber
c) Have a large diameter
d) They go to vertebrate spinal column

120- Mangenase monoamine oxidase enzyme is found in:


a) cellular skeleton
b) mitochondria
Located in the outer mitochondrial membrane
121- Doc of panic disorder:
a) Venlafaxine
b) Amitriptyline

122- Wich one is categorized as SNRI:


a) Mirtazapine
b) Bupropion
c) Duloxetine

123- Paroxetine has indication in all except:


a) GAD
b) Depression
c) OCD
d) Bipolar

124- Combination of low dose ASA and ibuprofen every day decrease benefit:
a) Ibuprofen decrease antiplatelet activity of aspirin
A & C but C more because A can be managed
b) Aspirin decrease anti-inflammatory of ibuprofen by dose spacing
c) Increase risk of duodenal ulcer

125- Usually when we change from one opioid to another one, we start second opioid in
50% of equianalgesic dose? Because:
a) prevent withdrawal symptoms of first opioid
b) check the cross-sensitivity (or sth like this)
c) prevent tolerance
d) to have an optimum analgesic effect

126- The respiratory depression and withdrawal symptoms in for what receptor?
a) MU
b) MU and kappa
c) Delta

127- Which one has the muscle relaxant property?


a) Diazepam
b) Gabapentin

128- Natural product for migraine use for prophylaxis of migraine:


a) Butterbur Not used anymore due to side effects
b) Feverfew
c) Evening primrose
The substances petasin and isopetasin are found in Butterbur. They work to reduce
spasms and reduce inflammation. These actions are thought to help prevent
migraines and reduce migraine headaches

129- A folic acid is converted by dihydrofolate reducates to tetrahydrofolic acid and


methyltetrahydrofolate. The cofactor?
a) Vitamin B12
b) Vitamin C
c) Vitamin A

130- The long term use of PPIs can lead to deficiency of?
a) Folate
b) Thiamin
c) B12

131- Vegeterian people normally have deficiency of which?


a) B12
b) Vitamin D

132- What is correct for monitoring of statins?


a) LFT
b) CK every 3 months
c) serum creatinine

133- Mechanism of selegline & resegline: inhibitor of MAO‐B

134- Which enzyme is responsible to attach phosphate to protein?


Kinase Facilitates transfer of PO4 from ATP to the substrate (Transferase)

135- What is true about tertiary reference? The disadvantage is:


a) a range lag time to be published
b) Not have references
c) Not being reviewed

136- What we should do for preventing cataract?


a) Multivitamin
D first then A
b) Use corticosteroid (hydrocortisone)
c) Artificial tears
d) Use of photo protective eye were

137- Which one lengthens the eyelashes and thicken them: Bimatoprost
It is a prostaglandin analog used topically (as eye drops) to control the progression
of glaucoma and in the management of ocular hypertension. It reduces intraocular
pressure (IOP) by increasing the outflow of aqueous fluid from the eyes

138- Sign of allergic conjunctivitis


a) lacrimation
b) pain in the eye
c) vision blurrediness
d) Both Eyes Are Involved

139- Which statement in true about diabetic patient:


a) Patient should check repeatedly (more than the other days) duration of illness
and stress
b) For using the home device, finger is not a reliable site for sampling
c) If someone has diabetic type 1 and his day blood glucose is >10 should….
d) If diabetic patients see ketones in his urine …….

140- Someone using premixing insulin lispro and regular (70%-30%) and use 32 unit in
the morning and ………unit before supper. Right now hospital pharmacy does not have
premixing insulin, what we can use instead for one month? Mix is not interchangeable

141- Which one is short acting insulin?


a) Lispro also aspart
b) Glargin
c) Nph
d) Regular

142- Which is correct for fixed insulin ratio


a) minimum nocturnal hypoglycaemia
b) minimum hypoglycaemia if missed meal
c) provide approximate smooth peak
d) need to regulate schedule for snacks and meals
e) allow flexibility for dose changing in case of exercise
143- What is correct regarding clopidogrel?
a) Cause neutropenia and need blood monitoring
b) Good option for preventing atherosclerosis

144- Which one decrease heart oxygen demand without vasodilation


a) Atenolol
b) Nifedipine
c) minoxidil
d) hydralazine

145-which one is a risk factor for thromboembolism?


a) Dyslipidemia
b) Malignancy

146- A patient goes to the patent and in ECG the P wave is absent and patients feel
butterfly heart beat
a) atrial fibrillation
b) atrial flutter
c) ventricular fibrillation

147- Which one increases the rate of survival in CHF patients?


a) Digoxin
b) ACE inhibitor

148- Which one is safe is using in chronic kidney disease:


a) Heparin
b) Dalteparin

149- a patient come to the pharmacy who is taking warfarin 5 mg and the INR is always
normal but today is 4 (I don’t know today or a week) what is an appropriate advice:
a) Skip the dose for 2 days and continue 5 mg
b) Stop taking warfarin for 1 week and continue for 2,5 mg
c) Call physician to get prescription for vitamin K
d) Go to the emergency room

150- What will happen in phase 3 of repolarization?


a) Decrease permeability to sodium
b) Increase permeability to potassium – potassium out!
c) Decrease permeability to potassium
d) Increase permeability to calcium

151- Which statement is correct for digoxin?


a) increase contractility
b) it reduces total sodium of body
Digoxin Effect: +Ve Inotropic Effect,‐Ve Chronotropic

152- Someone has asthma and she used to have salbutamol 2 times in a month but now it
become Bid every day what is the next step:
a) Fluticasone
b) Salmetrol
c) Combination of Salmeterol and ICS

153- Which one is not an Asthma triggers in children


a) Infection (not sure it was infection or flue)
b) Cold weather
c) Fever
d) Environmental allergen

154- Sounds coming for lung during disease called:


a) wheezing
b) Crackle
c) whistling

155- What is the side of Montelukast?


a) Headache
b) palpitation
c) Mouth thrush
d) Mouth dryness

156- Fecal occult blood test is used for what?


a) Chron’s disease
b) Colorectal Cancer

157- What is the difference between Urinary Tract Infection & Uncomplicated UTI
a) Pain sensation/abdominal pain during urination
b) Frequency of urination
c) Urgency of urination
d) Turbidity of urine Fever & Nausea & Vomiting

e) Fever

158- We use a Finasteride for a patient when we should assess a patient for improvement?
6 months

159- a 33 woman who smoke & usually forget stuff OC every day to take, what do you
recommend?
a) Depoprovera
b) Novo ring (vaginal ring)
c) Oral combination
d) Norethindrone

160- Patients had a breast cancer 8 years ago but now she is experiencing on going hot
flushes in day and night what you recommend
a) Venlafaxine
b) oral combination
c) dermal combination
d) benzodiazepine
e) vaginal estrogen cream

161- Testosterone structure was given metabolized by 5 ‐ Alpha reductase

162- Nonprescription for impetigo?


a) Bacitracin
b) Doconasol
c) Polysporin

163- Impetigo caused by:


a) S. Aureus
b) Fungal
c) Viral
Impetigo is a bacterial infection that involves the superficial skin. The most
common presentation is yellowish crust on the face, arms, or legs. Less commonly
there may be large blisters which affect the groin or armpitsThe lesions may be
painful or itchy. Fever is uncommon.It is typically due to either Staphylococcus
aureus or Streptococcus pyogenes
164- Which one is a trigger for Cold sore?
a) UV light Ans. Is A
b) Acidic food Food has nothing to do with it

c) Spicy food

165- In end stage CKD we use Calcium to what purpose? Decrease blood phosphate

166- What is the duration therapy of osteomyelitis?


a) at least 4 weeks
b) 6 months Ans. Is A
c) 3 months
d) 7 to 14 days
Osteomyelitis (OM) is infection and inflammation of the bone and bone marrow. It
can be usefully subclassified on the basis of the causative organism (pyogenic
bacteria or mycobacteria)

167- Which element is a cofactor of Angiotensin converting enzyme?


a) Iron
b) zinc
c) selenium
d) Cu

168- Which one of these is an antigen presenting cell?


a) Plasma cell
b) Dendrocyte
c) T- cell
Antigen‐presenting cells, including macrophages, B cells and dendritic cells

169- Emphysema means:


a) the cell wall of alveoli is destroyed
b) inflammation of Brunches
c) increase in mucus production of Brunches

170- The duration therapy with nitrofurantoin in uncomplicated UTI:


a) 1 day
b) Minimum 3 days 5 days
c) 10 days
d) 3-5 days
171- In a history profile of patients he has not any allergy to penicillin but has GI problem
with macrolide suffers from Otitis media. The patient already has wait 3 days for watch. As
a pharmacist what will you advice?
a) Acetaminophen PRN
b) A course of Azithromycin
c) A course of high amoxicillin
d) A course of cefuroxime

172- A child with meningitis can use all except:


a) Ceftriaxone
b) Cefotaxime
c) Ampicillin
d) Vancomycin
e) Ciprofloxacin

173- Someone with pneumonia with Influenza which one is not useful?
a) Azithromycin
b) Clarithromycin
c) Levofloxacin
d) Co-amoxiclave
e) Cephalexin

174- What we should use for traveller diarrhea


a) Hepatitis A
b) Hepatitis B
c) Cholera Vaccine

175- The recommendation of malaria prophylaxis by chloroquine 5mg/kg/week ot a


maximum of 250mg / week .The patient is going for 6 weeks and needs to start 1 week,
during the trip and 4 weeks after the trip if this patient weight is 100 kg,
How many tablets of 250mg should dispense for this patient
a. 22
b. 28
c. 34
d. 42
Patient will be using the drug for 11 weeks. Each week he will consume
5x100=500mg, total of 5500. 5500/250=22 tablet.
176- Which one not causes eye problem?
a) Hydroxychloroquine
b) Rifamycin
c) Ethambutol
d) Tobramycin
e) Amiodarone

177- The pregnant woman shouldn’t be in touch with cat poop because of:
‐Toxoplasmosis

178- Which one is correct in regard of chemotherapy induced nausea and vomiting?
a) the anticipatory nausea and vomiting cannot be managed usually
b) by appropriate acute therapy we can prevent the delayed N &V
c) metoclopramide and Dexamethasone can be used for preventing delayed N&V
d) Prochlorperazine can be used for monotherapy

179- Phenol will be metabolized by?


a) glucuronidation
b) Glutathione
c) Sulfonation
Phenols undergo sulfonation, glucuronidation, aromatic hydroxylation and O‐
methylation

180- The sturcture of fatty acid was given:


a) mono unsaturated
b) poly unsaturated
c) Saturated
Polyunsaturated fatty acids (PUFAs) are fatty acids that contain more than
one double bond in their backbone. This class includes many important
compounds, such as essential fatty acids Omega‐3, Omega‐6, Omega‐9
181- Which statement is correct regarding phosphorelation of proteins?
a) Post translation
b) Post transcription
c) Assign by DNA gen

182- What is the oligonucleotide probe?


A short sequence of nucleotides that are synthesized to match a specific region of
DNA or RNA then used as a molecular probe to detect the specific DNA or RNA
sequence. They are used in order to determine the presence of microbial species,
genera, or microorganisms.

183- What is correct about RT-PCR in comparison to PCR?


a) Done on DNA which obtained from RNA (cDNA)
b) production of RNA
c) Has higher temperature
d) Has lower temperature

184- What part of brain control respiratory system:


a) Cerebellum,
b) Medulla oblongata
c) Thalamus

185- Respiration rate per min:


a) 10/min
b) 25/min
c) 45/min
d) 15/min
Average resting respiratory rates by age are:
 birth to 6 weeks: 30–40 breaths per minute
 6 months: 25–40 breaths per minute
 3 years: 20–30 breaths per minute
 6 years: 18–25 breaths per minute
 10 years: 17–23 breaths per minute
 Adults: 12‐18‐breaths per minute[7]
 Elderly ≥ 65 years old: 12‐28 breaths per minute
 Elderly ≥ 80 years old: 10‐30 breaths per minute
186- What is true about the Sciatic nerve?
Whole leg
a) Innervate the Whole Foot
B) Innervate the Hips
C) Innervate the Buttocks Large Muscle
D) Innervate the Quadriceps

187- Which one cause bronco-constriction


a) Acetylcholine Epinephrine causes vasoconstriction & bronchodilitation
b) Prostaglandin
c) Epinephrine

188- What we use in alzhimer as choline esterase inhibitor


a) Neostigmine
b) Edrophonium
c) Galanthamine

189- What is best counseling for levothyroxine?


a) if you have palpitation it could be a sign of toxicity
b) take it in the morning to prevent insomnia
c) how manage the photosensitivity of it

190- Filgrastim is used for?


a) prevent systemic infection
b) prevent for anemia Bone marrow depression
c) Prevention of cancer Neutropenia

Filgrastim is a granulocyte colony-stimulating factor (G-CSF) analog used to stimulate the


proliferation and differentiation of granulocytes. it is a pharmaceutical analog of naturally
occurring G-CSF. It is produced by recombinant DNA technology. Filgrastim is used to
treat neutropenia, stimulating the bone marrow to increase production of neutrophils

191- Patient with gout should not take?


a) Thiazide
b) Ramipril

192- The transfer enzyme attaches


acetyl to CoA by make?
-Thioester bond
193- Which on can be transfer just once?
a) Temazepam
b) Narcotic
c) methyl phenidate (No transfer)

194- We should keep the record of narcotic for how long?


a) 2 years
b) 6 month
c) 1 year

195- We should stop taking metformin before doing radiography test?


a) Can cause severe renal damage
b) Can reduce the clearance of radio…
c) If we use together it can potentiate the hypoglycemia

196- Which gland will be destroyed after puberty?


a) Adrenal
b) Thymus
c) Pituitary

197- Which one does not empty their stuff in GI track?


a) Pancreas
b) Adrenal
c) Parotid gland

198- According to ISMP which one has dangerous abbreviation and also high alert
medication?
a) Warfarin 5 mg po daily
b) Methotrexate 7.5 mg po daily
c) Insulin 20 IU
https://www.ismp-canada.org/download/hnews/HNews0308.pdf

199-which one was humectant, antibacterial, and solvent (not sure for third one)
a) Glycerin
b) Ethyl alcohol
c) Urea
200- Which on is not correct about dry powder inhaler?
a) Should shake it vigorously
b) Lactose can be used as a filler

201-what is the role of PEG (10%) in diazepam injection?


a) humectant
b) solubilizer
c) stabilizer
Conc of polyvinyl more than 40%: solubilizer

202- The role of albumin in insulin preparation?


a) buffer
b) prevent adsorption
In vitro pharmacodynamic profiles for Albulin characterized by receptor binding,
inhibition of gluconeogenesis, induction of glucose uptake, and global regulation of
gene expression in relevant cell types showed that Albulin produced similar activity
profiles compared with that of recombinant human insulin

203-which one is not a preservative?


a) Thimerosal
b) ascorbic acid
c) ethyl alcohol
d) CMC

204- Patient receives food via umbilical tube or nasogastric (NG) tube for provide
nutrition under which treatment?
a) palliative
b) sedated
c) infants
Palliative care is specialized medical care for people with serious illness

205- conduct a study between 2 diabetic patients and compare blood sugar and the p
value was 0.08 .which one is correct?
a) Reject the null hypothesis and there is a significant different
b) Fail to reject the null hypothesis and the difference is significant
c) Accepted the null hypothesis and there is no significant difference
206- Which one increase water absorbtion?
a) Thiazide
b) Loop
c) Spironolactone
d) Desmopressin

207- What excipient in tablet can cause diarrhea and flatulence?


a) Mg
b) Lactose

208- In an experiment want to know the number of newly prescribed Oxycodone and refill
prescription was undergo under 4 physicians what is the name of the used test?
a) Chi-Square
b) Wilcoxon
c) Man Whitney
Wilcoxon signed‐rank test is a non‐parametric statistical hypothesis test used when
comparing two related samples, matched samples, or repeated measurements on a
single sample to assess whether their population mean ranks differ (i.e. it is a
paired difference test). It can be used as an alternative to the paired Student's t‐test,
t‐test for matched pairs, or the t‐test for dependent samples when the population
cannot be assumed to be normally distributed.

209- What is the meaning of profitability?


a) Get revenue to pay all the liability and reward the owner.
b) ust get money to pay liability

210- To increase effective work of employee:


a) Reward them
b) increase number of making error
c) strict rule

211- Capping and lamination caused by?


- Use of too pressure in manufacture

212- To avoid photodegradation what we should do?


a) Add binder
b) Micro granulation
c) Film tablet
213- In regard of dry heat sterilization which one is true?
a) can be done on plastic IV bag
b) the air will be used again without filtrations
c) it will be done for 30 mines by 100C
d) it can eliminate pyrogens
e) it is called depyrogenation method

214- In which chromatography solute can be separated from soln?


a) HPLC
b) Thin layer
c) GC
d) Partition chromatography
e) Gel separation.
The Only Chromatography in Which the Stationary Phase Is Not Interacting With the
Substance to Be Separated Is Gel Chromatography as Separation Takes Place by Size
Only

215- The role of public health agency of Canada?


a) Improve public health and well being
b) Buy vaccines for Canada
c) Give influenza vaccine
The role of the Public Health Agency of Canada is to:
 Promote health
 Prevent and control chronic diseases and injuries
 Prevent and control infectious diseases
 Prepare for and respond to public health emergencies
 Serve as a central point for sharing Canada's expertise with the rest of the world
 Apply international research and development to Canada's public health
programs
 Strengthen intergovernmental collaboration on public health and facilitate
national approaches to public health policy and planning.

216- What is true about province and territories?


a) every province has the same formulatory
b) They negotiate and come to agreement to have the same cost for generic price
c) All have vaccination plan
217- What is right about provincial authorities?
a) Set standard and enforces pharmacist and tech
b) create a fair scope of practice for pharmacists and tech

218- What we can find in CPS?


a) Latin abbreviation which is used in prescription
b) Guideline For Treatment Of Diseases

219- Pharmacy started independently its business, furniture is on lease, do central


purchase & doing mass activity of marketing & merchandising what is type of pharmacy:
a) Independent
b) Banner
c) Franchise
d) Chain

220- you have the following data, what is the gross margin:
Cost of goods = 480.000 $ Sales = 600.000$
Expenses = 400.000$ Net profit = 240.000$
a- 6%
b- 10%
c‐ 20%
d- 25%
e- 50%
600000‐480000/600000*100
o Sales = Units price x Unit volume
o Cost of Goods Sold = Unit volume x Unit cost
o Gross Margin = Sales – Cost of Goods Sold
o Profit = Gross Margin – Expenses
o % Gross Margin = [(Sales – Cost of Goods Sold)/ Sales] x 100%

221- Which one is correct for task assignment?


1-taking the prescription and check the demographics
2‐put the information to the computer
3- Check the appropriateness of therapeutic medications
4-fill the prescrption
5-check the prescription
6-councell
-pharmacy assistant 1, 2 / tech 3, 4/ pharmacis 5, 6
222- A pharmacist and a doctor are talking one the phone regarding a patient non
adherence. Which is not discussed during their conversation?
a) Information About Patient Cycle Of Care
b) Patients with similar pattern of behavior

223- Who initially set the patent drug price?


a) Manufacture
b) Wholesaler
c) Provincial
d) Price review board of parliament
Patented Medicine Prices Review Boar wasn’t in options!

224- SWOT, S means? Strength


SWOT stands for: Strength, weakness, opportunity, threat

225- Key symptom of nephrosis:


a) glucose in urine
b) protein in urine
c) ketones in urine
d) lipids in urine
e) Minerals in urine

226- Whenever we are doing medication review we should sometime pause and be silent
to do what?
a) To notice nonverbal clue of patients
b) To provoke patient to talk
c) To give patient time to understand what we said

227- What herbal remedy should be avoided in hypertensive patient?


a) Liqouorice
b) Roselle

228- In Microcytic Iron deficiency, you will see?


A. Increased MCV
B. Increase in serum ferittin
C. Increase in iron binding capacity
D. Decrease in serum folate
229- Which one is not true about charcoal?
a) The drug should be absorbed to charcoal
b) If a medication has enterohepatic cycle we should use multiple dose of
charcoal
c) It excreted in stool
d) The risk of toxicity of charcoal is a withdraw of using of it

230- The sig for a prescription is like this: 1 tab qid hs


How many tablet we should dispense for 3 month?
5 times daily * 90 day = 450 tablets

231- Look at the chart and select best option


Dose concentration
100 50
200 100
400 350
800 1200
a) Saturated clearance
b) Dose-dependent first pass metabolism
c) Dose dependent excretion

232- If we draw the first order concentration versus time on a semi logarithm paper the
diagram will be?
a) Exponential
b) Linear (straight)
233- This structure is a prodrug of what?
It is for codien prodrug of morphine

234- Someone is taking prednisolone what should be


checked?
a) Serum creatinine
b) Potassium and glucose
Increase renal exchange of K versus Na in the collecting tubule, causes
hyperglycemia

235- Which one increases the dose of digoxin?


a) Amiodaron
b) metoclopramide
Metoclopramide decrese digoxin absorbtion

236- What is the main cause of hands warts? Papilloma virus 2

237- Question regarding L-arginine (= nitric oxide)


L‐arginine is an amino acid." It is obtained from the diet and is necessary for the
body to make proteins.
‐Used in Chest pain (angina), erectile dysfunction (ED). High blood pressure.
Inflammation of the digestive tract in premature infants Nitrate tolerance Leg pain
associated with poor blood flow (peripheral arterial disease) Improving recovery
after surgery High blood pressure during pregnancy (pre‐eclampsia)
‐Side effects Allergies or asthma, Cirrhosis, Herpes, Low blood pressure,
Guanidinoacetatemethyltransferase deficiency, Recent heart attack, Kidney disease:

238- Out of 100 people who had a disease, 10 had been exposed to a risk factor, Of 100
matched controls, 2 had been exposed to a risk factor. Which of the following is true?
A: The relative risk is 5.
B: The odd ratio is 5.
C: The relative risk is 0.2.
D: The odd ratio is 0.2.
Odd ratio (10/100) /(2/100)=5

239- Which organization is responsible for Drug recall: Med effect

240- Ototoxcity caused by: Amikacin


241- Which is false concerning the “hydrate” of a drug?
a) Same core composition as the anhydrous
b) similar solubility as the anhydrous
c) may become anhyrous in dry atmosphere
d) hygroscopic materials tend to be hydrates
e) trap water molecules into its structure

242- Thalidomide is been proven to be effective in


a) cancer
b) glaucoma
c) multiple myeloma
Multiple myeloma, also known as plasma cell myeloma, is a cancer of plasma cells, a type
of white blood cell normally responsible for producing antibodies. Initially, often no
symptoms are noticed. When advanced, bone pain, bleeding, frequent infections,
and anemia may occur. Complications may include amyloidosis.
The precise mechanism of action for thalidomide is unknown, but possible mechanisms
include anti-angiogenic and oxidative stress-inducing effects. It also inhibits TNF-α, IL-
6, IL-10 and IL-12 production, modulates the production of IFN-γ and enhances the
production of IL-2, IL-4 and IL-5 by immune cells. It increases lymphocyte count,
costimulates T cells and modulates natural killer cellcytotoxicity. It also inhibits NF-
κB and COX-2 activity

243- The most common organisms associated with community-acquired pneumonia in


adults are
A. Chlamydia pneumoniae, Mycoplasma pneumoniae, and H. influenzae.
B. Streptococcus pneumoniae, H. influenzae, and Chlamydia pneumoniae.
C. Mycoplasma pneumoniae, Chlamydia pneumoniae, and S.pneumoniae.
D. Mycoplasma pneumoniae, Streptococcus pneumoniae, and H.influenzae.
E. Chlamydia pneumoniae, Staphylococcus aureus, and H. influenzae

244- Pharmacist want to increase activity and promotion:


a) discuss with prescriber about patient drug safety n issues
b) information leaflet in different languages

245- What is incorrect about acne topical preparations?


a) benzoyl peroxide is photosensitive preferably apply before bedtime
b) gel preparation for acne treatment is preferable than cream
c) isotretinoin is topical preparation is used if antibiotics are not effective
d) recommend oil free makeup products
e) isotretinoin can increase triglycerides, cholesterol and liver enzyme

246- What is true about the blot:


a) Northern blot detects the DNA
b) Southern blot detects the RNA
c) Eastern blot detects the protein
d) Western blot detects the protein

247- Drug of choice for the treatment of conjunctivitis in the newborn (ophthalmia
neonatorum) is:
a. Erythromycin, Ceftriaxone
b. Tetracycline or Doxycycline
c. Erythromycin or Azithromycin
d. No antibiotics
Tips: Conjunctivitis can be caused by all EXCEPT:
a) gonorrhea
b) chlamydia
c) S aureus
d) B fragilis
e) S pneumonia

248- Asymptomatic bacteriuria requires treatment in which of the following groups of


patients?
a) pregnant women
b) diabetic patients
c) over 65 years old
d) patients with turbid urine
e) children

249- Vit D decifencey causes: Osteomalacia


Osteomalacia is the softening of the bones caused by impaired bone metabolism primarily
due to inadequate levels of available phosphate, calcium, and vitamin D, or because
of resorption of calcium. The impairment of bone metabolism causes inadequate bone
remineralization.
250- Superoxide dismutase: found in
a) Mitochondria
b) Vacuoles
c) Cytoplasm

251- Suger in protein: Ribose

252- Which of the following are inappropriate to do to prevent hemorrhoid?


a) Avoid sitting for a long time on chair
b) Avoid sitting on toilet for long time
c) Avoid very acidic food

253- When a vagal stimulation occurs, which of these substances will be released?
a) Dopamine
b) Serotonin
c) Insulin
d) Vasopressin
e) Acetylcholine (also E & NE)

254- Drug Combination used To treat MRSA:


a) Vancomycin + Folic acid
b) Erythromycin + Doxycyclin
c) Metronidazole+ Ampicillin
d) Vancomycin + Metronidazole

255- Bacterial Vaginosis: Pruritus & malodor (no option of fishy order)

256- End Product of Protein metabolism: Urea


end product of purine metabolism: Uric acid

257- Angle of repose measures:


a- Solubility
b- Dissolution
c‐ Flowability
d- Stability
e- PH
258- Chemoreceptor is a sensory organ related to
a) Vision
b) Knee Jerk
c) Balance
d) Hearing
e) Smell

259- Right counselling for levothyroxin


a. palpitation indicates overdose
b. insomnia
c. wt gain
d. take in the morning

260- A 54 years patient was recently diagnosed with BPH, he is having the following
medications, captopril, sildenafil for his erectile dysfunction and hydrochlorothiazide, he
has no allergy and has postural hypotension but not high, what drug you should
recommend for this patient.
a) Tamsulosin
b) Finasteride
c) Dutasteride
d) Doxazosin
e) Prazosin
Tamsu. Is selective alpha 1 blocker > less risk of hypo

261- Eye drops what is wrong: gel has less duration in eye than drops

262- While counseling patient what will not discuss: Drug Interaction

263- While counseling anticancer drug council all expect:


avoid taking lots of fluid

264- Which of the following is least in decreasing error in prescription:


a. Appearance of the drug
b. Date of prescription
c. Dose of the drug

265- All causes alkalosis expect: increases ingestion of ASA


266- Tadalafil is different from Sildenafil because:
a) sildenafil is longer acting
b) Tadalafil is shorter acting
c) Tadalafil can be used safely with nitroglycerin
d) Tadalafil and Sildenafil are both equally active and have the same duration
e) Tadalafil is much longer acting than Sildenafil
sildenafil best work on empty stomach whereas tadalafil works best regardless of
meal. So if u takes tadalafil after heavy meal there is no impact on efficacy and onset
of action. Both have similar side effect profile

267- Pharmacist role in accounting: to control capital etc

268- Which of the following is the least preferable combination antihypertensive in a


patient whose K+ levels are 5.5 mEq/L?
a. ramipril/ hydrochlorothiazide
b. valsartan/amlodipine
c. perindropril/spironolactone
d. losartan/hydrochlorothiazide
e. amlodipine/atorvastatin

269- A patient is doing abuse of medication what will you do: call to do & discuss,
a) Ask the patient
b) Patient is illiterate & cant read how you will council him:
c) tell him written detail, provide verbal information,
d) tell his caregiver
e) Discuss every detail of the consent!

270- Css is equal to how many half lives: 5 half lives

271- 2 tabs tid ac & hs total number of tablets for a month: 240

272- Role of Cholecystokinin – Gastric Juice Secretion


it is a peptide hormone of the gastrointestinal system responsible for stimulating
the digestion of fat and protein. Cholecystokinin, previously called pancreozymin, is
synthesized and secreted by enteroendocrine cells in the duodenum, the first segment of
the small intestine. Its presence causes the release of digestive enzymes and bilefrom
the pancreas and gallbladder, respectively, and also acts as a hunger suppressant
273- A 50 year old women in menopause is planning to take Estraderm patch or estalis
patch which of these would you most commonly council on
a) Estrogen patch will cause redness and irritation on site of application
b) Make sure to apply to non hairy open area on the arms
c) Must take for 3 weeks and then remove for one week and keep cycling it regularly

274- What is use in tablet as filler?


a) Dicalcium phosphate
b) Lactose
c) Mg stearate
d) Mineral oil

275- Emulsion/suspension problem:


a) Frosting
b) Coalescence
c) Sedimentation
There are four types of instability in emulsions: flocculation, creaming, coalescence,
and Ostwald ripening

276- Nystagmus is ccc by all EXCEPT:


a) Oscillation
b) Repetitive
c) Involuntary
d) Eye purges(exopthalmic)

277- Secondary Hypogonadism is due to the deficiency of:


a. Anterior Pituitary gland Primary > testes
b. Hypothalamus Ans. Is A Secondary > anterior pituitary
Terierty > hypothalamus
c. Thymus gland Anterior pituitary can be considered primary if no testes in the
d. Adrenal Cortex choices

e. Thyroid gland
goes for hypothalamus as it controls pituitary gland

278- Thixotropic material exhibits:


a. Dilatant flow
b. Plastic flow
c. Pseudo‐plastic flow
d. Sol-gel-sol system
Thixotropy is going from Gel to Solution... this is for both plastic and pseudo.
While antithixotropy is going from solution to gel (increase in viscosity) and this for
dilatant

279- A pharmacist stops counseling a patient who repeatedly takes the medication
incorrectly. Which ethical principle does pharmacist violate?
a) Justice
b) beneficence
c) nonmaleficence
d) autonomy
e) veracity

280- Which diagnostic test would be most helpful with the diagnosis of a pulmonary
embolism?
a. chest x-ray (for pneumonia)
b. electrocardiogram
c. spiral CT of the chest
d. bronchoscopy
e. echocardiogram
A spiral CT of the chest or a ventilation‐perfusion scan would be necessary to
confirm the diagnosis of a PE. Chest x‐ray, electrocardiogram, echocardiogram, or
bronchoscopy would not assist with the diagnosis.

281- Alcohol metabolism (structure given and ask


for enzyme)
Alcohol dehydrogenase
EXTRAS:
1- A deceased patient's wife recently brought the patient's unused narcotic medications to
my pharmacy. Do I have to obtain authorization from Health Canada before I can destroy
these narcotics? No more authorization for both.. record and witness only

No. When destroying previously dispensed narcotic drugs, you do not need to
obtain approval from Health Canada. Authorization is only necessary to destroy
expired narcotic drugs that have not been dispensed, i.e., those that are part of your
inventory. Although you do not require approval to destroy returned medications,
you should still record the amount of narcotic to be destroyed, ensure the drugs are
rendered unusable, and have the destruction witnessed by a second person.

2- Can a pharmacist accept a faxed prescription for a narcotic or controlled drug to a


pharmacy?
All prescriptions (with the exception of prescriptions for drugs monitored by the
Controlled Prescription Program (CPP) formerly called triplicate/duplicate
prescriptions) may be faxed to a pharmacy. This includes written and verbal
prescriptions written for narcotics and/or controlled drugs. For detailed
information, see Bylaw 5‐38(2) Facsimile Transmission of Prescriptions.

3-I have a patient with cancer who wants to receive medical marijuana. What does the
patient need to do to?
The patient must apply to Health Canada's Office of Cannabis Medical Access.
Application forms are available online at Medical Use of Marihuana, by telephone toll-free
866.337.7705, or by mail:
Office of Cannabis Medical Access
Drug Strategy and Controlled Substances Programme
Healthy Environments and Consumer Safety Branch
Health Canada
Address Locator 3503B
Ottawa, Ontario K1A 1B9

4-Can a pharmacist accepts a prescription for a controlled prescription (formerly called


triplicate/duplicate) drug written by an out-of-province physician?
Yes and no. In certain emergency situations, a pharmacist can receive a prescription
for a controlled prescription drug from an out‐of‐province physician that's not on
the "approved" form. In these situations, as with any emergency, you should use
your professional judgment as to whether you dispense the prescription or not. You
can base your decision on the medical condition of the patient and surrounding
circumstances.
However, it's your responsibility to make sure the prescription is legitimate and the
physician is licensed to practise medicine in Canada. It may be reasonable for some
patients, for example those who live close to the B.C./Alberta border, to have a
physician in Alberta, and their pharmacy in B.C. In those cases, it may be
appropriate to continue to dispense a controlled prescription from outside of B.C.
In most other circumstances, however, if a particular patient makes a habit of
asking you to dispense prescriptions for controlled prescription drugs written by
out‐of‐province physicians, you should refer the patient to a physician licensed to
practise in B.C. You should not continue to dispense the prescriptions.

5-A patient brought in a prescription for two drugs, Amoxil® and Tylenol with Codeine
No.3®, written on the same prescription form. The patient only wants me to dispense one
of these drugs and wants another pharmacy to dispense the other drug. What do I need to
do?
The process you need to follow is different, depending upon which drug you
dispense.
Amoxil® dispensing (non‐narcotic) ‐ you need to return the original prescription to
the patient:
o Process the prescription for Amoxil®.
o On the original prescription, indicate that you have dispensed the Amoxil® and
note your pharmacy name, your initials, and the date.
o Photocopy the original prescription for your records.
o On the photocopy, make a note that the original prescription was returned to
the patient.
o Return the original prescription to the patient. The patient can then have the
undispensed narcotic prescription dispensed at the pharmacy of his/her choice.
Tylenol with Codeine No.3® dispensing (narcotic) ‐ you need to keep the original
prescription:
o Process the prescription for Tylenol with Codeine No.3®.
o Log the prescription for Amoxil®.
o Photocopy the original prescription for your records.
o On the original prescription, indicate that you have dispensed the Tylenol with
Codeine No.3® and logged the Amoxil®. Note your pharmacy name, your
initials, and the date.
o You may photocopy the original prescription and provide the photocopy to the
patient. Be sure to note your pharmacy name and telephone number on the
photocopy so that the other pharmacy can contact you to transfer the Amoxil®
prescription.

6-We were working on a prescription for 100mL of Tussionex suspension. As the


technician was attaching the label to the bottle, it tipped over and the contents spilled
onto the counter and floor. What should I do?
As required by the Regulations to the Controlled Drugs and Substances Act, you
should report the loss (or theft) of controlled drugs and substances directly to the
federal Office of Controlled Substances within ten days of discovery. You can order
loss and theft reporting forms from:
Compliance, Monitoring and Liaison Division
Office of Controlled Substances
Health Canada
Address Locator: 3502B
Ottawa, Ontario K1A 1B9

7-What is the proper way of documenting part-fills for controlled drugs and substances
(including narcotics)?
In the past, Health Canada has expected pharmacists to document part‐fills of
controlled drugs and substances (including narcotics) by recording the quantiity
dispensed on a given date on the reverse side of the original prescription, along
with the handwritten initials of the pharmacist responsible for dispensing the part‐
fill. In addition, a "paper trail" copy of the prescription, for information purposes,
had to be included in the daily prescription file on each part‐fill date.
Health Canada is now determined that the software commonly used in community
pharmacies in British Columbia has automated many recordkeeping functions. It is
now not necessary to add part‐fill documentation to original prescriptions when a
second and subsequent part‐fill is processed, provided that the software program
allows tracking between the part‐fills (quantity, date, prescription number) and the
original prescription. A "paper trail" copy of the prescription must continue to be
filed in the daily prescription file on each part‐fill date.
In the case of methadone prescriptions, pharmacists may continue to document
each part‐fill on the reverse side of the original prescription. A "paper trail" copy
filed on each part‐fill date is not required for methadone part‐fills.
January 2016

1‐ All of the following are true about the DNA cloning, EXCEPT:
a‐ Endonuclease enzyme for cutting at specific site.
b‐ Ligase enzyme for binding.
c‐ Cutting DNA to multi portion.
d‐ Using host cell (E. Coli) for duplication.
e‐ Using plasmid.
DNA cloning is a molecular biology technique that makes many identical copies of a piece
of DNA, such as a gene. In a typical cloning experiment, a target gene is inserted into a
circular piece of DNA called a plasmid.

2‐ Pernicious anaemia is due to deficiency of: 2011,2009


B1 (thiamin)
a‐ Vitamin A B2 (riboflavin)
b‐ Folic acid B3 (niacin)
B5 (pantothenic acid)
c‐ Vitamin E B6 (pyridoxine)
B7 (biotin)
d‐ Cyanocobalamin  vit. B12 B9 (folate [folic acid])
B12 (cobalamin)

3‐ In chromosomal DNA what is true about DNA:


a‐ Single long strand of DNA
b‐ Single long strand of RNA

4‐ Histones are related to:


a‐ DNA b-RNA
histones are highly alkaline proteins found in eukaryotic cell nuclei that package and order
the DNA into structural units called nucleosomes. They are the chief protein components of
chromatin, acting as spools around which DNA winds, and playing a role in gene regulation
5‐ The treatment of Lyme disease:
a‐ Doxycycline (10 dyas) or amoxicillin or cefuroxime DAC (14 days)

6‐ Drug of choice for acute sinusitis:


a- Ciprofloxacin
b‐ Amoxicillin

7‐ What cause endocarditis:


Gram‐positive bacteria staphylococci 40%, streptococci 20% and enterococci
10% account for the majority of pathogens causing IE.
8‐ All can cause conjunctivitis, EXCEPT
a- Streptococcus pneumonia
b- S. aureus
c- H. influenza
d‐ Entamoeba fragilis

9‐ What can cause acute sinusitis:


a‐ Staph. Aureus
b‐ H. influenza
c- Pseudomonas aeruginosa

10‐ What can cause syphilis:(chancre sores)


a- Neisseria gonorrhea
b- Clostridium difficile
c‐ Treponema pallidum

11‐ All are true about ACEI treatment, EXCEPT:


a- Can cause dry cough
b- If there are edema and swelling in the neck and chest area go to emergency
c‐ Take potassium supplement
d- There may be a syncope effect

12‐ All are useful for MI, EXCEPT:


a‐ Verapamil  may lead to AV block
b- Nitroglycerine
c- Streptokinase
d- Oxygen
e- Metoprolol

13‐ What can’t be used with seizures:


a- Carbamazepine
b- Valproic acid
c‐ Bupropion

14‐ AS is related to:


a‐ left ear
b- right eye
c- right ear
d- left eye

15‐ which of the following cause vasodilation? 2015


a- CO
b- NO2
c‐ NO
d- CO2
e- NH2

16‐ What is a nominal or categorical variable? 2015


a- Weight
b- High
c‐ Eye color
d- Range of movement
e- Temperature

17‐ Nephrolithiasis is caused by: 2014,2015


a‐ Calcium
b- Iron
c- Magnesium
d- Phosphate
e- Potassium

18‐ A woman prescribed codeine for pain and complained of no action, then she
increases the dose by herself to relief the pain but there is no action and there is no
abuse, this is due to:
a- Mild renal failure
b- Mild hepatic failure
c‐ Poor codeine metabolism
d- High codeine metabolism

19‐ A patient with runny nose, watery eyes and sneezing and there is no fever, he has:
2015,2012
a- Common cold
b- Influenza
c- Sore throat
d- Pharyngitis
e‐ Allergic rhinitis
20‐ A patient has 4 episodes of cough, fever and weight loss, may had:
a- pneumonia
b‐ tuberculosis (cough with blood, chest pain, orthopnea, weightloss, night
sweets, fever)
c- meningitis
d- influenza
e- common cold

21‐ what enzyme polymorphism may affect ACEI:


a- cytochrome P450
b- Cyclooxygenaze
c- Xanthin oxidase
d- 5 α-redctase
e‐ Angiotensin converting enzyme

22‐ What is the most abundant protein in the body? 2012


a- Keratin
b- Albumin most abundant protein in blood
c‐ Collagen  25 – 30 %
d- Glycoprotein
e- Globulin

23‐ What is the external liquid around blood cells that could take 30 seconds for
coagulation?
a‐ Plasma  protrombin time
b- Erythrocytes
c- Albumin
d- Platelets

24‐ The reverse of lateral is: 2015


a- Ventral
b- Horizontal
c‐ Medial

25‐ Upon eating protein, what is secreted from the stomach: 2013
a- Gastric acid
b‐ Gastrin
c- Secretin
d- Pancreatic enzymes = ttt of cystic fibrosis
e- Pepsinogen

26‐ What is the most one of the following


to decrease triglycerides?
a- Niacin
b- Cholestyramine
c- Statins
d- Ezetimibe
e‐ fibrates

27‐ A child (<12 months) with constipation, what you can give him:
a- Mineral oil
b- Enema
c‐ Glycerine suppositories
d- Enema of aqueous solution
e- Betadine

28‐ The drug that binds to the receptor and reverse its
effect is called: Reverse agonist
a- Atrial agonist
b‐ Physiological antagonist

29‐ What of the following can be taken as IV parenteral solution?


a- Lipophilic soltion
b‐ O/W emulsion
c- W/O emulsion
d- Suspension
e- Depo solution

30‐ Extravasation in IV infusion means: 2015


a- Leakage of blood to the surrounding tissue
b‐ Leakage of IV fluids to the surrounding tissues

31‐ A patient is taking oral contraceptives and she always starts her medication on
Sunday, she forgot to take two doses in the second week, what should you told her to
do: 2012
a- Discard the strip and begin with another one
b‐ Take the daily tablet until the next Sunday and start a new strip
c- Take two tablets today, then two tablets tomorrow and then continue the strip
d- Safely continue the strip without warring
e- Talk to the doctor

32‐ A patient which is on breastfeeding ask you about a method for contraception after
two months of delivery and she told you that she can’t remember to take the drugs
which depend on daily dose, what would you recommend for her:2012,14
a- Oral progesterone pills
b- Oral progesterone and estrogen pills
c- Progesterone patches
d- Vaginal ring
e- DMPA injection it's best option to increase compliance

33‐ What of the following is essential amino acid?


a- Glycine
b- Cysteine
c- Alanine
d‐ Lysine
e- Serine

34‐ A diabetic patient who is taking venlafaxine, he had exacerpation of COPD and he
had the following prescription: (There is some missing parts in the question)
- Salbutamol 18 µg TID
- Fluticasone 200 mg BID
- Metformin 500 mg TID
- Venlafaxine
- Glyburide
What of the following medication will attract your attention?
a‐ The dose of salbutamol is very small = should be 100 µg
b- The dose of metformin is too high
c‐ The possibility of drug interaction between metformin and glyburide

35‐ A person who doesn’t take essential amino acids in his diet, what will the body do
for getting these amino acids: 2015, 2014
a- Synthesis of the amino acids
b‐ Breaking protein to get the amino acids
36‐ Tadalafil is preferable over sildenafil :2015,2013,2012
a- When taking it after Nitroglycerine
b‐ After heavy meals as it decreases absorption of sildenafil
c- For cytochrome enzyme metabolism

37‐ All can write prescriptions for antibiotics, EXCEPT:


a- Dentist
b- Obstetrician
c‐ Chiropractor
d- Veterinary
e- Physician

38‐ What of the following medication can cause agranulocytosis? 2012, 2011, 2009
a‐ Clozapine
b- Haloperidol

39‐ A patient with 4 episodes of diarrhea and dizziness, he had decrease in the urine
flow without vomiting, what can we give him:
a‐ Oral rehydration solution
b- Antidiarrheal
c- Antiemetic

40‐ An Rx for weekly compliance package:


- Furosemide 40 mg 2-tab qam
- Enalapril 10 mg 1 tab with lunch
- Lipitor 20 mg 1-tab hs
- Glyburide 2-tab BID
- Acetaminophen every 6 hrs
What should trigger the attention of the pharmacist:
a- Frusemide 2 tablets in the morning
b- Enalapril 1 tablets daily
c- Lipitor before bed time
d‐ Glyburide 4 tablets daily

41‐ Dysgeusia is the loss of: 2014,2013,2012


a‐ Taste
b- Smell
c- Hear
d- Breathing
e- Vision

42‐ Dysphonia could be caused by:


a- Acetaminophen
b‐ Fluticasone
c- Methotrexate

43‐ Dysphasia is:


a- Difficulty of swallowing
b- Difficulty of breathing
c- Difficulty in the vision
d- Difficulty in tasting
e‐ Difficulty in speeking

44‐ A patient has COPD is taking salbutamol & tiotropium, what should be added on:
a- Fluticasone
b‐ Formoterol
c- Leukotrienes antagonists
d- Theophylline

45‐ All are true about the factors that trigger asthma, EXCEPT:
a- Emotional stress may increase asthma symptoms
b- Viral infection may increase asthma symptoms
c‐ Increase asthma symptoms at high is uncontrolled asthma
d- Pollen grains and specific drugs may increase asthma
e- Smoking in adult make worsening to symptoms

46‐ A patient with pruning in the vagina with white discharge had:2012
a- Viral vaginitis
b‐ Vaginal candidiasis  cottage discharge
c- Trichomonas
d- Bacterial vaginitis  fishy odour
e- Vaginal atrophy

47‐ All can be used for the treatment of onychomycosis, EXCEPT:


a‐ Metronidazole
b- Amorolfine
c- Terbinafine
d- Itraconazole
e- Fluconazole

48‐ If a patient took amiodarone with statin what will happen:2012


a- Increase amiodarone
b- Decrease amiodarone
c‐ Increase statin levels  leading to toxicity and rhabdomyolysis
d- Decrease statin
e- Nothing will happen

49‐ The following structure is found in a mixture with a similar compound, that
compound is: 2015
a- Isomer structure
b‐ Cis trans isomer
c- Tautomer
d- Diasteriomer
e- Geometrical isomer

50‐ The following structure (tetracycline) when


taken with metals such as Ca or Fe will make:
a‐ Drug‐metal complex

51‐ The following structure when we


separate the intermediate structure
this will be: (I am not sure about the
structure below):
a- Ecosahexaenoic acid
b- Arachidonic acid
c‐ Docosapentaenoic acid
d- Linolenic acid
e- Ascorbic acid
52‐ What is the antimetabolite of the following
structure? 2010
a- Acetoacetate
b‐ Para amino benzoic acid
c- 5-flurouracil
d- Trimethoprim
e- Sulfonamide

53‐ The following structure undergoes metabolism by:


a- Deacetylation
b- Oxidation
c- Dealkylation
d‐ Hydroxylation
e- Conjugation

54‐ Antioxidant in biphasic lipid layer is: 2011


a‐ Vitamin A
b‐ Butylene
c- Vitamin C
d‐ Vitamin E

55‐ What tablet we could crack to put into a solution in the mouth-stomach tube:
a- Enteric coated
b‐ Sugar coated
c- Sustained release
d- Film coated

56‐ The relation between B-cells and bone marrow: 2015


a- Bone marrow contains B-cells
b- B-cells differentiate to bone marrow
c‐ B‐cells derived from bone marrow

57‐ What of the following drugs may increase insulin secretion:2015,2013,2010


a- Pioglitazone
b- Acarbose
c‐ Natiglinide
d- Metformin
58‐ Topiramate side effect:
a‐ Weight loss
b- Weight gain
c- Nausea
d- Sedation

59‐ What could cause calf pain:


a- Sitting for long time
b- Heating in the leg
c‐ Claudication

60‐ Damage of the sciatic nerve:


a- Difficulty in moving the arm
b- Difficulty in moving the shoulder
c‐ Difficulty in walking and pending the knee
d- Difficulty in moving the elbow

61‐ All can cause hypercalcemia, EXCEPT:


a- Excessive vitamin D
b- Excessive parathyroid hormone
c- Malignancy
d‐ Malabsorption and chronic diarrhea

62‐ Tumor lysis mechanism, what is true: 2014


a‐ Hyperuricemia hyperkalemia, hyperphosphatemia, hypocalcemia, and acute
renal failure
b- Hypophosphatemia
c- Hypercalcemia
d- Hypokalemia
e- Hypoglycemia

63‐ The most nauseating chemotherapy:


a- Bleomycin
b‐ Cyclophosphamide
c- Trastuzumab Cisplatin is more nauseating then comes cyclophosphamide

d- Herceptin
e- Cisplatin
64‐ Granisteron is related to:
a‐ Serotonin  5HT3 receptor antagonist used as antiemetic
b- Epinephrine
c- Dopamine

65‐ N-acetylcysteine is used in acetaminophen toxicity to restore the level of:


a- Metabolite
b- Phenacetine
c‐ Glutathione
N-Acetylcysteine is the drug of choice for the treatment of an acetaminophen
overdose. It is thought to provide cysteine for glutathione synthesis and possibly
to form an adduct directly with the toxic metabolite of acetaminophen, N-acetyl-p-
benzoquinoneimine.

66‐ The most used treatment of chronic acquired pneumonia with pseudomonas
aeruginosa is:
a- Nitrofurantoin
b‐ Ciprofloxacin
c- Clarithromycin
d- Ceftriaxone
e- Penicillin

67‐ Which one is used for Treatment of cystitis?


a- Amoxicillin/clavulanic acid
b‐ Levofloxacin
c- Penicillin
d- Moxifloxacin
Syndrome Infecting organism & Symptoms Firs‐Line Second‐Line
Acute Escherichia coli (80– 90%), SMX/TMP PO × Fluoroquinolone
Uncomplicated Staphylococcus saprophyticus (5– 10%), 3 days or (ciprofloxacin,
Klebsiella pneumoniae, Proteus mirabilis
UTI (Cystitis) Trimethoprim levofloxacin,
Behavioural factors promoting infection
Occurs in include sexual intercourse and use of
PO × 3 day or norfloxacin), PO
females with spermicide or diaphragm. Nitrofurantoin × 3 days
normal Usual presenting symptoms include internal PO × 5 days or or
genitourinary dysuria, frequency, suprapubic discomfort Fosfomycin Cephalexin PO ×
tract. and urgency. Recurrences are common. tromethamine 7 days
as single dose
PO
68‐ What of the following organisms may cause malaria?
a- Clostridium difficile
b‐ Plasmodium ovule
c- Treponema pallidum
d- H. pylori

69‐ All are for prophylaxis from traveller’s diarrhea EXCEPT:


a- Peel fruits
b- Leaving the rest of the food
c‐ Eating salad
d- Cooked vegetables
e- Drink backed water

70‐ Omeprazole + tetracycline + metronidazole is used for the treatment of:


a- Crohn’s disease
b- Traveller’s diarrhea
c‐ Peptic ulcer  triple therapy for E. coli

71‐ The symptom of the ulcerative colitis:


a- Constipation
b‐ Bloody diarrhea
c- Fistula  crohns disease
d- Vomiting
e‐ Abdominal cramps

72‐ The most side effects of donepezil are on: Cholinesterase Inhibitor
a- Respiratory system
b- Liver
c- Kidney
d- Heart
e‐ Gastrointestinal tract

73‐ The function of corpucus collasum:


a‐ Connect the two brain hemispheres
b- Connect the fronted lobe with parietal lobe
c- Connect the two temporal lobes
d- Connect brain with spinal cord
74‐ In status epilepticus what should be given in the first 10 minutes:
a‐ IV lorazepam
b- IV phenytoin

75‐ What of the following drugs can cause flushing:2015.2013?


a- Fluconazole
b- Dexamethasone
c‐ Anastrazole  ttt of breast cancer

76‐ Treatment of endometritis pain:


a- Testosterone
b‐ NSAIDS & HORMONES (OCP‐GnRH AGONIST, DANAZOL)

77‐ What of the following is neuropathic pain? Shingles (Postherpetic Neuralgia)

78‐ What is true about hepatitis B transition:


a- Contact with saliva of infected animal
b- Direct contact
c- Sexual contact, blood transfusion

79‐ What can be added to tablets to delay its dissolution:


a‐ Carboxy methyl cellulose (or cellulose)

80‐ What can cause red man syndrome:


a‐ Vancomycin
b- Amoxicillin
c- Penicillin
d- Ciprofloxacin
e- Cephalosporin

81‐ Treatment of panic attack:


a‐ Venlafaxine  also SSRIs are 1st line
b- Amitriptyline

82‐ What of the following can decrease prolactin secretion?


a- Acetylcholine INCREASE PROLACTINE SECRETION
b‐ Dopamine agonist (bromocriptine)
83‐ What is the least one to cause dependence?
a- Morphine
b- Amphetamine
c‐ Guaifenesin

84‐ Angle of repose measures: 2015, 2014


a- Solubility
b- Dissolution
c‐ Flowability
d- Stability
e- PH

85‐ Aliskiren is:


a‐ Renin blocker
b- ACEI
c- Angiotensin receptor blocker

86‐ Losartan act on what receptor:


a- Adrenergic receptors
b‐ Angiotensin type 1A receptors
c- Cholinergic receptors

87‐ Nifidipine is:


a- Beta blocker
b‐ Dihyfropydridine CCB
c- Non-dihydropyridine CCB
d- ACEI

88‐ The use of polyvinyl alcohol in ophthalmic preparation is to: 2014


a- Increase contact of drug with cornea
b- Decrease viscosity (viscosity enhancer)
c- Lubricant
Polyvinyl alcohol (C2H4O) x is a water-soluble synthetic polymer. It is used to increase
viscosity in pharmaceuticals and as a lubricant and protectant in ophthalmic preparations.
Polyvinyl alcohol is often found in over-the-counter eye redness and eye lubricant ("Tears")
eye drops. Polyvinyl alcohol is the lubricant, and works by providing moisture to the eye,
which helps relieve dryness and protects the eye from becoming more irritated
89‐ Gauge number represents: 2015
a- Inner volume of the needle Inner diameter of needle

b‐ Outer diameter of the needle


c- The length of the needle

90‐ Why we press on lacrimal gland when we add ophthalmic drops:


Systemic absorption through nasal mucosa can be reduced through digital occlusion of
the nasolacrimal drainage system for several minutes following instillation of drops or
through eyelid closure for 3–5 minutes

91‐ A patient undergoes knee surgery and he had taken heparin, he is suffering from
heparin induced thrombocytopenia, what is the least drug can be used for prophylaxis
after the surgery:
a‐ Enoxaparin
b- Fondapurinox
c- Rivaroxaban
d- Dabigatran

92‐ What has positive inotropic effect:


a- Propranolol –ve inotropic and chronotrpic
b- Dobutamine +ve inotropic
c- Enalapril

93‐ What is not true about vitamin A: Deficiency could cause night blindness
(nyctalopia) This is true

94‐ What is a symptom of BPH?


a- Polyuria
b‐ Hesitant urination
c- Painful urination

95‐ Loflunomide used for treatment of rheumatoid arthritis


by inhibiting:
a‐ Selective inhibitor of de novo pyrimidine synthesis
that works by inhibiting dihydroorotate dehydrogenase

96‐ All are tests for diagnosis of cancer EXCEPT: 2014,2012


a- Pap smear
b- APGAR test was designed to quickly evaluate a newborn's physical condition
c- PAS PSA
d- Cervical colon antibody
e- Mammography  examine the human breast

97‐ Ovulation cycle takes place for how many days before the new menstrual cycle:
a- 7 days
b‐ 14 days
c- 21 days
d- 28 days

98‐ In DNA Guanine is equal in amount to:2015,2013 Chargaff's Rules


a- Adenine in DNA the amount of
b- Thiamin adenine always equals to
c- Uracil the amount of thymine.
d‐ Cytosine The amounts of guanine
e- Adenosine are always equale to the
amount of cytosine
99‐ What is true about cold chain technique:
a‐ Keeping drugs cool from manufacturing until the patient receive it
b- Keeping the vaccine in the refrigerator

100‐ Conversion from nucleoside to nucleotide by: 2014,2012


a) Double band
b) Sugar
c) Ribose
d) Phosphate

101‐ Long acting insulin:


a- Lispro
b- NPH
c‐ Detemir

102‐ All are true about insulin injection EXCEPT:


a- Cause lipoatrophy
b- Cause pain at site of injection
c‐ Cause weight gain
103‐ Primary deficiency in androgens is related to deficiency in secretion of:2011
a- Anterior pituitary
b‐ Adrenal cortex This q is confusing because u can mix up with primary and secondary
c- Posterior pituitary hypogonadism but here the site of production is testies and adrenal so both
primary and secondary is pituitary because it triggers the synthesis in
d- Thyroid gland primary organs
e- Adrenal medulla

104‐ Trastuzumab is used for the treatment of:


a- Ovarian cancer
b‐ Breast cancer HER2 RECEPTOR ANTAGONIST
c- Prostatic cancer
d- Colon cancer
e- Lung cancer

105‐ Which type of cytokines cause flue like syndrome: 2013


a‐ Interleukin
b- Erythropoietin Both does
c- Interferon
d- Filgrastim
e- Colony stimulating factor

106‐ All are true about psoriasis EXCEPT: 2013,2009


a- Autoimmune disease
b- Treated by UV
c- Treated by anthralin
d‐ 70‐80% of cases will get psoriatic arthritis  only 5 – 8% will develop
psoriatic arthritis
e- It is red patches covered with silver scales

107‐ Examples of biologic response modifiers:


a‐ Etanercept
b- Penicillamine
c- Hydrocloroquine
d- Azathioprine
e- Cyclosporine

108‐ Which of the following when excessively used will precipitate osteoporosis?
a- Calcium
b- Vitamin D
c‐ Anstrazole

109‐ What is used for toxicity from enteric coated tablet: 2015
a- Diuretics
b- Activated charcoal
c- Alkalizing agents
d- Acidifying agents
e‐ Complete gastric lavage

110‐ What is mainly secreted by the kidney:


a- Erythropoietin produced by interstitial fibroblasts in the kidney
b- Vitamin K BY INTESTINAL BACTERIA (COLON)

111‐ Warfarin is affected by metabolism of which enzyme:


a- Cyclooxygenase
b‐ Cytochrome P450
c- Vitamin K hydrogenase

112‐ Pathway of blood from liver to systemic circulation:


a‐ Mesenteric vein and portal vein
b- Portal artery and hepatic vein
c- Portal vein and hepatic artery
d- Hepatic vein and portal artery

113‐ Pathway of blood from lung to brain:


a- Right atrium - left atrium - lung - brain
b- Right atrium – right ventricle – lung –
brain
c- Left atrium – right ventricle – brain
d- Left ventricle – right atrium – brain
e‐ Left atrium – left ventricle – aorta –
brain

114‐ Normal peacemaker of the heart:2014


a‐ SA node
b- AV node
c- Purkinje fiber
d- Right ventricle
e- Aorta

115‐ Sleeping apnea is very dangerous in patient???


a- Corma
b‐ Stroke
c‐ Seizure
d- Dizziness
e- Asthma

116‐ Secretion of ADH is stimulated by: 2013,2009


a- Increase blood volume
b‐ Increase plasma osmolality (LOW WATER)
c- Decrease blood osmolality

117‐ Drug is 100% metabolised by liver when renal function is decreased toxicity is
due to:
a- Increase drug clearance
b- Decrease renal metabolism
c‐ Accumulation of toxic metabolite

118‐ What is the most drug that cause problems when wrongly dispensed:
a- Leflunomide
b‐ Methotrexate
c- Infliximab
d- Azathioprine
e- Mercaptoprine

119‐ Atypical antipsychotic has more effect than typical antipsychotic in:
a- Neuromalignant syndrome
b- Extrapyramidal side effect
c‐ Weight gain
d- Agranulocytosis
e- Sexual dysfunction

120‐ Which drug used in treatment of ADHS cause abuse:


a- Amphetamine
b- Methylphenidate
c- Atomoxetine
d‐ Dexamphetamine

121‐ Carbidopa used with levodopa in treatment of parkinsonism to:


a‐ Prevent peripheral conversion of levodopa to dopamine
b- To increase peripheral dopamine absorption
c- To increase central dopamine secretion
d- To decrease central levodopa conversion to dopamine

122‐ What is the most thing to monitor when using lithium:2015,2013,2012


a- Uric acid level
b- Triglycerides
c- Blood glucose level
d- LFT
e‐ TSH

123‐ In treatment of parkinsonism which drug is added for wearing off:


a‐ Amantadine
b- Haloperidol
c- Valproic acid
d- Phenytoin

124‐ What is the following is better for more improving dialysis?


a- PH
b‐ Large particle size

125‐ Vancomycin monitoring to prevent toxicity:


a- Weight
b- Liver enzyme
c- Blood count
d‐ Serum creatinine
e- Thyroid function

126‐ Gentamycin side effect:


a- Hepatotoxicity
b- Anemia
c‐ Tubular necrosis
death of tubular epithelial cells that form the renal tubules of the kidneys
127‐ Which of the following antibiotics can cause QT prolongation:2014,2015
a- Penicillin
b- Nitrofurantoin
c- Amoxicillin
d‐ Clarithromycin

128‐ In ECG, arterial fibrillation appears as: 2012,2013,2014


a‐ Disappearance of P wave
b- QT prolongation
c- QRS complex prolongation
d- Disappearance of impulse
e- Increase ventricular contraction

129‐ In QT prolongation which of the following is the most dangerous:2013


a- Ventricular tachycardia
b- Atrial fibrillation
c- Supraventricular arrhythmia
d‐ Torsade de points
e- Ventricular repolarisation

130‐ Meclobemide mechanism of action:


a- Cholinergic receptor inhibiter
b‐ Monoamine oxidase type A inhibitor
c- Serotonin reuptake inhibitor

131‐ St. john worts is an herbal drug and can be used with all of the following
EXCEPT:
a- Mild depression
b- Cause photosensivity
c‐ Can be used with MAOI

132‐ Tick typhus fever in rickettsia illness caused by:


a- West Nile virus
b‐ Rockey mountain virus
c- Familial Mediterranean fever
d- Puerperal fever
133‐ What is true about oxybutynin side effect:
a- Urination
b- Diarrhea
c‐ Dry mouth
d- Lactination
e- Bradycardia

134‐ What is true about active immunity:


a- It is mediated by vaccination
b- Antibodies transferred from another body
c- It is transferred from mother to fetus
d‐ The person is infected and make antibody in his body

135‐ Sympathetic heart stimulation can cause: Increase AV conductivity

136‐ Particle size affecting: Bioavailability

137‐ In hyper immunity reaction what is true: Vasodilation mediated by NO


release

138‐ Bacteriophages are: 2015.2013


a- Bacterial infection
b‐ Virus that could infect bacteria
c- Bacteria that are carried with virus

139‐ Mercaptopurine is metabolised by which enzyme: 2013,2012,2008


a- Dehydrogenase
b- Cyclooxygenase
c- 5 α reductase
d- Cytochrome P450
e‐ Xanthine oxidase

140‐ Isoniazid needs concurred administration with:


a- Vitamin C
b- Calcium
c- B carotene
d- Vitamin K
e‐ Pyridoxine
141‐ Parenteral preparation may cause foaming during reconstitution, what should
be done before administration:
a‐ Wait until the foam disappear
b- Pull the solution with the foam
c- Pull the foam first and discard it then take the solution
d- Clear the solution with a needle and leave the foam

142‐ Counselling for cephalexin suspension:


a‐ Shake well – refrigerate‐ expect mild diarrhea
b- Shake well – take on empty stomach
c- Refrigerate – take on empty stomach – expect diarrhea
d- Keep at room temperature – shake before use
e- Shake well – refrigerate – take on empty stomach

143‐ Insulin can be taken till the expiry date if:


a‐ Refrigerate
b- Cooled
c- Freezed
d- Left on room temperature

144‐ INR can be increased when taken warfarin with: Cimetidine

145‐ Vitamin D synthesised by sun exposure from the internal source which is:
a- Calcium
b- Vitamin E
c- Vitamin C
d‐ Cholesterol

146‐ What does collagen need to be synthesised? 2015,2013


a- Keratin
b- Vitamin A
c‐ VitaminC
Collagen contains specific aminoacids Glycine, Proline, Hydroxyproline and
Arginine.

147‐ Which of the following can be increased in bruising after heating: 2013?
a- Albumin
b‐ Bilirubin
c- Globulin
d- Platelets

148‐ The drug Primexin (propoxy phenoxy???????) ??????


a- The same drug
b- Generic drug
c- Therapeutic interchangeable

149‐ All of the following are viral infection EXCEPT:


a- Herps simplex
b- Chicken pox
c- Measles
d‐ Impetigo by two bacteria streptococcus pyogenes and staphylococcus
aureus

150‐ What is true about ophthalmic preparation in surgery: 2015


a‐ It is packed in single dose container
b- It is sterile and used with normal saline

151‐ What is true about sunscreen products: 2015


a- Minimum SPF is 15
b- Protect from UVA All sunscreen products are against UVA & UVB

c‐ Protect from UVB

152‐ Folic acid dose for pregnant with epilepsy:


a- 1 mg
b- 2.5 mg
c‐ 5 mg
d- 7.5 mg
e- 10 mg

153‐ Shelf life is:


a- T ½ HAIF LIFE
b‐ T 90
c- T 95

154‐ What is true about blood glucose: Fasting blood glucose > 7 is indication of
diabetes
155‐ What of the following is the safest prescription according to dangerous ISMP
abbreviations?
a‐ Latanoprost i gtt TID
b- Vitamin D 400 IU daily
c- ……. PO od
d- Warfarin .5 mg qd am
e- ……. 50 mg prn

156‐ What should be added to soft gelatin capsule to increase plasticity:


a- CMC
b‐ Glycerin
c- Starch
d- Water

157‐ Acetaminophen metabolised to


its toxic metabolite by:2010
a- Acetylation
b- Methylation
c- Hydroxylation
d‐ Oxidation
e- Glucoronidation

158‐ What is metabolised by O-demethylation to form active metabolite:2014


a‐ Codeine
b- Morphine
c- Meperidine
d- Heroin

159‐ Ketoconazole is a potent inhibitor to:


a- Cyt 2D9
b- Cyt 2D6
c‐ Cyt 3A4

160‐ Carboxylic acid undergoes metabolism by making conjugation with:


a- Serine
b- Glutathione
c- Cysteine
d‐ Glycine
161‐ Cold sores can be transmitted through: 2011
a- Blood transfusion
b- Body fluids
c‐ Direct contact with the sores

162‐ ASA inhibits COX enzyme by:2014


a‐ Acetylation
b- Oxidation
c- Hydrolysis

163‐ Low molecular weight heparin LMWH has more effect on;2011
a- Thrombocytopenia
b- Factor Xa
c‐ No more effect therefore no need to be monitored

164‐ Which is primary literature: 2014


a- Pubmed
b‐ Clinical trials
c- CPS
d- E-TC

165‐ Which of the following causes venous pooling: 2015?


a- Hydralazine
b- Clonidine
c‐ Sodium nitroprusside
d- Prazocin
e- Minoxidil

166‐ Antitussive contains codeine is suitable to be used in which age:2015


a- 6 years
b- 8 years
c- 10 years
d‐ 12 years
167‐ What is present in external layer of surfactant and when increased it makes
aggregation:2011
a‐ Micelles (Follicules of SAA)
It is an aggregate (or supramolecular assembly)
of surfactant molecules dispersed in a liquid colloid. A typical
micelle in aqueous solution forms an aggregate with
the hydrophilic "head" regions in contact with
surrounding solvent, sequestering the hydrophobic single-tail
regions in the micelle centre.

168‐ A Drug is classified as schedule F, what is true for advertising of this drug:2012
a‐ No advertisement is available
Schedule F part 1: Food and prescription drugs advertisement standards. Have
all prescription drugs that are present in NAPRA schedule 1

169‐ You are a pharmacist in a community pharmacy and you have told that a
technician in your pharmacy counselled about an OTC drug, what would you do:
a- Talk to him in a private area
b‐ Make an open meeting and review the roles of the pharmacy staff
c- Talk to other employee to make a supervision on him
d- Make a report about this

170‐ The role of the assistant in the pharmacy includes:


a- Counselling about OTC drugs
b- Suggesting drugs for the patients
c‐ Taking the prescription from the patients for preparing it
d- Answering the phone and taking orders
e- Review the patient’s information from their files that are kept on the system

171‐ The weakest clinical trial is:


a- Cochran library
b- Meta analysis
c- Systemic review randomised trials
d‐ Systemic review case study trials

172‐ You are a pharmacist and a person come to you and told you about the
appearance of some cases of pertussis, what is true to inform him about this disease:
a‐ Pertussis is a vaccinated disease and there are booster doses can be taken
b- Pertussis is a miner disease and no thing to worry about
c- Pertussis is a disease that affect children only
Pertussis vaccine is a vaccine that protects against whooping cough. There are two main
types: whole-cell vaccines and acellular vaccines dTaP, TDaP. The whole-cell vaccine is
about 78% effective while the acellular vaccine is 71–85% effective.

173‐ A patient recently had NSTEMI and has hypertension, atherosclerosis, and
hyperlipidemia, how long should he takes clopidogrel + ASA for prophylaxis from
further heart problems:
a- 1 month only
b- 3 months
c- 6 months
d‐ 1 year

174‐ A patient which make periodic check for his anticoagulant therapy in the clinic,
the doctor found that his INR = 5.2, the patient is on a therapy of warfarin 0.5 mg PO
daily. The doctor wants to know the cause of this. Which is the most initial question
that the doctor should ask to the patient:
a‐ Did you take the dose correctly today?
b- Do you have any symptoms of bleeding?
c- What did you eat today?

175‐178 ‐ 4 questions about type I and type II errors 2014.2013


a type I error is the incorrect rejection of a true null hypothesis (a "false positive"),
a type II error is the failure to reject a false null hypothesis (a "false negative").
More simply stated, a type I error is detecting an effect that is not present, while a
type II error is failing to detect an effect that is present

179- what is true about tremors of Parkinson:


a- It is initiated bilaterally - UNILATERAL
b- It affects mainly the upper extremities
c‐ It is happening during the rest and decreased by moving

180- you are in a community pharmacy and a patient comes and wants to measure his blood
pressure, what is true to be done:
a- Told the patient to sit and rest for 20 minutes before measuring
b- Told the patient to lay down for 10 minutes before the measuring
c‐ Tell the patient that he should not talk during the measuring
d- Take the average of 10 measurements

181- you are in a diabetic clinic and a patient comes to you with a prescription for insulin
pen written by a registered nurse, what would you do:
a‐ Tell the patient to get another prescription from a doctor
b- Make a report about this nurse
c- Tell the patient to sign the prescription from the chef nurse
d- Dispense the prescription after normal checking

182- what is true about Minoxidil treatment for alopecia: 2009


a‐ Minoxidil mechanism is by stimulating hair follicles for hair growth
b- Minoxidil is used for males only  2% FOR FEMALES
c- 5-20% of minoxidil reach systemic circulation
d- Minoxidil is left for 10 minutes on the hair then wash off

183- a lawyer came to your pharmacy and asked you for the medical history for a girl (16
years) for an investigation in a case between her parents, what would you do:
a- Give him the information he needs
b‐ Tell him that you should talk to the girl first and take her permission.
c- Tell him that you will give him the information if her parents agree
d- Tell him to get a court order first
e- Give him the information on a paper and write “confidential” on it

184- a patient in a hospital is prescribed for Lamisil but he received Lamictal instead, after
3 days, the patient asked why he is taking antidepressant drug, this kind of dispensing error
could be happened due to all of the following EXCEPT: Debating question between C, D & E
a- The nurse heard the order wrongly from the doctor
b- The nurse confused in the names of both drugs
c‐ The drugs are putted in the pharmacy beside each other that may cause
confusion
d- The nurse in this shift wasn’t know by the treatment of this patient
e- The doctor wrote the wrong prescription from the beginning

185- the patient refuses to take his medication and gets his autonomy when:
a- The patient knows the side effects for the drug
b- The patient knows the nature of his disease
c- The patient took the medication before and didn’t feel any improvement
d‐ You told the patient about all the benefits and the side effects of this drug
186- you got a phone call from the doctor who want an accurate and rapid information
about glucosamine, what is the most rapid source you will use to get the information:
a‐ CPS
b- Glucosamine is not found in Cochran library and there is no information about it

187- in counting antibiotics for dispensing sheet in the pharmacy containers, you can use a
sheet for counting the antibiotics for:
a‐ All antibiotics
b- Antibiotics that are similar in shape
c- Antibiotics that are contraindicated in pregnancy
d- Antibiotics that are taken on empty stomach

188- the pharmacist who is working in a hospital, his role is regulated by:
a- The manager of the hospital
b- The physician of the sift
c‐ The order of the hospital pharmacists

189- who regulate the role of the pharmacist in the community pharmacy:2012
a- Health Canada
b‐ NAPRA
c- Canadian association for drugs and technologies in health
d- Canadian advisory control

190- what regulation is responsible for the final formaulary for a new drug on the basis of
cost effectiviness and clinical effect:
a- Canada vigilance post-market surveillance program that collects and assesses
reports of suspected adverse reactions to health products marketed in Canada
b- Canadian association for drugs and technologies in heath CADTH delivers evidence,
analysis, advice, and recommendations to health care decision-makers, so that they
can make informed decisions about the optimal use of drugs and medical devices in
our health care system
c- Patented medicine price review board The PMPRB protects and informs Canadians
by ensuring that the prices of patented medicines sold in Canada are not excessive
and by reporting on pharmaceutical trends.
d- The Canadian Medication Incident Reporting and Prevention System

191- what is true about cost benefit analysis:2012


a- Both measurements and outcomes indicated with dollars
192- a patient of 52-year-old is in early stage of Alzheimer, he came to you in the pharmacy
with a prescription, what is true about counselling this patient:
a- Dispense the prescription and counsel him as usual
BOth r good options but c is better if possible
b- Speak to him loudly and clearly because he might take the paper and forget
c- Counsel the patient in presence of a caregiver about it

d‐ Give the patient a printed sheet contains information of the drug and limit the
verbal counselling

193- drug A and drug B, both are given IV, the dose of drug A = the dose of drug B. but AUC
of drug A is double AUC of drug B, so drug B has: 2015
a‐ More clearance than drug A
b- More volume of distribution
c- More absorption
The AUC is directly proportional to the dose when the drug follows linear kinetics.
The AUC is inversely proportional to the clearance of the drug.

194- a study takes place in chemical factory to find relationship between having cancer and
working in the factory. The results have been collected over a long period of time, the data
gathered proved that there is possibility of cancer with workers, what type is this study:
a- Cochran study
b- Meta analysis
c- Double blind study
d- Randomised study
e‐ Case control study

195- in drug clinical trials, phase II is done to detect safly and efficacy of the drug and is
done on:
a- Small group of healthy volunteers
b‐ Small group of people with the disease
c- Large group of healthy volunteers
d- Large group pf people with the disease

196- what is meant by merchandising:( ktype) 2011


a‐ Visual selling
b‐ Counting drugs in the shelfs
c‐ Advertising about a new drug
197- a patient had a recent myocardial infraction, he had to stop smoking, what is
contraindicated:
a‐ Nicotine replacement therapy
b- Vernacillin
c‐ Bupropion
he had to stop smoking till now he didn't stop smoking so he can't use nicotine
replacement therapy as it is contraindicated with smokers

198- a woman on progesterone oral contraceptive, she is suffering from spotted bleeding
after the day eight of the menstruation cycle. What should you suggest the doctor to do?
a- Increase the oral progesterone dose
b‐ Give her a combined therapy with higher dose of estrogen
c- Give her a back up method
d- Tell her that the bleeding will stop after a period of time
e- Stop the medication and make reassessment
S = strength
199- in SWOT analysis, W is referred to: 2012 W = Weakness
a- Working O = OPPORTUNITY
b‐ Weakness T = THREAT

200- the most inhibitory neurotransmitter in the body is: 2009


a- 5-hydroxy tryptamine
b‐ Gama amino butyric acid
c- Acetyl choline

201- when you are working in a community pharmacy, and one of the working team
informs the insurance company about something happened in the pharmacy, this is
called:2014
a‐ Whistle blower
b- Honesty

202- when you will discuss the prescription with a patient you ask him a propping question
to:
a‐ Check if he understands what you say
b- Make counselling about drugs written Differs according to the question details
c- Starting conversation with him
d- Ask him what he is suffering from
203- what is meant by empathy listening:
a‐ Feeling what the talker feels
b- A knowledge with what the other person says
c- Listening with open mind
d- Understand what the other say

204- what causes angioedema: 2015


a- Verapamil
b‐ Enalapril
c- Hydrocortisone
d- Spironolactone
e- Furosemide

205- you are a manager in a community pharmacy and you decide to give service with price
which only covers your cost and expenses, you decide to make this service with 10 cases
per month, therefore you will have:
a- Increase in net profit
b‐ Increase in the revenue
c- Increase in the inventory

206- the ophthalmic solution used in the ophthalmic surgery should be: 2015
a‐ Packed in single dose containers
b- Isotonic in normal saline
c- Stock solution

207- using sulfonylureas for treatment of diabetics, Gliclazide differ from Gliburide in:
a- It has longer duration of action
b‐ It has less hypoglycemic effect
c- It causes more weight gain

208- febuxostate is act by inhibition of which enzyme:


a‐ Xanthin oxidase
b- Cyclooxygenase
c- Alpha reductase
d- Folate reductase

209- all of the following are polypeptide in nature EXCEPT:


a- Insulin
b- Glucagon
c‐ Galactose
d‐ Sucrose

210- drug can cause hyperuricemia:


a‐ Hydrochlorothiazide
b- Ramipril
c- Allopurinol

211- which of the following drugs may affect bioavailability of OCP:2014,2012


a‐ Phenytoin
b- Lamotrigine
c- Gabapentin

212- what is considered the most dangerous cause of stroke:2013,2012


a- Supraventricular tachycardia
b- Ventricular arrhythmia
c‐ Atrial fibrillation

213- ceriatine kinase level is measured and used for diagnosis of:
a‐ Myocardial infraction
b- Congestive heart failure

214- kinase enzyme is considered as what type of enzymes:


a- Hydroxylase
b- Lyase dehydratase, aldolases, carboxylases
c‐ Hydrolase proteases/ peptidases, lipases, nucleases
d‐ Transferase also deaminases

215- dissolution of the drug affected by:


a- Wettability
b‐ Solubility
c- Stability

216- methotrexate is considered as:


a‐ Antimetabolite DMARDs

217- opioid agonist act on what type of receptors:2015,2012,2009


a‐ Mu receptors
b- Kappa receptors
c- Delta receptors

218- osmotic pump is considered as: 2014,2013,2011


a‐ Zero order
b- First order
c- Second order

219-what is true about triazepam prescription:


a- Only written prescription with no verbal allowing
b- Written and verbal prescription with no transfer
c- Written and verbal with no refill and part fill
d- Written and verbal with only written refill and part fill with no transfer
e‐ Written and verbal prescription with allowing refill and part fill and transfer
once

220- what is true about codeine 8 mg prescription (with 2 non opioids):


a‐ It is OTC drug and don’t need prescription BTC not OTC

b- It can be dispensed and refill easily


c- It needs only a written prescription with no verbal or written refill is allowed

221- bioavailability represents:


a‐ Rate and extent of absorption
b- Rate of elimination
c- Amount of drug in the body

222- which drug can cause dangerous side effects when a pregnant take it:
a- Clonidine
b‐ Ramipril  known teratogenic
c- Hydralazine
d- Metoprolol
e- Methyldopa

223- in osteoporosis, which drug may increase fracture risk:


a‐ Anastrazole
b- Alendronate
c- Infliximab
224- which vitamin is not recommended for a breast-feeding woman: 2007
a- Vitamin D
b‐ Vitamin A
c- Vitamin K
d- Vitamin E

225- overdose of levothyroxine can cause:


a- Weight gain
b‐ Tachycardia
c‐ Diarrhea
d- Sensitivity to cold

226- excessive sweeting is known as:


a- Salivation
b‐ Diaphoresis
c- Excessive water excretion

227- what is the organ that is not innervated by sympathetic stimulation:


a- Lung
b- Heart
c- Eye
d‐ Salivary gland
e- Genital gland

228- which of the following organisation is responsible for reporting for a drug side effect:
a- Drug review system
b- Canada vigilance
c‐ Institute of safe medication program

229- vitamin K is secreted from: 2008,2007


a- Lung
b- Kidney
c‐ Colon It is the highest PH
d- Spleen
e- Liver

230- which drug can cause esophagitis:


a‐ Alendronate
b- Fluticasone
c- Methotrexate

231- ions is important for muscle contraction and its deficiency may cause muscle spasm:
a‐ Calcium
b- Sodium
c- Potassium
d- Magnesium
e- Iron

232‐243 Calculations

244- the inventory at the end of the year was 203.000 $ and the cost of goods was 240.000$,
what is the inventory at the beginning of the next year:
a- 200.000$
b‐ 203.000$
c- 240.000$

245- you have the following data,


what is the gross margin:
Cost of good sold = 480.000 $, Sales =
600.000$, Expenses = 400.000$, Net
profit = 240.000$
a- 6%
b- 10%
c‐ 20%
d- 25%
e- 50%

246- an adult dose is 0.8 mg and a


child surface area 0.75m2, what is the
child dose:
a- 0.6
b- 0.35 = 0.8 x 0.75 / 1.73 =
c- 0.25
d- 0.2
247- a prescription for prednisolone tablets was written as follow: 40 mg prednisone and
decrese the dose 5 mg every 3 days until 5 mg for 7 days, if you have 5 mg tablets, how many
tablets will you dispense:
a- 49 tablets
b- 80 tablets
c- 96 tablets
d‐ 112 tablets

248- a new drug make cure in 20 % of patients in comparing with controlled drug which
make cure in 16% of patients, when improvement percentage is 25% this improvement is:
a‐ Relative improvement Relative risk reduction
b- Absolute improvement
c- Mean improvement

249- net profit of a pharmacy is 90.000$ per year, estimate its value assuming a rate of
return in net profit of 15%: 2011, 2007
a- 13500$
b- 90.000$
c- 480.000$
d‐ 600.000$ 90000/0.15 = 600000

250- you have to make a camphor ointment in petrolatum base 95%, you have only a
concentration of 65% and you have the following:
Camphor 65% 1.2 ml
Water up to 60 ml
What is the amount of camphor 95% you will need?
a- 0.5 mg
b‐ 0.8 mg
c- 1.2 mg
65×1.2=#×95 #= 0.8

251- a pharmacist decrease the expenses of his pharmacy by 12000 $ in compared with the
last year while achieving the same sales, what will be the percent of increasing net profit if
you know that: 2015
-Sales= 1.200.000 $ -Costs = 1.050.000 $ -Net profit this year = 120.000$
a- 6%
b‐ 10% same idea of the mentioned solution
c- 20%
d- 22%
e- 25%

252- anaphylactic shock can be treated with:


a- Penicillin
b‐ Epipen

253- a woman on levothyroxine and became pregnant, what should be done regarding to
dose of levothyroxine:
a- Stop taking levothyroxine
b- Decrease dose of levothyroxine
c‐ Increase dose of levothyroxine

254- which vitamin has a hormone origin: 2013,2009


a- Vitamin A
b- Vitamin B
c- Vitamin C
d‐ Vitamin D
e- Vitamin E

255- using fibers for the treatment of constipation what is true:


a‐ Taken with full glass of water
b- It doesn’t need much of water.

256‐who’s body contain more water?


a‐ neonates
b- elderly
c- young

257- Early warning signs of ischemic stroke include:


a) Hearing loss
b) Shoulder pain
c) Chest pain
d) Knee pain
e) Confusion

258- A pharmacist Joe is being restricted from practice because of history of substance
abuse. Based on what college make this dicission?
A. Incompetent
B. Incapacity
C. Negligence
D. Misconduct
E. accountability
July 2015

1) Which of the following causes vasodilation? jan 2015


a) CO
b) CO2
c) NO
d) NO2

2) Test to determine SPF value is done by:


a) UVA
b) UVB
c) UVC
d) Infrared
e) Visible light

3) Which is a nominal or categorical variable? (the other options are continuous


variables)
a) Weight
b) Height
c) Eye color
d) Range of movement
e) Temperature

4) Nephrolithiasis is caused by:2016


a) Calcium
b) Iron
c) Magnesium

5) This the structure of Sumatriptan, it is related to:


a) Tryptamine

6) Woman prescribed Codeine 200 pills/week complain of no action despite exceeding


the dose because:
a) Mild renal failure
b) Mild hepatic failure
c) Poor Codeine metabolizer
d) High Codeine metabolizer
(Note: there was no tolerance in the choices)
7) Drug A has 1% mortality; Drug B has 1.5% mortality. The relative risk drug A to B with
95% confidence interval is 0.64 (0.32-1.02).
a) Drug A is better than drug B and the clinical finding is significant
b) Drug B is better than drug A and the clinical finding is significant
c) Drug A is better than drug B and the clinical finding is non-significant
d) Drug B is better than drug A and the clinical finding is non-significant
Drug A is better than B and the clinical finding is significant, significance could be
concluded as long as the relative risk value lies in the confidence interval range. That
means we are by 95% confident that the relative risk lies between 0.32 and 1.02 and in
this case, it is equal to 0.64 so it is significant

8) Dose of drug = 160mg. Drug follow 1st order kinetics.


Time h Concentration mg/L
2 4
4 2
What is the volume of distribution?
a) 10
b) 20
c) 40
d) 50
e) 60
Volume of Distribution = Total Dose / Initial Concentration = 160 / 8 = 20
The conc after the 1st half life (2 hrs) is 4  so, the initial conc is 8 as the drug follow 1st
order

9) What is the drug of choice of myoclonic seizures?


a) Phenytoin
b) Gabapentin
c) Valproic acid
d) Ethosuximide
e) Topiramate

10) Patient with gonorrhea is treated with?


a) Doxycycline
b) Fluoroquinolone
c) Cefixime + Azithromycin
11) This is the structure of Clomiphene, it is found in a mixture with a similar
compound, that compound is its:
a) Structural isomer
b) Cis trans isomer
c) Tautomer

12) Mechanism of action of dorzolamide: carbonic anhydrase inhibitor


a) decreases aqueous humor secretion
b) increase aqueous humor drainage

13) Bone in the head:


a) Sphenoid
b) Scaphoid  hand

14) Which drug can cause Jaundice?


a) Isoniazid
b) Isotretinoin

15) Tacrolimus mechanism of action:


An immunosuppressive drug used mainly after allogeneic organ transplant to lower the
risk of organ rejection. It achieves this by inhibiting the production of interleukin‐2, a
molecule that promotes the development and proliferation of T cells, which are vital to the
body's learned (or adaptive) immune response.

16) Methadone is a:
a) Straight narcotic
b) Preparation narcotic
c) Controlled 2substance
d) Targeted substance

17) Which antibiotic change the urine color?


a) Ciprofloxacin
b) Norfloxacin
c) Nitrofurantoin
d) Doxycycline

18) Which drug cannot be taken on empty stomach?


a) Alendronate
b) Sucralfate
c) Omeprazole
d) Norfloxacin
e) Diclofenac

19) Which oral drug improves osteoporosis and prevents both vertebral and skeletal
fractures?
a) Alendronate ‐ Residronate also (1st choice)
b) Zoledronate
c) Etidronate
d) Raloxifene

20) Patient with hysterectomy and hot flashes is treated with:


a) Oral estrogen
b) Oral estrogen + progestin
c) Vaginal estrogen
d) Progestin only
A hysterectomy means she removed her uterus so in this case w can use estrogen only ok
and hot flashes means she has systemic symptoms so we use oral not topical

21) Insulin chains are linked by: Di sulphide linkage

22) What is true about the chemical structure of Immunoglobulin?


a) Contains heavy and light chain connected by disulfide bond
b) Consists of four subunits connected together by hydrogen bond
Antibody (or immunoglobulin) molecules are glycoproteins composed of one or more
units, each containing four polypeptide chains: two identical heavy chains (H) and two
identical light chains (L). Heavy and light chains are held together by a combination of
non-covalent interactions and covalent interchain disulfide bonds, forming a bilaterally
symmetric structure.

23) Which component is essential in PCR? polymerase chain reaction


a) Nucleotides
b) Endonucleases
c) RNA template

24) Which cause alopecia? Cyclophosphamide


25) Which is NOT an STD? SEXUALLY TRANSMITTED DISEASE
a) Hep B
b) HIV
c) Clamydia
d) Clostridium perfringens
e) Gonorrhea

26) What causes community acquired meningitis in Adults?


a) Streptococcus pneumonia
b) H. Influenza  only in Children ≥1 month
c) Strep. Pyogens
d) Moraxella Catarahlis

27) Which disease decrease blinking rate? EYELID MOVEMENT


a) Parkinson’s disease
b) Shingles
c) Hyperthyroidism
d) Rheumatoid arthritis
e) Psoriasis

28) What is correct about the guidelines of public health immunization?


a) Pneumococcal vaccine every year
b) Tetanus every 2 year
c) Meningococcal after injury
d) Measles, diphtheria, booster in adult

29) Risk factor in osteoporosis:


a) Obesity
b) Hyperthyroidism
c) Type one diabetes
d) Osteoarthritis

30) First line treatment for osteoarthritis: Acetaminophen

31) Risk factor in Osteoarthritis:


a) Obesity
b) Alcohol
c) Caffeine
d) Smoking
e) Aerobic Exercises

32) Tumor lysis syndrome causes:


a) hyperuricemia
b) hypokalemia
c) hypernatremia
d) hypoglycaemia
the syndrome is characterized by rapid development of hyperuricemia, hyperkalemia,
hyperphosphatemia, hypocalcemia, and acute renal failure

33) All cause metabolic acidosis except:


a) Vomiting in vomiting, the bicarbonates (mainly pancreatic and bile ‐ both are
alkaline), are lost...so this will give acidosis.... Same goes for diarrhea
b) Diarrhea Vomiting causes metabolic alkalosis while diarrhea causes metabolic
acidosis
c) Bicarbonate reabsorption
d) Decrease H+execration
e) Diabetes

34) What improves GERD symptoms and disease progress?


a) Misoprostol
b) Ranitidine
c) CaCO3
d) Sucralfate

35) A pharmacist knows that another pharmacist is doing something wrong in a


neighboring pharmacy.
a) Report him to the provincial authority
b) Report him to the board of pharmacist
c) Report him to the neighboring pharmacy manager

36) Pregabalin causes all EXCEPT:


a) Dependence
b) Weight loss
Sedation, dizziness, cognitive impairment, dry mouth, peripheral edema, weight
gain.
37) What is true about intranasal route?
a) Bypass first pass effect
b) No enzymatic metabolism
c) Used only for local action
d) Its bioavailability is equal to the oral rout of administration

38) Woman taking combined oral contraceptive come to the pharmacy complaining
about nausea, what is appropriate counseling:
a) Refer to doctor for further assessment because COC doesn’t cause nausea.
b) Tell patient nausea will go away in a year
c) Tell her it will go away in 3 months and to visit the doctor and ask for higher dose if
it doesn’t.
d) Tell her it will go away in 3 months.
e) Tell her the nausea will never go away as long as she taking COC

39) We take metformin with food to decrease risk of:


a) Lactic acidosis
b) Hypoglycemia
c) GIT symptoms

40) Registered Nurse at STD clinic wrote a patient an IM antibiotic, what do you do?
a) Tell the patient to get prescription from a physician
b) Write the patient a similar prescription following the pharmacist extended scope of
practice
c) Report nurse for going outside scope of her practice
d) Ask that the prescription be cosigned by the head nurse
e) Dispense prescription after usual check.

41) A caregiver for a cancer patient that comes with prescription for 2mg
hydromorphone and lactulose. You know this caregiver. Patient is now taking
sennosides with codeine/caffeine/acetaminophen. Your main concern as a pharmacist
is?
a) Effective pain control of this patient
b) The duplicate therapy of sennosides and lactulose
c) To verify if the prescription is a fake

42) HAMA in immunology stands for: Human anti‐mouse antibody


43) Taq DNA polymerase is the best used in PCR because of:
a) Great resistance to heat (THERMUS AQUATICUS BACTERIA)
b) It is fast
c) buffer

44) Net profit of a pharmacy is 90 000$ per year. Estimate its value assuming a rate of
return of 15%.
a) 13500
b) 90000
c) 600000 90000/0.15 = 600000

45) 1000 mg in 250 ml infused intravenously


in 4 hours. What is the rate of infusion in
mg/min?
1ml contain 4mg 250ml/240min=1.04
*4mg=4.17mg/min

46) What is the name of the reaction from


cholesterol to cholate bile acid?
a) Hydroxylase BY 7 ALPHA
HYDROXYLASES
b) Lipase
c) Oxidase
d) Desmolase

47) Pharmacist decided to provide a patient with a service covering only the costs.
Assuming the patient had this service 10 times per month. What of the following is
true?
a) The pharmacy had an increase in net profit
b) The pharmacy had an increase in revenue
c) The pharmacy had a decrease in inventory

48) Which of the following is responsible for motor function?


a) Frontal lobe conginitive function too by frontal
b) Parietal lobe
c) Occipital lobe
49) Woman of child bearing age on methotrexate, what is the recommended dose of
folic acid?
a) 5mg Per week
b) 3mg
c) 1mg

50) The Number of Theoretical Plates in HPLC indicate: 2012


measurement of column efficacy depends upon the substance being chromatographed
as well as the operating conditions, such as the flow rate and temperature of the mobile
phase or carrier gas. Used to Separate equilibrations of the sample between the
stationary and mobile phase.

51) Patient with early Alzheimer’s came with a prescription for donepezil. What is the
appropriate method to counsel this patient?
a) Limit the verbal interaction and give him written information
b) Ask if you can speak with a caretaker
c) Talk loudly and repeatedly
d) Counsel as usual

52) The following compounds are:


a) Enantiomers
b) Diastereomers
c) Geometrical isomers
d) Racemates

53) Phenelzine is a:
a) SSRI
b) TCA
c) MAOI

54) Regarding Phenelzine. What counseling point should you tell the patient?
a) Avoid consuming foods such as aged cheese

55) Patient taking Phenelzine, has cough. He can take:


a) Guaifenesin
b) Pseudoephedrine
c) Dextromethorphan
d) Xylometazoline
56) What is responsible of balance in the brain? 2012
a) Cerebellum
b) Hypothalamus

57) Phrenic nerve damage cause:


a) Stop of breathing
b) Inability to bend hip
c) Inability to stretch leg

58) What causes systemic lupus erythematosus? Hydralazine HIPP MCQ

59) Patient have rash while on allopurinol. An alternative for this patient is:
a) Febuxostat

60) If you put vaccines in your pharmacy, this is:


a) Primary prevention b) Secondary prevention

61) Opposite of lateral plane of anatomy:


a) Proximal
Medial×lateral
b) Medial
Frontal ×dorsal
c) Dorsal
Vertical×horizontal
d) Ventral
Proximal ×distal
e) Distal

62) Protrusion over the stomach: Hiatus hernia

63) The most that contribute to the production of seminal fluid:


a) Epididymis
b) Seminal vesicles
c) Corpus cavernosum
d) Bartholin gland

64) Function of Corpus callosum:


a) Connect the 2 lobs of the brain
b) Separates the spine and the brain
c) Separates the parietal and frontal lobe
65) What is true about the cerebrospinal fluid? (protect brain lobes)
a) Leukocytes are normally present
b) Produced from the choroid plexus (EPENDYMAL CELLS)

66) Duschenne muscular dystrophy syndrome is cause by:


a) Dopamine 4 receptor gene
b) Dystrophin 1 gene progressive muscle degeneration and weakness
c) Dystonia gene
d) Diciclogene 1

67) Which if taken with iron salts will increase its absorption?
a) Orange juice Acidic medium is required to convert ferrous into Ferric ion,
which is readily absorbed.
b) Antacid.
c) Food

68) Dexamethasone improve outcome of:


a) Endocarditis
b) Infective arthritis
c) Meningitis

69) Last year end inventory is 230 000. Starting inventory in the new year is:
a) 230 000

70) Which of the following is prescription medication?


a) Mupirocin
Both A & B are schedule 1..
b) Fusidic acid I checked the NAPRA website

c) ASA
d) Hyaluronic acid

71) Amphetamine mechanism of action: Indirect acting prevent reuptake to the vesicles
effects by elevating extracellular dopamine (DA) and prolonging DA receptor signaling in
the striatum. Traditionally, AMPH has been characterized as a DA releaser that elevates
DA by three major mechanisms. First, it is a substrate for the DA transporter (DAT) that
competitively inhibits DA uptake; second, it facilitates the movement of DA out of vesicles
and into the cytoplasm; and third, it promotes DAT-mediated reverse-transport of DA into
the synaptic cleft independently of action-potential-induced vesicular release
72) Fentanyl act on:2012,2016
a) Mu
b) Delta
c) Kappa
d) Mu and delta
e) Delta and kappa

73) A female patient with DVT. She can take all of the following EXCEPT: Estrogen

74) B amyloid protein is present in:2014,2013


a) Alzheimer’s
b) Parkinson’s

75) Rivastigmine is used in all EXCEPT:


a) Delirium
b) Alzheimer’s dementia
c) 3 other types of dementia

76) Which CANNOT be used in a pregnant woman with hypertension?


a) Clonidine
b) Methyl dopa
c) Hydralazine
d) Ramipril
e) labetalol

77) Which of the following cause angioedema?


a) ACEI
b) ARB
c) CCB
d) Thiazide

78) Which is NOT used in atrial fibrillation rate control?


a) Amiodarone
b) Amlodipine It can cause tachycardia
c) Verapamil
d) Digoxin
79) Patient with sleep apnea is at high risk of:
a) Stroke
b) Seizures
c) Venous embolism

80) Patient over 50 years old has shingles. He should take antiviral:
a) Within 72 hour of skin rash
b) After herpetic neurologic pain
c) After skin crusts rupture
d) Within 7 days of skin rash

81) Protease is under which enzyme class?


a) Lyase
b) Hydrolase proteases/ peptidases, lipases, nucleases
c) Esterase

82) High uric acid in patients for a long-time cause:


a) Osteoarthritis
b) Tophi
c) Osteoporosis

83) All can cause loose stool except?


a) Phenytoin – CAUSES CONSTIPATION
b) Meropenem
c) Methotrxate

84) Which drug increase fracture risk? Anastrazole

85) Natalizumab is used for multiple sclerosis to:


a) Prevent relapse
b) Decrease symptoms
c) Cure the disease

86) New drug cured disease in 20%, old dug cured disease in 16%. The manufacture of
the new drug promotes 25% improvement. This improvement is
a) Relative improvement comparing one drug with another drug in the same
dosage form.
b) Absolute improvement comparison of any drug with the same drug IV dos
87) Bioavailability represents:
a) Rate and extent of absorption
b) Maximum concentration in plasma

88) What is NOT a symptom of diabetic foot?


a) Neuropathic pain
b) Opening of skin
c) Erythema
d) Swelling
e) Purulent discharge

89) Isotretinoin monitoring: jan 2015, 2013


a) Liver and lipid
b) Glucose and Hemoglobin

90) Best way to prevent migraines: Decrease the triggers

91) Isotretinoin advice: Put sun screen

92) The counseling for patient taking Sertraline, that Sertraline causes:
a) Sexual dysfunction
b) Hyperstimulation

93) Patient taking carbamazepine, should monitor


a) White blood cells
b) Anemia

94) Absence seizures drug of choice: Ethosuximide

95) Insomnia is a side effect of:


a) Bupropion
b) Mirtazapine
c) Trazodone

96) Resistant schizophrenia drug of choice:


a) Quetiapine
b) Clozapine
97) Schizophrenia in a diabetic patient can be treated by all of the following except:
a) Olanzapine
b) Haloperidol
c) Resperidone
d) Aripiprazole

98) Test for TSH and T3 and T4 for assessing treatment every
a) 2-4 week
b) 4‐6 week
c) 3 months
d) 4 months

99) ISMP is concerned about which of the following abbreviations:


a) cc  Cubic centimeters Mistaken as “u” (units)
b) qid
c) PRN
d) L
e) Bid

100) Granisetron inhibit which receptor


a) Dopamine
b) Serotonin
c) GABA
d) Glutamate

101) Vitamin A is synthesized from beta-carotene by: 2012,2014

a) Oxidative cleavage
b) Hydroxylation

102) Which incorporates glycine as an intermediate complex in metabolism?


a) P450
b) Acetyl Co‐A
103) Which is used to assess HIV disease progression?
a) Plasma viral load
b) Tuberculin skin test
c) Chest x-ray
(CD4 was not in the choices)

104) Indinavir is an enzyme inhibitor. Fentanyl taken with indinavir. Then Fentanyl:2012
a) Will have decreased metabolism
b) Will have increased metabolism
c) Will have decreased plasma level

105) Methylphenidate monitor:


a) Weight
Weight loss
b) CBC

106) Digoxin + Clarithromycin


a) Increased Digoxin absorption The serum concentration of Digoxin can be increased when it is combined
with Clarithromycin. May lead to toxicity
b) Decreased Digoxin absorption

107) Patient has CHF. What improves survival with negative inotropic effect?
a) Carvedilol
b) Captopril
c) Digoxin
d) Amlodipine

108) According to ISMP, which of the following drugs have high risk of incorrect dose?
a) Methotrexate

109) Which of the following drugs cause sexual dysfunction?


a) Pantoprazole
b) Atenolol

110) Pregnant women with acute uncomplicated cystitis and sulfa allergy. What
medication she should take for 3 days? 2013
a) Ciprofloxacin
b) Moxifloxacin
c) Cotrimoxazole
d) Cephalexin
111) Drug that cause QT prolongation: Clarithromycin

112) Disulfiram reaction Metronidazole

113) All used to treat vaginal candidiasis EXCEPT: 2013,2012


a) Metronidazole
b) Other antifungal azoles

114) Treatment of Chlamydia: Azithromycin

115) Patient with Crohn’s disease has colovaginal fistula. It is treated by:
a) Metronidazole
b) Vancomycin
c) Cotrimoxazole
d) Amoxicillin

116) Sensitivity is:2013


a) Real positive test results in people who have the disease

117) Which if the following is a proven case for prevalence:(means current situation of
disease)
a) 117 died from community of 300 infected by salmonella
b) 1021 died in 2010
c) Diabetic patients in Canada
Tips: A is cross section data B is case control data & Because a prevalence study is a
cross‐sectional study, should be in the present time and should be pointed at a
special population.

118) Mutation of which enzyme occurs in Enalapril?


a) Angiotensin converting enzyme
b) CYP450

119) All the following organisms cause conjunctivitis EXCEPT:


a) B. fragilis
b) Chlamydia trachomatis
c) N. gonorrhea
120) Marijuana initial symptom: Medical Access Regulations:(Marihuana = controlled
substances)2012,2014
a) Mydriasis
b) Conjunctival reddening
c) Diarrhea
d) Shortness of breath

121) To avoid lactic acidosis, which is NOT a contraindication of metformin?2013


2012,2009
a) Chronic alcoholism
b) Dehydration
c) Concurrent Insulin use
d) Hepatic failure
e) Renal failure

122) Side effect of acarbose:


a) Flatulence
b) Constipation

123) What is used for glaucoma?


a) Dorzolamide
b) HCTZ
Tips Oral drug for glaucoma acetazolamide

124) NPH insulin is taken: 2013,2014


a) Twice daily
b) Once daily
c) Before food
d) After food

125) Quaternary ammonium compounds are used as surfactant and:


a) Antimicrobial

126) Proteins biotechnology products are put in all except: (contaminated by culture
media)
a) Capsule
b) Microsphere
c) Nanoparticle
d) Lysosomes
Tips: Drugs made from biotechnology techniques are all the following, EXCEPT:
a) Human insulin
b) Hepatitis B vaccine
c) Filgrastin
d) Interferon
e) Heparin

127) Gauge number indicates (SMALL GAUGE NUMBER INDICATES LARGE OUTER
DIAMETER)
a) Outer diameter of needle
b) Inner diameter of needle For me it is inner diameter
c) Bevel size
d) Syringeability
Tips: if say gauge number is outer but if say gauge number and thickness is
inner

128) What is true about ocular suspensions:


a) Ocular suspension has faster action than solution
b) Nonaqueous and non-aqueous are used
c) Solution is more effective than suspension
d) Agitation to form homogeneity and uniformity of resultant particles

129) Vial with biotechnology medication after reconstitution was foaming:


a) Wait until it settles
b) Withdraw solution without foam
c) Shake vigorously till foam disappears
Tips: For biotechnology products and others of protein nature. If foam occurs, u
must wait because if foam doesn't subside most probably the product denaturated
and shouldn't be injected.

130) For long term effective gene therapy, gene needs to be:
a) Integrated into nuclear membrane
b) Fully integrated into chromosome

131) Levonorgestrel taken as emergency contraceptive causes: Breast tenderness


132) In studying relation between cancer and chemicals. We took a sample of the
population exposed to chemical hazard and continued to follow them for ten years.
This is a:2013
a) Cohort study
b) Case control study
c) Randomized study
d) Cross sectional study

133) Best possible medical evidence is:


a) Systemic review of cohort studies
b) Systemic review of randomized controlled trials
c) Randomised controlled trials

134) Use of hospital formulary (What is the objective of the hospital formulary?)
a) To help physicians to know what the cost‐effective medications
b) To manage the inventory of the pharmacy
c) To help the pharmaceutical companies

135) Drug A cost $1200 and 80% effective and Drug B cost $1200 and 80% effective.
What pharmacoeconomic analysis help to determine the better drug for hospital
formulary?
A) CUA B) CMA C) CBA d) COI E) CEA
Tips: When therapy has same cost and same result then alway check quality
adjusted life year gain. In this cas if we had different cost of therapy and same
outcome then CMM

136) Drug when administered 5mg, its AUC 0.5mg/L. Drug when administered 10mg, its
AUC 3mg/L. it is because: Saturation of liver enzymes

137) Patient you know want to be served first. Tell him to wait

138) Patient wants you to wave him copayment. If you do you violate
a) Justice
b) Veracity
c) fidelity

139) Count cis Trans+ chiral carbons in the structure. it was:


a) 1 cis trans + 2 chiral carbons
140) Terminal NH2 is metabolized by: Acetylation (The only phase II reaction in the
choices)

141) Which is NOT metabolized by glucuronidation?


a) NO2
b) NH2
c) SH
d) OH phenolic
e) OH Alcohlic

142) Structure with –SH changed to –SCH3. The name of reaction is: Transmethylation

143) Which of the following is classified as antimetabolite? 5‐fluorouracil

144) How many chromosomes from one parent?


a) 22
b) 23

145) Which of the following drug causes Steven Jonson syndrome?2013


a) Sulfamethoxazole SASPAN
b) valproic acid
c) isoniazid

146) Neutrophils’ first immune reaction:


a) phagocytosis by activated macrophage
b) presenting antigen
c) engulfing pathogen

147) What is a vitamin K reductase inhibitor? warfarin

148) Dabigatran mechanism of action: direct thrombin inhibitor

149) Anti-hypertensive drug for diabetic patient: ACE i

150) K-type: Sympathetic system affect all of the following


i. Decrease heart rate
ii. Facilitate AV node conductivity
iii. Increase contractility
151) Patient with MI history and no risk for GI stress, which NSAIDs is appropriate for
him?
a) Celecoxib
b) Ibuprofen
c) Naproxen
TIPS: Very important note
Safest NSAIDS when there is heart problem is naproxen
Cox 2 inhibitors the safest on GIT but cause heart problem
What is the safest NSAIDS when there is kidney problem? Sulindac

152) Flumazenil is an antidote for:2011


a) Benzodiazepines
b) Barbiturates

153) Patient treated with Lithium we should monitor:2013,2012


a) TSH
b) Glucose
c) Lipid

154) Patient taking isoniazid has neurotoxicity treated with:


a) Pyridoxine
b) Cobalamin
c) Niacin

155) Which vitamin essential for collagen synthesis?


a) Vit C
b) KERATIN

156) Which drug is taken after food to increase release of insulin?2016,2013,2010


a) Nateglinide Taken just before meal, skip medication if skipped meal
b) Metformin
c) Acarbose
d) Pioglitazone Medication that work by stimulating insulin release after meal

157) Which of the following is a lubricant in tablets?


a) Cellulose
b) Potato starch
c) Croscarmellose
158) Angle of repose is used to measure:
a) Flowability
b) Solubility
c) Dissolution
d) stability

159) The use of glycerin in ointment:


a) Solubilizer
b) Levigating agent
c) Emulsifier

160) Most common organism in COPD exacerbation:


a) Staph aureus
b) Strep. Pneumonia
c) Pseudomonas aeruginosa

161) Patient taking salbutamol and newly diagnosed with emphysema. Which is the
appropriate drug to add?
a) Formoterol
B then A
b) Ipratropium
c) Fluticasone

162) In acute asthma all are used except


a) Theophylline
b) Fluticasone
c) salbutamol

163) What is extravasation?


a) Leaking of blood into surrounding tissue
b) Leaking of IV fluid into surrounding tissue

164) What is involved in the reaction from kinase to phosphokinase?


a) ADP
b) ATP
c) cGMP

165) What is used for the prevention of stroke with patient with a history of atrial
fibrillation?
a) ASA
b) ASA +Dipyridamole
c) Warfarin
d) Clopidogrel

166) What would increase solid solubility in solvent?


a) Increase in solvent viscosity
b) Increase in solid particle size
c) Increase in pH
d) Increase in solid surface area
e) Decrease in temperature

167) When to repeat the treatment of head lice? After 7‐10 days

168) What disease treatment should be applied to the entirehousehold?


a) Scabies  also pinworm
b) Impetigo
c) Lice

169) Which is a primary source?


a) Clinical study
b) PubMed
c) Compendium of Pharmaceuticals and Specialties
d) Cochrane Review

170) Doctor is calling to ask about evidence-based information about cholestyramine. He


needs it immediately. Where do you look?
a) Compendium of Pharmaceuticals and Specialties Detailed monographs for all
prescription pharmaceuticals available in Canada
b) Cochrane Review

171) What characteristic should the drug have to pass into breast milk?
a) Acidic
b) Highly bound to plasma protein
c) Low lipid solubility
d) Low molecular weight

172) Anastrazole side effect: Hot flushes


173) When should we give DVT prophylaxis? After hip replacement surgery

174) When a patient takes a toxic dose of a sustained release product we should:
a) Give him psyllium
b) Give him Kaolin
c) Give him Bismuth
d) Give him Senna
e) Do a bowel irrigation

175) Drug A and drug B are given intravenously. Dose of drug A = Dose of drug B. But
AUC of drug A > AUC of drug B because of:
a) Clearance of drug B is more than drug A
b) Clearance of drug A is more than drug B
c) Drug A is more absorbed then drug B
Tips: The AUC is directly proportional to the dose when the drug follows linear
kinetics. The AUC is inversely proportional to the clearance of the drug.

176) What is a bacteriophage?2013


a) Bacteria that infect human
b) Bacteria that attack virus
c) Virus that infect bacteria

177) Who can benefit most from prefilled insulin syringes?


a) Patient taking high dose of insulin
b) Patient taking mixed insulin
c) Visually impaired patients

178) Patient taking NSAID for his rheumatoid arthritis. He can take all of the following
drugs except:
a) Sulfasalazine
b) 5-ASA
c) Etanercept
d) Hydroxychloroquine
e) Leflunomide
Tips: all these DMARD taken in rheamtoid and NSAD used as bridging therapy for
short time in order to give chance for the medicines to work but 5‐ASA used for IBS.
179) What is true about megaloblastic anemia?
a) It is caused by deficiency in folic acid or B12
b) It is treated by iron

180) What is the mechanism of action of bisphosphonate?


a) inhibits osteoclast’s bone resorption
b) inhibits osteoblast’s bone resorption

181) What are the aspects of schizophrenia diagnosis?


a) Hallucinations + delusions
b) Hallucinations + motor dysfunction

182) If there is a lack of one essential amino acid in the diet, the human body will:
a) Synthesis the amino acid
b) Breakdown a protein

183) What is the symptom or patient complaint that makes us look for prostate cancer?
NOCTURIA, WEAK URINARY STREAM, POST VOID DRIPPING AND URINARY
RETENTION

184) What is equal in amount to Guanine?


a) Uracil
b) Adenine
c) Cytosine

185) What of the following is a property of ophthalmic solution used in surgery?2016


a) Isotonic with normal saline
b) Packed into a single dose container

186) Patient with blood type A has a Rh antigen. That makes him:
a) A positive
b) A negative

187) Naltrexone is used to treat:


a) Alcohol dependence
b) Benzodiazepine dependence
188) What is true about acne rosacea?
a) Characteristic dilation of the capillaries
b) Common in <30 years old

189) Cisplatin mechanism of action:


a) Alkylating agent
b) Antimetabolite

190) Cisplatin bond type with DNA: ALKYL BOND WITH CELLULAR COMPONENT OF
DNA

191) Know what drugs are used in treatment of chronic renal failure but contraindicated
in acute renal failure and what drugs used to treat acute renal failure

192) Herbal medication used for rheumatoid arthritis is:


a) Evening primrose oil
b) Feverfew

193) Rickettsia cause: Rocky Mountain Spotted Fever

194) What is true about testosterone prescriptions?


a) Written prescription only, no verbal prescription accepted
b) Verbal prescription accepted with no refills
c) Verbal prescription accepted with refills

195) What is true about benzodiazepines prescriptions?


a) Verbal and written Rx with refills accepted
Benzodiazepine is controlled substance, send patient to the doctor so he
assesses patient, he needs refill or not if patient needs refill then Dr can write
about early refill in prescription

196) Which of the following is a non-essential Amino acid?2011,2009


ESSENTIAL PVT TIMHALL

197) We can give codeine as antitussive starting from age:


a) 10
b) 12
c) 14
198) Which can be used as prophylaxis for osteoporosis? 2012 Raloxifene

199) Raloxifene mechanism of action:


SELECTIVE ESTROGEN RECEPTOR MODULATOR SEMR
SERM

200) Disulfiram reaction happen while taking: Metronidazole

201) Which drug can cause venous pooling? NITRATES

202) The drug of choice in dystonia: JAN 2015 Benztropine

203) All cause QT prolongation EXCEPT: tetracycline

204) All can decrease errors in pharmacy EXCEPT? Decrease number of technicians

205) Know the new drug licensing


process

206) What is true about phase 2


study?
a) Testing is done on 300
people with the disease to
assess efficacy and safety

207) Which NSAIDs NOT used in treatment of rheumatoid arthritis?


a) Ibuprofen I don’t think it’s right question but we should note that selective cox2 has
b) Acetaminophen poor role in acute pain management so if question is about acute pain we
can exclude celecoxib
c) Naproxen
d) Celecoxib

208) Aspirin increase side effects of: Methotrexate

209) Drug excreted 10 % unchanged. When patient with 50% renal function is taking this
drug. What should be done to the dose?
a) No change in dose or dose interval
b) Increase the dose interval of the drug
c) Decrease the dose of the drug
210) Alpha is: The probability that we have a type I error

211) Which is not considered a body physiological barrier?


a) Skin  anatomical (physical) barrier
b) Mucous membrane
None of these
c) Tears The fifth choice : may be inflammation
d) Gastric secretions barriers include the skin, mucous membranes, tears, earwax, mucus, and
stomach acid.

212) Which patient is more at risk of hypoglycemia?


a) Patient taking insulin before meals
b) Patient eating small meals instead of large meals
c) Patient taking insulin before exercise

213) Patient taking rosiglitazone needs to monitor:


a) Renal function
b) Liver function
c) Cardiac function
January 2015

1) Which of the following is one of the ankle bones?


a) Carpals
b) Tarsus
c) Patella

2) Intaraarticular injection is in: 2008


a) Bone
b) Joint
c) Spine
d) heart
e) artery

3) Which of the following organs found in the left side of the human body?
a) Liver
b) Spleen
c) Appendix
d) Bladder

4) Milk allergy is an adverse immune reaction to :2013,2009


a) Calcium
b) Lactose
c) Protein (caseine)
d) Vitamin D

5) When a vagal stimulation occurs, which of these substances will be released?


a) Dopamine
b) Serotonin
c) Insulin
d) Vasopressin
e) Acetylcholine

6) Famciclovir is used in:


a) Herpes simplex
b) west Nile virus
c) Bacterial infection
d) Fungal infection
7) All of the following are a sexual-transmitted disease Except:
a) Trichomoniasis
b) Herpes simplex
c) Herpes Zoster
d) Gonorrhea

8) The most common GIT disorder is:


a) Zolinger-Ellison syndrome
b) Peptic Ulcer
c) GastroEsophogeal Reflux Disease
d) Acid Hyper secretion
e) Ulcerative Colitis

9) For a patient taking Acetylsalicylic Acid 350 mg, which of these patients is the least to
cause gastric bleeding?
a) Take it with food
b) Patient over 70 years old
c) Taken Naproxen at the same time

10) All of the following can be taken with or after food Except:
a) Sucralfate
b) Predinsolone
c) Losartan
d) Carbamazipine

11) Which of the following medication for treatment of glaucoma is taken orally?
a) Acetazolamide
b) Dorzalomide
c) pilocarpine
d) latanoprost

12) Which of these medications cause iris pigmentation?


a) Carbonic anhydrase inhibitors
b) Prostaglandins F2 α analog Eg Latanoprost

c) Adrenergic antagonist
d) Cholinergic agonist
13) Linoleic acid & Linolenic acid are structurally differing in: 2010
a) Number of double bonds
b) Stereochemistry
(N.B: The position of the double bond was not one of the choices)

14) Which of the following is not a symptom of Rheumatoid arthritis?


a) Muscle pain
b) Fatigue
c) morning stiffness
d) swelling

15) H. Pylori treatment include all the following Except :2012,2011,2013


a) Metronidazole
b) Clarithromycin
c) Tetracycline
d) Vancomycin

16) All are used in treatment of acute gout Except :2013


a) Acetaminophen
b) Prednisone
c) Celecoxib
d) Colchicines
e) Indomethacin

17) Lysosomes are responsible for cellular:


a) Protein formation
b) Energy production
c) Enzyme digestion
d) Transport of DNA
e) Permeability
18) Mitochondria is an organelle that responsible for:
a) Formation of protein
b) Production of ATP
c) Digestion of enzymes
d) Transportation of DNA

19) Which of the following enzymes is responsible for phosphorylation of amino acids?
a) Kinase
b) Lyase
c) Esterase
d) Synthetase

20) How many cis double bonds can be


found in that structure?
a) 1
b) 2
c) 3
d) 4

21) Metformin is found in 2 chemical structure which are: 2014

&
what is the relationship in between these 2 structures?
a) Tautomers
b) Conjugate acid/base
c) Polymorphism
d) Structural isomers
Tautomers are isomers of a compound which differ only in the position of the protons
and electrons. The carbon skeleton of the compound is unchanged. A reaction which
involves simple proton transfer in an intramolecular fashion is called a tautomerism.

22) This structure is? 2012


a) Phenothiazine
b) Tricyclic Antidepressant
c) Benzodiazepines
d) Antihistaminic
23) During the metabolism of purine bases which of these compounds is formed? 2013
a) Hypoxanthine
b) Azathioprine
c) Mercaptopurine

24) A patient with high urinary Urobilinogen, that indicate a disease in:
a) Bladder
b) Kidney
c) Liver
d) Heart
e) Pancreas

25) Oral Vancomycin is used in case of:


a) MRSA – IV vancomycin
b) Pseudomembraneous colitis caused by Clostridium difficile

26) Which of these vaccines is advised to prevent traveller diarrhea?


a) Cholera vaccine
b) Hepatitis A Vaccine
c) Hepatitis B Vaccine
d) Rota Vaccine
e) Gardasil Vaccine

27) The most common cause of Cervix epithelia cancer is? 2012
a) Human papilloma virus
b) Having no children
c) Multiple birth

28) Pneumonia-causing bacteriapathogens include all of the below Except :


a) Streptococcus pneumoniae
b) Moraxella catarrhalis
c) Bacteroides fragilis
d) Haemophilus influenzae

29) Which of the following is a symptom of chronic bronchitis?


a) Productive sputum cough
b) Dyspenia
c) Non productive cough
30) What is the treatment of chronic sinusitis? 2014
a) Amoxicillin-clavulanic acid
b) Levofloxacin
c) Azithromycin

31) A 30-year-old man has gonorrhea, what is the appropriate treatment?


a) Cefixime + azithromycin
b) Ceftriaxone
c) Nitrofurantoin
d) Ampicillin
Canadian STI guidelines list ceftriaxone (in combination with azithromycin) as a
preferred treatment for pharyngeal infections.

32) A 60 years old woman with gout, which of the following antihypertensive agent
must not be used:
a) Hydrochlorothiazide
b) ACE-I
c) ARBs
d) B-Blockers

33) what antihypertensive agent used for mild to moderate hypertensive patient with
congestive heart failure:
a) Nifedipine
b) Verapamil
c) Labetolol
d) Hydrochlorothiazide

34) Which of the following cause pneumonia?


I. Streptococcus Pneumonaie
II. Haemophilus Influenzae
III. Staph. aureus
I Only III Only I, II Only II, III Only All answers

35) Lipase elevation indicates a disease in:


a) Pancreas – amylase also
b) Liver
c) Heart
d) Kidney
36) Patient with a problem in valve of the heart can be diagnosed by:
a) Echocardiogram
b) Electrocardiogram
c) Blood pressure measure
d) Exercise Electrocardiogram

37) The deoxygenated blood goes to the right atrium of the heart from all the body
through:
a) Pulmonary vein
b) Carotid artery
c) Aorta
d) Superior Vena Cava

38) Dehydration can cause all Except:


a) Thirst
b) Dizziness
c) Headache
d) Hypertension
e) Dry skin

39) Pre-eclampsia is characterised by:


a) Hypertension
b) Hypotension
c) Hyperglycemia
d) Hypoglycemia
Pre-eclampsia is characterised by: is a disorder of pregnancy characterized by high blood
pressure and a large amount of protein in the urine. The disorder usually occurs in the
third trimester of pregnancy and worsens over time
Eclampsia: seizures during pregnancy that occurs due to hypertension. MgSO4 for
prevention and here only mother’s life matter there is high rise of BP as well as
convulsions so no chance of continuation of pregnancy this is as obstetric management

40) Regarding high blood pressure, what is a true statement? 2012


a) BP reading of 160/100 at home requires referral to Emergency Room
b) BP reading of 160/100 in the pharmacy requires referral to Emergency Room
c) 160/100 is a sufficient for a diagnosis of hypertension
d) Cuffs should be placed at heart level when measuring
e) BP reading at home may be inaccurate due to untrained use
41) Which can reduce cardiac oxygen demand without causing vasodilatation?2012
a) Atenolol
b) Felodipine
c) Nitroglycerine
d) Minoxidil
e) Hydralazine

42) Which of the following cause venous pooling?


a) Nitrates
b) Nifedipine
c) Digoxin

43) NADP is needed for: 2013


a) Biotin
b) Niacin
c) Riboflavin
d) Thiamine
e) Cyanocobalamin

44) Which vitamin may cause flushing?


a) Vitamin B1
b) Vitamin B2
c) Vitamin B3
d) Vitamin B6
e) Vitamin B9

45) Which is true about Niacin Except:


a) Niacin dose as lipid lowering agent is so small
b) Hepatic Enzyme elevation cause by niacin in tablet form

46) All of the following are an appropriate advice for a lady suffers from hot flash
Except:
a) Light Exercise
b) Take 0.5 mg Aspirin
c) Light cloths
d) Use fan
47) All are appropriate advices for haemorrhoid patient Except:
a) Avoid taking acid fruit juice No restriction about acidic food
b) Avoid heavy lifting
c) Avoid straining
d) Avoid sitting for long periods

48) Smoking in pregnancy may affects fetus by:


a) Neuro disorder
b) Lung not formed well
c) Interrupt Oxygen transfer through the placenta

49) what is the property of the drug that can be found in breast tissue with high
concentration?
a) Low Molecular Weight
b) Low Lipid Solubility
c) High Protein Binding
d) High Acidic product

50) When a drug circulating in blood & highly perfuse organs, we name it :2013
a) Central Compartment
b) Peripheral Compartment
c) 2- Compartment
d) Multi Compartment

51) In end stage renal failure Calcium is given to :2013


a) Reduce hyperphosphatemia
b) Reduce hyperalbuminuria
c) Adjust Calcium level in the blood

52) Interleukins & TNF are considered:


a) Cytokines
b) Enzymes
c) Prostaglandins
d) Leukotrienes

53) OATP & OCTP are considered:


a) Cytokines
b) Transporters
c) Enzymes
Tips: An organic anion-transporting polypeptide (OATP) is a membrane transport protein
or 'transporter' that mediates the transport of mainly organic anions across the cell
membrane. Therefore, OATPs are present in the lipid bilayer of the cell membrane, acting
as the cell's gatekeepers. OATPs belong to the Solute Carrier Family (SLC), more
specifically the Solute Carrier Organic Anion (SLCO) gene subfamily
An organic cation transport protein (OCTP) mediates the transport of organic cations
across the cell membrane. These proteins are members of the solute carrier family,
subfamily 22. This family of proteins can also transport zwitterions and anions, though it
is a different subfamily of solute carrier proteins than the organic anion transporters

54) The test to detect pyrogen in the parenteral preparations is called:


a) Limulus test
b) Coombs test
c) Schilling test

55) Demyelination(myelin sheath of neurons is damaged) is a characteristic for :


2014.2013,2012,2010
a) Multiple Sclerosis ,Vitamin B12 deficiency can cause demyelination
b) Diabetic foot
c) Rheumatoid Arthritis
d) Polycystitis
e) Multiple Myeloma

56) What is true about the Sciatic nerve?


a) innervate the whole foot  NOT FOOT BUT THE WHOLE LEG AND HEAL OF FOOT
b) innervate the Hips
c) innervate the Buttocks large muscle
d) innervate the Quadriceps

57) Destruction of the radial nerve may cause:


a) Difficulty straightening the elbow
b) Difficulty moving the shoulders
c) affect knee function

58) Glutathione is made up of Glutamate, Glycine and: 2013,2011.2009


a) Cysteine
b) Lysine
c) Alanaine
d) Tryptophan
e) Phenylalanine

59) In the ECG, Ventricular depolarization is indicated by:


a) P Wave
b) PR Interval
c) QRS Complex
d) ST Segments
e) QT Interval

60) Angiogenesis means: 2014, 2012, 2010, 2009


a) Formation of new blood vessels
b) Formation of new bone
c) Formation of new Red Blood Cells
Tips: Angiogenesis is the physiological process through which new blood vessels form
from pre-existing vessels.This is distinct from vasculogenesis,which is the denovo
formation of endothelial cells from mesoderm cell precursors.The first vessels in the
developing embryo form through vasculogenesis,after which angiogenesis is responsible
for most, if not all, blood vessel growth during development and in disease.

61) ACTH is secreted from: 2013, 2011


a. Anterior Pituitary Gland
b. Posterior Pituitary Gland
c. Thymus Gland
d. Adrenal Cortex
b) Pineal Gland

62) All of the following used in treatment of breast cancer Except :2011
a) Anastrozole - oral non-steroidal aromatase inhibitor
b) Letrozole - oral non-steroidal aromatase inhibitor
c) Tamoxifen - used for Albright syndrome also , prodrug
d) Flutamide - synthetic,non-steroidal, pure antiandrogen used primarily to treat
prostate cancer
e) Exemestane - aromatase inhibitors

63) Levothyroxine is used in the treatment of hypothyroidism by:


a) Act as a replace hormone
b) Increase conversion of T4 to T3
c) Increase synthesis of Thyroxin

64) Which is true about Diabetes?


a) Glucagon & Insulin have opposite effect on glucose metabolism
b) Metformin causes hypoglycemia
c) Repaglinide increase the secretion of insulin from B-cells in response to glucose

65) What is true about Regular Insulin (globin insulin)?


a) Its onset of action is less than 1 hour
b) It's to be taken after food

66) Rx for weekly compliance package 2012


Furosemide 40 mg 2 Tabs q am
Enalapril 10 mg 1 Tab With lunch
Lipitor 20 mg 1 Tab hs
Glyburide 2 Tabs bid
Which should trigger the attention of the pharmacist?
a) 1 Lipitor Tab. in the bed time slot
b) 2 Glyburide Tabs. in the morning and Supper slot
c) only 1 Tab. in lunch slot
d) 4 Tabs. in the morning slot
e) 1 Furosemide Tab. with breakfast and 1 Tab. with the Supper

67) The relation between B-Lymphocyte & Bone marrow is:


a) B-Lymphocyte produce antibody in the bone morrow
b) B-Lymphocyte formed from the bone marrow

68) A patient is on glargine insulin daily at bed time & takes Lispro three times daily
with meals, what is the best way to protect this patient from hypoglycemia during
sleep?
a) Take a snack at bed time with no change in doses of insulin
b) Decrease the dose of lispro
c) Increase the calories in his dinner
d) Skip the dose of lispro at night

69) What is the mechanism of action of PropylThiouracil in the treatment of


hyperthyroid?
a) Inhibit the synthesis of TSH
b) Inhibit the synthesis of thyroxin
c) Increase the conversion of T3 to T4

70) Which of the following is a symptom of hypothyroidism?


a) Heat intolerance
b) Dry Skin
c) Tachycardia
d) Diarrhea
e) Weight loss

71) Decrease in ADH may cause:


a) Ploy urea
b) Oligurea
c) PolyPhagia
d) Protein in urine

72) All of the following are a symptoms of Septic shock Except occurs when sepsis,
which is organ injury or damage in response to infection
a) Fever
b) Tachycardia
c) Increase Urine Volume
d) Hypotension

73) Which of the following products acts as 5- α reductase inhibitor?2009


a) Finasteride
b) Tamsulosin
c) Prazosin

74) Which of the following products used to shrink the size of the prostate in BPH?
a) Finasteride
b) Tamsulosin
c) Prazosin

75) All of the following are factors that affect the creatinine clearance Except:
a) Age
b) Gender
c) Weight
d) serum creatinine
e) Urine volume

76) What is true about aminoglycoside?


a) Used in the treatment of pneumonia caused by pseudomonas aeruginosa
Tips: they are deactivated when combined with penicillins due to a reaction
between the amino groups in aminoglycosides and betalactam ring in the penicillin

77) What is true about Post Antibiotic Effect?


a) The drug still has the power to prevent the growth of microorganisms after
duration of administration
b) Amino glycosides are an example against anaerobic microorganisms

78) Which of the following products is suitable for a patient suffer from depression and
neuropathic pain?
a) Amitriptyline
b) Bupropion
c) Selegiline
d) Paroxetine
e) Mirtazapine

79) What is the duration of treatment needed for a patient with a first episode of major
depression?
a) 3 months
b) 6 months
c) 1 year
d) 2 years
e) No time limits

80) Drug X has a clearance of 300 ml/min while the normal clearance rate is
120ml/mint his indicate:
a) Increase GFR and active excretion
b) Increase GFR and reabsorption
c) Increase active excretion
d) Increase GFR

81) What is the appropriate advice for a patient will use oral corticosteroid for a month?
a) Rinse with water after taking the dose
b) Take the dose on an empty stomach
c) He may experience a drowsiness
d) Don't stop the medication suddenly

82) The eastern blot is used to detect :2009


N ‐ RNA
a) Protein
E ‐ Analysis Of
b) RNA
protein
c) DNA
W ‐ Separation or
d) Polysaccharides
detection of
Protein
83) What is true about the blot:
S ‐ DNA
a) Northern blot detects the DNA
b) Southern blot detects the RNA
c) Eastern blot detects the protein
d) Western blot detects the protein

84) A lady comes into your pharmacy and is taking Captopril, Digoxin,
hydrochlorothiazide, and Furosemide. What should she monitor?
a) K levels and Mg levels
b) K levels and serum Creatinine
c) Serum TG
d) Glucose levels
e) Serum Phosphates

85) Morphine cause respiratory depression due to its action on which receptor
:2012,2016
a) Mu
b) Kappa
c) Delta
d) Mu & Kappa
e) Mu & Delta

86) All of the following cause hyperkalemia Except:


a) Vomiting
b) Spiranolactone
c) ARBs
d) ACE-I
e) Renal insufficiency
87) What is the mechanism of action of Spiranolactone? Aldosteron antagonist

88) Respiratory acidosis caused by :2012


a) Decrease in HCO3
b) Increase in CO2 pressure
c) Decrease ventilation

89) Naturally occurring programmed cell death is known as:2013


a) Necrosis - Cell death due to injury
b) Apoptosis
c) Atrophy - is the partial or complete wasting away of a part of the body
d) Mitosis
e) Cytosis - transport mechanism (endocytosis (into the cell), exocytosis (out of the
cell), and transcytosis (through the cell, in and out).

90) The process by which the neutrophils kill the strange cells in the serum by engulfing
is called
a) Phagocytosis

91) What are the most abundant white blood cells in the blood?
a) Monocytes
b) Basophiles
c) Neutrophils
d) Platelets
e) Macrophages

92) Which is responsible for the metabolism of mercaptopuric acid?2011


a) CYP3A4
b) Glucuronic acid
c) Glutathione S Transferase
d) NABQI
e) Bilirubin

93) Which of the following glucournide metabolite is active?


a) Codeine
b) Digoxin
c) Beta Blocker
94) Jaundice in newborns is caused from a deficiency of:
a) Glucose
b) UDP glucose
c) Glucuronate
d) UDP glucuronate
e) UDP glucuronyl transferase

95) All of the following agents used to treat the nausea and vomiting caused after
chemotherapy Except:
a) Metochlopramide
b) Dexamethazone
c) Ondansetrone
d) Perchloperazine
e) Doxylamine

96) Which of the following chemotherapeutic agents acts by inhibit the synthesis and
formation of DNA:
a) Cisplatin
b) 5-Flurouracil
c) Vincristine
d) L-Aspraginase
e) Etopsoide

97) Patient with chronic COPD and had pneumonia, before he had pneumonia and took
amoxicillin Clavulonate since 2 months, what is the empirical treatment: (Check)
a) Amoxicillin Clavulonate
b) Clarithromycin
c) Doxycycline
d) Levofloxacin

98) In Candida we can use all the following agents Except: 2013,2012
a) Fluconazole
b) Metronidazole
c) Ketoconazole
d) Nystatin
e) Amphotericin B
99) The drug of choice for treatment of athlete’s foot is:
a) Clotrimazole
b) Polymixin B
c) Germicidin
d) Bacteriocin

100) Treatment of impetigo in a 10 years patient : contagious bacterial skin


infection most common among preschool children primarily caused by Staph. aureus,
and sometimes by Strept. pyogenes
a) Oral erythromycin
b) Oral amoxicillin
c) Topical mupirocin

101) Patient with severe ulcerative colitis, what will be used to reduce remission:
a) Infliximab
b) Prednisone
c) Azathioprine

102) This structure is found in:


a) Cephalosporins
b) Quinolone
c) Tetracycline
d) Macrolides

103) Which antibiotic works by inhibiting cell wall synthesis?


a) Aminoglycosides
b) Cephalosporins
c) Macrolides
d) Lincosamines
e) Tetracyclines

104) Which drug causes dry mouth?


a) Tiotropium
b) Pilocarpine
c) Acetylcholine
d) Physostigmine
e) Neostigmine
105) Which of the following give Parkinsonism effect:
a) Paroxetine
b) Haloperidol
c) Lithium
d) Clozapine

106) Acute Dystonia caused by antipsychotics treated by: Benzotropine

107) A pharmacist plans a strategic plan so he develops a SWOT analysis, what


does the O stand for in this case? 2012,2016
a) Opportunity
b) Outcome
c) Objectives

108) Which is not a symptom of rhinitis JAN&JULY 2012


a) Runny nose
b) Hives
c) Sneezing
d) Watery eye
e) Fever

109) What is the difference between Urinary Tract Infection & Uncomplicated
Urinary Tract Infection?
a) Pain sensation during urination
b) Frequency of urination
c) Urgency of urination
d) Turbidity of urine
e) Fever Fever, chills, bleeding IN COMPLICATED UTI

110) Which of the following herbal remedy can be used in treatment of migraine?
a) Licorice
b) Butterbur  also feverfew (tanacetum parthenium)
c) St. John's wart
d) Guava
e) Ginkgo

111) Sumatriptan is used in treatment of migraine due to its effect as 2011,2010


a) Dopamine antagonist
b) 5HT1b/1d agonist
c) Acetylcholine antagonist
d) 5HT1b/1d antagonist
e) Dopamine agonist

112) Rods in the eye are responsible for:


a) Peripheral vision
b) color vision
c) Night vision
d) Acuity of vision
e) Rapid eye movement

113) Which of these factors affect on the passage of a drug through the stratum
corneum? 2013,2014,2009
a) Solubility
b) Dissolution
c) PKa
d) Partition Coefficient
e) Particle size

114) Too much pressure in tablet during manufacture causes:


a) Capping
b) Lamination
c) Picking
d) Sticking
e) Mottling

115) Which of the following is a water-soluble ointment base?


a) Lanoline
b) Yellow wax
c) Sodium Monostearate
d) PEG 3000 & PEG 6000 Mixture

116) Salbutamol is:


a) B1 Agonist
b) B2 Agonist
c) B1 Antagonist
d) B2 Antagonist
117) Filgrastim used in patients receiving strong chemotherapy to:
a) Protect them from Alopecia
b) Reduce the risk of Cardio toxicity
c) Reduce the risk of infection
d) Reduce the risk of pulmonary toxicity
e) Protect them from stomatitis

118) Cytochrome oxidase & Oxygen transport contain:


a) Myosin
b) Myoglobin
c) Heme
d) Albumin
e) Transferrin

119) Which of the following blood component which are important in drug
binding?
a) Albumin & VLDL
b) Albumin & Glycoprotein
c) RBCs& WBCs

120) High Extraction ratio is present in:


a) High first pass effect High extraction ratio directly proportional to hepatic
elimination
b) Zero Order
c) Second Order
(N.B: No first order among the choices)

121) The Kaplan-Meier equation is applied for new therapies (such as dialysis or
antineoplastics) to measure patients’: 2013,2012
a) Preferences
b) Adverse reaction rate
c) Mortality rate
d) Morbidity rate
e) Survival rate

122) What is true about the chemical structure of Immunoglobulin?


a) Contains heavy and light chain connected by disulfide bond
b) Consists of four subunits connected together by hydrogen bond
123) Which of the following drugs may require monitoring the eye of the patient?
a) Phenytoin
b) Hydroxychloroquine
c) Carabamazepine
d) Haloperidol

124) All of the following products cause pulmonary fibrosis Except :


a) Amiodarone
b) Digoxin
c) Bleomycin
d) Bromocriptine

125) What is best treatment of ADHD (Attention Deficit Hyperactivity Disorder):


a) Methylphenidate
b) Amphetamines
c) Rivastigmine
d) Cocaine

126) Venlafaxine acts on which receptors :2013


a) Serotonin
b) Dopamine
c) Norepinephrine
d) Serotonin & Dopamine
e) Serotonin & Nor epinephrine

127) Which of these medications not to be taken with Levothyroxine?


a) Ferrous gluconate Fe causes chelation and reduces levothyroxin absorption
b) Omeprazole
c) Phenytoin
d) Carbamazepine

128) Vitamin E after Oxidation in lipid bi layer, returns to its reduced form by the
action of :2011
a) Vitamin A
b) Vitamin B12
c) Vitamin C
d) Vitamin K
e) Vitamin D
129) Vitamins are:
a) Organic substances that are needed in small quantities to sustain life.
b) Inorganic substances that must be taken daily to act in biological functions
c) Inorganic substances that act as a cofactor in the enzymatic reactions

130) Dysphasia is impaired in:


a) Speech
b) Taste
c) Dyspepsia
d) Smell

131) A patient taking Warfarin, u should monitor :


a) Appt
b) INR
c) Bp
d) Heart rate
e) Vitamin K Conc.

132) What is true about Troponin?


a) Used to diagnose Cardiac infarction diseases
b) Found in Cardiac and smooth muscles
c) Used to detect hepatic diseases
d) Used to evaluate the renal function

133) What is true about Nicotine Replacement Therapy?


a) It should be used for 8-12 weeks
b) Attending behavioural courses may help the patient
c) If the Nicotine dose increased the patient will suffer from headache
d) If there is an itching happened during usage of patches patient can't use gum

134) While using Isotretonin, we should monitor :2014,2013


a) Haemoglobin & Lipid level
b) Lipid & Liver
c) Kidney

135) A Rx contains Sodium Bisulfite which is used as :


a) Anti-Oxidant
b) Lubricant
c) Humectants
d) Solubilising Agent
e) Preservative

136) To increase the solubility of organic salt and protein by increasing the ion
strength this is called:
a) Salting In
b) Salting Out
c) Co Solvency

137) Which of the following can be used to prevent DVT in a pregnant woman?
a) Vitamin K antagonist therapy
b) Dalteparin
c) Heparin then Warfarin
d) Warfarin
e) Fondaparinux

138) What is meant by Functional food?


a) Cereals contain multivitamins
b) Jelly contain mineral oil
c) Tabs or caps for vitamins and mineral oil

139) Which of the following can be used in treatment of B. fargilis Infection?


a) Metronidazole
b) Clarithromycin
c) Tetracycline
d) Ketoconazole

140) Staph. Aureus can be found in colonies in chronic infection of:


a) Ear
b) Eye
c) Nose
d) Fingernail
e) Sore

141) Rhabdomyolysis is accompanied by: 2013,2012


a) Liver Failure
b) Kidney Failure & lead to urine discoloration
c) Heart Failure
d) Pulmonary Failure

142) A mother of a 3 years old baby came to the pharmacy with a prescription of
Amoxicillin to treat otitis media for her baby but you notice that the date of the
prescription was 3 days later, what is the appropriate action you have to follow?
a) Refuse to dispense the prescription as it's useless to treat the baby
b) Call the Physician and discus with him the case
c) Fill the prescription as the 3 days period was watching period for the case

143) Drug of choice for the treatment of meningitis in adult is: 2013
a) Ceftriaxone
b) Cefixime
c) Cephalexin
d) Cefuroxime

144) The first line treatment for Pain in Osteoarthritis is:


a) Morphine
b) Ibuprofen
c) Acetaminophen
d) Indomethacin

145) What is true about Codon:


a) Present in tRNA
b) Consists of 3 nucleotide bases that express 1 amino acid
c) It's a segment of DNA express genes

146) SNP (single nucleotide polymorphism) Means:


a) Single Non-Polymorphism
b) Change in one base in DNA

147) RT-PCR is differing from PCR in: 2013


a) It needs high temperature
b) It needs low temperature
c) Restricted in RNA
d) Reverse Transcriptase in cDNA
148) The process in which DNA is converted to mRNA is called 2013
a) Translation
b) Transcription
c) Reverse Transcription
d) Denaturation

149) The following is contraindicated with celecoxib:


a) dyspepsia
b) sulfa allergy
c) prostatic hyperplasia
d) renal impairment
e) acetaminophen concurrent use

150) In the Northern Hemisphere the most suitable period to take flu vaccine is:
a) Mid September till Mid December
b) Mid September till Mid October
c) Mid October till Mid December
d) Mid November till Mid January
e) Mid November till Mid April

151) In case of Heparin overdose, what is the correct antidote?


a) Protamine Sulfate
b) Vitamin K
c) Warfarin
d) Atropine

152) Tadalafil is different from Sildenafil because:2013,2012


a) sildenafil is longer acting
b) Tadalafil is shorter acting
c) Tadalafil can be used safely with nitroglycerin
d) Tadalafil and Sildenafil are both equally active and have the same duration
e) Tadalifil is much longer acting than Sildenafil

153) The Steady-State Concentration is a factor of:


a) AUC
b) Half life
c) Plasma Protein binding
d) Dose
154) By dividing the amount of the drug in the body by the concentration of the
drug in the plasma, we will get:
a) Volume of Distribution Vd=Db/Cp
b) Clearance Rate
c) Rate of Infusion
d) Steady-State Concentration

155) What is the clearance rate for a product with T1/2 = 3 hours & volume of
distribution is 9 L?
a) 2 L / Hr
b) 2 ml / Hr
c) 2 L / Min
d) 2 ml / min

156) What does ISMP Stand for?


a) Institute of Safe Medication Practice

157) What the most symbol take the pharmacist attention in RX:
a) I.U
b) PRN
c) mg
d) Kg

158) Rx
Substance X
Hydrocortisone 1 % AA 25 mg
Mitte 50 mg
What does AA refer to?
a) Equal amount of each
b) Acceptable amount up to
c) Appropriate amount up to

159) 250 mg of Substance X 50 % needed to be diluted to be 10 %, how many mg


diluents should be added?
a) 1000 mg
b) 1250 mg
c) 750 mg
160) Which of the following preparation must be completely sterilized?
a) Total Parenteral Nutrition
b) Oral Suspension
c) Nasal preparation
d) Rectal Suppository

161) How many mill equivalents of CaCl2 is found in 1000 mg (Ca atomic weight
40)?
a) 25
b) 50
c) 100
Tips: MEQ= Weight * Valance / molecular weight. Calcium valance is 2.
Meq= 1000*2/40 Meq= 50

162) A product with T1/2 = 2 hr. was given as 250 mg every 12 hr, we notice that
that product is not accumulated in the body because of:
a) Rapid Excretion
b) High Volume of distribution
c) Low plasma protein binding
d) Short T1/2 in compared to dose interval

163) Lung thrombus (pulmonary embolism also) is mainly due to:2013,2012,2011


a) Fibrin
b) Platelets
c) C protein
d) RBCs
e) WBCs
PE is related to VTE (Venous thromboembolism) which the main component is
fibrin unlike Aterial thrombosis which the main component is platelts.
164) Thalidomide used to treat: 2013,2012
a) Multiple hereditary exostosis
b) Multiple endocrine neoplasia
c) Multiple system atrophy
d) Multiple sclerosis
e) Multiple myeloma

165) Mycophenolate mofetil is an:


a) Antifungal
b) Antihelminthics
c) Anti Arrhythmia
d) Immunosuppressant

166) If Product A is shown to has AUC of 50 When given as a dose of 50 mg orally,


While Product B has AUC of 90 when given as a dose of 100 mg orally, what is the
relative bioavailability of product B to Product A?
a) 0.9
b) 1.1
c) 1
Tips: Relative bioavalability = AUC b/AUC a at the same dose
For drug b AUC = 90 for 100 mg So = 45 for 50 mg
So relative bioavalability B to A = 45/50=0.9

167) If Product A is given as a dose of 150 mg shows AUC of 50 with C max 50 mg &
T max 1hr. & Product B is given as a dose of 300 mg shows AUC of 50 with C max 50
mg & T max 6hr.
What is true about Product B?
a) Sustained Release
b) Has high extensive pass metabolism
c) Has high clearance rate

168) It was found that when we increase the dose of a product, the rate of
elimination doesn't affect, so that product obey:
a) Zero Order
b) First Order
c) Second Order

169) In a clinical trial the absolute risk reduction was 5 % Then what will number
needed to treat :2013
a) 2
b) 20
c) 200
d) 100

170) All of the following are enzyme inducer Except:


a) Valproic Acid
b) Rifampin
c) Phenobarbital
d) Phenytoin
e) Carbamazepine

171) What is the relation of Codeine to Morphine?


a) Codeine is a pro-drug of Morphine
b) They are functional isomers
c) They are structural isomers

172) All the following drugs decrease the effects of oral contraceptives, Except
:12,14,16
a) Rifampicin
b) Carbamazepine
c) Tetracycline
d) Phenytoin
e) Ibuprofen

173) Warfarin acts in:


a) Liver
b) Kidney
c) Heart
d) Blood

174) Which of the following medication may cause erectile dysfunction? 2013
a) Corticosteroid
b) α Adrenergic Blocker Opioids more
c) Opioid
d) ACE Inhibitors

175) Which of the following medication is used in Congestive Heart Failure and
increase the survival rate?
a) Enalapril
b) Digoxin
c) Dobutamine
(N.B: No Beta Blockers among the choices)

176) Class I ant arrhythmic products act by blocking:


a) Na Channels
b) Ca Channels
c) Potassium Channels
d) Beta Receptors

177) What is true about Clopidogrel?


a) It acts by inhibition of Cox-II
b) It acts by inhibition of TXA2
c) It acts by antagonise ADP on its receptors

178) Patient takes aged cheese and was treated with MAO-I, he had a hypertensive
crisis because of:
a) Tyramine
b) Serotonin
c) Histamine

179) Which of the following medication may cause serotonergic syndrome if taken
with MAO-I:
a) Dextromethorphan
b) Pseudoephedrine
c) phenylephrine

180) Typical antipsychotics act on:


a) Dopamine
b) Serotonin
c) Acetylcholine
d) Norepinephrine

181) Which structure can be formed from ephedrine and pseudoephedrine?


a) Amphetamine
b) Codeine
c) Morphine
d) Niacin

182) Acetyl Salicylic acid gets metabolized into salicylic acid by:
a) Salicylase
b) Esterase
c) Amidase
d) Acetyltransferase
e) Carboxylase

183) Which is used as local anesthetic for hemorrhoids? 2013,2012


a) Lanolin
b) Witch Hazel
c) Zinc Oxide
d) Hydrocortisone
e) Pramoxine

184) You take Fluticasone inhaler and also take Salbutamol inhaler. What
instructions must your pharmacist tell you regarding these two medications?
a) Take Fluticasone, then Salbutamol then rinse.
b) Take Salbutamol, then Fluticasone then rinse.
c) Take Salbutamol, rinse, and then Fluticasone.
d) Take Fluticasone, then rinse then take Salbutamol
e) Take Salbutamol, then Fluticasone.

185) Why do we take protein at IV Injection?


a) Cause it binds faster with the plasma protein
b) To avoid first pass effect This is the reason why isulin is not in Oral tablet till now
c) To get a controlled serum concentration

186) Methylation occurs by the aid of: 2012


a) S-adenosyl methionine  methyl group transfers
b) Glutamyl Transferase
c) Monoamine Oxidase
d) Esterase

187) Which vitamin is found only in animal-derived foods?


a) Vitamin A
b) Vitamin B1
c) Vitamin B12
d) Vitamin D
e) Vitamin K

188) Which enzyme is responsible for the metabolism of sildenafil?


a) CYP1A2
b) CYP2C9
c) CYP2D6
d) CYP3A4

189) Process which works by removing the small impurities from plasma across a
semi-permeable membrane is termed:
a) Dialysis
b) Osmoses
c) Concentration Gradient
d) Dispersion
e) Diffusion

190) Photo toxicity of a medication appears in the form of:


a) Urticaria
b) Allergy
c) The skin response resembles an exaggerated sunburn

191) Amorphous form differs from crystalline form as it is:


a) Crystalline Is generally less soluble than amorphous
b) Amorphous is more soluble than Crystalline
c) Amorphous is more susceptible to polymorphism
d) Amorphous differs in the chemical composition

192) What is true about the critical micelle concentration in emulsion


a) Is defined as surfactant concentration that cause assembly
b) It needs high amount of surfactant when increase lipophilicity
Tips: CMC is the conc of surfactant above which micelles are formed

193) Which drugs cause hemorrhagic cystitis:


a) Cyclophosphamide
b) Vincristine
c) 5-Flurouracil
d) Bleomycin
e) Methotrexate

194) When we add a salt to a solvent, which is true:


a) Increase freezing point
b) Increase vapor pressure
c) Increase boiling point
d) Decrease osmotic pressure

195) What is true about aerosol?


a) It can be prepared for internal and external usage
b) It contains preservatives
c) The same Concentration will give the same effect as oral form

196) What is meant by lavigation?


Reduce particle size by triturating in a morter

197) All of the following instruments need caution during handling Except:
a) Spatula
b) Syringe
c) Needle

198) What is true about Coated Tablets?


I. To protect from gastric PH
II. To mask the taste
III. To Protect it from moisture in the atmosphere
a) I Only
b) III Only
c) I & II Only
d) II & III Only
e) All answers

199) To test for a pharmaceutical’s ability to withstand abrasion we do :2012,2013


a) Friability Test
b) Thickness Test
c) Hardness Test
d) Dissolution Test
e) Disintegration Test

200) Chromatography for heat labile substances, all Except:


a) Thin Layer Chromatography
b) High Performance Liquid Chromatography
c) Reverse Phase Chromatography
d) Gas Chromatography
e) Paper Chromatography
201) Polysorbate 80 acts as :2012
a) Surfactant
b) Preservative
c) Humectants
d) Suspending agent
e) Lubricant

202) What is true about eutectic mixture? 2011


a) Degrade at room temperature
b) Segregates upon storing
c) Liquefies at room temperature
d) Cause Effervescent
e) Absorb moisture from the atmosphere

203) What is the role of surfactant in suppositories?


a) Increase water content
b) Increase contact of suppository with the surround
c) Enhance the melting of the suppository

204) Sedimentation rate is determined by:


a) Stoke's Law
b) Fick's Law

205) Which of the following medication is used to treat non-complicated diaper


rash?2012
a) Zinc Oxide Cream
b) Hydrocortisone 1 %
c) Acetyl Salicylic acid
d) Nystatin

206) What is true about Elixirs? 2011


a) It's a sweated hydro alcoholic solution
b) It's for external use only

207) What ethics means "prevent harm"?


a) Beneficence
b) Non-maleficence
c) Justice
d) Autonomy
e) Veracity

208) If you waive the co payment for one of your clients because he is a regular
customer to you and you didn't do that with other customers, So you violate:
a) Beneficence
b) Non-malfeasance
c) Justice
d) Autonomy
e) Veracity

209) Canada's health act includes all the following except:


a) Universality
b) Portability
c) Public Adminstration
d) Accessibility
e) Speciality

210) If you would like to search for more information regarding Fentanyl
regulation, you'll find it in
a) Schedule H in FDA
b) Schedule I in NAPRA
c) CDSA Schedules (Controlled Drugs and Substances Act)

211) Where can you find information's about medication outside Canada?
a) Martindale
b) CPS
c) AHFS
d) Therapeutic Choices
e) Patient Self Care

212) Where can you find the monograph of a drug in Canada?


a) Compendium of patient self care products
b) Therapeutic choice for Minor Aliments
c) Canada drug database
(N.B: No CPS among the choices)
213) Who publish CARN (Canadian Adverse Reaction Newsletter)? 2013
a) Med effect (Health Canada)

214) Which of the following drugs you cannot advertise by a banner on the counter:
a) A new herbal drug
b) A new supplement
c) A new OTC
d) A new anti arrhythmic prescription drug

215) The most expensive Burden on Canada’s health insurance: 2013


a) Dental
b) Outpatient medication
c) Physician fees
d) Hospitalization

216) A pharmacist wants to increase his pharmacy sales, he can do all Except:
a) Make discount for prescription came from local physician
b) Make arrangement with nurse to make regular blood glucose monitor

217) A pharmacist want to make an offer for patients who don't cover with
province insurance, So he makes it under the name of discount, he did 10 cases
during a month, when he calculate it he found that:
a) Increase net profit
Increase in revenue
b) Decrease net profit

218) The key symptom of nephrosis:2012


a) glucose in urine
b) protein in urine
c) Stone in kidney
d) Increase in urination frequency
e) minerals in urine

219) Increase the risk of bleeding may occur when combining clarithromycin with:
a) Aspirin
b) Clopidogrel
c) Dipyridamole
d) Warfarin
e) Heparin
220) Carbohydrates stored in the animal cells in the form of:
a) Glycogen
b) Glucose
c) Lactose

221) Specificity means:


a) To have negative results with people who don't have the disease
b) To have positive results with people who have the disease

222) Sensitivity means: To have reproducible results

223) The simplest form of modern community pharmacy is:


a) Sole proprietorship
b) Partnership
c) Incorporations
d) Franchise

224) In an experiment want to know the number of newly prescribed Oxycodone


and refill prescription was undergo under 4 physicians what is the name of the used
test?
a) Chi-Square
b) Wilcoxon
Tips: Wilcoxon used when comparing 2 related samples OR repeated measurements
on a single sample, Chi s used for multi group comparison.
225) A prescription for codeine Give 60, I tab bid 30 days, R = 1. Is this prescription
accepted?
a) No, because refills are not allowed
b) No, because part fill are not allowed
c) Yes, because the total amount is clear
d) Yes, because the amount is easily identifiable

226) Which of the following statement is of the responsibilities of a regular


authorized pharmacy technician?
a) Give a counsel for an OTC medication
b) Give a counsel for a new prescription
c) Receive and document a verbal order from a physician about a refill prescription
d) Dispense a narcotic prescription

227) Which of the following antibiotic must not refrigerated after reconstitution?
a) Amoxicillin
b) Clarithromycin
c) Ciprofloxacin

228) A pregnant woman has penicillin allergy and suffers from urinary tract
infection, the most appropriate medication for her case is : 2013
a) Cephalexin
b) Ciprofloxacin
c) Nitrofurantoin
d) Sulfamethoxazole

229) Phenytoin cause : 2013


a) Gingival Hyperplasia
b) Alopecia
c) Kidney stones
d) Dyskinesia

230) The main screening test for colorectal cancers is : JAN & JULY2013,2011
a) Blood tests
b) Ultrasound scans
c) PPD test
d) Pap smear
e) Fecal occult blood
231) In order to protect protein when freeze drying which should be
add:2014,2012
a) Humectant
b) Protectant
c) Lyoprotectant

232) Polyvinyl alcohol is used in ophthalmic preparations as :2014.2016.2013


a) Viscosity Enhancer
b) Solubilising agent
c) Preservative
d) Lubricant

233) A person came to the pharmacy asking for Marihuana to treat his insomnia,
what is the most appropriate action?
a) Advise him to make application in special access program in health Canada
b) Tell him it's not allowed to use Marihuana in Canada
c) Tell him that Marihuana is not indicated to treat insomnia and advise him for a
suitable treatment

234) What is meant by profitability?


a) To cover the costs with a little revenue for the owner

235) What is true about phase II trials?


a) Done on animals
b) Done on small healthy human population to test the safety
c) Done on small disease human population to test the efficacy
d) Done on large disease human population to test the efficacy and safety
e) Done while the medication in the real world

236) What is true about phase III trials?


a) Done on animals
b) Done on small healthy human population to test the safety
c) Done on small disease human population to test the efficacy
d) Done on large disease human population to test the efficacy and safety
e) Done while the medication in the real world

237) Which is not a component of pharmacy-economics?


a) International
b) Payer
c) Hospital
d) Society

238) OU means :
a) Right Eye
b) Left Eye
c) Both Eyes
d) Both Ears
e) Left Ear

239) What is true about Parallax error? 2013


a) Position of eye at all volumetric vessels must be at the same level as the meniscus
Parallax is a deceptive change of the position of an object which is observed
while the position of the observer changes. Position of eye at all volumetric
vessels must be at the same level as the meniscus. If not, the parallax will
cause an error while reading the position of the meniscus of a liquid in a
burette. It will be a positive mistake if the eye is lower, and negative if the eye
is higher than the meniscus plane

240) Which of the following medications cause QT prolongation?


a) Flouroquinolones
b) Amoxicillin
c) Digoxin
d) Nitrates

241) Dysphonia is an impairment in :


a) Taste
b) Voice
c) Smell

242) When a drug circulating in blood & highly perfuse organs, we name it
a) Central Compartment
b) Peripheral Compartment
c) 2- Compartment
d) Multi Compartment
243) the presence of alternating residues of β-(1, 4) linked N-acetyl glucosamine
and N-acetylmuramic acid is found in:
a) Plasma membrane
b) Cell wall
c) Cytoplasm
d) Mitochondria

244) When you take the Weight, age and diseased state of patients, this is
comparing?2012
a) Statistical variable
b) Biological variable
c) Collective variable

245) In your pharmacy a technician made a mistake and gave a wrong medication
to the patient, when you notice that mistake you called the patient and you knew that
he didn't take his dose, what is the next step?
a) Report the case
b) Exchange the medication

246) In an ointment preparation containing water-removable bases, polyethylene


glycol is often added as: 2013,2012
a) Humectant
b) Stabilizer
c) Antioxidant
d) Preservative
e) Surfactant

247) The cost of a product is 2.00 $ and you sell it for a price of 2.50 $. Which is
true?2012
a) Margin percent is 20%
b) Mark up of cost is 20%

248) A pharmacist decreased the expenses of his pharmacy by $ 30,000 in


compared to the last year , while achieving the same sales, what will be the percent of
increasing his net profit if you know that : 2016
Sales = $ 3,000,000
Costs = $ 2,400,000
Net profit this year = $ 150,000
a) 6%
b) 8%
c) 10%
d) 20%
e) 25%

249) Bisphosphonate mechanism of action:


a) Inhibit osteoclast resorption
b) Inhibit osteoblast resorption
c) Decrease Ca execration
d) Increase Ca absorption

250) Which of the following is a humanistic outcome:2011


a) decrease patient visits to physician
b) Patient satisfaction
c) Decrease in the Blood pressure of the patient
d) Decrease in Blood Glucose level of the patient

251) What is the minimum quantity that can be weight on a balance with sensitivity
requirements of 15 mg of a 5 % error is permissible?
a) 300 mg
b) 600 mg
c) 900 mg
Tips: Sensitivity requirement = min. weighable wt × acceptable error %
=15*100/5=300mg
Remember if they give you 95 accuracy you should convert it to 5 percent error

252) In a hospital pharmacy, a patient suffers from side effects of a medication and
you found that the nurse gave him a wrong dose of his medication, what is the most
appropriate action?
a) Try to cover her mistake
b) Report the mistake to the physician
c) Call one of patient care to report the mistake
d) Don't deal with this nurse again

253) In research cost $ 100,000 cause increase in patient life by 0.5 year, so what is
the true to express the Cost-Effectiveness analysis of that research? (Check)
a) $ 100,000 / QALY
b) $ 200,000 / LGY (Life Gained per Year)
c) $ 50,000 / QALY
d) $50,000 / LGY

254) Counselling for patient taking VIMOVO (naproxen + esomeprazol) 2013


a) Drink a lot of fluids
b) Don’t take antacid while take vimovo
c) Take vimovo at least 30 minutes after meal
d) Take vimovo at least 30 minutes before meal
e) Vimono should be taken with milk

255) All can be done to an ankle injury except?


a) Rest
b) Heat
c) NSAIDS
d) Compress
e) Elevate

256) Which of these treat Alzheimer’s disease?


a) Bromocriptine
b) Benztropine
c) Rivastigmine
d) Nitrosamine

257) Structure of acetylcholine and acts indirectly by


inhibiting?2014,2012,2009
a) Acetyl cholinesterase

258) Structure of NAPQI and it's a metabolite of: 2014


a) Acetaminophen

259) This is:


a) Sulfacetamide
b) Acetazolamide
c) Furosemide
d) Hydrochlorothiazide
e) Spironolactone
260) What is the mechanism of action of Orlistat? 2010,2009,2008
a) Inhibit gastric Lipase

261) What is the use of egg-yolk? 2012


a) Emulsifying agent
b) Humectant
c) Suspending agent
d) Lubricant

262) What is the enzyme name that found in viruses that enables them to be
released from the host cell?2012
a) Neuraminidase

263) Two drugs with equivalent therapeutic effects are best compared by: 2009
a) Cost-Benefit Analysis
b) Cost-Effectiveness Analysis
c) Cost-Minimization Analysis
d) Cost-Utility Analysis
e) Willingness to Pay

264) What is the mechanism of action of Senna as Laxative?


a) Stimulate the bowel wall to contract
b) Allow the stool to pass easier through the bowel
c) Increase the bulk of the stool which stimulate peristalsis

265) Misoprostol is an example of: Eicosanoids

266) What is the most appropriate advice to a patient will start Levothyroxin?
a) Talk it in the morning to avoid insomnia
b) Feeling palpitation is an indicator of overdose

267) The process by which the virus transfer DNA from one bacteria to another is
called:
a) Transduction
b) Translation
c) Transformation
d) Transportation
268) When a pharmaceutical company starts a research for a new medication, it
has to protect her rights by making registration for: 2008
a) Logo
b) Copy Right
c) Trade Name
d) Patency
e) Brand Name

269) What is true about Crohn's disease? inflammatory bowel disease that may
affect any part of the gastrointestinal tract from mouth to anus

270) A taxi driver came to you with a prescription of sumatriptan for his migraine
and you know that it may cause dizziness, what is the most appropriate action to be
done?
a) Call the physician to explain to him your fears from side effect of the medication
b) Refuse to dispense the prescription
c) Dispense the medication and give the patient a verbal counsel about the side effect
of the medication
d) Dispense the medication and give the patient a printed copy of the side effect of the
medication

271) Rx for a medication as follow:


Start 7.5 mg then 2.5mg BID for 14 days
Calculate the dispensed amount?
a) 77.5 mg
b) 72.5 mg
c) 75 mg

272) A product with T1/2 One year, Concentration 400 mg what will be the
concentration after 3 years?
a) 50 mg
b) 100 mg
c) 150 mg

273) A patient who is 3 years old experiencing sore throat. His Dr. prescribed for
him: Zithromax 200 mg 1QD for 5 Days. In your pharmacy you have 2 bottles of
azithromycin suspension 600 mg (200mg/5ml) & 900 mg (200mg/5ml). What is the
best bottle to be dispensed to fill this prescription?
a) One Bottle of azithromycin 600 mg (200mg/5ml)
b) One Bottle of azithromycin 900 mg (200mg/5ml)
c) Two Bottles of azithromycin 600 mg (200mg/5ml)
d) Two Bottles of azithromycin 900 mg (200mg/5ml)
e) One Bottle of azithromycin 600 mg (200mg/5ml) & One Bottle of azithromycin 900
mg (200mg/5ml)

274) Differential diagnosis of Rosacea from Acne vulgarise is?


a) Erythema
b) Telangiectasia
c) Papules
d) Comedones
e) Seborrhea.

275) Buccal Route is characterized by:2011


a) No membrane barrier
b) Less metabolic activity
c) For high dose drugs
JULY 2014

1- how many (h) atom/s required for the reduction of flavin


a-1 b-2 c-3 d-4 e-5
TIPS: You need to break N= And you have 2 so U need one
hydrogen for each one.

2-when a new drug goes to the real world for testing its safety, in which phase does that
happen. JAN2014
a-phase I
b-phase II
c-phase III
d-phase IV- It goes after phase III
e-phase zero

3-When a tumor lysis mechanism takes place in human body what characteristic you
would find after doing lab tests
a-hyperuricemia  hyperkalemia, hyperphosphatemia, hypocalcemia, acute renal
failure
b- hyponatremia
c- hypocalemia
d-hypokalemia
e- hypernatremia

4-A patient was using allopurinol as a treatment for his Gout disease, but now he can no
longer use it. what would be the suitable alternative uricosuric drug for his case.
a-febuxostate
b- colchicines

5- A 54 years patient was recently diagnosed with BPH, he is having the following
medications, captopril, sildenafil for his erectile dysfunction and hydrochlorothiazide, he
has no allergy and has postural hypotension but not high, what drug you should
recommend for this patient
a-tamsulosin
b- finasteride
c- dutasteride
d-doxazosin
e- prazosin
tips: tamsulosin is actually selective for alpha receptors on prostate, it has no effect on
blood vessels & Pt has erectile dysfunction so finastrride and dutrasteride are
contraindicted

6- side effects of blucetamide (Bicalutamide) prostate anticancer: 2008


a- hot flush b- migraines
7- the indicator for prostate cancer
a- PSA prostate specific antigen b-decrease in prostate size

8-A hypertensive and nephropathic patient taking lisinopril, he complained to his


prescriber about experiencing cough, which of the following drugs you should
recommend.
a- losartan b- indapamide

9- structure of N-acetyl-p-quinone imine (NAPQI)


(metabolite) and he asking about from which parent
compound
a- Acetominophin

10- B carotene converted to vit A by 2012


a-oxidative cleavage
b-reduction
c-hydrolysis

11- the most common form of CYP450 in GIT and liver


a- cyp 3A4
b- cyp2d6

12-A patient is suffering from acute gout attack, all of the following medications can be
used for him EXCEPT
a-naproxen
b-acetaminophen

13-what is the disease by rickettsia tick porn


a- rocky mountain disease

14- which bacteria in dental and harm valve of heart 2013


a- strep viridans group b- s. aureus
15- which of these enzymes considered as precursor
a- pepsin
b- pepsinogen – breack down to pepsin (aprotease)

16- what is the mechanism of dabigatran


a- increase anti thrompin - direct thrompin inhibitor

17- which drugs used in parkinsonism?


a- pramipexol

18- which drug undergo nonlinear pharmacokinetic?


a- phenytoin

19-which drug cause steven johnson syndrome?


a- hydralazine b-sulfamethoxazole. SASPAN
20-q about effect of valproic acid on drugs which increase its effect?
a- lamotrigine
Inhibits glucuronidation; may decrease clearance of lamotrigine and lorazepam.

21- effect of metoclopramide on digoxin


a- decreases its absorption
b- increases its reabsorption
metoclobramide decrease absorption of some drugs e.g digoxin. While increase the
absorption of other drugs e.g acetaminophen, tetracycline, levodopa, ethanol, cyclosporin

22- this structure is related to? Anticholinergic (quasi ring)

23- which alkaloid cause dry skin, blurred vision, dry mouth, and hyperthermia
a- belladonna
b- colchicine
c- digitalis
24-which drugs cause cystitis, alopecia? cyclophosphamide

25-antidote leucovorine used for which one? methotrexate

26-testosterone  controlled substances part III, written, verbal, refill, partfill, no


transfer

27- methadone is classified as


a- straight narcotic b- controlled c- targeted

28- lorazepam targeted drugs  accepted written, verbal, refill, partfill, transfer

29-In lice treatment, patient should repeat the dose after


a-7-10 days b- every 2 days c-every one month

30-which not used in dandruff treatment


a- selenium sulfide
b- salicylic acid
c- ketoconazole
d- corticosteroid
e- cool tar

31-os  left eye

32-probing question we use it to clarify certain missing information with patient


PROBING: is the use of questions to elicit needed information from patients or to help
clarify their problems or concerns. Several things should be considered before asking a
question:
 The phrasing of the question
 Avoid “why” type questions; For example, people might become defensive if asked
◦ “Why do you miss doses of medication?” instead of “What causes you to miss
doses of medication?”
 It is usually better to use “what” or “how” type of questions.

33 - cortisone monitor: glucose


Glucose should be monitor from the day of starting It's immediate as well as prolong used
side effect could be increased from day1

34- what is the wrong in prescription


a‐cc b-mg c- prn

35-which bacteria causes meningitis in adult


Streptococcus pneumoniae, Neisseria meningitidis and Haemophilus influenzae.

36- which bacteria causes conjunctivitis in children


a- E coli b- staph aureus
Neonates: Chlamydia trachomatis, Staphylococcus aureus, Haemophilus influenzae,
Streptococcus pneumoniae, Neisseria gonorrhoeae, Neisseria meningitidis
Children: H. influenzae, S. pneumoniae, S. aureus
Adults: S. aureus, coagulase-negative Staphylococcus organisms, H. influenzae, S.
pneumoniae, N. gonorrhoeae, N. meningitidis

37- mechanism of montelukast


a- leukotriene inhibitor b-lipoxygenase inhibitor

38- mechanism of candesartan:


angiotensin II receptor blocker  block AT1 receptors

39- which is nifedipine nucleus: 1,4 dihydropyridine

40- lithium side effect:


TREMORS, DIARRHEA, CONFUSION, HEADACH, NEPHROGENIC
DIABETIS INSIPIDUS.
41-LH & FSH secretion:
a- Hypothalamus b- anterior pituitary gland

42- secondary hypogonadism related to 2013


a‐ anterior pituitary gland b-hypothalamus
Hypogonadism occurs when your sex glands produce little or no sex hormones.
primary hypogonadism: problem in the testicles
secondary hypogonadism: anterior pituitary & hypothalamus
What is secondary hypogonadism?
These abnormalities can result from disease of the testes (primary hypogonadism) or
disease of the pituitary or hypothalamus (secondary hypogonadism). The distinction
between these disorders is made by measurement of the serum concentrations of
luteinizing hormone (LH) and follicle-stimulating hormone (FSH).

43- the most drug causes erectile dysfunction of these drugs? alpha blocker Opioids

44-patient with symptoms chestpain, abdominal pain 2013,2012  GERD

45-prodrugs of ACE inhibitors are converted to active forms by


a-oxidation b‐hydrolysis (because its ESTERASE)

46- the aim of metabolism to make compound


a‐decrease oil/water partition coefficient
b- increase o/w partition coefficient

47- Vit A defficiency cause all of the following except:


a‐ glaucoma
b- night blindness
c- keratitis

48- which the toxicity of over dose of ascorbic acid


a‐ urate stones
Megadoses of ascorbic acid may cause: diarrhea, nausea, vomiting, heartburn, abdominal
bloating and cramps, headache and insomnia, kidney stones.
NICOTINIC ACID ALSO (VIT. B3) CAUSE URIC ACID INCREASE

49- which is the name of crystals deposited in organs in gout  urate crystals
50- which of these drugs is antimetabolite
a‐ 5‐fluorouracil
b- cisplatin
c- bleomycin

51- high risk of DVT  knee replacement surgery

52- question related which drug increase metabolism?


a‐Rifampin  potent CYP3A4 inducer

53- which drug is considered biological drug in rheumatoid arthirites


a- leflunomide
b- hydroxycholorquie
c- sulfpyarzone
d‐ etanercept
e-methotrexate

54-which transport iron in plasma


a- ferritin b‐transferritin

55- dextrmetrophan with MAOI


a‐Serotonin syndrome
56- which mechanism of Moclobamide
a‐ selective REVERSIBLE MAO A inhibitor
Moclobemide is a benzamide, derivative of morpholine

57- which is used to decrease prolactin


a-dopamine agnist (Levodopa, Bromocriptin, Ergot alkaloids)
b- Acetylcholine

58-calculation iv= 500 ml and rate of infusion 15 dps/min and standared of drops is 15
drop/ml
so how many times need to finish starting from 1100 hr
answer = 1920 hr

59- A pharmacist decreased the expenses of his pharmacy by $ 30,000 in compared to the
last year , while achieving the same sales, what will be the percent of increasing his net
profit if you know that :
Sales = $ 3,000,000 Costs = $ 2,400,000
Net profit this year = $ 150,000
a) 6%
b) 8%
c) 10%
d) 20%
e) 25%
answer net profit (120000) + 30000= 150000 so new net profit increased by 20%

60-sedimentation rate  stoke’s law

61- today i intend to start medication to loss weight in next 30 days in model change state
is:
1- actions
2- preparatin
3- precontamplation
4- maintenance
Stages of “model change state”
Precontimplation – contemplation – preparation – action – maintenance - relapse

62- vit D is converted from 7 cholesterol by effect of (k type)


a- 7 alpha hydroxy chlesterol
b-sunlight exposure
c-UV

63- which drug cause blue pigmentatuion amidoraone

64- which drug cause photosensitivity amiodarone


Drugs causing photosynsitivity: quinolons, cotrimoxazole, tetracyclins

65-metformin + glubiride interaction  Increase risk of Hypoglycemia

66- rosiglitazone is related to  oxazolidinedione (Thiozolidinedione)

67- a type 1 diabetic patient using NPH, what is the right dose? 2013
a- twice
b- once at bed time
c- after each meal
68- which drugs taken on empty stomach
a- ampicillin b-fluoxetine
penicillins (ampicillins, cloxacillin, pen v), tetracycline, quinolons, iron and ca preparation,
bisphisphonates, levothyroxin

69- ASA is acting as antiplatelet by mechanism


a- acetylation
b- oxidation
c-reduction
d-hydrolysis
antiplatelet effect by acetylation of the platelet cyclooxygenase leading to TX A2 inhibition

70- strucure contain CH2CH3 after metabolism loses it, so what is metabolism
a- dealkylation b- deacetylation

71- warfarin one is least effect


a- polymorphism of CYP2C9
b- allergy
c- taking oral vit k
d- eating green vegetables
e- vitamin k epoxidase polymorphism
if it is asking, what is least effecting warferin, then allergy...if it is asking what will make
warferin least effective then C.

72- female after getting her first baby 2 week she wants a contraceptive and cannot take
pills daily and she wishing starting her contraceptive which one 2012
a- patch
b- vaginal ring
c- DMPA (depo provera)
d- oral esrogen
e-oral progestronn (norethindron)
unlike estrogen, progestron does not bind to prolactine so it does not inhibit milk like
estrogen …its preferred than estrogen

73-infliximap is considered tumer necrosis factor inhibitor

74- which one is rapid insulin


a- aspart (and lispro and glulisin)
b- regular
c-detemir

75-You are a pharmacist and see something opposite to your province’s rules doing by
other pharmacist in a pharmacy beside yours what can u do
a- tell your manager
b-tell his manager For me it’s B
c- tell ……province
Your manger will deal with his manager and it would be more formal than you talk to his
manager. any legal stuff your pharmacy manager is responsible for it

76 -B amyloid protien found in which disease  Alzheimer

76-phrenic nerve to  breathe (nerve to the diaphragm)

77- all contains glucans except


a- bone - these are fungal glycoproteins and fungi does not have bones
b- neurons
c- muscle

78- CK IN all k type 2013


1- skeletal muscle 2- heart 3 -smothmuscle
a- 1 only b- 2 only c- 1,2,3 d-1,2 only

79- when we add aminoglycoside to cephalosporin it becomes inactive due to


a-amide bond b-two methyl group

80- drug used to ttt neuropathic pain


a- pergablin
b-allopurinol
c-azithromycin

81- which is risk factor for osteoprosis


a- hyperthrodism. hyperparathyrodism
b- obesity

82- rituximab is made from


a- human mouse b- mouse c-human
83- which is not found in DNA
a-thyamine instead of uracil

84- what is main duty role of pharmacy manger in hospital: jan2014


a- toward pharmacist b- toward patient

85- which is used in lubricant in tablet manufacure


a- starch
b‐ cellulose
The most famous lubricant is mg. stearate

86- which is used in suspension to improve sedimentation


a- adding flocculating agent…this inhance the particles dispersability
b- increase viscosity of suspension (suspending agent)
increase viscosity of the suspending agent, decreasing the sedimentation rate.

87- Rx asking for role of polysorbate 80  Its nonionic surfactant and emulsifier

88-the best time for female ovulation start after which time
a-7 days
b-14 days
c-21 days

89-you are the manager of hospital pharmacy and assistant come to work in your
pharmacy how u can select him he must have:
a-experience in a hospital pharmacy
b-have a university degree
c-have a diploma

90-which drug increase QT interval  clarithromycin (torsade de pointe)

91- what the role of neutrophil in primary immune respone:


a- engulfing parasite (monocytes and macrophages)
b- opposing antigen (beta cells)
c-inflammatory lipid mediators
basophil and mast cells ………produce histamine
Oesinophils produce antihistamine, involved in protozoal infection, and in allergic Rx
neutrophil … involved in viral, bacterial, COPD
92- which is immunity classified (not remmebr) Innate immunity

a-hummoral and cell mediated


b-innate and humoral
c-humoral and indogenous
93- acetylcystien as antidote for: acetaminophine.

94-what increase survival rate in CHF


a-lisinopril
b-digoxin
c-metoprolol

95-question about peroxisomes


a- oxidation of organic substance
breakdown of the long chain fatty acids through beta oxidation

96-mechanism of action of Alisikrin; renin inhibitor

97-Pharmacist left his break to dispense prescription for patient this is


a-Fidelity
b-Veracity
c-Beneficence

98-drug which cause red man syndrome?


a-vancomycin
b-ampicillin

99-acording to ISMp which is very dangers to mark in prescription


a-qid
b-qod
c-hrs

100-when pharmacist didn't tell patient about side effect violate


a-nonmalficence
b-veracity

101-for what used atamoxiten ADHD disease


Atomoxitine is norepinephrine reuptake inhibitor for ttt of ADHD
102-question to calculate the percentage of codiene and if it needs written prescription.

103-Structure of metformin and belong to what group.2012


Biguanide

104-Structure of omega 3

105-what is the functional group in HCT and acetazolamide.


Acetazolamide: thiadiazole = sulphinamides, HYDROCHLOROTHIAZIDE (HCT):
sulpha gp and halogen

Thiazide diuretics chemistry:


Benzene ring with sulfonamide in position 7, and halogen or trifluoro methyl group in
position 6.
Saturation of3,4 double bond (increases potency with hydrochlorothiazide)
106- (progressive discipline) is the actions that
a pharmacy manager takes towards a
pharmacist who commits a dispensing error
that would be:
a‐ warning
b-telling the responsible authorities to take
actions
c- telling the police
d- applying some sort of punishment now then increase punishment if the pharmacist
recommits the error another time

107- Which of these drug


decrease effect of Oral
contraceptives:
a‐ phynytoin

108- a drug with the following


characteristic:
- low water solubility
- low first pass metabolism
- senstive to light and humidity
- half life 5 hours
would be better formulated as:
a- sublingual tablet
b‐ coated tablet
c- nasal aerosol
d- enteric coated tablet

109- a pharmacist in a community pharmacy give a report to x about Adverse drug effects.
Its better if :
a‐ reported and predicted
b-happened frequently Depends on the question details

c- unpredicted and unreported


d- low incidence but reported
e-high incidence even not reported
110- when giving anti cancer drug what of blood component is to be closely monitored
jan2014,2012
a‐ neutrophils then platelets
b- basophils
c- RBCs
d- Eosinophils

111- pt. want to have co-payment in pharmacy


a-illegal
b‐justice
c-autonomy

112 - drug interaction between Valproic acid and Digoxin: There is no interaction.

113- drug used in lewy body dementia


a‐rivastigmine
b-galanthamine
c- donepezil

112- patient suffering orthostatic hypotension so measure


a- standing then supine blood pressure
b‐ Supine then standing BP
C- setting then standing BP
d- setting BP
e-standing blood pressure

113- Patient with congestive Heart Failure suffered angioneuretic oedema from ACE
inhibitor change to
a- Lisinopril
b‐ Losartan
c- Amlodipine
d- Hydrochlorothiazide

113- Nucleoside differs from nucleotide by: Phosphate


A nucleoside consists of a nitrogenous base covalently attached to a sugar (ribose or
deoxyribose) but without the phosphate group.
A nucleotide consists of a nitrogenous base, a sugar (ribose or deoxyribose) and one to
three phosphate groups.
114- What is used in injectable lipids as surfactant 2013,2010
a‐ Phospholipid (we momken EDTA & Lecithin)
b-Sodium lauryl sulphate
c-Acacia
d- Edentate

115- Child weight is 35 Pound. Toxic dose of acetaminophen is 200mg/Kg. what is the
maximum dose in ml to be adminstrated to this child using a soln containing 160mg/5ml?
a- 20 ml
b 100 ml
c- 90 ml
d- 80 ml

116- Conversion from codeine to


morphine by
a- Dealkylation
b- Deacetylation
c- O-Demethylation

117- Man has renal failure and need to


take anticoagulant which one:
a‐ heparin
b- LMWH

118- what is the peacemaker:


a‐SA node
b-Av node

119- Dysphonia caused by: oral corticosteroid

120- primary refrence is case study

121-person don’t take essential amino acids in his diet what will happen:
1. the body will synthesis amino acids from sources other than protein
2. amino acid synthesis will be impaired Break proteins

122- patient has kideny failure with hepatitis >>>>


a‐ the drug has high concentration in blood
123- GABA the most inhibitory neurotransmitter, jan 2014

124 - Dysphagia means: difficulty in swallowing

125- the pharmasict in hospital>>>> staff

126-increase the dose >>>> increase AUC, jan2014


Cl = k. Vd = F.D/AUC

127- increase Vd >>> decreas AUC 2012


Cl = k. Vd = F.D/AUC

128- why we put surface active agent in the suppositories


a-Increase contact surface area (wetting properties)

129 - methydopa for BP  DOC pregnant

130 - SWOT analysis: S= strenght

131 question for glaucoma MOA for medication…….

132- P value 2012


Regarding P value: the more the confedince interval (100 % - p value), the more the
significance

133- error type 2 false negative

134- error type 1 false positive

135- if we eat chocolate >>>> which enzyme stimulate???


Chechlolate aggravate the secretion of serotonin

136 – Treatment of MRSA  vancomycin

137 - diffusion in semi membrane……… dialysis

138- glycolysis  breakdown of glucose into pyruvate (aerobic) or lactate (anaerobic)


139- monitor TSH serum … for hypothyrodism

140- structure of tetracycline antibiotic

141- Patien on warfarin His INR=4.5 has Not life thread bleeding what should be your
advice him
a) stop warfarin and take Oral K
b) stop warfarin and avoid vegetable
c) skip or reduce dose and monitor

142- question on cost benefit or cost effectiveness

143 -increase the absorption with increasing the surface area

144- used to give informations about molecular weight of compound  Mass


spectrophotometry

145 -health canada advice vaccine (repeated question)

146 - depression with insomnia  trazodone or mirtazipine


Because of severe daytime sedation, trazodone is rarely prescribed at therapeutic
antidepressant doses (300–400 mg daily) but is often prescribed at lower doses (50–100
mg) as a hypnotic in combination with other antidepressants.

147 dispill (repeated question) what is the wrong Phrase

148- patient chronic asthma take salmetrol bid and salbutamol qid and cortisone
149- Calculation: 1 g of X in 5% ointment, we need to prepare 60 mg in compined
ointment. what is the total volume?

150-Calculation: calculate ARR

151- Calculation: Community pharmacy the End inventory = 0000 and the cost of goods =
00000
what is the beginning undetemanet inventory??

152- calculation: C1V1 = C2 V2

153- which drug doesn’t make conjunctivis:


a-morphin
b-dps containing theomersal and benzalkonium
c-drugs causing steven –johonson syndrome

154- What is the vaccine for otitis media? Pneumococcal vaccine and influenza

155-Drug make tremors? Lithium

156-Drug most cause Insomnia if taken night?


A) bupropion
B) trazodone
C) mitrazepine
D) gabapentine

157-Which make change in urine color? rifampicin

158-Mode of action of Fenatanyl patch? Fentanyl is an opioid analgesic.


Fentanyl interacts predominately with the opioid mu‐receptor. These mu-binding sites
are distributed in the human brain, spinal cord, and other tissues.

159-TSH realese from? anterior pituitary gland

160- Filtration pores? 0.22 Micron

161-Warfarin decrease blood coagulation >? 2013


a-inhibit vit K epoxide reductase
162. Li toxicity Increase in case of: 2011
a) Renal failure
b) Increase sodium
c) Old age
d) Enalapril ‐‐‐‐decrease Na/H2O retention so increase sodium excretion
therefore inc Li

163. amiodarone : grapefruit is avoided

164. autoclave: steam steralization under pressure

165. ear wax gland? cerumen

166. Simple syrup contain 85% sugar:


a) support bacterial growth
b) act as preservative.

167. child toxic with paracetamol-first symptom:


a- diarrhea
b‐ nausea and vomiting

168. A woman carrying hemophilia and has a child from a man with hemophilia...which of
the following is Not her child?
a..a girl with hemophilia
b..a boy with hemophilia
c..a girl having no gene of hemophilia
d..a girl having the gene of hemophilia
e..a boy having no hemophilia

169. to minimize S.E of ASA  take with food

170. pt. With ulcerative colitis take all except;


a‐ naproxen
b- salfasalazin
c- infleximab
d- corticosteroid
171. which antifungal available as oral suspension.
A. Fluconazole
B. Itraconazole
C. Nystatin
D. Clotrimazole

172. lymphatic circulation all except; connected with general circulation

173. drug highly excreted:


a‐Glomerular filteration
b-activ tubular reabsorbtion

174. lactating mother take penicillin hypersensitivity in child due to;


a-high dose
b‐dose independent

175- ribozymes
a-classified as a polynucleotide catalyst
Ribozymes (ribonucleic acid enzymes) are RNA molecules that are capable of
catalyzing specific biochemical reactions, similar to the action of protein enzymes.
The ribozyme catalyses specific reactions in a similar way to that of protein
enzymes. Also called catalytic RNA, ribozymes are found in the ribosome where they
join amino acids together to form protein chains.
Day 1 Jan 2014
1. Which drug used for treatment Neutropnia with anticancer drugs :2012
a) Filgrestim
b) Cyclophtsphamide
c) Oprelvekin  prevention of thrombocytopenia and reduce the need for platelet
transfusion

2. What is the main component of blood protein:


a) Albumin
b) Glycoprotein
c) Creatinin

3. The difference between Nucleoside and Nucleotide:


a) po4
b) Ribose
c) Quinine base

4. What make lactic acidosis in the following drugs: 2012


a) Metformin
b) ASA
c) Insulin

5. The best treatment of H. Influnanza:


a) Ofloxacin
b) Tetracycline
c) Amoxicillin and Clavulanic acid

6. Which isn’t used in treatment of Chlamydia:


a) Amoxicillin  treatment of chlamydia in pregnancy
b) Ofloxacin
c) Doxycyclin  preferred treatment of chlamydia, also azithromycin

7. The 2 following compounds are:


a) Optical isomers
b) Geometrical isomers
c) Structure isomers
Geometric isomers are chemical species with the same type and quantity of atoms as
another species, yet having a different geometric structure. Atoms or groups exhibit
different spatial arrangements on either side of a chemical bond or ring structure.

8. in ttt f a heart problem the drug costs 500$ this is considered as:
a) Cost benefit analysis
b) Cost utility analysis
c) Cost effective analysis

9. A patient received prednisolone for 3 weeks what the best monitoring for him:
a) Glucose monitoring
b) Hypoglycemia
c) ECG
d) Creatinine clearance

10. What is the best tttt of ADHD (Attention Deficit Hyperactivity Disorder):
a) Methylphenidate
b) Brufen
c) ASA

11. What is the main side effect of pioglotazone :2012


a) Skin rash
b) Increase liver enzyme
c) Diarrhea

12. Microcytic anemia is characteristic by: 2013,2012


a) Increase iron binding capacity
b) Serum ferritin
c) Vit. B12
deficiency in iron It characterized also by decreased V.B6

13. what is the drug safe (use) in pregnancy:


a) Gentamycin
b) Ofloxacin
c) Insulin
14. A women usually forget oral contraceptive pills (smoker) what is suitable ttt for her:
a) Ethinyl estradiol
b) Progestin
c) Depo provera IM

15. one of the following drugs if take by wrong can-do dangerous side effects:
a) Methotrexate  narrow therapeutic window
b) Leflunomide
c) Ciprofloxacin

16. one of the following drugs can use as


anthelmintic:
a) Cotrimisaxazole
b) Metronidazole
c) Mebindazole

17. Portal vein initiated to liver from:


a) Heart
b) Kidney
c) Small intestine

18. This is the following structure (Ach) Is inhibited by:


a) ACH cholinesterase
b) αreductase
c) Βlactamase

19. Popliteal pain is characterized in:


a) D.V.T
b) Skeletal
c) Headache
Popliteal vein thrombosis is a condition in which a blood clot develops in
the popliteal vein. Popliteal vein thrombosis is a type of venous thromboembolism
(VTE), which is also called deep vein thrombosis (DVT)

20. Dysphasia is impaired in:


a) Speech
b) Taste
c) Dyspepsia
21. Which of the following herds is rick factor in hypertension: 2012
a) Liquorice
b) Escherichia
c) Era berry

22. Which of the following of the main cause of CAP (community aquiral pneumonia):
• S. pneumonia
• S. aureus
• Clorstridum difficle
(There are 3 causes: S. pneumonia, M catarrhialis, H. influanza) thats also causes of
otitis media ...

23. Which drugs cause hemorrhagic cystitis:


a) Cyclophosphamide
b) Amikacin
c) Gentamycin

24. Patient receiving venlafaxine for long time must be monitored for:
• Hypertension
• Glucose
Monitoring blood pressure and heart rate once a week is recommended during the 1st
month of treatment and when increasing doses.

25. What is the main side effects of Felodipine:


a) Angioeodema
b) Pruritis
c) Headache
S.E: Ankle edema, flushing, headache and palpitations).

26. Which drug can be used for topical treatment of Alopecia:


a) Hydralazine
b) Finistride
c) Minoxidil

27.ttt for MRSA (DOC): (MRAS: Methicillin Resistant Staphylococcus Aureus)


a) Metronidazole
b) Vancomycin
c) Amoxacyllin
28. Which drug of the following can cause Tremors, Diarrhea, and headache:
a) Lithium
b) Ofloxacin
c) Propronalol

29. Atrial fibrillation is characteristic by: 2013,2012


a) P wave b) QT c) QRS
Atrial Fibrillation (AF) is the most common abnormal heart rhythm.it
characterized clinically by the absence of P wave in the ECG.

30. In which reference we can find newly recent drug:


a) CPS
b) Canada drug database
c) TC

31. The aim of using (enteric) coated tablet is:


a) Delay the effect by protecting from gastric acid
b) SR (sustained release)
c) Controlled release
An enteric coating is a polymer barrier applied on oral medication that prevents
its dissolution or disintegration in the gastric environment. This helps by either
protecting drugs from the acidity of the stomach, the stomach from the
detrimental effects of the drug, or to release the drug after the stomach (usually
in the upper tract of the intestine)

32.In collegative properties which is true:


a) Increase freezing point
b) Increase vapor pressure
c) Increase boiling point

33. Which drug can be used as a laxative for children > 1 y:


a) Senna
b) Biscodyl
c) Polyethylene glycol 3350
Infants Rectal disimpaction of infants can be achieved with pediatric glycerin suppositories.
< 1 Year Barley malt extract, corn syrup or sorbitol can be used as stool softeners
Use of enemas, Heavy mineral oil and stimulant laxatives are not recommended.
Children PEG (superior), lactulose or sorbitol are considered first-line agents for the treatment of
≥1 Year constipation whereas magnesium hydroxide is second line.
Heavy mineral oil is considered third line to avoid risk of lipoid pneumonia.
Senna and bisacodyl can be used as rescue medication when other agents have failed.
Disimpaction with enemas is recommended after diagnosis of impaction via rectal
examination.
After disimpaction, maintenance therapy should be initiated for 1–2 months once the child
has >3 stools/week.

34. The death of cell is called:


a) Necrosis
b) Growth
c) Replication
Necrosis is a form of cell injury which results in the premature death of cells in living
tissue by autolysis.

35. Which of the following symptoms cause burning sensation:


a) GERD
b) Peptic ulcer

36. A patient is receiving MAOIs contraindicated to take:


a) Dextromethrophan  risk of serotonin syndrome
b) ASA
c) Metronidazole

37. liver is the main source of synthesis of: Vita K (wrong answer)
Coagulation factors I (fibrinogen), II (prothrombin), V, VII, VIII, IX, X, XI, XII, XIII, as
well as protein C, protein S and antithrombin are proteins that synthesized by liver.
Vitamin K is found in a number of foods, including leafy greens, cauliflower and, if you
consider it a food, liver. However, the chief source of vitamin K is synthesis by bacteria
in the large intestine, and in most cases, absence of dietary vitamin K is not at all
deleterious. Vitamin K is a fat-soluble vitamin and both dietary and microbial vitamin K
are absorbed into intestinal lymph along with other lipids. The fetus obtains vitamin K
from its mother by transplacental transfer.

38. What is the enzyme responsible for transfer DNA to mRNA:


a) Reverse transferase enz
b) RNA polymerase
c) DNA
39. Protein C is used as: 2012
a) Clotting factor (its anti coagulant)
b) Growth H
c) Collagen

40.For which reason Ab used in a high dose:


a) Tolerance
b) Resistance
c) Inoculation
Inoculation, process of producing immunity and method of vaccination that consists of
introduction of the infectious agent onto an abraded or absorptive skin surface instead
of inserting the substance in the tissues by means of a hollow needle, as in injection.
Of the common vaccines, only smallpox vaccine is routinely inoculated. The term
inoculation is also commonly used more broadly to mean any introduction of antigenic
substances into the tissues.
Immunization refers to the use of all vaccines but also extends to the use of antitoxin,
which contains preformed antibody such as to diphtheria or tetanus exotoxins.

41 which components can be used in ear preparation:


a) Borax
b) Glycerin
c) Polyvinyl alchol

42. Bacterial sterilization can be done by:


a) Filtration b) Radiation c) Gaz

43. A women is forgotten to taken 2 pills in the 2nd week:


• Take 1 pill today and 1 pill in the other day
• When she remembers take 2 pills today and 2 pills in 2nd day
• Take 1 tablet ASAP, continue the strip, discard
the free hormone tablets and start new strip.

44. the difference between anhydrous and hydrate


except:
• Both have the same solubility
45. Enkaphalin act mainly in which of the following receptors to produce analgesic effect:
a. Mu‐receptor
b. Delta (the agonist is enkaphalin)
c. Kappa
Enkephalins are endogenous opioid pentapeptides that are produced mainly in the central
nervous system, adrenal medulla, and other peripheral tissues. involved in
regulating nociception in the body. two forms of enkephalin have been found, one
containing leucine ("leu"), and the other containing methionine ("met"). Both are products of
the proenkephalin gene.
The receptors for enkephalin are the delta opioid receptors and mu opioid receptors. Opioid
receptors are a group of G‐protein‐coupled receptors, with other opioids as ligands as well.
The other endogenous opioids are dynorphins (that bind to kappa receptors), endorphins (mu
receptors), endomorphins, and nociceptin/orphanin FQ.

46. Which drug used for prophylactic of travel diarrhea:


a) Ciprofloxacin
b) Ampicillin
c) Vancomycin
Bismuth subsalicylate, Cipro, levo, norfloxacin can be used for prophylaxis. Azithro for treatment only.
Consider antibiotic prophylaxis only for those who are at greatest risk of TD or its complications (e.g.,
patients who are immunocompromised or have comorbid disease), those who undertake critical
travel (e.g., diplomatic missions) or those who have diarrhea every time they travel.
When used for prevention of TD, antibiotics should be prescribed for short courses only (<3 weeks)
Cipro  Start prophylactic on the 1st day in the area of risk and continued for 1-2 days after return
home, to maximum 3 wk total.

47. When Levothyroxine is used in an overdose what will it cause?


a) Bradycardia
b) Hypertension
c) Wt loss

48. In elderly patient all the following happen except:


a) Increase in BUN
b) Decrease Cr cl
c) Increase in GIT motility

49. In DNA strand binding:


a) AC-GT
b) AG-CT
c) AT‐GC
50. Kaplan miere equation used for:
a) Survival rate
b) Apoptosis

51. What is the characteristic in asthma:


• Wheezing
• Constipation
• Skin rash

52. NA lauryl sulphate is 2013


• Cationic surfactant
• Ionic
• Anionic

53. Lorazepam can be found in which:


• Straight narcotic
• Exempted
• Benzodiazepines and targeted drug

54. Which drug can’t make (cause) bleeding (less bleeding):


• Naproxen
• ASA
• Celecoxib

55. Which can’t take in last 3rd trimester:


• ASA
• Warfarin  contrindicated in pregnancy also
• Acetaminorhen

56. What drug causes gingivitis (gingival hyperplasia):


• Valproic  neural tube defects
• Phenytoin
• Erythromycin  safe in pregnancy

57. Which drug having aromatic ring:


• Progesterone
• Estradiol
• Testerone
58. What is effect of carbidopa with L. dopa:
• Prevent metabolism peripherally of L. dopa
• Prevent metabolism centrally of L. dopa

59. What is the mechanism of Triptan:


• 5HT 1B/1D agonist
• Dopamine
• NE reuptake inhibitor

60. Which drug prolonged life and having –ve inotropic effect in CHF:
• Ramipril (it acts on vesseles as its an ACE inhibitor AND cause reflective
tachycardia)
• Digoxin
• Carvedilol
Carvedilol acts by blocking b1 and b2 and alfa1. The
vasodilatory effects of carvedilol reduce afterload, and the resulting decrease in
impedance to left ventricular ejection offsets the negative inotropic effect that
would normally result from beta‐blockade.
(It could be ramipril if he does not mention the –ve inotropic effect)

61. Which of the following drug can be used as divided dose:


• Losartan
• Capoten
• Enalapril

62. Smoker having deficiency in what vit.: 2013,2012


Vit C Vit B12 Vit D

63. What is mode of action Aiskiren:


• Calicium channel blocker
• ACE I
• Inhibitor of Renin Release

64. Which of the following is least in decreasing error in prescription:


• Appearance of the drug
• Date of prescription
• Dose of the drug
65. Which of the disadvantage of sole proprietorship pharmacy:
• More liabilities
• Owner should take opinion by himself
• Governmental interruption

66. Which study is the most credible study:


• Randomized double blind study
• Case study
• Cohort study

67.What is the reaction between morphine and


codeine:
a. codeine considered a produg of Morphine
• Acetylation
Morphin with one methyl group, codien with 2
methyl gps, heroin is 2 acetyl gp so the most
basic one so the most potent as it is the most that pass BBB is HEROIN.

68. plan –B can be taken with from intercourse to be effective: jan&july2013


• 48 hours
• 5 days
• 72 hours

69.Define inventory turn over rate


ITS THE COST OF THE GOODS SOLD /AVERAGE OF INVENTORY CAPITAL

70. ABC pharmacy has following figures appearing on its balance sheet.
Cost of goods sold $500,000 Beginning inventory $200,000
Ending inventory $220,000 Total current assets $150,000
Total fixed assets $40,000 Total liabilities $75,000
Inventory turnover rate for ABC pharmacy is
a) 4.3 b) 2.3 c) 3.5 d) 6.0 e) 8.0

T.O = 500,000 / (200,000 + 220,000)/2) = 2.3


71. What is the enzyme responsible for methylation:
• S-adenyl methionine
• S-transfer
• Sulphoxidation

72. What is the most burden in health Canada expense:


• Hospitalization

73. Fasting blood glucose threshold


a.7mmol/L (NORMAL 5‐6 mmol/L)

74.What is indicate for patient with hypertension No risk factor:


• 140/90
• 120/80
• 130/90
THAT’S IN CASE HE HAS NO CONCOMMETANT RISK FACTORS e.g DM or CHF … IF SO,
130/90)

75. Which is least interaction with oral contraceptive: 2012


• Gabapentine
• Valploric acide
• Methotrexate

76. What protein stimulate:


Gastrin Amylas Lipase

77. What is the mechanism of pioglotazone :2013,2011


• Inhibit PPAR (Peroxisome Proliferator Activated Receptor gama)

78. Isotretinoin must be monitored:


Baseline & monthly pregnancy tests must be performed by women of child‐
bearing age.

79.which drug treatement of influenza B


• Ribavirin
• Oeslemivin Oseltamivir
• Amantadine
80. Patient admitted in alcoholic toxicity having a deficiency in which vitamin:
• Thiamie (B1) and vitamin 2 also (ribavirin)
• Vita C  smoking
• Vita B12

81.what the most symbol take the pharmacist attention in RX:


• I.V (IT CAN BE CONSIDERED I.U)
• PRN
• Kg

82. Which drug don’t make photosensitivity:


• Penicillin
• Isotretenoin
• Ciprofloxacin
• Gentamycin

83. The telangiectisia is symptom of:


• Acne roscea
• Acne vulgaris

84.what is 2nd line treatement of UTI: check nitrofurantoin


• Fluroquinolone
• Nitrofurantion
• Gentamycin
Syndrome Infecting organism & Symptoms Firs‐Line Second‐Line
Acute Escherichia coli (80– 90%), SMX/TMP PO × Fluoroquinolone
Uncomplicated Staphylococcus saprophyticus (5– 3 days or (ciprofloxacin,
UTI (Cystitis) 10%), Trimethoprim levofloxacin,
Occurs in Klebsiella pneumoniae, Proteus PO × 3 day or norfloxacin), PO
females with mirabilis Nitrofurantoin × 3 days
normal Behavioural factors promoting PO × 5 days or or
genitourinary infection include sexual intercourse Fosfomycin Cephalexin PO ×
tract. and use of spermicide or diaphragm. tromethamine 7 days
Usual presenting symptoms include as single dose
internal dysuria, frequency, PO
suprapubic discomfort and urgency.
Recurrences are common.
85. What is the side effect of phenytoin:
• Diahrrea
• Skin rash
• Convulsion
S.E: Gingival hyperplasia; rash 5–10%, which rarely can be very serious; increased liver
enzymes; blood dyscrasias; dose-related encephalopathy; coarse facial features.
Idiosyncratic: Skin rash and Steven Johnsons syndrome. Birth defects like cleft palate

86. Molarity of sol.is [0.02M] what is Ph?


a.1.2 b.1.7
PH = - LOG (MOLARITY) = - LOG (0.02) = 1.7
To find the pOH take the pH value and subtract it from 14.
14 − 1.7 = 12.3 = Poh

87. Which of the following cause nephrolithiasis


a. Ca b. K c. Mg d. lithium e. Po4

88. A patient having overdose of phenytoin which symptoms occur:


a. thrist b. diarrhea c. gait disturbance d. Nystagmus
Toxic concentration of are between (7.5 mcg/ml to 20 mcg/ml). Overdose symptoms due
to saturation zero order (saturated) kinetics is Nystagmus (repetitive, uncontrolled eyes
movements)

89. Which of the following is not fiber:


a. strach b. cellus c. lignon

90.Cystic fibrosis (& Atelectasis) is ttt by drug DORNAZE ALPHA taken by:
a. IV b.IM c. Inhalation e. Nasal inhaler
Recombinant human deoxyribonuclease I (rhDNase), an enzyme which selectively cleaves DNA.
Dornase alfa hydrolyzes the DNA present in sputum/mucus of cystic fibrosis patients and reduces
viscosity in the lungs, promoting improved clearance of secretions. This protein therapeutic agent
is produced in Chinese hamster ovary cells.

91. What the following drugs used for diabetic patient to protect Kidney:
a. ACEIs b. Furosemide

92. What of the following disease cased rapid destruction of platelets:


a. SLE  cause thrombocytopenia (low platelets)
93. CARN journal is published by:
a. Medeffect (health Canada)
New S.E: the pharmacy makes a report to “Med effect” which inform “CANADA
VIGILENCE” TO INFORM “HEALTH CANADA” AND “CADRIS”
“CADRIS” PRINT THE “CARN” (CANADA ADVERSE REACTION NEWSLETTER)

94. What type of bonds join disaccharide: glycoside

95. Hydroxylapatide is considered as: 2012


a. mineral in bones a naturally occurring mineral form of calcium apatite

96.Equation of calculate number need to treat: 1/ARR x 100 = 9%

97.What the following type of irregular rhythm (beats) check


• Paroxysmal rhythm
• Atrial fibrillation
• Ventricular fibrillation
• Supra ventricular tachucardia
A fib is typically irregular beats. In ECG ull find p wave missing...
V fib is prolonged and fast beats....

98.biphosphonate mechanism of action:


• Inhibit osteoclast resorption
• Inhibit osteoblast resorption
• ↓ Ca execretion
• ↑ Ca absorption

99. Deficiency of folic acid during pregnancy causes teratogenicity:


a. spinabifida (NTD neural tube defect)

100. ttt of impetigo for child at 10 years and has 2 spots under nose:
• Oral erythromycin
• Oral amoxicillin
• Topical mupirocin
Imptigo is a highly contagious bacterial skin infection most common among pre
school children. mild cases ttt with bactericidal ointment e.g mupirocin. more sever
cases require oral antibiotics e.g dicloxacillin, flucloxacillin, or erythromycin
101.Patient with prostate cancer what following is sign of cancer: check
• Urine increase
• ↑PSA the question is asked for signs. If it was a symptom then it would be C.
• ↑urine frequency
Symptoms: difficulty in urination, blood in urine, pain in pelvic or back or when
urinate

102.Nystagmus (dancing eye) is characterizrd by all except: (K type)


a. Retire b. promusing eye c. Oscillation
Promising eye mean dry eye
Retire eye mean eye injury

103.The stage where the drug goes to! Real world w phase: Phase 4

104. Counseling for patient taking VIMOVO (naproxen + esomeprazol)


• Don’t take antacid while take vimovo
• Take vimovo at least 30 minutes after meal
• Take vimovo at least 30 minutes before meal  should be taken with food as per
CTC
• Vimono should be taken with milk
VIMOVO is drug used to ttt osteoarthritis, rheumatoid arthritis ..

105. All used for breast cancer except:


• Tamoxifen
• Anastrazol
• Letrozole
• Becalutamide for prostate cancer

106. Stereoisomers effect all of following except:


• Passive diffusion
• Active diffusion
• Facilitated diffusion

107. Cohich septal defect means: 2012,2009


Opening or ahole in the septal wall between right and left side (could appear between
ventricle VSD and atrium ASD)

108. Pregnant women with dental abscess which is recommended: Amox/clav


109. In order to protect protein when freeze drying which should be add:
• Humactant . Lyoprotectant

110.Prednisolone 5 mg tab taken for 30 days Decrease tablets by 5mg every other day.
The Insurance company will only cover a 30-day supply so how many tablets will u
dispense for the thirty-day supply?
40 mg for 2 weeks - 30 mg for 2 weeks - 20 mg for 2 weeks
a.230 b.204 c.196 d.184

111.The cost of product is 2 $ and retail price is 2.5$


a. Gross margin is b. mark up is
A-Gross Margin is 2.5-2=0.5 = 20%
B-Mark up is ((2.5/2)-1) *100 = .25 =25%

112. O2 toxicity due to accumulation of: free radical OH

113. Add solute is ↑ solubility of product increase: cosolvent


Salting in is the addition of salt leading to the increase solubility of another salt
Cosolvent: addition of solvent less than 5% leading to increase the solubility

114. A pharmacy manager heard from some of workers that a technician was discussing
with patient on OTC drug what would be the appropriate action?
• Make a meeting for all pharmacy members to discuss issue
• Tell the pharmacist to deal with him
• Make a report
• Talk with him in private area about issue

115.The eye adaptation to see near vision is called: accommodation


116. Secondary prevention of cardiogenic shock (Post MI):
• ASA
Cardiogenic so warfarin
• ASA, dipyrimidol (AGGRENOX)R
• Warfarin

117- What is the property of the drug that can be found in breast tissue with high
concentration.
a) Low Molecular Weight
b) Low Lipid Solubility
c) High Protein Binding
d) High Acidic product

118- Prescription adaptation is ...


When a pharmacist renews a prescription for a patient under the pharmacist name And
we can only adapt those prescription which patient is taking from past 6 months if less
then that we can't adapt it ....and with new adapted prescription we have to get a copy of
patients old original prescription and fax a documentation to Dr that we adapted .....one
more thing if pt got prescription from other pharmacy and we transferred from other
pharmacy we can't adapt that coz we don't have original prescription for
documentation......NO adaptation for narcotics and controlled even benzodiazepines.
Day 2 Jan 2014
1. In smoking patient what is the vitamin deficiency:
• vit. E
• vit. C
• vit. D

2. Rutiximab is taken from (biological derivative):


• mouth
• human / mouse
• rabbit
Mouse Chimeric Humanized Human
100% 25% mouse 10% mouse 100%
mouse human
Muromonab Infliximab Trastuzumab Adalimumab
Rituximab Omalizumab
Abciximab Daclizumab
Cetuximab
Basiliximab

3. Storage of glucose in animals is the:


• glycogen
• sucarose
• fructose

4. Trihelix DNA structure in which three oligonucleotides wind around each other
and form a triplehelix (Triple‐stranded DNA)

5. Which type of capsule having aqueous preperation:2013


• Soft capsule
• Hard capsule
• Controlled

6. What increase absorption of uric acid: ascorbic acid


High vitamin-C intake shows an inverse relation with UA as such vitamin has a uricosuric
effect due to competition of UA renal resorption through a change in the anion
transportation system in the proximal tubule  increasing renal fractional clearance
of uric acid, thereby reducing SUA
7. This structure act on :2009
• Hanthiene
• Hydroxyxanthiene
• Xanthane oxidase

8. Which drug can’t be taken during pregnancy:


• Isotretinoin
• Erythromycin
• Vit B

9. Which disease is not autoimmune disease:2012


• Osteoarthritis
• RH. Arthritis
• Multiple sclerosis

10. Glutathione conjugation produce the product called?


A. Mercapturic acid
B. Sulfonation
C. Oxidation
D. Glutathione conjugation
E. Reduction

11. in which disease myelin sheath degeneration (demylenation)


• Multiple sclerosis

12. Test for pharmaceutical ability to withstand abrasion: friability

13. Tranformation (metabolism) of prodrug to drug is:2011


• Bioactivation (Prodrug to active drug)
• Bio transformation

14. Which hormone control in Ca release:


• Parathyroid H
• TSH
• FSH

15. Bisphosphonate related in structure to:


• Vit D
• Calcitonin
• PyroPhosphate
Bisphosphonates are structurally similar to pyrophosphate, but with a central carbon
that can have up to two substituents (R1 and R2) instead of an oxygen atom.
Bisphosphonates, when attached to bone tissue, are released by osteoclasts, the bone
cells that break down bone tissue.

16. What is the major inhibitory neurotransmitter in the brain:


• GABA
• Glycine
• Ach

17. What drug is uricosuric agent:


• Naproxen
• Sulfinpyrazone
• Colchicine
URICOSURIC IS SUBSTANCES INCREASE THE EXCRETION OF THE URIC ACID IN URINE,
e.g sulfinpyrazone, probenicid

18. Vitamin different with alcoholic patient toxicity:


• Thiamine (Vitamin B1)
• Vit.B12

19. In patient smoker using patch suffering from itchy at night:


• Replace with gum
• Remove the patch at night
• Put 2 patch
Rash is different from Itching...Itching means that once the patch is removed, he will
restore normal state. He will remove the batch over night and replace it again once he is
awake to stop craving for nicotine

20- Smoking cessation symptoms all except:


• Techyarcardia
• Headache
• Confusion
• increased appetite
Withdrawal Symptoms: Dysphoric or depressed mood, irritability, anxiety, difficulty concentrating,
restlessness, increased appetite/weight gain, GI symptoms, headaches and insomnia. No myalgia.
21. Anaerobic bacteria is mostly found in: 2012,2011
• Duodenum
• Colon
• Jeujenum

22. What is the side effect of isoniaziade:


• Jaundice
• Headache
• Diarrhea

23. What is the key word in cross sectional


study: Wash out

24. What is the treatment of hepatitis B:


a) antibiotic
b) interferon alpha
c) amphetamine

25. Effervesent tablet: must be dissolved in the water

26. Patient suffering HIV and his CD4 is more than 150 you must check:
• Pneumonia (carrini)
• TB
• Hepatitis

27. In an adequately powered, randomized controlled trial conducted over 3 years, a


specific serious side effect (i.e., reduction in leukocytes) with conventional therapy is seen
in 0.5% of the study sample. In patients who receive a newly discovered drug, only 0.45%
experience the same side effect. Based on these results, the minimum number of patients
that would have to receive the new drug for 3 years to statistically demonstrate the
prevention of one episode of this side effect in at least one patient is:
a. 15. b. 20. c. 150. d. 200. e. 2000
ARR= 0.5(%) - 0.45 (%) =0.05 %=0.0005
NNT=1/ARR=1/0.0005=2000

28. which drug used in decrease size of prostate:


a) fenestride
b) α –blocker
29. What is side effect of Acarbose:
Flatulence, diarrhea, abdominal pain, cramps, nausea

30. Spirometer detected?


• total lung capacity
• blood pressure
• gravity

31. Glyburid belong to:


• biguanide
• meliginatide
• sulfonylureas

32. sulbutamol belong to or act as:


• β –blocker
• β 1-stimulant
• β 2‐stimulant (β 2‐agonist)

33. Which of the following don’t cause hypoglycemia:


• glybiride
• metformin
• meglitinide

34. What is Tacrolimus used for:


• atopic dermatitis
• psoriasis
• cellulites

35. Structure of …
• tricycline antidepressant
• phenothiazine
• benzodiazepine

36. Which inhaler need shake before use:


• fluticazone
• salbutamol

37. Hyperplasia means: 2010


• cell division
• cell death
• increase in cell number

38. How to treat dry skin: Apply emollient after the bath

39.What is increase QRS prolongation:


• Vancomycin
• Fluroquinolone  Azithromycin & clarithromycin
• ASA
• NA channel blockers

40.In community pharmacy elderly patient want to pass the line to take his medications,
this patient is a known neighbor for pharmacy:
• Dispense the prescription direct
• he must wait in line and respect the roles
• Excuse from the other patient

41. Which of the following in liver microsomal inducer CYP 450:


• Grape fruit
• Clarithromycin
• Rifampin  Rifampin is a very potent enzyme inducer of CYP450 3A4

42. Which is of the following not advertising:


• schedule F
• schedule A
• schedule B

43. Which is not taken with Sildinafil:


• Nitroglycerin
• β –blocker
• α -blocker

44. Osmotic pump is following to :2013,2011


• zero‐order
• 1 st order
• 2nd order
Osmotic pumps are most promising systems for controlled drug delivery. These systems
are used for both oral administration and implantation.
Osmotic pumps consist of an inner core containing drug and osmogens, coated with a
semipermeable membrane.
As the core absorbs water, it expands in volume, which pushes the drug solution out
through the delivery ports. Osmotic pumps release drug at a rate that is independent of
the pH and hydrodynamics of the dissolution medium.
For osmotic drug delivery system, highly water-soluble drugs would demonstrate a high
release rate that would be of zero order

45. Which is not can be transferred:


• Methylphenidate
• Benzodiazepine

46. What is the prophylactic of migraine:


• Sumatriptan this is for ttt
• Propranolol
• Carvedilol

47. J.M. is a patient in last stage renal failure, drug excreted 10% unchanged in urine.
What should we do regarding the dose:
a) No need to adjust the dose
b) Reduce the dose to half
c) Reduce the dose to quarter
d) Increase interval to double

48. Which is stable against β – lactamase:


• Cloxacillin
• Pencillin V
• Amoxicillin

49. Acetic acid conjugated with:


• Glutathione
• Glycine
• Glutamate

50. Leucovorin is antidote of:


• Benzodiazepam
• Methotrexate
• 5 –flurocuracil
Leucovorin (folinic acid) is adjuvant used in cancer therapy involving in the drug
methotrexate.it differ from folic acid.its however used as methotrexate antidote.

51.Definition of disclosure: (whistle blower)


‐the act of releasing all relevant information pertaining to a company.
‐aperson who informs on a person or organization engaged in an illicit activity.

52.Rynaud disease symptoms except


Rynaud’s disease: Refers to blanching of the fingers, mainly when exposed to a cold environment,
followed by a second cyanotic phase then by a reactive hyperhemia phase. The combination of
these phases, from white to blue then red constitutes classic Raynaud phenomenon.
Reynaud's Phenomenon is NOT caused by plaques but result of over sensitive blood vessels in the
body extremities which become vasospastic because of atherosclerosis, connective tissue disease,
ingestion of ergot alkaloids, or frequent use of vibrating tools.

53. End metabolite of protein: 2013


• Urea
• Lactice acid
• Uric acid

54.Which of the following factors affect on passage of a drug through the stratum
corneum? 2013,2015
a) Solubility
b) Dissolution
c) PKa
d) Partition Coefficient
e) Particle size

55. What is the difference between lispro (insulin) and glyburide:


• The peak of lispro come 1st
• Both marking hypoglycemia

56. What is the accurate test of hypothyroidism:


• Serum TSH
• Sensitive TSH
• T4
57. Which of the following is taken as divided dose:
• Loratadine
• Miclizine
• Diphenhydramine 1st generation

58. Nifedipine is classified as:


• Dihydropyridine
• Non-dihydropyridine
• Benzodiazepine

59. VLDL is derived from:


a. APO‐C b. APO-D c. APO-E
abo B  LDL & VLDL, abo C  HDLP & VLDL

60. Creaming problem is in:


• Tablet
• Suspension
• Emulsion

61. Mechanism of action of Metoclopramide:


• MAO inhibitor
• Ca-channel-blocker
• Dopamine antagonist
a dopamine D2 antagonist but also acts as an agonist on serotonin 5‐HT4
receptors and causes weak inhibition of 5‐HT3 receptors.

62. Which of the following is not a viral disease:


• Rubella
• Cold sore
• Impetigo (S. aureus bacterial)

63. Aqueous humor is secreted from:


• Granulamous
• Ciliary gland

64. polyvinyl alcohol in eye drops is used as:


• Viscosity agent  also lubricant
• Surfactant
• Preservative

65. What is the measure of flowability:


• Angle of repose
• Wetting agent
• Decrease particle size

66. What is the meaning of angiogenesis:


• Formation of new blood vessels
• Formation of arteries
• Formation of veins

67. Fishy odour is characterized in: 2009


• Candida  cottage discharge
• Bacterial (bacterial vaginosis)
• Trichmoniasis  frothy, wet discharge, pruritus

68. Which of the following need vertical laminar:


• Cyclophosphamid
• Doxorubicin (drug used in cancer chemotherapy)
• Bleomycin

69. Dysgeusia mean: also known as parageusia, is a distortion of the sense of taste.

70. Unit of K-constant:


• Time ¯¹
• e-minute

71. The least drug excreted from breast milk:


• basic drug
• weak acid drug
• strong acid with high protein binding

72. Unit of creatinine clearance: ml/minute

73. What is the process of adding insoluble powder to solvent for decrease particle size:
a. levigation b. pulverization c. grinding
74. What is not risk factor for osteoporosis
a) low Ca intake b) low vit D intake c) obesity

75. Which type of patient can’t use compressive stocks:


• pregnant women
• coronary heart disease
• hypertension
Indications Contraindications
Edema. Suspected or proven peripheral arterial disease, including history of
Chronic venous insufficiency. peripheral arterial bypass grafting
Varicose veins. Severe peripheral neuropathy or other cause of sensory impairment
Deep vein thrombosis. Allergy to stocking material
Lymphedema. Massive leg edema or pulmonary edema from congestive cardiac failure
Phlebitis. Local skin or soft-tissue condition, including recent skin graft, fragile
Lipodermatosclerosis. “tissue paper” skin, gangrene, oozing dermatitis and severe cellulitis
Pregnancy. Extreme deformity of the leg, or unusual leg shape or size preventing
correct fit

76. Which of the following increase the bulk formation in the tablet manufacturing:
• Diluent binder glidant

77. Pharmacy manager in hospital pharmacy is resp. towards: 2012


• pharmacy stuff
• nurse stuff
• patient

78. Which measure increasing the pressure stand leading to abrasion:


• Friabilator
• sticking
• molting

79. for a pharmacist which is more accurate to identify the medication:


• DIN
• Batch No.
• Price

80. Why we take influenza vaccine (flu) every year:


• The virus changes every year
• Efficacy is decreased
• Resistance
81. What is the initial symptoms for acetaminophen toxicity:
• Nausea
• Diarrhea
• Confusion

82. Which of the following cause dry mouth (has anticholinergic effect):
• Benzotropine
• Pilocaprine
• Neostogmine

83.what is the mechanism of candesartan:


• Angiotensin II receptor blocker
• Ca channel blocker

84. What is the treatment of hypertension diabetic:


• Ramipril (ACEI)
• Propanolol
• Thiazide

85. Which of the following is Not used as antiemetic in cancer:


• Diphenhydramine
• Dexamethazone
• Ondansetron

86. What is the meaning profitability: Cover the cost with little revenue for pharmacist

87. PCR meaning:


Polymerase chain reaction
Polymerase chain reaction (PCR) is a
method widely used in molecular biology to
make several copies of a specific DNA
segment. Using PCR, copies of DNA
sequences are exponentially amplified to
generate thousands to millions of more
copies of that particular DNA segment.

88. What happen when increasing in AUC:


• Decrease in Vd
• Increase in Vd
• Increase in K contant

89. What happen when increasing in dose of drug: Increase in Vd

90. Definition of incidence:2012  New case certain period


a measure of the probability of occurrence of a given medical condition in a population
within a specified period of time
Incidence proportion (also known as cumulative incidence) is the number of new cases
within a specified time period divided by the size of the population initially at risk

91. Definition of merchandizing: Visual selling

92. Calcium contraindication with:


• Quinolone
• ASA
• Penicillin

93. What is suitable for patient like grape fruit juice and prescribed for him amiodarone:
• Separate between grape fruit and amiodarone
• Stop grapefruit  separation won’t work as amiodarone has long half life

94. Enteric coated tablet for protect from: stomach Hcl

95. End product of purine: uric acid

96. Patient take prostatic anticancer drug has hot flushing in face which of the following
medication causes this side effect: Bicalutamide : anti prostatic cancer and causes hot flushes
• Sildenafil
• Anastrazole
• paclitaxol
Anastrazole is an Aromatase P450 inhibitors causes
hot flushes and sweating, osteoporosis, insomnia,
body aches, loss of appetite.

97. Measurement of room temperature:


Arrhenius
98. Counseling for hemorrhoid all except:
• Avoid setting for along time
• Taking diet high with fibre
• Avoid consuming a diet high in white flour
• Avoid taking acid product

99.Which reference contains SE:


NEW SIDE EFFECT REFERENCE: MEDEFFECT, CANADA VIGILANCE, CARN
COMMON S.E REFFERENCE: CPS, USP‐1, AHES

100.Definition of Canada Vigilance:


it’s the health canada’s post market surveillance program that collects and assesses
reports of suspected adverse reactions to health products marketed in Canada.

101. Structure of Rifampicin this structure is used for: TB tuberculosis

102. IgE released in :2012: Asthma and allergic rhinitis

103. Hypertension affects all the following organs except:


a. lung b. heart c. eye d. kidney e. brain

104. According to the monoclonal antibody ramming synthetic:


a. rabbit b. mouse c. mouse / human d. goat e. frog

105. Moclobemide classified as:


a-SSRI b. Selective MAO A inhibitor (reversible)

106-To prepare a 75g of paste contain: 1% merthal 2% HC


a. 0.75 HC & 1.5 m
b‐1.5 HC & 0.75 merthal
c- 2 HC & 1.75 m

107. responsible of ca absorption: parathyroid hormone


108. Wich receptor opioids mostly act on:
a. Mu b. Kappa c. delta

109. Which of the following cause Parkinson:


• Lithium
• Metoclopramide
• Levodopa

110. according to ISMP what is most dangerous abbreviation:


• ml
• IU
• Kg
• Qid

111. Methadone is considered as: straight narcotic

112. What of the following test Not use for detecting cancer:2012,2016
• PAP smear
• mammography
• colonoscopy
• PSA (prostate specific antigen)
• APGAR (test used to assess the health of newborn)

113. in UTI caused by Pseudomonas aeruginosa the most effective will be:
• Levofloxacin
• Clotrimazole
• Sulfamethoxazole

114. what enzyme responsible for protein digestion: 2013


• Pepsin
• Gastrin

115. What of the following used in enteric coated capsule as plastizer:


• glycerin
• PEG
• Ethyl alcohol
116. Asymptomatic bacteriuria require to treat:
• Pregnant women
• Over 65 y old
• Diabetic patient
• Children

117. the most affected leukocytes by chemotherapy:2012


• Lymphocyte
• Basophil
• Monocyte
• Neutrophils

118. Wich organization responsible for funding public drugs;


• Medicare
• Provincial government

119. What of the following carry VLDL: apo B and apo c

120.what is initial symptoms of marijuana:2012


• Mydriasis
• Conjunctival redding
• Fatigue

121.fluticasone MDI what is the paper label: rinse mouth after use

122. What is used in acute asthma: salbutamol

123. lutein is: carotenoid


Lutein is a xanthophyll and one of 600 known naturally occurring carotenoids.

124. Type I error is:


type I error is the incorrect rejection of a true null
hypothesis (a "false positive").
type II error is the failure to reject a false null
hypothesis (a "false negative"). More simply stated, a
type I error is detecting an effect that is not present,
while a type II error is failing to detect an effect that
is present
Evaluation ‐ July 2013
2. Alpha Lipoic Acid (ALA) is
a) Vitamin
b) Minerals
c) Protein
d) Amino Acid
e) Anti‐Oxidant
Alpha-lipoic acid is an antioxidant also known as Acetate Replacing Factor, ALA, Biletan, Lipoicin,
Thioctan, and many other names. Alpha-lipoic acid is a naturally occurring fatty acid that can be found in
many foods such as yeast, spinach, broccoli, potatoes, and organ meats such as liver or kidney.

3. Hypertension affects all the below organs EXCEPT


a) Lung
b) Brain
c) Eyes
d) Kidneys
e) Heart

4. S-Adenosylmethionine (SAM) plays a pivotal role in biological and biochemical


events through
a) Oxidation
b) Hydrolysis
c) Methylation – conversion of NE to E
d) Reduction

5. Propylene Glycol used in ophthalmic preparations as


a) Buffer
b) Preservative
c) Lubricant
d) Solvent
e) None of the above
(MS) is an unpredictable disease affecting vision,
hearing, memory, balance and mobility often disabling
6. Demyelination is a characteristic for disease. The disease attacks the myelin which is a
a) Multiple Sclerosis protective covering wrapped around the nerves of the
CNS. Canadians have one of the highest rates of
b) Diabetic foot multiple sclerosis in the world. MS is the most common
c) Schizophrenia neurological disease affecting young adults in
Canada.MS last a lifetime. There is no cure
d) Attention deficit hyperactivity
disorder (ADHD)
7. Differential diagnosis of Rosacea from Acne vulgaris is
a) Erythema
b) Telangiectasia (widened venules that cause red lines or patterns on the skin)
c) Papules
d) Comedones
e) Seborrhea

8. Long-acting basal insulin used to control blood sugar level


a) Glulisine
b) Lispro
c) NPH
d) Detemir
e) Novolin

9. Excessive sweating, is a common disorder known as


a) Nocturia
b) Hyperhidrosis
c) Seborrhea
d) Xeroderma
e) Hyperpiesis ( High Blood-Pressure Without Evident Cause)

10. When your body can’t make enough red blood cells due to destruction of bone
marrow known as
a) Megaloblastic anemia
b) Pernicious Anemia
c) Sickle Cell Anemia
d) Aplastic Anemia
e) Thrombocytopenia

11. Which of the following not used as Acne treatment?


a) Erythromycin
b) Clindamycin
c) Ciprofloxacin
d) Tetracyclines
e) Isotretinoin

12. Osteoporosis patient can’t do all the below activities EXCEPT


a) Basketball
b) Horse ridding
c) Cycling
d) Walking
e) Swimming
Osteoprosis patient can’t do risky excercises as this may increase risk of fall and fracture.

13. Warts are caused by a viral infection,


Mirtazapine has a dual mode of action. It is a
specifically by noradrenergic and specific serotonergic
a) Enterovirus antidepressant (NaSSA) that acts by antagonizing
b) Herps Zoster the adrenergic alpha2‐autoreceptors and alpha2‐
heteroreceptors as well as by blocking 5‐HT2 and 5‐
c) Herpes simplex HT3 receptors. It enhances, therefore, the release
d) Human Papillomavirus (HPV) of norepinephrine and 5‐HT1A. This dual mode of
e) HIV action may conceivably be responsible for
mirtazapine's rapid onset of action

14. Selective irreversible MAO-B I is:


a) Nortriptyline
b) Azathioprine
c) Benzatropine
d) Mirtazapine
e) Selegiline

15. The below organism is responsible for Sexual Transmitted Disease EXCEPT
a) S. Aureus
b) Hepatitis B
c) Treponema pallidum
d) Chlamydia trachomatis
e) Trichomonas vaginalis

16. What is the end product of Protein breakdown?


a) Amino Acid
b) Nitrogen
c) Urea
d) Uric Acid
e) Citric acid

17. ADH (Antidiuretic Hormone known as Vasopresin) release is triggered by


a) High Plasma osmolality (Low water)
b) Low viscosity
c) High PH
d) High salt and water concentration
e) Increase blood pressure

18. Misoprostol is a synthetic prostaglandin analog


a) PGI2 (Platelets)
b) PGH
c) PGE1 (Alprostadil and misoprostol)
d) PGF2α (latanoprost for glucoma)

19. All are correct about the Dry Powder Inhaler (DPI)EXCEPT
a) Use lactose as bulking agent
b) DPIs are an alternative to Metered-Dose Inhaler
c) Propellant provides the force to generate the aerosol cloud
d) Humidity may degrade the ability of the device
e) DPI medication must be stored in a sealed packaging

20. Which of the following medication may cause erectile dysfunction?


a) Corticosteroid
b) α Adrenergic Blocker
c) Opioid
d) ACE Inhibitors

21. Using Insulin pen by diabetic patients can be best described as


a) More than 2 different insulin can be mixed
b) Suitable for large amount of insulin
c) Using pens is usually less expensive
d) Need someone to help Diabetic patient
e) Easier to use for those with visual impairments

22. Which test should be done before starting Infliximab?


e) Haemoglobin
f) Creatinine Clearance
g) TB
h) TSH
i) Hepatitis

23. The Superior Mesenteric Vein (SMV) is a blood vessel that drains blood from
a) The small intestine
b) Left Ventricle
c) Liver
d) Lung
e) Spleen (splenic vein)

24. Chemoreceptor is a sensory organ related to


a) Vision
b) Knee Jerk
c) Balance
d) Hearing
e) Smell

25. All of the below microorganism may cause Traveller’s Diarrhea EXCEPT
a) Shigella
b) E. coli
c) Salmonella
d) Campylobacter
e) Staph

26. Milk allergy is an adverse immune reaction to


e) Calcium
f) Lactose
g) Protein
h) Vit D
i) Amino Acid
A milk allergy is an immune reaction to one of the many proteins in animal milk. It's most
often caused by the alpha S1‐casein protein in cow's milk.
A milk allergy is sometimes confused with lactose intolerance because they often
share symptoms.

27. The most common laboratory markers used for diagnosis of acute pancreatitis is
a) Total Bilirubin
b) Lipase  pancreatitis increase both lipase and amylase level
c) Erythrocyte Sedimentation Rate (ESR)
d) Soluble Liver Antigen (SLA)
e) C-Reactive Protein (CRP) Level

28. The Absolute Refractory Period including phases 1 and 2 in which


a) The membrane returns to the resting status
b) The cell can’t be stimulated by any impulse
c) The membrane is hyperpolarized (phase 3 occurs with relative not absolute)
d) It lasts till the membrane return to resting state (relative)

29. Which is the precursor of Heroin


a) Marijuana
b) Cocaine
c) Amphetamine
d) Codeine
e) Morphine  BY ACETYLATION

30. In treating acute exacerbation of COPD, all can be used


EXCEPT
a) Levofloxacin
b) Ipratropium
c) Montelukast
d) Salbutamol
e) Prednisone

31. All are correct about Metformin uses with diabetic patients EXCEPT 2012
a) Can’t be used if someone has kidney problems
b) Usually no Hypoglycemia is reported
c) Has diarrhea or stomach cramping as a common side effect
d) Patient may experience weight gain

32. All the following are vitamin K–dependent coagulation factors except
a) Factor X
b) Factor VII
c) Protein C
d) Protein S
e) Factor VIII

33. Treatment with Lithium required the monitoring of 2012,2015,2016


a) Glucose
b) Lipid
c) Serum Potassium
d) T4 & TSH
a) Creatinine
34. All the following can be used in Migraine prophylaxis EXCEPT
a) Carvidolol
b) Nadolol
c) Atenolol
d) Propranolol
e) Timolol

35. Which one of the following medications can’t be used in Psoriasis?


a) Topical retinoids
b) Salicylic acid
c) Vitamin D3 derivatives
d) Oral Isotretinoin  acitretin
e) Topical corticosteroids

36. In When a solute is added to the solvent


a) The vapor pressure of the solvent decreases
b) The osmotic pressure decreases
c) No change in vapor pressure
d) Boiling point depression
e) Freezing point elevation

37. The process by which a solute forms a solution in a solvent is known as


a) Diffusion
b) Dissolution
c) Liquidation
d) Sublimation
e) Deposition

38. All of the following may cause Metabolic Acidosis EXCEPT 2012
a) When the kidneys are not removing enough acid from the body
b) Excessive increase of sodium bicarbonate
c) Poisoning by acetylsalicylic Acid
d) Excessive increase in ketone bodies
e) Excessive loss of sodium bicarbonate from the body

39. Mechanisms of Cisplatin cytotoxicity involve the binding of the drug to


a) tRNA
b) mRNA
c) RNA
d) DNA
e) Cell wall receptors

40. The vitreous humor is the clear gel that fills the space in
a) Brain
b) Lung
c) Bone
d) Eye

41. Glidant is a substance that is added to a powder to improve its


a) Stability
b) Flowability
c) Friability
d) Lubricity

42. Breastfed Babies Need Extra Vitamin


a) B6
b) A
c) B12
d) D

43. In Phototoxicity,
a-Skin changes are similar to those of allergic contact dermatitis (photo allergy)
b-Eliciting a response of the immune system to attack the foreign antigen
c-UV rays cause the shape of a molecule to transform into a new substance
d‐Skin response like exaggerated sunburn

44. Tacrolimus is an immunosuppressive drug that is mainly act on


a) Cyclooxygenase-1(COX-1)
b) Synthesis of nucleic acids
c) Interleukin‐2 (IL‐2) through T cell
d) Synthesis of tetrahydrofolate

45. SOAP stand for


a) Small Operational Assessment Project
b) Short Oligonucleotide Analysis Protocol
c) Sexual Offender Apprehension Plan
d) Subjective, Objective, Assessment, Plan
e) Simple Object Access Package
46. An example of 5 alpha-reductase 5‐alpha reductase inhibitors interfere with the effect of androgens
on the prostate. This slows the growth of the prostate and can even
inhibitors cause it to become smaller, which may help improve the symptoms
a) Amiloride of benign prostatic hyperplasia (BPH). But since prostate size does
not always correspond to the severity of a man's symptoms, this
b) Fensteride group will not give satisfactory results in every case. When stop
c) Gliclazide taking, symptoms usually return. They are used to keep symptoms of
BPH from getting worse, reduce the risk of being unable to urinate
d) Loperamide (urinary retention) and to prevent surgery for BPH. They are not
recommended for men who have BPH symptoms without a
e) Metopramide noticeably enlarged prostate. It may take 6 to 12 months before
notice symptoms improvement.

47. Difference between Glimepiride &


Glyburide
a) Glyburide has higher risk of hypoglycemia compared with Glimepiride
b) Glimepiride has higher risk of hypoglycemia compared with Glyburide
c) Reduction in A1c is more with Glyburide
d) More weight loss with Glimepiride
e) Reduction in A1c is more with Glimepiride

48. Side effect with Varenicline (Champix) use is


a) Chest pain
b) Arthralgia
c) Muscle Cramp
d) Mood changes
e) Polyuria
S.E: Nausea (30%); may be mitigated by taking on a full stomach, increasing water intake
or reducing dose. May cause insomnia; take second daily dose at suppertime.
Neuropsychiatric side effects such as suicidal/homicidal ideation have been reported;
monitor closely for changes in mood/behaviour.

49. Pulmonary Embolism (PE) most commonly results from 2012,2011


a) Fibrin
b) Platelet
c) Coagulation factors

50. Metformin is contraindicated with all EXCEPT 2012,2009


a) Patient with renal insufficiency
b) Patient with sever congestive heart failure
c) Patient with age (≥ 80 years),
d) Excessive alcohol intake
e) Use with Insulin
51. Increase the risk of bleeding may occur when combining clarithromycin with
a) Aspirin
b) Clopidogrel
c) Dipyridamole
d) Warfarin
e) Heparin

52. Drug commonly associated with SJS


a) Simvastatin Stevens‐Johnson syndrome SJS is a rare but serious
b) Gapabentin condition that causes sores that are like boils on the
mucous membranes of the mouth, nose, genitals, and
c) Omeprazole eyelids. A widespread skin rash of irregularly shaped, flat,
purplish red spots also develops, which may be painful.
d) Venlafaxine Stevens‐Johnson syndrome is usually caused by a reaction
e) Sulfamethoxazole to a medicine. The condition may also result from an
infection or a vaccination. Recovery from Stevens‐Johnson
syndrome may be slow, taking up to 6 weeks. Treatment
53. Spironolactone side effects may include includes a soft diet and oral and IV fluids to prevent
dehydration
a) Hypokalemia
b) Gynecomastia
c) Constipation
d) Insomnia
e) Hirsutism

54. Taking Plan-B may be accompanied with side effect


a) Breast Tenderness
b) Dark urine
c) Unusual weight loss
d) Increased sweating
e) Irregular heartbeat
CTC - S.E: Nausea, vomiting, dizziness, fatigue.

55. What is the medication may cause bradycardia?


a) Clonazepam
b) Diltiazem non dihydropyridine CCBs
c) Cocaine Hydrochloride
d) Clonidine
e) Chlorpheniramine

56. All of the following considered as polymers EXCEPT


a) Cellulose
b) Protein
c) RNA
d) DNA
e) Glucose

57. What can be used for Secondary Prevention of Stroke


a) Aspirin
b) Clopidogrel
c) Aspirin plus Dipyridamole
d) Warfarin
e) All of the above

58. Gingival hyperplasia is a common side effect of


a) Doxycycline
b) Dapigatran
c) Olmesartan
d) Phenytoin

59. Most suitable oral dosage form for aqueous solution is


a) Soft Capsule
b) Wafer
c) Hard Capsule
d) Enteric coated tablet

60. Which of the following can be used in the first few days to prevent DVT in patients
on knee surgery with acute Heparin-Induced Thrombocytopenia (HIT)?
a) Vitamin K antagonist therapy
b) Low Molecular Weight Heparins
c) Warfarin
d) Fondaparinux indirct factor xa inhibitor

61. The aPTT test is a performance indicator used to monitor the treatment effects with
a) Aspirin
b) Vit K
c) Unfractionated Heparin
d) Warfarin
e) Clopidogrel
62. The Drug of Choice (DOC) for pregnant Trimethoprim alone or TMP‐SMX combinations should be
women suffer from Cystitis (UTI) avoided during the first trimester of pregnancy. Macrolides
(clarithromycin, erythromycin, and azithromycin) are not
a) Amoxicillin for 3 days first‐line agents for UTI in pregnancy. Fluoroquinolones
b) TMP/SMX for 7 days (Ciprofloxacin, Norfloxacin) are contraindicated in pregnancy.
Nitrofurantoin restriction is limited to the last several weeks
c) Cephalexin for 7 days of pregnancy. Cephalexin is among the most commonly
d) Ciprofloxacin for 3 days prescribed oral cephalosporin for pregnant women with UTI.
e) Norfloxacin for 7 days

63. Gastrointestinal absorption of Indinavir may be reduced by concomitant


administration of
a) Furosemide
b) Venlafaxine
c) Omeprazole
d) Naproxen
e) Corticosteroid

64. Medication has been safely used to treat hypertension during pregnancy
a) Furosemide
b) Angiotensin Converting Enzyme Inhibitors (ACE-I)
c) Methyldopa
d) Thiazide
e) Angiotensin Receptor Blocker (ARB)

65. Drug used to treat symptoms of Crohn's disease Crohn's disease is a chronic condition of inflammatory
bowel disease (IBD) where it affect any part of the
a) Sertraline gastrointestinal tract from mouth to anus. Another type
b) Diclectin of IBD is ulcerative colitis where inflammation affect the
lining of the rectum and large intestine (colon). Signs
c) Lorazepam are usually abdominal pain and diarrhea after eating
d) Morphine and symptoms may include skin lesions or arthritis.
While there is no cure, medications can help control the
e) Infliximab symptoms: Amino salicylates, Corticosteroids, Immune
modifiers, Antibiotics & Anti‐Tumour Necrosis Factor
alpha (TNF‐α) also known as biologics (infliximab)
66. Prophylaxis of traveler's diarrhea could be
achieved by
a) Ciprofloxacin
b) Loperamide
c) Bismuth Sub Salicylate
d) Iodo-chlorhydroxyquin
e) TMP-SMX
67. All the below tips could be beneficial to reduce acid reflux EXCEPT
a) Relax and eat your food slowly.
b) Don't lie down after eating
c) Avoid anything tight clothing around your middle
d) Quit smoking
e) Incorporate vigorous exercises like running.

68. Candida is generally treated with a number of options to choose from EXCEPT
a) Fluconazole
b) Nystatin
c) Ketoconazole
d) Miconazole
e) Metronidazole

69. Sinusitis-causing bacterial pathogens include all of the below EXCEPT


a) Streptococcus pneumonia
b) Moraxella catarrhalis
c) Staphylococcus aureus
d) Bacteroides fragilis
e) Haemophilus influenza

70. Treatment of patients with absence seizures with


a) Benzodiazepine
b) Ethosuximide
c) Phenytoin
d) Vigabatrin

71. Stomach exposure to protein stimulate the pyloric glands to secret


a) Pepsinogen
b) Amylase
c) Secretin
d) Gastrin

72. A variety of drugs may be used to manage Congestive Heart Failure EXCEPT
a) Loop diuretics
b) Quinapril
c) Digoxin
d) Carvidolol
e) Verapamil
73. Phenytoin is highly bound (~90%) to
a) Glubulin
b) Plasmin
c) Albumin
d) Fibrinogen
e) Glycoprotein

74. Symptoms of hypothyroidism may include all EXCEPT


a) Cold intolerance
b) Soft skin
c) Weight gain
d) Constipation
e) Slowed heart rate

75. Pinaverium bromide act by


a) Soften the fecal mass by osmotically drawing water into the GI tract
b) Inhibits small intestinal bacterial overgrowth
c) Relaxing the smooth muscle of the bowel
d) Blockade of serotonin on 5HT3 receptors in the gut
e) Metabolic fermentation in the large intestine
USED IN IBS‐Constipation Predominant IBS‐C as Antispasmodic

76. Which vitamin may cause flushing?


a) Vitamin B1 (Thiamine)
b) Vitamin B2 (Riboflavin)
c) Vitamin B3 (Niacin)
d) Vitamin B6 (Pyridoxine)
e) Vitamin B9 (Folic acid)

77. An example of Serotonin–Norepinephrine Reuptake Inhibitors (SNRIs) is


a) Sertraline
b) Fluvoxamine
c) Phenelzine
d) Fluoxetine
e) Venlafaxine

78. All are used in the treatment of Tuberculosis EXCEPT


a) Ethambutol
b) ciprofloxacin
c) rifampicin
d) Isoniazid
e) Cyclosporin

79. Dysgeusia is a distortion of the sense of


a) Touch
b) Smell
c) Taste
d) Hearing
e) Vision

80. Warfarin decrease blood coagulation by


a) Inhibiting thrombin & factor X
b) Inhibiting the binding of ADP to its platelet receptors
c) Vitamin K epoxide reductase
d) Direct thrombin inhibitor
e) Inhibition of the platelet enzyme (COX-1)

81. The differences between the omega-6 and omega-3 fatty acids are
a) Omega-3 absorption is faster than Omega-6
b) Only omega-3 fatty acids can be found in the food
c) Only omega-6 fatty acids may be taken as a supplement
d) Position of double bond
e) Number of double bonds
Multiple myeloma is a cancer that starts in plasma cells (type of white
blood cell that make antibodies, made in the bone marrow). Myeloma
82. Thalidomide used to treat begins when a plasma cell becomes abnormal and begins to divide
uncontrollably, making abnormal plasma cells. Abnormal plasma cells are
a) Multiple hereditary exostosis called myeloma cells. Over time, the myeloma cells crowd out the normal
b) Multiple endocrine neoplasia blood cells in the bone marrow and prevent them from working properly.
Myeloma cells can also spread to the solid part of the bone and cause pain
c) Multiple system atrophy or fractures. As the number of myeloma cells increases, they can upset the
d) Multiple sclerosis balance of calcium in the body & prevent kidneys and nerves, from working
properly. The disease is called multiple myeloma because it affects many
e) Multiple myeloma bones. If myeloma cells form a tumour in only one bone, it’s called a
plasmacytoma

83. Naturally occurring programmed of cell death is known as


a) Necrosis
b) Apoptosis
c) Atrophy
d) Mitosis
84. The most important protective barrier over the body's surface is
a) Dermis
b) Epidermis
c) Stratum corneum
d) Basement membrane
e) Hypodermis

85. In Phase II trials, the experimental treatment is given to


a) 10-20 healthy volunteers to assess safety & efficacy
b) 20-50 healthy volunteers to assess safety
c) Large groups of people (1,000–2,000)
d) 100‐300 patients to assess efficacy
e) Small group of animals

86. The most common refill medications used in Canada


a) Cardiovascular
b) Steroids
c) Anti-diabetics
d) Analgesics
e) Antidepressants

87. Best diuretic used in heart failure and edema is


a) Thiazide
b) Hydrochlorothiazide
c) Spironolactone
d) Acetazolamide
e) Furosemide

88. Deficiency in ADH cause


a) Polyuria Polyuria is a urine output exceeding 3
L/day in adults and 2 L/m2 in children.
b) Nocturia It must be differentiated from the more
c) Decrease in plasma osmolority common complaints of frequency or
nocturia, which are not associated with
d) Hyponatremia an increase in the total urine output.
e) Hypokalemia

89. Catalysis of methyl group transfers Involving Tetrahydrofolate include


a) Vit C
b) Vit B 12
c) Vit D
d) Vit A
e) Vit K

90. PEG is used in ophthalmic preparations as


a) Filler
b) Surfactant
Viscosity enhancer and lubricant
c) Humectant
d) Preservative
e) Buffer

91. Drug side effects includes symptoms of jaundice, N&V may be linked with
a) Sertraline
b) Amiodarone
c) Digoxin
d) INH

92. Local anaesthetic that undergo rapid distribution in tissue and has a short acting
effect
a) Procaine ESTER TYPE
b) Lidocaine
c) Thiopental
d) Tetracaine

93. What is considered the biggest burden on health care cost


a) Continuing Education
b) Research and Development
c) Drug spending
d) Physician services
e) Medical services provided by hospitals

94. The federal government program where the government transfer payments to be
used for provincial and territorial health insurance programs called
a) Territory care
b) Total Health care
c) Medicare
d) Primary health care
95. The long QT syndrome increases the risk of episodes of
a) Brady arrhythmias
b) Cardiomyopathy
c) Atrial fibrillation
d) Torsades de pointes
e) Congestive Heart Failure

96. Absence of the P wave in the ECG may indicate


a) Heart attack
b) Heart block
c) Valve disorders
d) Atrial fibrillation
e) CHF

97. Azotemia is a medical condition characterized by a decrease in the


a) Glomerular Filtration Rate (GFR) of the kidneys
b) Serum Glutamic Oxaloacetic Transaminase (SGOT)
c) Blood Urea Nitrogen (BUN)
d) Serum creatinine concentrations
e) International Normalized Ratio (INR)

98. Chronic Bronchitis could be described as


a) A common complication of the cold or flu
b) Airways react by getting red, swollen & making extra mucus
c) Viruses attack the insides of airways and infect them
d) Reversible with Salbutamol
e) Extra mucus is in the lungs & blocks the airways

99. DIN is related to


a) Drug Interaction Nature
b) Drug Information Number
c) Data Information Network
d) Drug Identification Name
e) Drug Identification Number

100. Counseling for patient taking VIMOVO (Naproxen + Esomeprazol)


a) Drink a lot of fluids
b) Don’t take antacids while taking VIMOVO
c) Take VIMOVO at least 30 minutes after meal
d) Take VIMOVO at least 30 minutes before meal
e) VIMOVO Should be taken with milk
CTC - Naproxen / esomeprazole Vimovo: 1 tablet BID PO (contains 375 mg or 500 mg
naproxen + 20 mg esomeprazole). Take with food.

101. The most appropriate counseling for patient taking Estrogen patch
a) When replacing patch, make sure to apply the new patch to the same area
b) Apply the patch to a clean, dry and open area of the arm
c) Do not apply the patch on the breast
d) It may cause redness for the skin
e) Keep the patch for 3 weeks and then replaced

102. When a teenager girl asking for OCP, you can do all EXCEPT
a) She should take it at the same time, every day.
b) Inform her that no form of birth control is 100% effective
c) Inform her parents about her request.
d) Highlight that the best way to avoid pregnancy is to not have sex
e) Check when the last period started and ensure the girl is not pregnant.

103. Which of the following micro-organisms is included in Oral Vancomycin spectrum?


a) Streptococcus species
b) E. coli
c) Staphylococcus aureus
d) Clostridium difficile  CAUSE Pseudomembraneous colitis
e) Moraxella catarrhalis

104. Treating head lice for 3 years old girl with history of seizures and no allergy
a) Dimethicone
b) Benzyl Alcohol
1%
c) Permethrin 2%
d) Lindane shampoo
e) Tea tree oil’s

105. All about Best Possible Medication History (BPMH) are correct EXCEPT
a) A 'snapshot' of the patient's actual medication use
b) Complete documentation includes drug name, dosage, route and frequency.
c) A review to obtain & verify all of a patient's medication use (RX and non-RX).
d) Created using a systematic process of interviewing the patient/family
e) Review the triggers of event lead to hospitalization

106. Medication that work by stimulating insulin release after meal


a) Rosiglitazone
b) Nateglinide
c) Acarbose
d) Metformin
e) Saxagliptin E is the right answer
Nateglinide  Taken just prior to meals to reduce postprandial glucose elevations and
should be omitted if meal is missed.
Rosiglitazone  Taken with or without MEALS.

107. Which medication inhibit Peroxisome Proliferators Activated Receptor gamma


(PPARγ) jan 2013
a) Gliclazide
b) Pioglitazone
c) Metformin
d) Sitagliptin

108. Initial symptom for patient with liver toxicity


a) Polydipsia and polyphagia
b) Shortness of breath
c) Abdominal pain and swelling
d) Gastrointestinal bleeding
e) Metallic taste in the mouth

109. What kind of study name when all physicians in a hospital assess future goal and
their healthcare thoughts?
a) Randomized control
b) Case control
c) Cohort
d) Cross sectional
e) Meta-analysis

110. All are correct about Methadone RX EXCEPT  STRAIGHT NARCOTIC


a) Faxed prescription permitted
b) Transfer not permitted
c) Verbal prescription permitted
d) Refill not permitted
e) Part fill permitted

111. Amoxicillin used before dental extraction against


a) Staphylococcus auricularis
b) Streptococcus pneumonia
c) Escherichia coli
d) Staphylococcus aureus
e) Viridans streptococci

112. All are most commonly identified pathogen in community-acquired pneumonia


EXCEPT
a) Moraxella catarrhalis
b) Haemophilus Influenzae
c) Legionella pneumophila  NOSOCOMIAL (hospital acquired)
d) Pseudomonas aeruginosa
e) Streptococcus pneumonia

113. The most important diagnostic criteria for diabetes are


a) Unexplained weight loss
b) Unquenchable Thirst
c) Fasting Plasma Glucose (FPG) level of 7.0 mmol/L
d) Increased Urination
e) Sudden weight loss

114. When a drug circulating in blood & high perfused organs, we name it
a) Central Compartment
b) Peripheral Compartment
c) 2- Compartment
d) Multi Compartment
Pharmacokinetic two-compartment model divided the
body into central and peripheral compartment.
The central compartment (compartment 1) consists of
the plasma and tissues where the distribution of the
drug is practically instantaneous.
The peripheral compartment (compartment 2) consists of tissues where the distribution
of the drug is slower.
115. A compound with 3 chiral carbons atoms can have a maximum of
a) 3 stereoisomers
b) 4 stereoisomers
c) 6 stereoisomers
d) 1 stereoisomer
e) 8 stereoisomers

116. The term "cytokine" has been used to refer to


a) Endocrine hormones
b) Prostanoid
c) Interferon
d) Leukotriene
e) kinase enzyme
Cytokines include chemokines, interferons, interleukins, lymphokines, and TNFs.

117. Why do I need a flu vaccine every year?


a) Flu viruses are constantly changing
b) Vaccine expire after 1 year
c) Vaccine efficacy decrease by 50% after 1 year
d) It takes about I year after vaccination for antibodies to develop

118. Which related to non heme iron-containing


a) Ferritin
b) Transferrin
c) Superoxide dismutase
d) Cytochrome oxidase
e) Methane mono-oxygenase

119. The federal government's roles in health care include all the below EXCEPT
a) Regulation of pharmaceuticals
b) Consumer safety
c) Disease surveillance and prevention
d) Planning and implementation of health promotion and public health
initiatives
e) Primary care and emergency services

120. In Zero order reaction


a) Rate is directly proportional to the concentration
b) Rate is independent of the concentration
c) Rate is proportional to the square of the concentration

121. Nephrosis is characterized by


a) Anaemia
b) Haemoglobinuria (High concentrations of protein hemoglobin in the urine)
c) Proteinuria (Excess of serum proteins in the urine)
d) Haematuria (Presence of red blood cells in the urine)
e) Oliguria (Reduction of urine output)

122. Major enzymes involved in drug metabolism and bioactivation is


a) CYP 3A4
b) CYP 2C9
c) CYP 1A2
d) CYP P450
e) CYP5A1

123. Pharmacists role includes all the below EXCEPT


a) Oversight of dispensing medicines on prescription
b) Education and counseling for patients on optimal use of medicines
c) Decision to accept or refuse the prescription
d) Clinical medication management
e) Supervising pharmacy technicians and other staff

124. A nurse mistakenly administered medication incorrectly, what is pharmacist role?


a) Report this error to someone who is in charge of the care for this patient
b) Cover up for her mistake
c) Dispense the correct medication
d) Apologies for patient & his family
e) Write a disciplinary report for hospital management.

125. Which of the following cause Hyperuricemia?


a) Cholchicine
b) Atorvastatin
c) Thiazide Diuretic
d) Probenicid
e) Losartan
126. Trigeminal neuralgia characterized by
a) Intermittent claudication
b) Fainting
c) Radiating pain starting in the lower back and going down the leg
d) Seizures
e) Intense facial pain

127. Sensitivity measures the proportion of


a) Actual negative which are correctly identified as such (True negative)
b) Actual positives which are correctly identified as such (True positive)
c) Actual negative which are incorrectly identified as such (False negative)
d) Actual positives which are incorrectly identified as such (False positive)
e) Confidence intervals

128. All may occur with aging EXCEPT


a) Urinary retention
b) Shortness of breath
c) Increase gastric motility
d) Ear wax becomes drier
e) High blood pressure

129. The most diagnostic symptom of ascites is


a) Poor appetite
b) GIT bleeding
c) Frequent urination
d) Constipation
e) Abdominal distension

130. Extravasation can be defined as


a) Abnormal and irregular dilations in superficial veins
b) A period in which the blood vessel cells multiply excessively
c) Discharge or escape of blood from a vessel into the tissues.
d) Abnormal bulge in the wall of a blood vessel
e) A blood clot occurring in a deep vein.

131. Following unprotected intercourse or a contraceptive accident, Plan B can prevent


pregnancy if two tablets are taken together within
a) 24 hours
b) 12 hours
c) 72 hours
d) 48 hours
e) 36 hours

132. What is correct about Meperidine RX


a) Written and no transfer
b) Verbal or written prescription permitted
c) Refill or part fill not permitted
d) Refill or part fill are permitted
e) Transfer is permitted

133. Regarding the DNA bases, which is similar to Guanine in size


a) Cytosine
b) Thymine
c) Adenosine
d) Adenine
It should be Adenine as both are purine bases (pure as gold)
If they asked about the same amount so it will be A

134. Pharmacist should warn patients on Leflunomide about


a) Dark urine
b) Stomatitis
c) Infection
d) Liver toxicity
e) Hypertension

135. All true about Methyldopa side effect EXCEPT


a) Teratogenicity
b) Dry Mouth
c) Headache
d) Mental Impairment
e) Fluid Retention in the Legs

136. The most suitable medication for hypertension patient having diabetes is
a) Loop diuretics
b) Beta Blocker
c) Diuretic
d) ACE Inhibitor
137. Which structure is related to Macrolides?

Amphetamine Macrolides Cephalosporin Morphine

138. The main screening test for colorectal cancers is 2015


a) Blood tests
b) Ultrasound scans
c) PPD test
d) Pap smear
e) Fecal occult blood

139. The compounds that lower the interfacial tension


a) Lubricant
b) Surfactant
c) Humectant
d) Preservative
e) Buffer

140. Pramoxine belongs to a class of drugs known as


a) Anti-leukotriene Pramoxine is in a class of medications
called topical anesthetics used to treat
b) Chondroprotective soreness, burning, itching, and pain
c) Skeletal muscle relaxant from hemorrhoids (''piles'') and other
minor rectal irritations or itching
d) Local anesthetic
e) Bronchodilator

141. Drug of choice for pregnant with UTI is


a) Nitrofurantoin
b) Cephalexine
c) Isoniazide
d) Trimethoprim
e) Methenamine
142. Which immunoglobulin plays an essential role in allergic rhinitis jan2013,2012
a) IgG
b) IgM
c) IgA
d) IgE
e) IgD

143. During Isotretinoin treatment patient should monitor


a) CD4 Cell Count
b) Blood Glucose Test
c) Liver Function Test (LFT)
d) Blood count
e) Electrocardiogram (ECG)

144. Which drug can be used in hyperthyroidism,


a) Prednisone
b) Midodrine hydrochloride
c) Thiazide
d) Levothyroxine
e) Beta Blocker

145. Rhabdomyolysis characterized with all the below EXCEPT 2010


a) Muscle weakness
b) Kidney failure
c) Dark urine
d) Nausea & vomiting
e) Bone damage

146. Which drug will work on vasodilatation for both artery & vein?
a) Hydralazine
b) Sodium Nitroprusside
c) Beta Blocker
d) Sildenafil

147. All can be used in treating prostate carcinoma EXCEPT


a) Orchiectomy  a surgical procedure in which one or both testicles removed.
b) Hydrotherapy
c) Hormone therapy
d) Chemotherapy
e) Immunotherapy

148. Heartburn is the major symptom that most likely to occur with
a) Gastroenteritis
b) IBD (Inflammatory Bowel Disease)
c) GERD (Gastro‐Esophageal Reflux Disease)
d) Acute MI (Myocardial Infarction)
e) COPD (Chronic Obstructive Pulmonary Disease)

149. Different weight, ages in clinical trials could be related to


a) Biological Variables
b) Ordinal Variables
c) Behavior Variables
d) Nominal Variables
e) Categorical Variables

150. Definition of profit is


a) The difference between the cost of a good or service and its selling price
b) The percentage of selling price that turned into benefit
c) The percentage of cost price that one gets as benefit on top of cost price.
d) The amount of revenue gained from a business activity exceeds the
expenses, costs and taxes.
e) additional revenue that will be generated by increasing product sales by 1 unit

151. On bringing a product or service to market, the 4 P’s is related to 2012


a) Product. Package. Purchase. Promotion
b) Principle. Purchase. Price. Profit
c) Preparation. Package. Price. Profit
d) Product. Place. Price. Promotion
e) Preparation. Place. Purchase. Promotion

152. According to the Food and Drug Regulations, Schedule F is 2012


a) Medical Equipment
b) Prescription drugs
c) Narcotic Control Act
d) Over The Counter
e) Controlled Drugs
153. Which drug used in treating Glaucoma? Glaucoma is an eye disease that causes peripheral vision loss
associated with an increase of pressure inside the eye because
a) Dorzolamide of too much production or not enough drainage of the aqueous
b) Vitamin A palmitate humour cause compression on the optic nerve which is
responsible for relying visual information to the brain.
c) Homatropine Treatments include: Prostaglandin analogues (Travoprost),
d) Fluorometholone Beta Blockers (Timolol), Alpha Agonists (Brimonidine), Carbonic
Anhydrase Inhibitors (Dorzolamide) & Miotics (Pilocarpine).

154. The plasma levels of Indinavir dropped when taking with


a) Probiotics
b) Echinacea
c) St. John Wort
d) Ginkobiloba
e) Fever few

155. What is your advice for patient taking warfarin and his INR is 5.4 but there no
bleeding?
a) Continue with warfarin
b) Change warfarin with another anticoagulant
c) Give Vit. K
d) Stop warfarin
e) Skip warfarin dose

156. Post antibiotic effect of aminoglycoside is


a) Mild photosensitivity reaction
b) White patches on the tongue
c) Severe allergic reactions, called anaphylaxis
d) Discolor urine to red-brown
e) Drug eliminated but still have effect

157. Data about U.S., British, and international approved brand names and corresponding
generic name could be found in
a) Drug Information Handbook
b) Martindale’s
c) American Hospital Directory
d) Drugs medical journal
e) Medical and health information directory

158. Why did patient advised to put sunscreen while taking tetracycline?
a) Sunscreen use can slow the aging effect of tetracycline on skin
b) Tetracycline has a negative effect on collagen, elastin, and keratin
c) Tetracycline can make the skin more sensitive to the sun's rays and
result in sunburn
d) Effective combination in protecting the skin against cancer
e) Tetracycline increase UV-intensity that acts on the deeper layers of the skin

159. The Kaplan-Meier (K-M) equation can be used to estimate


a) Survival rate
b) Population growth rate
c) The median lethal dose
d) The therapeutic index

160. What is true about amorphous form


a) Condition where a liquid and its vapor are in equilibrium with each other
b) Natural-gas condensate
c) Non‐crystalline solid Amorphous is more soluble than crystalline

d) Powder granulation
e) Removal of dissolved gases from liquids

161. Pharmacist role about patient abuse his analgesic medication when ask early refill
a) Dispense the lower concentration medication
b) Talk to doctor about this concern
c) Refuse dispensing the analgesic medication
d) Give half quantities to patients
e) Dispense what is patient ask for

162. The Hospital care fund in Canada could be summarized by


a) Primarily by federal and provincial funding their taxation
b) Completely by provincial
c) Partly by federal and provincial funding as well as cash payments, and
insurance payments
d) The provinces and territories must meet the requirements in order to
receive the full federal cash transfer
e) Completely by federal government
0.02
163. What is the PH of 0.2 M of Acetic Acid?
a) 1.7
b) 2.9
c) 3.1
d) 12.3
e) 6.7

164. 1 gm of epinephrine and sodium equivalent 0.23 in 100 ml how much sodium
chloride needed to make isotonic solution
a) 0.34
b) 0.67 0.9‐0.23
c) 1.9
d) 4.5
e) 3.1

165. If rate constant is 0.35, how long will it take to reach Css?
a) 2 Hours
b) 9 Hours
c) 3 Hours
d) 7Hours
e) 6 Hours

166. If CER was 4% & EER was 2 % and they advertised on pamphlet that the RR (Risk
Reduction) is 2%. What does this 2 % RR represent
a) Relative Risk
b) NNH
c) Absolute Risk
d) NNT
e) Odd Ratio

167. If pharmacy sales are $ 1,500,000 and cost of goods $1,125,000. What is the Growth
Margin?
a) 6%
b) 10 %
c) 15 %
d) 20 %
e) 25 % sales-cost/sales =? *100 = answer

168. All 3 mg of active ingredient in 2 g suppository, what is the percent weight?


a) 1.5
b) 0.015 =3/2=1.5/100=0.015
c) 0.0015
d) 0.15 3/2000*100=0.15%
e) 15

169. Specific gravity of a compound is 0.917 and the mass is 25 mg? What is the volume
needed to be added? WRONG
this is to calculate mg not ml
a) 24 ml The question is asking about ml added to dilue solution to 5%
20 gm ——-> 100 ml
b) 25 ml 250 gm ——-> X
c) 26 ml X = 1250 ml of the 20% solution

d) 27 ml = 25*1000/917=27ml 5 gm ——-> 100 ml


250 gm ———> X
e) 29 ml X = 5000 ml for the 5% solution

Amount to be added 5000-1250=3750 ml


170. 250 g in a 20 % solution how much ml we need to add to make it 5 % solution?
a) 800 ml
b) 1000 ml  C1V1=C2V2‐‐‐‐‐–20*25=V2*5
c) 900 ml
d) 100 ml
e) 1250 ml

171. At renal function 44 % of the drug is excreted unchanged with half-life of 8 hours.
What will be the half-life for the dose of drug with 50% renal function? 2010
a) 14 hours b) 10 hours c) 12 hours d) 9 hours e) 4 hours
#44% unchanged means 22% excreted in 50% renal function. So, total its
44%+22%=66% More simple : 8+(8*50%)= 12 hours

Now. For 44% excreted unchanged its 8 hours then for 66%. How much? 12 hours
#half life would be increased by 50% so we should dose the patient ever 8+4=12hrs

172. A very angry customer comes to your pharmacy and complains that you have
dispensed wrong medication. What is initial appropriate action?
a) Promise to punish the technician responsible for the mistake
b) Report the error to ISMP
c) Establish if the patient has taken any of the incorrect medicine
d) Offer to him the full payment of the wrong medication
e) Try to minimize the seriousness of an error.

173. All about Iron Deficiency Anemia (IDA)are correct EXCEPT


a) Reason for IDA includes: if you lose more iron than your body can replace
b) As the anemia gets worse, patient could experience shortness of breath & a
sore tongue.
c) The risk factor includes chronic kidney disease
d) Iron in storage is not affected.
e) IDA is a condition where your body does not produce enough red blood cells

174. When Glycosylated part of Glycated hemoglobin (HbA1c) removed, it is a


reflection for 2012
a) High blood glucose level
b) High risk for anemia
c) Controlled blood glucose level
d) Ineffective anti-diabetic treatment
e) Immunosuppressive effects

175. What is the Bacteriophage?


a) Protective shell made of proteins symmetrical shapes
b) Represses transcription of all phage genes
c) key enzyme in lytic infection
d) Is a virus that infects and replicates within bacteria
e) Cell contain plasmids, which are extra-chromosomal DNAs that may contain
genes

176. All may cause dry mouth EXCEPT?


a) Parkinson's disease PD cause of high Ach in parkinson's so no reason for
dry mouth
b) Cholestasis  it's in gall bladder
c) Sjogren’s syndrome
d) Uncontrolled diabetes
e) Crohn’s syndrome
CTC - Nonmotor features of PD: Dementia, psychosis and autonomic dysfunction (excessive sweating &
salivation, constipation, bladder frequency/urgency, orthostasis) become more disabling features as
disease progresses, difficulty in speaking and swallowing.

177. All can be used in prostate cancer treatment EXCEPT?


a) Cabazitaxel
b) Denosumab (Prolia)
c) Prednisone
d) Anastrazole breast cancer
178. Filgrastim (Neupogen)
a) Used in substance abuse treatment
b) Help in treatment of Smoking Cessation
c) Granulocyte‐Colony Stimulating Factor (G‐CSF).
d) Lowers serotonin levels
e) Increase libido in women

179. The most common cause of granuloma is


a) Papilloma virus
b) Mycobacterium
c) Shigella
d) Salmonella
e) Staph
A granuloma is a structure formed during inflammation that
is found in many diseases. It is a collection of immune
cells known as macrophages.
Granulomas form when the immune system attempts to
wall off substances it perceives as foreign but is unable to
eliminate. Such substances include infectious organisms including bacteria and fungi, as
well as other materials such as foreign objects, keratin and suture fragments.

180. The role of Cerebrospinal fluid (CSF) is


a) Nutrient and waste transportation
b) Protect brain lobes
c) Shock absorption
d) Lubricates the articulating joints
e) Provides eye protection

181. Which one is nociceptive rather than neuropathic pain?


a) Diabetic foot pain
b) Shingles Pain
c) Phantom limb pain
d) Trigeminal neuralgia
e) Skeletal muscle pain

182. Calcium may chelate with which of the below antibiotics results in absorption delay?
a) Amoxicillin
b) Ciprofloxacin
c) Clarithromycin
d) Cefixime
e) Vancomycin

183. All of the below are flu symptoms EXCEPT


a) Headache
b) Chills
c) Fatigue
d) Diarrhea
e) Fever

184. Sodium lauryl sulphate used in pharmaceutical preparation as


a) Filler
b) Anionic Surfactant
c) Humectant
d) Preservative
e) Buffer

185. Which of the below can be used as lubricant


a) Propylene glycol
b) Magnesium stearate
c) Sodium lauryl sulfate
d) Lactose
e) Propyl Paraben

186. The most fastest acting oral formulations


a) Soft gel caps
b) Spray
c) Sublingual
d) Solid crystals
e) Waver

187. SOAP stand for


a) Specific Outline Procedure
b) Social Operation Philosophy
c) School of Pharmacy
d) Subjective – Objective ‐ Assessment ‐ Plan
188. Mass spectrometry is referred to
a) Separation of an incoming wave into a frequency spectrum
b) Visual representation of the spectrum of frequencies in a sound
c) Measurement of radiation intensity
d) Atoms of a particular element are bombarded with electrons

189. suitable recommendation for adult patient having nausea with no vomiting is
a) Fluid administration only
b) Fluid & juice administration
c) Oral rehydration therapy
d) Electrolyte (specially Sodium & potassium)
e) Reduce physical activities

190. Drinking plenty of water will increase the activity of the below medication EXCEPT
a) Allopurinol
b) Phenytoin
c) Doxycycline
d) Sulfasalazine
e) Clindamycin

191. All are often given before chemotherapy treatment to control emesis EXCEPT
a) Ondansetron
b) Granisetron
c) Dolasetron
d) Dexamethasone
e) Dimenhydrinate

192. Patient with pulmonary edema and acute heart failure can use
a) Spironolactone
b) Felodipine
c) Fursamide
d) Digoxin
e) Nifedipine

193. Fecal Occult Blood Test (FOBT) is used as a diagnostic test for all of the below
EXCEPT
a) Crohn's disease
b) Ulcerative Colitis
c) Colorectal cancer FOBT is a screening test for hidden (occult) blood
in the stool (feces) of risk people between the
d) Prostate cancer ages of 50 and 74‐could be done every 2 years.
e) Hemorrhoids Newer tests look for globin, DNA or transferrin

194. The most common side effect with Infliximab is Infliximab is an antibody‐ used for treating chronic
inflammatory diseases‐ Infliximab works by blocking the
a) Faster heart rate effects of tumor necrosis factor alpha (TNF alpha), a
b) Pneumonia substance made by cells of the body which has an important
role in promoting inflammation. Infliximab is used for
c) Lymphoma treating Crohn's disease, rheumatoid arthritis, psoriasis,
d) Gastrointestinal bleeding ankylosing spondylitis, and psoriatic arthritis.

e) Meningitis

195. The most important effect for Cholestyramine


a) Decrease triglycerides
b) Decrease LDL
c) Increase HDL
d) Decrease HDL
e) Increase LDL

196. All are used in warts treatment EXCEPT


a) Podophyllin solution
b) Trichloroacetic acid
c) Salicylic acid
d) Cimetidine
e) Amantadine

197. Sexual Transmitted Disease (STD) associated with alopecia is


a) AIDS
b) Genital herpes
c) Syphilis
d) Gonorrhea
e) Chlamydia

198. First line treatment for Osteoarthritis is


a) Acetaminophen
b) Celecoxib

199. Mechanism of anti-diabetic action of Rosiglitazone is


a) Acts on liver to cause decrease in insulin resistance
b) Alpha-glucosidase inhibitors that reduces glucose absorbance
c) Inhibits Peroxisome Proliferators Activated Receptor gamma (PPARγ)
d) Dipeptidyl Peptidase-4 (DPP-4) inhibitors
e) Sulfonylureas that stimulates insulin release by pancreatic beta cells

200. Which is considered effective and safe in the management of constipation in infants
a) Senna
b) Bisacodyl
c) Mineral oil
d) Docusate sodium
e) Polyethylene glycol 3350

201. When a 16-y girl asking for plan-B, the appropriate action by pharmacist is
a) Ask for written approval from her parent first
b) Discuss with her parent over the phone.
c) Ask for physician RX
d) Dispense
e) Refuse to dispense

202. Dexrazoxane may chelate


a) Calcium Dexrazoxane is a cardio protective drug used to reduce the undesired side
b) Iron effects of doxorubicin, a chemotherapy medication.

c) Aluminium
d) Magnesium
e) Cupper

203. Cephalosporin act by


a) Interferes with DNA Synthesis
b) Inhibits an Enzyme
c) Interferes with Cell Membrane Permeability
d) Interferes with Cell Wall Synthesis
e) Interferes with DNA Synthesis

204. Type-1 error is


a) A true positive
b) Rejecting the null hypothesis when it is in fact true
c) A false negative
d) Accepting the null hypothesis when in fact the alternate hypothesis is true
205. Smokers are at higher risk for deficiency of vitamin
a) B 12
b) B6
c) C
d) E

206. Bioavailability for oral drug could be measured by


a) Area under curve (AUC) intravenous minus the AUC (oral)
b) Area under curve (AUC) oral multiply by AUC intravenous
c) Area under curve (AUC) intravenous divided by AUC (oral)
d) Area under curve (AUC) oral divided by AUC intravenous
e) Area under curve (AUC) oral minus by AUC intravenous

207. What should be monitored when adding alpha blocker to ACE inhibitor
a) Hypotension
b) Fainting
c) Cough
d) Glucose level
e) Serum level

208. Nystatin MOA is? 2012, 2009


a) Binds to ergosterol
b) Inhibits microtubule polymerization
c) Inhibits glucan synthase
d) Inhibits DNA synthesis
e) Disrupting the formation of β-glucan in the cell wall

209. All could be used in treating candidiasis EXCEPT


a) Amphotericin
b) Metronidazol
c) Fluconazole
d) Itraconazole
e) Miconazole

210. Deltiazem side effects includes


a) Increase peripheral resistance
b) Increase heart rate
c) Bradycardia
d) Tachycardia
-Impedes formation of thyroid hormone. -Inhibits the
incorporation of iodine molecules into the amino acid
tyrosine (required to make the precursors of T3 and
211. Propylthiouracil (PTU) MOA is T4) so the formation of the thyroid hormone is
a) Stimulate thyroid secretion impeded.

b) Decrease the amount of thyroid hormone


c) Increase T4 level
d) Increase T3 & T4 level Tardive Dyskinesia symptoms are side effect of medications that
are targeted at the specific dopamine receptor involved in
emotion and lower cognitive function. These medications,
212. Phenothiazine side effect including metoclopramide, chlorpromazin & Phenothiazine.
a) Mood swing Tardive Dyskinesia is characterized by sudden, uncontrollable
movements of voluntary muscle groups (mouth, tongue, cheeks,
b) Hair fall jaw, body, arms, or legs). The patient may move their jaw
c) Muscle tenderness laterally or up and down, as if chewing. The tongue may suddenly
d) Dysphagia protrude or move about in a squirming, twisting manner.
Symptoms may disappear completely when the patient is asleep.
e) Tardive dyskinesia

213. Example for Benzodiazepine receptor antagonist


a) Chlordiazepoxide
b) Flumazenil
c) Flurazepam
d) Clonazepam
e) Tizanidine

214. The mother of a 4-year-old child presents to the pharmacist with a written
prescription for amoxicillin that was ordered by the physician 3 days earlier. She
states that her child was diagnosed with otitis media. Which of the following is the
most appropriate pharmacist response?
a) Fill the prescription as written.
b) Refuse to fill the prescription as the antibiotic order is no longer current.
c) Indicate a need to contact prescriber before filling the prescription at this time
d) Fill the prescription for a quantity that ordered amount less 3 days’ supply.
e) Explain that, at this late date, antibiotic therapy will likely be ineffective for
the child.

215. All considered as correct instructions on giving a preschool child eye drops EXCEPT
a) Gently pull down your child’s lower eyelid
b) Hold the dropper with the other hand, as close to the eye without touching it
c) Place a finger on the tear duct and apply gentle pressure
d) Close child eye for 10 minutes and tip your head down
e) If you are using another type of eye drop, wait at least 5 minutes before giving
it
216. Which one is Acid Related Disease?
a) Chronic diarrhea
b) GERD (Gastro Esophageal Reflux Disease)
c) Gastroparesis  delayed gastric emptying, due to partial paralysis of stomach
d) Crohn's disease

217. Alprazolam is metabolized via


a) CYP2A6
b) CYP2C9
c) CYP5A1
d) CYP2D6
e) CYP3A4

218. All considered as standard first line treatment for TB (Tuberculosis) EXCEPT
a) Ethambutol (EMB)
b) Isoniazid (INH)
c) Pyrazinamide (PZA)
d) Rifampin (RMP)
e) Cycloserin

219. Metastasis is
a) Process when cancer spreads from its original location (primary tumor)
to a new part of the body
b) An abnormal tissue growth or mass that can be benign or malignant
c) Removal of fluid or tissue
d) Form of cancer that develops in the tissue of the body such as the lung or skin.
e) The use of a beam of energy to kill cancer cells

220. Para-Amino Benzoic Acid (PABA) is an intermediate in the bacterial synthesis of


a) Ribosome
b) Plasmid
c) Cytoplasm
d) Cell membrane
e) Folate

221. Canada's national health insurance program, often referred to as


a) OHIP
b) Trillium
c) Medix
d) Medicare
e) CHIP

222. The microorganism responsible for endocarditis after dental procedures is


a) Clostridium perfringens
b) Legionella pneumophila
c) Streptococci viridans
d) Yersinia pestis
e) Staphylococcus aureus

223. A blister pack of 5 medications ordered by the doctor for a patient to be given for
the patient and for 30 days of each medication. The major concern of pharmacist:
A) Blister packs are only for 1 week
B) Blister packs cannot be ordered by the doctor
C) The consent of the patient should be taken
D) None of the above

224. Which of the following agent is the most indicate to treat Athlete’s foot?
a) Tolnaftate
b) Terbinefrine
c) Fluconazole
d) Bethamethasone
e) Clotrimazole
Treatment of the skin with a topical antifungal agent (e.g., ciclopirox 1%,
clotrimazole 1%, ketoconazole 2%, miconazole 2%, terbinafine 1%) twice daily is
the mainstay of therapy.

225. Two drugs with equivalent therapeutic effects are best compared by:
a) Cost-Benefit Analysis
b) Cost-Effectiveness Analysis
c) Cost‐Minimization Analysis
d) Cost-Utility Analysis
e) Willingness to Pay
Here they mentioned both drugs have same therapeutic effects so, when we use
costminimization analysis..so go for least cost drug..n with same therapeutic effect..this
kind of analysis useful in hospital formulary.
226. An obese patient has CHF, renal disease and drinks alcohol. Recently diagnosed with
type II DM, what is drug of choice?
A-Metformin
B-Chlorpropamide
C-Rosiglitasone
D‐Acarbose
E-Glyburide

227. The following applies for inventory turnover rate except:


A. It is used for financial analysis.
B. Cost of goods sold divided by the average inventory at cost.
C. This ratio will increase or decrease in direct proportion with purchase
volume.
D. A ratio of 3 is considered a minimum rate of turnover.

228. Randomized controlled trials can not adequately address the risk of adverse drug
events at drug approval for all of the following reasons EXCEPT?
A: Small number of patients
B: Short study duration
C: Strict inclusion criteria
D: Randomization

229. Q about Marijuana


230. Q about factor affecting emulsion stability (all except Frosting)
231. Q about paternalism
JANUARY 2013
1. To determine the accuracy of a pharmaceutical analysis, instrument must be: (k
type): 2010
a) Sensitive
b) Specific
c) Sensible

2. You are a member of the hospital committee of a hospital. You go to a conference


where pharmaceutical software company man offers you CD worth $75. You would:
a) accept the CD, its free
b) take it and tell your boss
c) don’t take it because of your position in the committee
d) don’t take it because you don’t deal with that company
Take it and give it to the boss or tell the higher in charge that I get this cd it's not
agift in order not
to take it it's free for everyone although it cost 75 dollars in addition it might has
good infos on it.

3. PGF2 is used for glaucoma. You use it when the patient:


a) has hypertension
b) has asthma
c) has blue yes
d) diabetes
e) is using beta blockers
N.B. PGF2 alpha  can be used with or as add on therapy with BBs.

4. Iron can be taken with the following:


a) Lithium
b) Tetracycline
c) Rifampin
d) Alendronate
e) Penicillin v
N.B. Lithium is the drug with the smallest molecular weight

5. Patient loves grapefruit juice; doctor wants to start him on amoidarone. Patient is
aware of the interactions between grapefruit juice and amiodarone. You counsel
the patient:
a) Advising the doctor to switch to another drug
b) Tell the patient not to take grapefruit juice
c) Tell the patient to take the drug 2 hours before the juice or 1 hour after

6. Structure is part of:


Cephalosporin and Penicillin  B‐lactam ring

7. Fluoxetine structure contains: secondary amine

8. Warfarin advice:
a) Watch for diet
b) avoid green leafy vegetables
As a "rule of thumb", green vegetables, especially leafy green vegetables, and certain oils
have a high content of vitamin K. they can decrease the effect of warfarin. However, rather
than avoiding vitamin K, you should maintain a consistent intake of vitamin K by
maintaining a consistent diet.

9. INR stands for: international normalization ratio

10. B. fargilis treatment: metronidazole


Bacteroides fragilis is an obligately anaerobic, Gram-negative, rod-shaped bacterium.
It is part of the normal microbiota of the human colon and is generally commensal, but can
cause infection if displaced into the bloodstream or surrounding tissue following surgery,
disease, or trauma
In general, B. fragilis is susceptible to metronidazole, carbapenems, tigecycline, beta-
lactam/beta-lactamase inhibitor combinations (e.g., Unasyn, Zosyn), and certain
antimicrobials of the cephalosporin class, including cefoxitin. The bacteria have inherent
high-level resistance to penicillin. Production of beta lactamase appears to be the main
mechanism of antibiotic resistance in B. fragilis. Clindamycin is no longer recommended as
the first-line agent for B. fragilis due to emerging high-level resistance (>30% in some
reports).

11. Which has to be sterile solution?


a. otic solution
b. nasal solution

12. What is not a disease because of a virus:


a. rubella
b. mumps
c. hepatitis
d. impetigo
N.B. Impetigo: skin infection  blisters and sores. It is caused by S. aureus or
Streptococcus pyogenus

13. OU stands for


a. both ears
b. right eye
c. left eye
d. both eyes

14. Biotechnology procedures are contaminated by:


a. protein fragments
b. culture media
c. the solution

15. RT‐ PCR (Reverse transcription polymerase chain reaction)


a. higher temperature
b. starts with RNA
c. start with RNA to get to cDNA (complementary)
Reverse transcription polymerase chain reaction (RT‐PCR) is an laboratory technique
combining reverse transcription of RNA into DNA (in this context called complementary
DNA or cDNA) and amplification of specific DNA targets using polymerase chain
reaction (PCR).
It is primarily used to measure the amount of a specific RNA. This is achieved by
monitoring the amplification reaction using fluorescence, a technique called real-time
PCR or quantitative PCR (qPCR). Combined RT-PCR and qPCR are routinely used for
analysis of gene expression and quantification of viral RNA in research and clinical
settings.

16. NADP is required for:


a. Biotin
b. niacin
c. vitamin a
d. vitamin c
N.B. NAD and NADP are precursors of Niacin
17. Patient is trying to quit smoking; which of the following is withdrawal symptoms:
a. loss of appetite
b. lacrimation
c. sweating
d. insomnia
CTC - Withdrawal Symptoms: Dysphoric or depressed mood, irritability, anxiety, difficulty
concentrating, restlessness, increased appetite/weight gain, GI symptoms, headaches and
insomnia. No myalgia.
Symptoms generally peak 24–72 h after the last cigarette and subside after about 2 wk.
Cravings can continue for years, and are probably related to behavioural and
psychological aspects of nicotine addiction.

18. Patient is suffering from redness with the patch, all are true except:
The gum will give the same redness

19. Which of the following does NOT give Parkinsonism effect:


a. Metoclopramide
b. Haloperidol
c. Quetiapine
d. Carbidopa
e. Olanzapine

20. How do you differentiate between acne vulgaris and acne rosacea?
a. Acne vulgaris has papules and pustules on the face while acne rosacea does not.
b. Acne Rosasea occurs at a young age.
c. Acne Rosasea commonly associated with diffuse erythema and flushing and
blushing.
d. Acne Rosacea has comedones and scarring is frequently seen.
e. Acne vulgaris has prominent telangiectasias, phyma formation, and ocular
involvement.
Physical examination of rosacea:
Presence of papules or pustules along with erythema of
the central face; absence of comedones.
Presence of telangiectasia
Evidence of conjunctivitis, blepharitis, stye formation
or complaint of dry eyes rhinophyma.
21. An 18-year-old male with acne tried all kinds of topical treatments and antibiotics and
they all failed. What should he use now to treat his acne?
a. Isotretinoin
b. tretinoin
c. Clindamycin
d. erythromycin
e. benzoyl peroxide
It remains the most powerful anti-acne agent, with the majority of patients achieving clearing and
sustained remission, even in the most severe cases.
It is recommended as first-choice therapy for severe papulopustular or moderate nodular acne and for
nodular or conglabate acne for many reasons: clinical effectiveness, prevention of scarring and quick
improvement of a patient’s quality of life, including minimizing depression.

22. Bacterial acne is due to which bacteria?


a. S. aureus
b. C. Diphtheria
c. P. auregenosa
d. HSV1
e. Yeast
The damage caused by C. acnes and the associated inflammation make the affected tissue more
susceptible to colonization by opportunistic bacteria, such as Staphylococcus aureus. Preliminary
research shows healthy pores are only colonized by C. acnes, while unhealthy ones universally include the
nonpore-resident Staphylococcus epidermidis, amongst other bacterial contaminants. Whether this is a
root causality, just opportunistic and a side effect, or a more complex pathological duality between C.
acnes and this particular Staphylococcus species is not known.

23. What is NOT considered a drug metabolizing enzyme?


a. CYP 450
b. Alcohol dehydrogenase
c. CYP 2D6
d. CYP 3A4

24. How much elemental iron is found in iron sulfate?


a. 10 %
b. 20% Fumarate: 33% (100mg of 300 mg tab)
Sulfate: 20% (60mg)
c. 30% Gluconate: 11.6% (35 mg)
d. 40%
e. 50%
25. Which drug works on Peroxisome proliferator-activated receptor gamma (PPAR-γ) as
a mechanism of action?
a. Metformin
b. Clarithromycin
c. Pioglitazone
d. Acarbose
e. Vancomycin

26. Which of these treat Alzheimer’s disease?


a. Levo-dopa
b. Bromocriptine
c. Benztropine
d. Rivastigmine
e. Nitrosamine

27. Secondary Hypogonadism is due to the deficiency of: 2011


a. Anterior Pituitary gland
b. Hypothalamus
c. Thymus gland
d. Adrenal Cortex
e. Thyroid gland
N.B. Hypogonadism: decrease function of gonads (testes and ovaries)  decrease
testosterone and estrogen due to
decrease FSH and LH in anterior pituitary gland tertiary hypogonadism: hypothalamus

28. ACTH Adrenocorticotropic hormone is secreted from:2011


a. Posterior Pituitary gland
b. Anterior Pituitary Gland
c. Thymus gland
d. Adrenal cortex
e. Pineal gland

29. Which antibiotic should you not refrigerate?


a. Amoxacillin/ Clavulin
b. Cephalexin
c. Cephaclor
d. Clarithromycin
e. Erythromycin Ethinyl Succinate
30. You take Fluticasone inhaler and also take Salbutamol inhaler. What instructions must
your pharmacist tell you regarding these two medications?
a. Take Fluticasone, then Salbutamol then rinse.
b. Take Salbutamol, then Fluticasone then rinse.
c. Take Salbutamol, rinse, then Fluticasone.
d. Take Fluticasone, then rinse then take Salbutamol
e. Take Salbutamol, then Fluticasone.

You must rinse always after taking


corticosteroids- to prevent candidiasis.
SABA > CST Inhaler > LABA > LTRA > CST
l CST IV

31. A woman in her mid 50’s started to be in her post menopausal phase. Her doctor
decided to place her on Hormonal therapy. What basic information must she be told?
a. Progesterone may be given for women who had a hysterectomy (uterus removed)
b. Progesterone causes break through bleeding 
c. Combining estrogen with progesterone to reduce the risk of cancer
Estrogen is the one used in hysterectomy pts and also causes breakthrough bleeding.
Combination reduces the risk of bleeding and the risk of endometrial hyperplasia or
carcinoma in women with a uterus.

32. An obese patient can have all except:


a. Hyperthyroidism
b. Type 1 Diabetes
c. Alzheimer’s disease
d. cold
e. Gout

33. Ketoconazole inhibits this enzyme:


a. Cyp P450
b. Cyp 1A2
c. Cyp 3A4
d. Cyp2C9
e. Cyp 2D6

34. Ozone toxicity is due to: 2012


a. Reduction
b. Oxidation
c. Glucuronidation
d. Decarboxylation
e. Oxidative deamination

35. A little girl had ingested a large amount of organophosphorous compounds and
entered into toxicity. Which drug will be the most suitable antidote?
a. Atropine
b. Pralidoxime
c. Flumezanil
d. Acetyl cysteine
e. Selenium
N.B. we use atropin at first as live saving agent, then we use pralidoxime before aginst
of the enzyme Pralidoxime is no longer available in canada

36. In case of Heparin overdose, what is the correct antidote?


a. Protamine Sulfate
b. Vitamin K
c. Warfarin
d. Na bicarbonate
e. Atropine

37. Treatment of Cystic Fibrosis: (disorder of lung function & AFFECT PANCREAS)2011
a. Pancreatic enzymes
b. High dose Antibodies
c. Prtyolitic Enzymes
d. Amylase enzyme
e. Maltase enzyme
N.B. Cystic fibrosis patient needs lung & pancreas transplantation but the lung is first
because it is life-threatening

38. Short form SF-36:


a. It is used in QALY
b. Humanistic goals
N.B The SF‐36 is a measure of health status, is commonly used in health economics as a variable in
the quality‐adjusted life year calculation to determine the cost‐effectiveness of a health treatment.

39. Cost Minimization Effectiveness: It is a tool used in pharmacoeconomics and is applied


when comparing multiple drugs of equal efficacy and equal tolerability
40. The difference between a Gm -ve and a Gm +ve bacteria:
a. The Cell wall of a gm +ve has Lipopolysaccharide
b. Cell wall of a gm-ve has techoic acid
c. Gm-ve gives a blue stain
d. Gm+ ve has techoic acid thick wall
e. Both Gm+ and Gm-ve are the same
N.B. Opposites of all statements are correct

41. Glutathione is made up of: 2009.2011


a. Cysteine, Glutamate and Glycine
b. Lyseine, alanaine, cytosine
c. Phenylalanine, tryptophan, Glutamate
d. Tyramine, Glutaric acid, Lyseine
e. Tryptophan, Tyramine, Glycine

42. Unmutated Enzyme is Called: 2011


a. Wild
b. Silent (no change in amino acid production)
c. Recessive ‫ﻣﺘﻨﺤﻰ‬
d. Major ‫ﺳﺎﺋﺪ‬
e. Regulatory

43. Canadian Adverse Reaction news letter is released from (CARN)? med effect to health
Canada
Health Product Info Watch (MedEffect™ Canada):
 A monthly publication intended primarily for healthcare professionals.
 Provides factual information on serious or unexpected side effects or adverse reactions
suspected of being associated with health products, such as prescription and non-prescription
medications, natural health products, biologically derived products, and medical devices.
 The Health Product InfoWatch replaced Health Canada's quarterly Canadian Adverse Reaction
Newsletter (CARN) in 2015.

44. Which does not cause Hypertension?


a. Ibuprofen
b. Celecoxib
c. Naproxen
d. Diclofenac
e. Aspirin
N.B. NSAID cause hypertension, but aspirin probably decreases blood pressure.
45. Doubling the dose increased the t1/2 by more than double. This may be due to:
a. saturation of the enzymes
b. higher elimination
c. Lower elimination
d. Higher metabolism
e. Lower metabolism
N.B. Saturation means that all of the enzyme molecules are occupied with substrate.
Adding more substrate will not increase the rate of reaction because the enzymes are
already working full bore.

46. Metformin when taken reduces what levels in your body?


a. Vitamin B12
b. Iron
c. Folic acid
d. Blood urea
e. Niacin
N.B. Vitamin B12 malabsorption is a common and well-documented complication of long-
term metformin use.

47. When a drug passes through the stratum corneum, it depends on:
a. diffusion
b. dissolution
c. pKa
d. partition coefficient
e. size

48. Rosuvastatin works on: HMG‐coA reductase enzyme

49. What can be taken with grapefruit juice?


a. Atorvastatin
b. Rosuvastatin
c. Simvatatin
d. Fluvastatin
e. Lovastatin
N.B. Avoid grape fruit juice with Atorvastatin, Lovastatin, Simvastatin (ALS)
Fluvastatin also can be taken with grape fruit
50. A female comes and takes Oral contraceptives and asks for dimenhydrinate. This is
given for:
a. nausea and vomiting
b. sleep apnea
c. allergy
d. headache
e. vision improvement

51. Biotechnology methods: gene cloning, recombinant DNA, antisense technology


Biotechnology is the use of living systems and organisms to develop or make useful
products

52. Mg Stearate is used for:


a. Humectant b. Effrorescence c. Lubricant d. Diluent e. sweetner
N.B. Magnesium stearate is often used as an anti-adherent. It has lubricating properties,
preventing ingredients from sticking to manufacturing .

53. Anionic Surface-active agent:


a. Sodium Lauryl sulfate b. Tween c. Spans e. Lactose

54. What is not a polysaccharide:


a. Cellulose
b. HPMC hydroxyl propyl methyl cellulose
c. HMC hydroxyl methyl cellulose
d. Bentonite absorbent aluminium phyllosilicate, essentially impure clay consisting
mostly of monomorillonite
e. HEC hydroxyl ethyl cellulose

55. Which of these measures the iron deficiency anaemia?


a. High iron binding
b. low iron binding
c. high MCV
d. High Hb count
e. Large Blood cell size
N.B: iron is low in your blood so the body tries to keep it as much and increases its
binding
Higher‐than‐normal TIBC and increase serum transferritin and decrease ferritin
level.
56. Term for a group of anaemia
Megaloblastic anaemia  folic acid + vit b12
pernicious anaemia  vit b12
iron deficiency anaemia
aplastic anaemia  bone marrow doesn't make enough new blood cells
Haemolytic anaemia  red blood cells are destroyed and removed from the
bloodstream
Thalassaemias  inherited blood disorders cause the body to make fewer healthy
red blood cells and less haemoglobin
Sickle cell anaemia  abnormal haemoglobin that causes the cells to have a sickle
shape, which don’t move easily through the blood vessels
Fanconi anaemia  rare, inherited blood disorder that leads to bone marrow failure

57. BANNER IS A TYPE OF: FRANCHISE


Anything with the word banner = franchise

58. Confounding means to be mixed up and not able to distinguish


Confounding: is interference by a third variable so as to distort (give a misleading
or false account) the association being studied between two other variables,
because of a strong relationship with both of the other variables.

59. Which can’t be used for Community Acquired Pneumonia?


a. Penicillin V b. Doxycyclin c. Moxifloxacin d. Nadifloxacin e. Ceftriaxone
Azithromycin or clarithromycin although a fluoroquinolone such as levofloxacin can
substitute. Doxycycline is now the antibiotic of choice in the UK for complete coverage of
the atypical bacteria. This is due to increased levels of Clostridium difficile seen in hospital
patients being linked to the increased use of clarithromycin.
Children should receive intravenous ampicillin, ceftriaxone, or cefotaxime.

60. In end-stage renal failure, Calcium is given to:


a. Reduce phosphate
N.B. calcium carbonate can control hyperphosphataemia in dialysis patients

61. Cyclosporin Mechanism:


a. Inhibits IL-1
b. Inhibits IL‐2
c. Inhibits uric acid formation
d. Inhibits PG formation
e. Inhibits 5HT 1b/d receptors
In medicine,the most important effect of cyclosporin is to lower the activity of T
cells and their immune response. Cyclosporin binds to the cytosolic protein
cyclophilin (immunophilin) of lymphocytes, especially T cells. This complex of
cyclosporin and cyclophilin inhibits calcineurin, which under normal
circumstances, is responsible for activating the transcription of interleukin 2

62. Spirometer is used for:


a. Measuring blood pressure
b. Measures the amount of Haemoglobin
c. Measuring volume of air inspired and expired by lungs
d. measures the rate of sedimentation
e. Measures the rate of turnover rate of cells

63. Total lung capacity? Total lung capacity (TLC) is the maximum volume of air
present in lungs

64. Diabetic nephropathy cannot have??


Diabetic nephropathy is progressive disease with slow & progressive increase in
albuminuria, a decline in estimated glomerular filtration rate (eGFR) to less than 60
mL/min/1.73 m2 and can lead to end-stage renal disease requiring dialysis or renal
transplantation.
They develop in late stages and may be a result of excretion of high amounts of protein in
the urine or due to renal failure, edema, swelling, usually around the eyes in the mornings;
later, general body swelling may result, such as swelling of the legs foamy appearance or
excessive frothing of the urine (caused by the proteinuria), unintentional weight gain
(from fluid accumulation), anorexia (poor appetite), nausea and vomiting,
malaise (general ill feeling), fatigue, headache, frequent hiccups.
The first laboratory abnormality is a positive microalbuminuria test. Most often, the
diagnosis is suspected when a routine urinalysis of a person with diabetes shows too
much protein in the urine (proteinuria). The urinalysis may also show glucose in the urine,
especially if blood glucose is poorly controlled. Serum creatinine and BUN may increase as
kidney damage progresses.

65. Serotonin comes from:


a. Tryptophan
b. Histidine
c. Lysine
d. alanine
e. glycine

66. A structure is given. Count the number of Cis and chiral centers:
Cis: double bond which is symmetrical around the double bond
Chiral centers: 4 different components around the C

67. A female comes to the pharmacy to get plan B (emergency contraceptive pills). Which
of these are correct?2009
a. Give it to her as long as it’s been 7 days since her sexual intercourse.
b. If she is pregnant, you may dispense it.
c. She may have them as long as she hasn’t exceeded 72 hrs since her sexual activity
d. you may not dispense it if you don’t want to.
e. Let her go and tell her she may not need it.

68) Which drug causes dry mouth?


a) Tiotropium
b) Pilocarpine
c) Actylcholine
d) Physostigmine
e) Neostigmine

69) A short acting BZ is:


a) Lorazpam
b) Oxazepam
c) Diazpam
d) Triazolam
e) Temazepam

70) Which of these drugs give a bronchoconstriction effect?


a) Propanolol
b) Tiotropium
c) Metoprolol
d) Carvedilol
e) Fluticasone
N.B. Propranolol: This drug may reduce blood flow to your hands and feet, causing them to feel
cold (peripheral vasoconstriction). B blockers cause bronchoconstriction
71) A patient has a severe migraine and requests to take an herbal remedy. Her doctor
gave her Feverfew. When counselling her, what should you tell her?
a) Take with sumatriptan every hour, and if there is no effect repeat dose every two hours
b) Tell her not to take it and take Sumatriptan as a prophylaxis
c) Ask her if she has allergy to Chrysanthemum parthenium, and if she is, counsel
her that she may be allergic to this herbal remedy also
d) Tell her to ask the doctor for another drug
e) Call the doctor

72) A 4-year-old child with otitis media is not started immediately on amoxi/clav
treatment. The reason why the doctor told the parents to wait and watch for a period of a
week is because:
a) It may become viral after this period of watching
b) It’s asymptomatic and that’s why the period of waiting is necessary
c) Spontaneous healing will occur
d) You must wait and get a bacterial culture to identify the microorganism
e) The doctor misdiagnosed

73) Digoxin Toxicity occurs in all EXCEPT:


a) Pulmonary Fibrosis
b) Quinidine Treatment
c) Hypokalemia
d) Corticosteroid treatment (HYPOKALEMIA)
e) Dehydration

74) Lysosomes are responsible for cellular:


a) Protein formation
b) energy production
c) enzyme digestion
d) permeability
e) transport of DNA
N.B: Lysosomes are the cell's waste disposal system and can digest some
compounds.

75) This drug is used for children enuresis and has an antidiuretic action was recently
discovered to have a new action in the treatment of neuropathic pain:
a) Imipramine
b) Neostigmine
c) Gabapentin
d) Minoxidil
e) Amoxacillin
N.B. Imipramine (Tofranil), also known as melipramine, is an a tricyclic
antidepressant(TCA) of the dibenzazepine group. It is mainly used in treatment
of major depression and enuresis (inability to control urination).

76) A patient is started on Fenasteride treatment for his BPH. In order to check the
efficacy
of his treatment, how often should he go for periodic checks?
a) Every 1-week b) Every 2 weeks c) Every 3 m d) Every 6 months e) Every year

77) This tablet should NEVER be crushed, because once it is crushed, its action is
completely lost:
a) Sugar coated tablets
b) Chewable tablets
c) Lozenges
d) Osmotic tablets
e) Effervescent tablets
OROS (Osmotic [Controlled] Release Oral [Delivery] System) is a controlled release oral
drug delivery system in the form of a tablet. The tablet has a rigid water-permeable jacket
with one or more laser drilled small holes. As the tablet passes through the body, the
osmotic pressure of water entering the tablet pushes the active drug through the opening
in the tablet. e.g. adalat, cardura XL

78) A lady comes into your pharmacy and is taking Captopril, Digoxin,
hydrochlorothiazide, and Furosemide. What should she monitor?
a) K levels and Mg levels
b) K levels and serum Creatinine
c) Serum TG
d) Glucose levels
e) Serum Phosphates

79) Which of the following is used as a humectant?


a) Magnesium Stearate
b) Triethanolamine
c) Glycerine
d) Mineral oil
80) Alcohol is converted into what metabolite by alcohol dehydrogenase?
a) Alcohol
b) Methanol
c) Aldehyde
d) Acetic acid
e) Ketone body

81) Sulfasalazine becomes an active drug metabolite called


5-Aminosalicylic acid (5-ASA) and is used mainly to treat:
a) Inflamatory Bowel Disease (ulcerative colitis)
b) GERD
c) Mouth ulcers
d) Pain relief
e) Headaches
Crohn's disease: Inflammation is present from the esophagus to the anus but
predominantly in the small bowel or colon, Obstruction of the bowel & abscess formation.
Drug of choice to treat mild and moderate oral budosenide and severe iv steroid.
Ulcerative colitis: Relapsing inflammatory condition in the colon with symptoms of
bleeding, urgency, diarrhea and tenesmus, Erosion and ulceration of the mucosa, Decrease
in the number of goblet cells, Frequent infection secondary to fever and anemia
• Drug of choice in mild to moderate is 5ASA
• Drug of choice in severe ulcers is oral prednisone

82) In the CPS you may find:


a) Latin abbreviations
b) Disease guidelines
c) Other options
CPS Content:
1.1 Brand and Generic Name Index
1.2 Therapeutic Guide
1.3 Product Identification
1.4 Clin‐Info
1.5 Directory
1.6 Monographs
‐1.6.1 Product Monographs
‐1.6.2 CPhA Monographs
1.7 Information for the Patient
1.8 Drug Updates
1.9 Appendices: Health Canada drug regulatory and monitoring programs’
reporting and request forms on such topics as Narcotic and Controlled Drugs,
Benzodiazepines and Other Targeted Substances, and Adverse Events Following
Immunization: Surveillance and Reporting.
1.10 Glossaries: including Medical Abbreviations, and Latin Prescription Terms

83) ARR, long question 2012


absolute risk reduction, risk difference or absolute effect is the change in the risk of
an outcome of a given treatment or activity in relation to a comparison treatment or
activity. It is the inverse of the number needed to treat. In general, absolute risk
reduction is the difference between one treatment comparison group's event rate
(EER) and another comparison group’s event rate(CER)
RRR means (Big‐small)/big *100, ARR means just big‐small = EER‐CER
NNT means 1/ARR*100, Absolute risk is drug/(drug+placebo)

84) Find the Molarity of 27 mg dissolved in 100ml volume


a) 0.27 M
b) 2.7 M
c) 27 M
d) 0.027 M
N.B. Molarity: one mole of a substance dissolved in a litre of water
𝒎𝒐𝒍𝒆𝒄𝒖𝒍𝒂𝒓 𝒘𝒆𝒊𝒈𝒉𝒕 𝒊𝒏 𝒈𝒓𝒂𝒎𝒔
Molarity =
𝒍𝒊𝒕𝒓𝒆𝒔

85) Pharmacy with net profit margin 15%. Net profit is $ 90000 to retain on his
investment
a) $ 600000 = (90000/15) * 100 = $600,000 $
b) $ 200500
c) $ 900000
d) $ 450000

86) Autoclave sterilization may be used for all EXCEPT:


a) Parentral solutions
b) ophthalmic solutions
c) Oily substances and wax
d) Glassware
e) Ampoules
N.B. Oily substances and wax are sterilized by dry heat (oven)
87) What is NOT a function of the skeletal system?
a) Blood vessel innervations and soft tissue
b) Transport food and oxygen through the body
c) Shape and support
d) Protection of vital organs
e) Bone marrow production and blood formation
N.B. Innervations: to stimulate (a bodily organ or part) with nerve impulses

88) COPD is a respiratory disease NOT characterised by what?


a) Reversible action and easily treated
b) Widely related to smoking as its number one cause
c) Emphysema and Chronic bronchitis are the manifestations of COPD
d) Cough, sputum and Dyspnea
e) It is a progressive destruction of the alveolar sac
N.B. Asthma is reversible

89) Dihydropyridine – Na sensitive


Voltage‐dependent calcium channels (VDCC) are a group of voltage‐gated ion
channels found in with a permeability to the calcium ion Ca2+. These channels are
slightly permeable to sodium ions, so they are also called Ca2+‐ Na+ channels, but
their permeability to calcium is about 1000‐fold greater than to sodium under
normal physiological conditions.

90) Which of the following heart conditions is the most dangerous?


a) Atrial fibrillation
b) Ventricular arrhythmia
c) Torsade de Point
d) Other names

91) During the relative refractory Period, phase III, there’s a repolarized state.
During this period, what occurs?
a) K goes back into the cell
b) Na goes back into the cell
c) K goes out of the cell
d) Ca goes into the cell
e) Na gets out of the cell
92) Which of the following drugs cannot be transferred from a pharmacy to another?
a) Propanolol
b) Lorazepam
c) Benztropine
d) Meperidine (straight narcotic)
e) Spiriva (tiotropium)

93) A patient comes into your pharmacy wanting to purchase sunscreen.


The pharmacist should advise him to:
a) Purchase an oil-based sunscreen
b) Purchase a water-based sunscreen
c) Tell him to purchase a sunscreen of SPF 15 at least
d) Tell him to buy a sunscreen that is waterproof
e) Tell him that sunscreen will prevent him from tanning

94) Which does NOT cause urinary retention?


a) oxybutynin
b) imipramine
c) Amytriptilline
d) Atropine
e) Propranolol
N.B. urinary incontinence: DOC adults: oxybutinin. DOC children: desmopressin
(ADH). Imipramine (TCA) is also used

95) When a person is bruised, what heme breakdown product is found?


a) Bradykinin b) Biliverdin c) b-carotene d) Creatinine
Biliverdin is a green tetrapyrrolic bile pigment, and is a product of heme catabolism. It is
the pigment responsible for a greenish color sometimes seen in bruises.

96) The following are risk factors of osteoporosis:


a) Obesity b) Cold c) Calcium deficiency d) Cholesterol e) Hypothyroidism

97) What is the ionization of aspirin when the pH is 8.9 and the pKa is 8.9?
a) 100% b) 75% c) 50% d) 25 % e) 0%
𝟏𝟎𝟎
N.B: % ionized = (Handelsen – Haselback equation)
𝟏 𝟏𝟎 𝒄𝒉𝒂𝒓𝒈𝒆 𝑷𝑯 𝑷𝑲𝒂
Charge = ‐1 in case of acid, charge = 1 in case of base
If PH–Pka = 0 50% ionized
If ph‐pka= 0.5 75% ionized and 25% unionized or vice versa
If ph‐pka=1 90% ionized and 10 unionized or vice versa
If ph‐pka=2 99% ionized or unionized
If ph‐pka=3 99.9% ( 100%) ionized or unionized

98) A person having a cold sore can use which of the following medications?
a) Abreva (Docosanol). b) Salbutamol c) Aspirin d) Zinc oxide e) Burrow’s solution

99) What is affected by CYP enzyme?


a) Sequinavir
b) Gentamycin
c) Vancomycin
Vancomycin is rapidly excreted in the urine without significant hepatic metabolism
Gentamicin is not metabolized. It is excreted by glomerular filtration in an active,
unchanged form. Sequinavir is an antiretrovirus drug used to ttt HIV.

100) Thyroxin-Globulin
a) osmosis regulation b) thyroid transport
Thyroxine‐binding globulin (TBG) binds thyroid hormone in circulation. It is responsible for
carrying the thyroid hormones thyroxine (T4) triiodothyronine (T3) in the bloodstream.

101) All are tablet making errors except:


a) Mottling unequal distribution of colour on a tablet
b) Capping the top of the tablet (upper cap) wanting to split
c) Sticking Sticking = picking
d) Creaming separation of an emulsion or cream into 2 phases

102) Hard Capsule coat has a material made of JC‐CAPS®. The purpose of using this is for:
a) To prevent tampering of the dose
b) To deliver a consistent amount of dose each time
Prescription sedatives are usually taken in pill form; however, some are available as
suppositories or prepared as a solution for injection. Some people tamper with the
medication for non-medical use for the drug’s euphoric effects. Tampering involves
changing the form of the medication or the route by which it is taken or both.
There are 3 main approaches to deter tampering with opioids, each based on a
presumption that abusers will tamper with intact tablets. These include agents with
physical barriers to crushing, chewing, and extraction; agents with sequestered aversive
agents; and agents with sequestered opioid antagonists.
103) Antisense Technology
Antisense therapy is a form of treatment for genetic disorders or infections. When the
genetic sequence of a particular gene is known to be causative of a particular disease, it is
possible to synthesize a strand of nucleic acid (DNA, RNA or a chemical analogue) that will
bind to the messenger RNA (mRNA) produced by that gene and inactivate it, effectively
turning that gene "off". This is because mRNA has to be single stranded for it to be
translated.

104) Drug alteration of patient with weak elimination and a liver disease:
a) high drug concentration in the blood
b) low drug concentration in the blood
c) ionized drug
d) unionized drug
e) drug will not be found in the blood

105) A purine metabolite found in the metabolism pathway of uric acid:


a) Hypoxanthine
b) Mercuric acid
c) lactic acid
d) allopurinol
e) azathioprine

106) A calculation question where you try to solve for the volume. Once you have it you
try to measure the concentration. Drops/ml question. Aerosol OT (AOT) is used in
veterinary medicine as a laxative. If 250 g of AOT is dissolved in 1000 ml of glycerine
(density of glycerine is 1.25 g/ml), the concentration of AOT in the solution is:
a. 12.5% w/w.
b. 16.7% w/w.
c. 20.0% w/w.
d. 23.8% w/w.
e. 25.0% w/w
You have the density for the glycerine and you have the volume of glycerine use the
density equation.
(d=w/v, and it will be 1250g).
To change the glycerine from volume to weight because he is asking for the percentage
w/w then (1250+250=1500g the total weight)
To find the percentage of AOT 250g in (1500g the total) ... 250/1500*100= 16.6%
107) Trimethoprim/Sulfa dose is:
a) Synergistic b) Additive c) No relationship

108) sphygmomanometer use:


a) Lipid level measurement
b) Blood pressure reading
c) Measures the lung capacity
d) Measures BMI
e) Measures the uric acid levels
N.B. A sphygmomanometer or blood pressure meter (also referred to as a sphygmometer)

109) Schizophrenia symptoms are all EXCEPT:


a) Hallucinations b) Social withdrawal c) Delusions d) Sleep apnea e) Paranoia

110) A pregnant lady has a dental abscess. She may take what drug therapy?
a) Amoxacillin
b) Penicillin V
c) Tetracyclin
d) Erythromycin
e) Vancomycin

111) All these drugs cause hypoglycemia except:


a) Insulin
b) Gliclazide
c) Glyberide
d) Prednisone
e) Chlorpropamide
N.B. Corticosteroids cause hyperglycemia, hypertension, osteoporosis, depression,
increase Na, decrease K, increase BUN, Chlorpropamide is a sulfonyl urea drug

112) This is NOT a part of the Canadian Health Act:


a) Universality
b) Portability
c) Private administration
d) Public administration
e) comprehensiveness
113) The pharmacist’s role is all except:
a) Patient counselling
b) Diagnosis of the disease
c) Finding and fixing prescription errors
d) Contacting the insurance company for patient issues
e) Delegating jobs and managing the pharmacy

114) Tadalafil is different from Sildenifil beacause:


a) sildenafil is longer acting
b) Tadanafil is shorter acting
c) Sildenifil is not potent
d) Tadanafil and Sildenifil are both equally active and have the same duration
e) Tadanifil is much longer acting than Sildenifil

115) The process of distributing tasks and dividing up the work is: 2010
a) Delegation
b) Managing For me this is Managing
c) Assisting
d) Autocratic
e) Bossing people around

116) All are true about dispensing errors except:


a) Increasing the number of technicians increases the risk of errors during
dispensing
b) Increasing the number of pharmacists on duty decreases the risk of error
c) Counselling the patient and follow up prevent dispensing errors
d) Can be avoided by looking at the DIN number
e) Can be avoided by keeping clear counter tops

117) What drug should you NOT give with H. Pylori?2011,2012


a) Amoxacillin b) Metronidazole c) Ciprofloxacin d) Omeprazole e) Clarithromycin

118) In CHF, what drug has inotropic action?


a) Dobutamine b) Digoxin c) Carvedilol
dobutamine and digoxin are +ve inotropic drugs while carvedilol is a –ve inotropic drug

119) Thixotropic agents:


a) Consolidate on shaking from solution to solid
b) Liquify on shaking from solid to solution
c) Follow Newtonian Solutions
d) Have a shear thickening action
e) Have no relation to viscosity
N.B. In thixotropy system, with shear stress ‐‐ decrease in viscosity and increase
flow ability e.g. transformation of gel to solution
In antithixotropy system, with shear stress ‐‐ increase in viscosity and decrease flow
ability e.g. transformation of solution to gel

120) An elderly lady 70 years of age may have all the following disorders EXCEPT:
a) Malnutrition
b) Alzeheimer’s disease
c) Cardiovascular disease
d) Bone disorders
e) Bulemia

121) 4% Ethanol and Water may be mixed together. In this case, Ethanol is:
a) Solvent
b) Co‐solvent
c) Viscosity enhancer
d) Humectant
e) Solute

122) A child has fever. All of the following are true EXCEPT:
a) Aspirin is completely contraindicated with a viral infection symptoms present giving
rye’s syndrome
b) Ibuprofen is completely contraindicated with a viral infection symptoms present
giving rye’s syndrome
c) Applying Cool compress will reduce the fever
d) Sweating and removal of clothes and blankets is necessary
e) Fever must be controlled to prevent any neurological complications
2nd day
1. The manager of a pharmacy is partalking in some illegal acts. The technician is aware
of it and notifies the body responsible for law enforcement. What is this referred to?
1) Whistle Blowing
2) Paternalism
3) Conscious Clause

2. HAMA stands for:


a) Human Antigen Mouse Antibody
b) HUMAN ANTI MOUSE ANTIBODY
An antibody found in humans which reacts in immunoglobins found in mice. Antibody treatment is a
type of therapy that is used to treat certain types of cancer and immune disorders.

3. SPO stands for:


a) Structure, Product, Outcome
b) Structure, Person, Outcome
c) Student, Place, Outcome
d) Structure, Process, Outcome

4. Toxicity of Gemfibrozil is associated with:


a) Nausea
b) Choleithiasis  gallstone disease
c) Vomiting
d) GIT Bleeding

5. What is involved in PCR? Endonucleases

6. Lipophilic emulsifying agent in parenteral IV:


LICETHEN, PHOSPHOLIPID, EDTA
Propofol (milk of magnesia) in a mixture of water, soy oil, and egg lecithin (phospholipid).

7. Which drug is known for its interactions with Digoxin?


a) Vit. D
b) Vit. B12
c) Warfarin
d) Heparin
Cholecalciferol can increase the levels of calcium in your body. Since calcium can enhance the effects of
digoxin on heart, using too much cholecalciferol can cause digoxin toxicity and irregular heart rhythm.
8. Protrusion of the upper part of the stomach through the thorax as a result of tear or
weakness through the diaphragm is referred to as: 2007
a) Inguinal Hernia
b) Liver Ascites
c) Sciatic Hernia
d) Hiatus Hernia

9. Which receptor does Sumatriptan work on? 2007


a) Benzodiazepine receptors
b) 5HT3 receptors
c) 5HT 1B/1D receptors
d) 5HT2 receptors

10. Streptococcus Viridans is the main causative organism of which of the following:
a) Endocarditis
b) Pneumonia
c) Skin infections
d) Allergic Rhinitis

11. An alternative Choice for treatment of Pseudo membranous Colitis second to


Metronidazole:
a) Clindamycin
b) Vancomycin
c) Floxacin
d) Erythromycin
NB. Vancomycin is DOC for MRSA.
Pseudomembranous colitis, a cause of antibiotic-associated diarrhea (AAD), is an inflammation of
the colon. It is often, but not always, caused by the bacterium Clostridium difficile.

12. A patient has HIV associated with a CD4+ count over 120. Prophylaxis is needed for
which disease?
a) Pneumocitis Jiroveci pneumonia (pneumocystitis carinii pneumonia)
b) Tuberculosis
CD4 cells or T-helper cells are a type of white blood cell that fights infection and their
count indicates the stage of HIV or AIDS in a patient.
350  M. tuberculosis, 275  Kaposis sarcoma
200  Non-Hodgkins lymphoma, 100  P.carinii pneumonia
50  Cytomegalovirus and M avium intracellular
13. Which drug causes Retinopathy?
a) Heparin
b) Chloroquine
c) Isoniazid
d) Quinolones

14. The below structure is associated with which of the following drugs:
a) Tricyclic Anti-depressants
b) Quinolones
c) Tetracyclines
d) Diuretics

15. The following drug is called?


a) Benzthiazole
b) Benzimine
c) Benzofuran
d) Benzimidazole

16. S - Phase in the cell cycle is associated with which of the following:
a) Mitosis
b) DNA Replication
c) Resting Stage
d) DNA synthesis
S‐phase (synthesis phase) is the part of the cell cycle in which DNA is replicated

17. Ventricular Arrhythmia will have: Prolonged QRS

18. What is the mechanism of action of Calcium Channel Blockers?


a) Reduce Myocardial Oxygen Demand and have vasodilatation effect
CCBs are ‐ve inotropic and ‐ve chronotropic

19. Which drugs help protect the heart and prolong life?
b) Diogoxin
c) ACE Inhibitors
d) Diuretics
e) Calcium Channel Blockers
20. Cardioselective Beta Blocker:
f) Propranolol
g) Timolol
h) Nadolol
i) Atenolol

21. Auxillary label for Prednisone  taper the dose when discontinuing

22. Which enzyme is responsible for breakdown of Protein in the stomach?


a) Pepsin
b) Gastrin
c) Amylase
d) Lipase All amino acids except glycine are chiral because
they all contain at least one chiral centre. The central
carbon has four different groups attached. So the
23. Amino Acids- something about chiral centre… compound can exist as a pair of nonsuperimposable
mirror images. Thus, we can have D- and L-isomers
of amino acids.
24. Which is considered a short acting Benzodiazepene?
j) Diazepam
k) Clonazepam
l) triazepam
m) Alprazolam
Long acting: Diazepam (longest half life), Flurazepam, Clonazepam, Chlordiazepoxide 
Used in seizure, alcohol withdrawl
Intermediate acting (TemOLAN): Temazepam, Oxazepam, Lorazepam, Alprazolam and
Nitrazepam  Used in REM sleep insomnia (to maintain sleep) & Anxeity ↑ REM sleep,
alcohol withdrawal.
Short acting: Triazolam, midazolam (shortest half-life). Have no phase I metabolism. Used
in insomnia (to fall in sleep), ↑ REM sleep (rapid eye movement)

25. A patient using smoking cessation aid found that Nicotine patch gives him
unpleasant side effects mainly associated with rashes. What should the patient do to
minimize those Side Effects?
a) Stop using the nicotine patches
b) Refer to the doctor and seek a treatment for the allergy associated rashes.
c) Removal of the nicotine patches at night as they are not required
d) Use the patches for one week then stop them every other week.
26. Which of the following drugs is permitted a transfer if requested:
a) Lorazepam  all BDZ are allowed for one transefer
b) Morphine
c) Amphetamines
d) Secobarbital

27. Which type of Study is evidence-based analysis and is


the most reliable and detailed form of Analysis?
a) Randomized
b) Meta Analysis
c) Quantitative Analysis
d) Qualitative Analysis

28. A pregnant woman is diagnosed with Deep Venous Thrombosis. Which is the most
appropriate
drug to administer?
a) Give Heparin followed by oral Warfarin
b) Give Warfarin
c) Give Clopidogrel
d) Give Dalteparin

29. A patient experiencing paralysis in


their ciliary muscles, resulting into loss
of accommodation, leading into vision
problems. What is the most appropriate
diagnosis for such condition?
a) Myopia
b) Cycloplegia
c) Conjunctivitis
d) Blindness

30. Which is the most appropriate anaesthetic used for treatment of hemorrhoids?2012
a) Calamine Lotion
b) Pramoxine
c) Burrow’s Solution
d) Zinc Oxide
31. Which drug is most appropriate antiperspirant in treatment of hyperhidrosis? 2012
a) Sodium Acetate
b) Ammonium Nitrate
c) Aluminum Chloride
d) Dibasic Sodium Phosphate

32. Which test is used for Colorectal Cancer Screening?


a) Pap smear test
b) Fecal Occult Blood Test
c) Prostate Specific Antigen (PSA)
d) Colon Specific Antigen

33. Psoriasis is characterized by? scaly silver (Red borders with silver scales)
Psoriasis is a common, chronic immune-mediated skin disease which may also affect
the joints. Psoriasis is characterized by scaly, erythematous (reddened)
patches, papules, and plaques which are usually pruritic (itchy).

34. The transformation of RNA from DNA is known as:


a) Reverse transcription
b) Transcription
c) Translation
d) Translocation
Transcription  first step of gene expression, in which a particular segment
of DNA is copied into RNA
*Reverse Transcription is the process of formation of cDNA (complimentary DNA)
from RNA*

35. Meniere’s disease should be treated with which drug? Betahistine


Ménière's disease is a disorder of the inner ear that can affect hearing and balance to a
varying degree. Treatmant: motion sickness drug, anti nausea medication, diuretic
Betahistine dihydrochloride is an antivertigo drug.
Wax
36. Which gland in the ear produces sweat?2012
a) Parotid gland
b) Ceruminous gland
c) Sebacious gland
d) Salivary gland
37. Which vitamin can be administered orally in treatment of Acne:
a) (13cis)‐retinoic acid (isotretinoin)=(ACCUTANE)R
b) 13 trans- retinoic acid (topical)= (RETIN-A) R
c) Vit. B12
d) Vit. C

38. In a person experiencing Gastric Acid Reflux, what should he take before food to
relieve him from such symptoms?
a) Sucralfate a mucosal protective
b) Ranitidine
c) Omeprazole
d) Clarithromycin

39. What is the main effect of occlusive dressings that are applied to the skin?
a) Softens the skin by providing an oily layer
b) Softens the skin by maintaining hydration to the skin
c) Prevents harmful microorganisms from penetrating the skin
d) Cools the skin in cases of burns and wounds
An occlusive dressing is an air- and water-tight trauma dressing used in first aid. These
dressings are made with a waxy coating so as to provide a total seal, and as a result do not
have the absorbent properties of gauze pads. Occlusive dressings come in various forms,
including Vaseline Gauze, which sticks to the skin surrounding the wound using Vaseline.

40. What is the main Structure Activity Relationship of the addition of an ethinyl group
in estradiol?
a) increases its toxicity
b) Inreases its solubility
c) Decreases its t ½
d) Increases its resistance to degradation
N.B Substitution at C17 of the estrane steroid with an ethinyl group served to
provide an estrogen that is much more resistant to degradation

41. Which vitamin supplement is not to be administered in pregnant women?


Vit A Vit B Vit C Vit D

42. Which antipsychotic should be avoided in diabetes? Olanzepine


it may produce non-trivial hyperglycemia in patients with Diabetes mellitus.it cause
wight gain and hyperlipidemia
43. Tapering in usually done in SSRIs except with which of the following?
n) Fluoxetine
o) Paroxetine
p) Sertraline
q) Citalopram

44. B-amyloid bodies are characteristic of which disease?


a) Parkinsonism
b) Alzheimer’s
c) Psychosis
d) Depression

45. A patient is diagnosed with stable angina. What is most likely associated with his
condition?
a) Sleep apnea
b) Raking too much leaves
c) Shortness of breath during exertion
d) Experiencing stressful and emotional situation

46. There is increase in dose of venlafaxine. What will be needed to be monitored most:
a) Temperature
b) Blood glucose
c) Serum Creatinine
d) Blood pressure
Monitoring blood pressure and heart rate once a week is recommended during the
1st month of treatment and when increasing doses.

47. Which immunoglobulin is most associated with Allergic Rhinitis?


a) IgA
b) IgD
c) IgE
d) IgM

48. Surfactants are used in contact lenses for which reason? Increase Viscosity
Surfactants are compounds that contain both a hydrophilic and a hydrophobic
portion and serves to increase wettability on a lens surface.
49. What is the reduced form of Glutathione?
a) Cysteine
b) Glutamate
c) Glycine

50. Which drug is most appropriate in treating chemotherapy induced nausea and
vomiting?
a) Bleomycin - antibiotic
b) Diclectin – for N&V in pregnancy
c) Scopolamine – anti muscarinic
d) Dexamethazone
 The lowest emitogenic drugs nausea and vomiting can be treated by
Dexamethasone PRN
 High and very high emitogenic drugs associated acute: nausea and vomiting
can be treated by Dexamethasone + Ondansetrons
 DOC for delayed nausea and vomiting  Dexamethasone
 Anticipatory nausea and vomiting  Benzodiazepine.

51. Citalopram + MAOi Can give? serotonin syndrome

52. Budesonide intra nasal spray


Budesonide is a glucocorticoid steroid for the treatment of asthma, COPD and non-
infectious rhinitis (including hay fever and other allergies), and for treatment and
prevention of nasal polyposis (masses arising mainly from the mucous membranes of
thenose and paranasal sinuses). In addition, it is used orally for Crohn's
disease (inflammatory bowel disease).
Budesonide nasal spray is used to prevent and relieve rhinitis.

53. Which does not have vaccine?


a) H. influenza H. Influenza has vaccine Hib
b) Malaria Malaria has prevention only .. No vaccine

INFLUENZA A & B VIRUS HAVE VACCINE, H. INFLUENZA IS BACTERIA HAS NO


VACCINE
Malaria vaccine is a vaccine that is used to prevent malaria. The only
approved vaccine as of 2015 is RTS, S, known by the trade name Mosquirix. It requires
four injections, and has a relatively low efficacy.

54. Metoclopramide known uses: anti‐nausea, anorexia (antiemetic)


55. Acetyl salicylic acid broken down by which enzyme? Esterase

56. Levodopa + MAO inhibitor Selegline


Levodopa and selegiline (selective MAO b inhibitor) are used to treat parkinson’s
disease

57. Neuropathic pain which drug not used 


tricyclics and selective serotonin-norepinephrine reuptake inhibitors (SNRI's),
anticonvulsants, especially pregabalin (Lyrica) and gabapentin (Neurontin), and
topical lidocaine. Opioid analgesics and tramadol are recognized as useful agents but
are not recommended as first line treatments.

58. Taxi driver with migraine, what to recommend?


Sumatriptan causes drowsiness and dizziness, PROPRANOLOL IS USED IN THIS CASE.

59. Marijuana  how to get it prescribed  through Health Canada's Marijuana


Medical Access Regulations (MMAR) program.

60. Tramadol: to check its schedule?


1‐check CSA (controlled substances act)
2-Narcotic Regulatory

61. Infliximab S.E? infection or TB


N.B. Infliximab should be administered with methotrexate to prevent the formation of
antibodies to infliximab

62. Enteric coated tablet? Delayed, extended, controlled


Its release is in small intestine

63. Epigastritis which Drug causes it? most commonly caused by alcohol use,
tobacco use, and prolonged use of aspirin and (NSAIDS).

64. Non-terminal sterilization? Filtration


i.e we have to do another type of stralization after it.
Terminal sterilization is the process of sterilizing a product in its final container. It
is an important process as it ensures the product remains sterile. All medical,
ophthalmic and parenteral equipment are sterilized in batches, and
usually sterilized using heat.
65. 32-year-old pregnant woman with clotting, use? LMWH (eg. Dalteparin)

66. Vegetarians lack which vitamin? Vit. B12 it is from animal source only

67. Collagen made from?  Vitamin C

68. What is not part of statistical tests?  Clinical significance

69. Which is a hormone?  D3

70. Digoxin with metoclopramide interaction.


Metoclopramide may decrease the concentrations of digoxin in blood.
Metoclopramide reduce the time to reach peak concentration for digoxin.
The effect of metoclopramide on digoxin absorption is minimized by administration of
digoxin in capsules.

71. what increases warfarin INR? Its the clarythromycin and erythromycin

72. How many ½ lives to reach Css?  5 half lives (4 – 5 half lives)

73. Sole proprietorship disadvantage? ‫ﻣﻠﻛﻳﺔ ﻓﺭﺩﻳﺔ‬


A sole proprietor will be responsible for all the costs and debts of their company.
*Unlimited Personal Liablility
*Limited Ability to Raise Capital
*Limited Expertise and Growth Potential
*Limited Life Expectancy

74. a woman on progesterone oral contraceptive, she is suffering from spotted bleeding
after the day eight of menstruation cycle. What should you suggest the doctor to do:
a- Increase the oral progesterone dose
b‐ Give her a combined therapy with higher dose of estrogen
c- Give her a back up method
d- Tell her that the bleeding will stop after a period of time
e- Stop the medication and make reassessment
progestin‐only pills (POP) contain only synthetic progestogens (progestins) and do
not contain estrogen. Lacking the estrogen of combined pills, they are not associated
with increased risks of DVT or heart disease.
The progestin-only pill is recommended over regular birth control pills for women who
are breastfeeding because the mini-pill does not affect milk production (estrogen
reduces the amount of breast milk).
With no break in the dosage, menstrual flow does not initially occur at a predictable
time. Most women tend to establish, over a few months, light spotting at approximately
regular intervals.
Side effects:
 May cause mastalgia (breast tenderness) and mood swings.
 Weight gain is less commonly experienced than on COCP.
 Some women may experience abdominal cramps and heavy bleeding.

75. In pregnancy which hormone continues to increase? (e.g hormone- first sign of
pregnancy): The hormone human chorionic gonadotropin (better known as hCG)
is produced during pregnancy. In about 85% of normal pregnancies, the hCG level will
double every 48 – 72 hours.

76. What cause cardiomyopathy? Doxorubicin Use dexrazoxane as cardio protective

77. Responsible for Calcium homeostasis? PTH parathyroid hormones

78. patient has Renal failure, with hypertension, DOC  loop diuretic (furosemide)

79. Made a dispensing error, what to do?


mistake fix it by  tell patient / tell health care provider

80. AIn treating acute exacerbation of COPD, all can be used EXCEPT
a) Levofloxacin
b) Ipratropium
c) Montelukast
d) Salbutamol
e) Prednisone

81. All are used in treatment of acute gout Except:


f) allopurinol
g) Prednisone
h) Celecoxib
i) Colchicines
j) Indomethacin
82. PIPEDA (PERSONAL RIGHTS): 2010
Personal Information Protection and Electronic Documents Act (PIPEDA). This is
a law that was enacted by the federal government to regulate the collection, use,
disclosure, retention, access and security of personal information in the course of
commercial activity. it is provincial act.

83. Insurance pays some, and Patient pays the rest?  Co‐payment
 Coinsurance: It is one type of cost sharing plan in which patient pay a
specific percent of all loss occurred.
 Co‐payment: It is one type of cost sharing plan in which patient has to pay
fixed amount each time a service is provided.
 Deductible: It is one type of cost sharing plan in which patient has to pay a
specified amount during a specific period of time. Before benefits are paid
by third party.

84. Drug of choice for the treatment of meningitis in adult is:


e) Ceftriaxone
f) Cefixime
g) Cephalexin
h) Cefuroxime

85. NPH DOSE  2 times a day


(morning 2/3 of the daily dose, evening 1/3 of the daily dose)
NPH is intermediate acting insulin (isophane)

86. Which in Diabetes is true? Metformin NOT cause hypoglycemia

87. Dabigatran works as?  direct Antithrombin (anticoagulant)


It is an oral anticoagulant drug that acts as a direct thrombin (factor IIa)
inhibitor.

88. What protects B lactam ring and decrease hydrolysis: isoxazole or thiazole ring?
In clavulanic acid: isoxazole ring, In salbactam and tazobactam: thiazole ring
β‐lactamase inhibitor is a molecule used in conjunction with a β‐lactam antibiotic to extend its
spectrum of activity. They inhibit the activity of β‐lactamases, a family of enzymes that break the
beta‐lactam ring that allows penicillin‐like antibiotics to work, thereby conferring bacterial
resistance. Ex:Clavulanic acid, Sulbactam, Tazobactam and Avibactam.
89. Parkinson’s with BPH, don’t give  Benzotropine  causes urinary retention

90. Calcium (constipation)  Mg (diarrhea) antacids

91. osmotic pump is considered as:


d‐ Zero order
e- First order
f- Second order

92. Aminoglycosides after dose effect? Post antibiotic effect

93. Creatinine in seniors? Increase or decrease


Creatinine is the product of the metabolism of muscle so as the muscle mass
decreased with old age, the creatinine level decreased

94. CK Kinase high in? MI Myocardial infarction


 CK‐MB increase in case of MI
 CK‐MM increase in skeletal muscle disorders
 CK‐BB increase in bone disorders

95. Meperidine not used why?  affect on sphincter of oddi (The sphincter of
ampulla)
N.B. meperidine is an opoid analgesic also called “pethidine”

96. Dextrometharphan? Don’t give to children less than 6 years


IT is the dextrorotatory enantiomer of levomethorphan, which is the
methyl ether of levorphanol, both opioid analgesics. Dextromethorphan is
an antitussive (cough suppressant) drug.
It causes hallucinations at high doses. dextromethorphan can trigger
a histamine release.
Dextromethorphan should not be taken with (MAOIs) due to serotonin syndrome.
Caution should be exercised when taking dextromethorphan when
drinking grapefruit juice as this can lead to excessive accumulation and prolonged
effects.

97. Healthy human clinical trial? PHASE 1


98. Intention to treat? Intention to treat is a strategy for the analysis of randomised
controlled trials that compares patients in the groups to which they were
originally randomly assigned.

99. Which artery Supply Brain with most blood? Carotid artery

100. Not allowed to prescribe schedule F drugs?  Chiropractor ‫ﺍﻟﻤﻌﺎﻟﺞ ﻟﻼﻣﺮﺍﺽ ﻳﺪﻭﻳﺎ‬

101. Western Blot checks for  Protein monoclonal antibodies

102. What is used in Soft capsules to increase plasticity? Glycerin

103. Parallax error?


Parallax is a deceptive change of the position of an
object which is observed while the position of the
observer changes. Position of eye at all volumetric
vessels must be at the same level as the meniscus. If
not, the parallax will cause an error while reading the
position of the meniscus of a liquid in a burette. It
will be a positive mistake if the eye is lower, and
negative if the eye is higher than the meniscus plane.

104. What doesn’t affect diffusion? Atmospheric pressure


Factors that affect cell diffusion:
particle size, temperature, surface area, concentration gradient, permeability &
diffusion distance.
Diffusion according to Fick’s law is affected by:
Concentration difference, thickness of the barier, surface area of the barrier

105. Clearance = dose. F/AUC

106. Glycogenolysis? Glycogen  Glucose

107. Rods in eye? For night vision

108. Whistle noise with breathing? Wheezing

109. Adrenal medulla releases? Epinephire


110. Lung cancer
a‐influenced by environment
b-mortality in prostate cancer is higher than lung cancer
c-Japan is the least country with lung cancer
d- men are at high risk than female

111. Calcium gluconate injection (US Pharmacopeial standard) contains 0.1 g Ca


(C6H11O7) 2/mL of solution. If the NaCl equivalent of calcium gluconate is 0.16, the
injection is:
a. Hypotonic b. Iso-osmotic c. Isotonic d. Hypertonic e. Opisthot
JULY 2012

1) Patient on warfarin, INR found to be 7.5 yet not bleeding. Proper action is to:2008
a) stop warfarin
b) stop warfarin and give vitamin K intramuscular
c) stop warfarin and give protamine sulfate orally
d) stop warfarin and give vitamin K orally
e) stop warfarin and vitamin K intravenously
N.B. Why would you give vit K for the patient if he is not bleeding. The cutoff for giving Vit K is no
bleeding, condition is INR=9, you also can give oral vit K if INR 5-9 and the patient is BLEEDING.

2) What is true regarding mixed human insulin?


a) It generally requires scheduling meals and snacks
b) It has low risk of hypoglycaemia
c) It gives smoother peak
d) It is easy to adjust the dose as after exercise
e) It is given more frequently than short acting insulin

3) All the following drugs decrease the effects of oral contraceptives, EXCEPT:
a) rifampicin
b) carbamazepine
c) tetracycline
d) phenytoin
e) ibuprofen

4) Symptoms of atrial fibrillation include:


a) widening of QRS wave
b) atrial arrhythmia
c) prolongation of QT interval
d) absence of P wave
e) decreased action potential

5) Digoxin:
a) decreases intracellular calcium
b) increases conduction
c) increases contractility
d) decreases blood potassium
e) potentiates action of sodium
6) All are symptoms of chronic renal failure EXCEPT:
a) hyperphosphatemia
b) hyperkalemia
c) anemia
d) hypertension
e) Encephalopathy
High blood pressure can damage the kidneys and is one of the leading causes of kidney failure.

7) All are true regarding elderly EXCEPT:


a) reduced water volume
b) reduced blood urea nitrogen
c) reduced creatinine clearance
d) reduced lean body mass
e) reduced microsomal enzyme action
In elderly
(1) Decrease in lean body mass (muscle)
(2) Decrease in total body water (10-15%)
(3) Decrease in serum albumin (0.5-1 g/dl change)
(4) Increase in alpha-1 glycoprotein
(5) Increase in total body fat.
(6) Protein binding may be altered.

8) Patient comes in for a refill of her salbutamol puffer. Upon checking her history, you
discover this is her 3rd refill in the last 2 months. All are appropriate EXCEPT:
a) checking profile for history of inhaled corticosteroids
b) warn the patient about salbutamol overuse
c) review with the patient the method of use of the puffer
d) suggest to her prescribing physician to switch to salmeterol
e) ask the patient about her asthma triggers

9) Due to N-binding something... which antidepressant requires an unusually long period


of time before being able to switch to another?
a) fluvoxamine
b) fluoxetine – wash out period 5 weeeks
c) paroxetine
d) sertraline
e) citalopram
10) Which antidepressant can cause seizures at both therapeutic and more than
therapeutic doses?
a) bupropion
b) sertraline
c) topiramate
d) nortriptyline
e) imipramine

11) Gynecomastia, hyperkalemia, and menstrual cycle irregularities are all common side
effects of:
a) amiloride
b) triamterene
c) spironolactone
d) furosemide
e) indapamide

12) All the following are preservatives EXCEPT:


a) benzalkonium chloride
b) methylparaben
c) ethanol
d) benzyl alcohol
e) lauryl sulfate – anionic surfactant

13) A patient says she wants to reduce risk of osteoporosis without taking medication.
The following are helpful EXCEPT:
a) walking
b) swimming
c) smoking cessation
d) vitamin D supplementation
e) calcium supplementation

14) Methylation occurs by the aid of:


a) S‐adenosyl methionine
b) glutamyltransferase
c) monoamine oxidase
d) esterase
e) the transporter
15) Jaundice in newborns is caused from a deficiency of:
a) glucose
b) UDP glucose
c) glucuronate
d) UDP glucuronate
e) glucuronyl transferase

16) Oxygen toxicity can cause seizures in the brain which is linked directly to the
inhibition of:
a) monoamine oxidase
b) catechol-orthomethyl transferase
c) lactate dehydrogenase
d) glutathione reductase
e) glutamate oxidase
Superoxide dismutase, catalase, glutathione peroxidase, and glutathione reductase
provide the enzymatic defence against oxygen toxicity.

17) The part of the brain responsible for balance is:


a) cerebellum
b) occipital lobe
c) brain cortex
d) frontal lobe
e) medulla

18) TSH is secreted by:


a) hypothalamus
b) adrenal medulla
c) anterior pituitary gland
d) posterior pituitary gland
e) thyroid gland

19) All the following are secreted by the pancreas EXCEPT:


a) lipase
b) insulin
c) amylase
d) gastrin
e) glucagon
20) Sulfonamides exert their action by inhibiting:
a) DNA synthesis
b) cell wall synthesis
c) protein synthesis
d) folate synthesis
e) cholesterol synthesis

21) Early warning signs of ischemic stroke include:


a) hearing loss
b) shoulder pain
c) chest pain
d) knee pain
e) confusion but if there is headch we must choose it first

22) Which of the following can NOT be crushed for an IV infusion?


a) omeprazole tablets
b) digoxin tablets
c) enteric coated aspirin tablets
d) furosemide tablets
e) clopidogrel tablets

23) Nystatin is selective due to its affinity to:


a) cholesterol
b) ergosterol
c) phosphatidyl serine
d) lecithin
e) mycoplasma
nystatin, amphotericin B, natamycin

24) Olanzapine differs from typical antipsychotics such as haloperidol in:


a) more extrapyramidal side effects
b) more sexual dysfunction side effects
c) more weight gain side effects
d) more renal function side effects
e) more visual disturbance side effects

25) A Type II diabetic patient is taking hydrochlorothiazide for hypertension, yet still his
blood pressure reads 160/100. An appropriate medication to be added is:
a) ramipril
b) amlodipine
c) atenolol
d) diltiazem
e) hydralazine

26) Patient on oral contraceptives, Sunday


start schedule, missed 2 consecutive doses in
the second week. Advise her to:
a) take 1 immediately and continue
b) take 2 immediately and continue
c) take 2 immediately and 2 tomorrow and
continue
d) take 3 today and wait till Sunday and
continue
e) throw pack away, start again on Sunday

27) Patient has history of lactic acidosis. All are fine to administer EXCEPT:
a) glyburide
b) pioglitazone
c) sitagliptin
d) metformin
e) glimepiride

28) Herb contraindicated in uncontrolled hypertension:


a) ginger
b) echinaceae
c) liqurice
d) garlic
e) rosemary

29) Patient on pioglitazone. Required to monitor:


a) liver enzymes
b) serum creatinine
c) cardiac output
d) blood glucose
e) muscle tissue
30) Indinavir’s levels have been known to decrease upon initiation of:
a) acetaminophen
b) omeprazole
c) SSRIs
d) ginseng
e) St. John’s wort
PPIs  Monitor for decreased efficacy of drugs requiring an acidic medium for dissolution
or absorption, e.g., itraconazole, atazanavir, indinavir.

31) Side effects of lithium include:


a) tremors ‐ common
b) weight gain – and weight loss  also
c) oliguria
d) eryhtrocytosis
e) hypertrichosis

32) All the following cause photosensitivity EXCEPT:


a) sulfamethoxazole
b) trimethoprim
c) penicillin
d) tetracycline
e) minocycline

33) All are found in DNA EXCEPT:


a) pentose sugar
b) nucleotide base
c) deoxyribose sugar
d) ribose sugar
e) purines
Sugar present in an RNA molecule is ribose
Sugar present in DNA is deoxyribose

34) Differences between gram +ve and gram –ve bacteria include:
a) gram –ve bacteria has techoic acid
b) gram –ve bacteria has thicker peptidoglycan layer
c) gram –ve bacteria has endotoxins
d) gram –ve bacteria stains blue or purple
e) gram +ve bacteria has lipopolysaccharide membrane
Gram +ve Gram –ve
Stain blue or purple Stain red
Techoic acid lipopolysaccharide
Peptidoglycan layer is Peptidoglycan layer is thin
thick
Exotoxin endotoxin

35) Antiperspirant for hyperhidrosis includes:


a) Triclosan- antibacterial
b) aluminum chloride
c) sodium benzoate
d) zinc oxide
e) parabens
All antiperspirants have an aluminum-based compound as their main ingredient.

36) The following is contraindicated with celecoxib:


a) dyspepsia
b) sulfa allergy
c) prostatic hyperplasia
d) renal impairment
e) acetaminophen concurrent use

37) Which can prolong life in chronic heart failure?


a) enalapril
b) digoxin
c) metoprolol
d) aspirin
e) amlodipine

38) The following are helpful in the treatment of stable angina EXCEPT:
a) beta blocker
b) ACE inhibitor
c) nitrate
d) diuretic
e) antiplatelet

39) Which can reduce cardiac oxygen demand without causing vasodilation?2015
a) atenolol
b) felodipine
c) nitroglycerine
d) minoxidil
e) hydralazine

40) Hospital funded primarily by:


a) provincial and federal taxes
b) provincial taxes only
c) provincial taxes with some user pay
d) federal taxes with some user pay
e) provincial and federal taxes with some user pay

41) Patient has been abusing pain medication for several months, says he spoke to his
doctor about it and the doctor says it’s not a problem. You should:
a) believe him and fill the prescription
b) refuse to fill the prescription
c) tell him to fill the prescription somewhere else
d) talk to the doctor about your concern
e) suggest he takes OTC analgesics to reduce his pain

42) The ethical law whereby the pharmacist can refuse to fill a prescription for moral or
religious reasons is called:
a) personal clause
b) notwithstanding clause
c) conscience clause ‫ﺿﻤﻴﺮ‬
d) freedom of faith clause
e) mutual respect clause

43) In an ointment preparation containing water-removable bases, polyethylene glycol is


often added as a (n):
a) humectant
b) stabilizer
c) antioxidant
d) preservative
e) Surfactant

44) In prescription mixture, 0.1% polysorbate 80 was included. Its role is most likely a(n):
a) preservative
b) Surfactant
c) solubilizer
d) antioxidant
e) diluent

45) In a urinary tract infection caused by Pseudomonas aeruginosa, the most effective
agent would be:
a) nitrofurantoin
b) sulfamethoxazole
c) levofloxacin
d) amoxicillin/clavulinate
e) metronidazole
parentral aminoglycosides are the DOC although quinolones are often used

46) Initial dose of methotrexate is usually:


a) 7.5 mg weekly
b) 25 mg weekly
c) 50 mg weekly
d) 7.5 mg daily
e) 25 mg daily

47) Concerning the frequency of dosing of bisphosphonates, all are true EXCEPT:
a) daily
b) weekly Answer is C
c) biweekly
d) monthly
e) Yearly

48) Which enzyme is responsible for the metabolism of sildenafil?


a) CYP1A2
b) CYP2C9
c) CYP2D6
d) CYP3A4
e) CYP3A5

49) phenytoin can reduce levels of which of the following:


a) thiamine
b) niacin
c) cyanocobalamin
d) vitamin D
e) folic acid

50) Asymptomatic bacteraemia requires treatment in which of the following groups of


patients?
a) Pregnant women
b) diabetic patients
c) over 65 years old
d) patients with turbid urine
e) children

51) Patients taking infliximab require monitoring for:


a) renal function
b) leukocyte count
c) infections
d) skin rash
e) nystagmus
infliximab is an immunosuppressant which causes leukopenia and neutropenia

52) Periodic liver enzyme tests are done serially to:


a) test for liver disease only
b) diagnose liver disease
c) differentiate between acute and chronic liver disease
d) determine efficacy of treatment of liver disease
e) monitor the progression of liver disease

53) Demyelinization occurs in:


a) rheumatoid arthritis
b) polycystitis
c) multiple myeloma
d) multiple sclerosis
e) osteoporosis

54) Auxiliary label “plenty of water” is appropriate for:


I co-trimoxazole II allopurinol III losartan
a) I only
b) III only
c) I and II only
d) II and III only
e) I, II, and III

55) In clinical trials, an undesired effect occurred 2% of the time in patients taking the
drug, and 4% in patients taking the placebo. In the brochure, they mention a 2%
reduction. This is:
a) absolute risk
b) relative risk
c) risk rate
d) Absolute risk reduction
e) relative risk reduction

56) Papain is often added to contact lens solution. Its role is:
a) surfactant
b) Hydration of soft lens
c) Antimicrobial
d) Remove fatty material
e) Remove proteinous material

57) Mechanism of surface-active agents:


a) increase interfacial tension
b) Reduce interfacial tension
c) disperse nonpolar phase
d) increase miscibility
e) form foamy layer on surface

58) All is true regarding polymorphism EXCEPT:


a) polymorphics are generally less soluble than amorphous
b) Chemical composition changes
c) crystalline structure changes
d) physical properties changes
e) the least stable form is the first to crystallize

59) Which is false concerning the “hydrate” of a drug?


a) Same core composition as the anhydrous
b) Similar solubility as the anhydrous
c) May become anhydrous in dry atmosphere
d) Hygroscopic materials tend to be hydrates
e) Trap water molecules into its structure
Hydrate indicates that a substance contains water.

60) Something about the laws governing pharmacy technicians, all EXCEPT:
a) Pharmacist is liable for their actions
b) Work is delegated by pharmacist
c) Must seek pharmacist if questioned about scheduled drugs
d) Licensed by the Ontario College of Pharmacists
e) Solely responsible for what they do

61) All the following are indicated for the treatment of acute gout EXCEPT:
a) ibuprofen
b) indomethacin
c) prednisolone
d) colchicine
e) allopurinol

62) Which has the most antioxidant action?


a) tocopherol alpha
b) tocopherol beta
c) tocopherol delta
d) tocopherol epsilon
e) tocopherol gamma

63) The following is an example of:


tetracyclineepi-tetracycline
a) isomers
b) racemization
c) enantiomers
d) conformers
e) optical isomers

64) Patient comes in with a prescription for a steroidal anti-inflammatory


(corticosteroids). It is appropriate to check her history for all EXCEPT:
a) osteoporosis
b) breast disease
c) diabetes
d) infections
e) Cushing’s

65) All are appropriate when counselling a patient on clarithromycin EXCEPT:


a) interaction with statins
b) what to do if GI upset occurs
c) possible photosensitivity
d) how long before symptom improvement expected
e) how to take

66) It is appropriate to tell a patient to avoid sexual activity or encourage their partner to
use protection in all EXCEPT:
a) gonorrhea
b) syphilis
c) AIDS
d) trichomonas
e) vaginal candidiasis

67) Conjunctivitis can be caused by all EXCEPT:


a) gonorrhea
b) chlamydia
c) S aureus
d) B fragilis
e) S pneumonia

68) Initial symptom of marijuana use:


a) mydriasis
b) dysphoria
c) polyphagia
d) conjunctivas reddening
e) fatigue

69) Morphine overdose can cause all EXCEPT:


a) constipation
b) respiratory depression
It causes pinpoint pupil
c) pupillary dilatation
d) encephalopathy
e) hypotension
70) Symptoms of vaginal candidiasis include all EXCEPT:
a) fishy odour
b) local itching
c) local burning
d) local inflammation
e) cheesy discharge

71) Increases risk of erectile dysfunction:


I atherosclerosis II diabetes III hypertension
a) I only b) III only c) I and II only
d) II and III only e) I, II and III

72) Symptoms of diabetic foot ulcer, all EXCEPT:


a) erythema
b) swelling
c) neuropathic pain
d) opening of skin
e) purulent discharge

73) What is the second most abundant cell


after erythrocytes in the blood? 2009
a) monocytes
b) basophils
c) neutrophils
d) platelets
e) lymphocytes
RBC's would be most abundant, followed by
platlets, followed by WBC.

74) What is a codon?


a) section of ribosome responsible for initiating protein synthesis
b) sequence of nucleotides corresponding to amino acids
c) terminal nucleotide where RNA polymerase ends transcription
d) responsible for phenotypic expression of genetic code
e) int RNA for collecting amino acids in translation
A codon is a sequence of three DNA or RNA nucleotides that corresponds with a specific
amino acid or stop signal during protein synthesis
75) All are appropriate antibiotics for the treatment of acne EXCEPT:
a) tetracycline
b) ciprofloxacin
c) clindamycin
d) doxycycline
e) erythromycin

76) The effect of genetics on drugs is called:


a) pharmacogenomics
b) interpatient variability
c) gene expression
d) genetic engineering
e) hereditary influence

77) Which is LEAST likely to cause chronic liver disease?


a) Hepatitis A
b) Hepatitis B
c) Hepatitis C
d) Hepatitis D
e) Epstein-Barr

78) The Kaplan-Meier equation is applied for new therapies (such as dialysis or
antineoplastics) to measure patients’:
a) preferences
b) adverse reaction rate
c) mortality rate
d) morbidity rate
e) survival rate

79) Drug has shown that it contains the same potency even at 90% the original
concentration. (There was another fact about the drug). What final detail do you need to
know to determine the expiry date of the drug?
a) the drug’s concentration after one year on the shelf
b) t 90
c) k or t1/2

80) Which vitamins are heavy smokers known to be physiologically deficient?


a) Vitamin A
b) Vitamin B12
c) Vitamin C
d) Vitamin D
e) Vitamin E

81) Where are clotting factors synthesized?


a) bone
b) kidney
c) blood
d) liver
e) pancreas

82) Two drugs with equivalent therapeutic effects are best compared by:
a) cost-benefit analysis
b) cost-effectiveness analysis
c) cost‐minimization analysis
d) cost-utility analysis
e) willingness to pay

83) When both the drug and the effect are measured in dollars:
a) cost‐benefit analysis
b) cost-effectiveness analysis
c) cost-minimization analysis
d) cost-utility analysis
e) willingness to pay

84) Which is/are indicator(s) of worsening asthma?


I salbutamol puffer required before exercise
II daytime symptoms once or twice a week
III night time symptoms causing wakening once or twice a week
a) I only b) III only c) I and II only d) II and III only e) I, II and III

85) Patient asks for a non-drowsy antihistamine; it is ok to recommend:


a) chlorpheniramine
b) dimenhydrinate
c) diphenhydramine
d) loratidine
e) hydroxyzine
86) IgE has been linked to:
a) asthma
b) emphysema
c) chronic bronchitis
d) pneumonitis
e) pharyngitis

87) Which can be classified as a serotonin norephinephrine reuptake inhibitor?


a) mirtazepine
b) imipramine
c) fluoxetine
d) bupropion
e) venlafaxine

88) Some question about confidence interval. What is true about confidence interval?
a) its width increases as sample size increases
b) its width decreases as sample size increases
c) its width increases as sample size decreases
d) its width decreases as sample size decreases
e) its width is independent of sample size

89) Patient asks you about sunscreen SPF 15. SPF 15 means:
a) it takes 15 times longer to burn
b) it protects for 15 hours
c) must be reapplied every 15 minutes
d) it contains 15% of the active ingredient
e) helps reduce burning by 15%

90) Heartburn experienced after heavy meals and lying down is most consistent with
which diagnosis?
a) Zollinger-Ellison syndrome
b) Gastroenteritis
c) gastroesophageal reflux disease
d) Peptic ulcer
e) Acid hypersecretion

91) All are appropriate to counsel patients with nausea/vomiting EXCEPT:


a) eat small meals
b) avoid fluid intake
c) avoid spicy foods
d) loose clothes
e) avoid hot meals

92) Which organism can increase the risk of shingles?


a) varicella zoster
b) Herpes simplex 1
c) Herpes simplex 2
same as chicken pox

93) Modified sweat gland in the ear:


a) sebaceous gland
b) sudoriferous gland
c) montgomery gland
d) areolar gland
e) ceruminous gland

94) Vancomycin can be used as alternative treatment for:


a) C. Difficile
b) MRSA 1st choice in MRSA - 2nd in C. difficile
c) N. Meningitis
d) S. Aureus
e) P. Aeruginosa

95) Something about fentanyl... structurally related to or derivative of...


a) hydromorphone
b) codeine
c) morphine
d) sufentanil
e) pentazocin
Sufentanil (R30730, brand name Sufenta) is a powerful synthetic opioid analgesic drug,
approximately 5 to 10 times more potent than its parent drug, fentanyl.
Both of them are used in spinal analgesia, but sufentanil is more lipid soluble

96) 100% bioavailability can be attained


I subcutaneously II intramuscularly III intravenously
a) I only b) III only c) I and II only d) I and II only e) I, II and III
97) Drug shows concentration 16mg/L with a dose of 75mg, but 54mg/L when a dose of
150mg is given. This may be due to:
a) saturation of hepatic enzymes
b) saturation of renal clearance
c) saturation of plasma protein binding
d) saturation of intestinal absorption
e) saturation of drug dissolution

98) Chromatography for heat labile substances, all EXCEPT:


a) Thin Layer Chromatography
b) High Performance Liquid Chromatography
c) Reverse Phase Chromatography
d) Gas Chromatography
e) Paper Chromatography

99) AUC profiles for a drug given in three separate routes: (ip – intraperitoneal)
AUC iv = 128, AUC po = 28, AUC ip = 28
Which is/are true?
I drug has significant hepatic first pass
II drug properties play a role in absorption
III drug has significant gut first pass
a) I only b) III only c) I and II d) II and III e) all

100) Gel separation works by separating according to:


a) pKa
b) polarity
c) molecular weight
d) osmotic pressure
e) electrostatic charges
If only 1 answer is to be chosen, then choose: molecular weight (size)

101) Non-terminal sterilization process:


a) dry heat sterilization
b) moist heat sterilization
c) filtration
d) radiation
e) gas
Terminal sterilization is the process of sterilizing a product in its final container. It is an
important process as it ensures the product remains sterile. All medical, ophthalmic and
parenteral equipment are sterilized in batches, and usually sterilized using heat.
Can be done using:
 Sterilization by moist heat (heating in an autoclave)  suitable only for water-
wettable materials and aqueous formulations.
 Sterilization by dry heat  suitable for non-aqueous liquids or dry powder
products.
 Sterilization by radiation is used mainly for the sterilization of heat-sensitive
materials and products.
 Sterilization by gases and fumigants: should only be used for products where there
is no suitable alternative.

102) Process which works by diffusing substances across a semi-permeable membrane is


termed: Movement of solute is diffusion Diffusion is a spontaneous movement of particles from an area of high
Movement of solvent is osmosis concentration to an area of low concentration. (ex. tea flavoring moving from
a) dialysis an area of high to low concentration in hot water.)
b) osmosis He should specify.. Osmosis is the spontaneous net movement of water across a
c) concentration gradient semipermeable membrane from a region of low solute concentration to a
more concentrated solution, up a concentration gradient. This equalizes
d) dispersion concentrations on both sides of the membrane.
e) diffusion  diffusion is the opposite of osmosis

103) Which is not used to determine physical properties of a structure:


a) melting point
b) pKa
c) partition coefficient
d) molecular weight
e) size

104) Something like “which of the following would NOT suppress dissolution of a drug”...
Factors affecting dissolution of a drug:
Drug solubility (partition coefficient)
Salt formation (salts dissolve faster
Particle size
State of hydration, polymorphic structure, crystalline state
Purity
Additives (SAA increase dissolution while lubricants decrease dissolution rate)
Granulation generally enhances dissolution
Compression force increases dissolution rate
PH, Temperature
Agitation
Amount of solvent used for dissolution
Pressure (crushing of tablet)

105) Something about liquid-liquid extraction:


Liquid–liquid extraction also known as solvent extraction and partitioning, is a
method to separate compounds based on their relative solubilities in two
different immiscible liquids, usually water and an organic solvent. It is an extraction
of a substance from one liquid into another liquid phase.

106) All cannot be taken with iron EXCEPT:


a) ciprofloxacin
b) risedronate
c) tetracycline
d)lithium
e) levothyroxine

107) Expected effect of levothyroxine overdose:


a) bradycardia
b) weight loss
c) hypotension
d) bradyarrhythmia
e) anaphylaxis

108) Which is not involved in making mixtures?


a) levigation
b) trituration
c) Pulverization
d) Grinding
e) Agitation

109) regarding high blood pressure, what is a true statement?


a) BP reading of 160/100 at home requires referral to ER
b) BP reading of 160/100 in the pharmacy requires referral to ER
c) 160/100 is sufficient for a diagnosis of hypertension
d) Cuffs should be placed at heart level when measuring
e) BP reading at home may be inaccurate due to untrained use
110) Which has long been known to increase risk of atherosclerosis?
a) tryptophan
b) phenylalanine
c) glutamine
d) homocysteine
e) lysine

111) Define profitability... Ability to cover cost with a little extra for the owner

112) Rx for weekly compliance (blister) packaging


Furosemide 40mg 2-tab qam
Enalapril 10mg 1 tab with lunch
Lipitor 20mg 1-tab hs
Glyburide 2 tabs bid
Which should immediately trigger the attention of the pharmacist?
a) 1 lipitor in bedtime slot
b) 2 glyburide tablets with breakfast, 2 with supper
c) only 1 tablet in lunch slot
d) 4 tablets in morning slot
e) 1 furosemide with breakfast, 1 with supper

113) Which is considered a primary source of information?


a) PubMed
b) CPS
c) Clinical study
d) Patient Self Care
e) Science Journal
primary source: Scientific journals
secondary source: indexes and abstracts, pubmed, cochrane
tertiary source: text books

114) Total Sales: $1.5 million, Cost of Goods Sold: $1.41 million, Profit: $90,000
What is the profit margin?
a) 6.0% = 900000/1500000 = 0.06 * 100 = 6%
b) 6.4%
c) 8.0%
d) 21%
e) 90%
115) Gluten can be found in:
a) corn
b) rice
c) soya
d)wheat
e) potato
Gluten is found in wheat, rye, barley

116) Which actually reduces the size of the prostate gland?


a) tamsulosin
b) mifepristone
c) danazol
d) dutasteride
e) cyproterone acetate

117) Rx Ibuprofen 200mg/cap (density of ibuprofen = 1.1 g/mL)


Lactose qs 30 caps (density of lactose = 0.8 g/mL)
What quantities do you use and which size capsule is most suitable?
a) 5.45 g ibuprofen, 0.7g lactose, 0.21mL capsule
b) 6g ibuprofen, 0.7g lactose, 0.21 mL capsule
c) 5.45 g ibuprofen, 1.2g lactose, 0.45 mL capsule
d) 6g ibuprofen, 1.2g lactose, 0.45 mL capsule
e) 5.45 g ibuprofen, 0.3g lactose, 0.30 mL capsule

118) Half-life of a drug is 2 hours. Concentration in plasma after 1 hour is 64 mg/L. What
would the concentration be after 5 hours? (16mg/L)
1hr 64 3216

119) Cp = 28e ^ (-4t) + 12e ^ (-3t). What was the initial concentration? 40
Initial concentration occurs at t=0. e ^ 0 = 1. So, then it’s Cp = 28(1) + 12(1) = 40

120) Want 60 ml of a 1:10000 mixture. Stock solution comes in 1:750. How many ml of
stock solution are required? 4.5 ml
60 x 1/10000 =? mL x 1/750 60 x 750 /10000 = 4.5 mL

121) Rx 10mg/kg on day 1, then 5mg/kg on days 2 to 5. Patient weighs 30kg. Suspension
comes in 200mg / 5mL. How many ml of the suspension needed? 22.5 ml
10mg/kg x 30kg = 300 mg. 5mg/kg x 30kg = 150 mg... x 4 days = 600mg. Total 300 + 600 =
900 mg. 200mg / 5mL = 900mg / ?mL ?mL = 900mg x 5mL = 22.5mL 200mg

122) Half life of a drug is 16 hours. How long will it take to reach steady state
concentration? 69.12

123) Structure of beta-carotene. Beta-carotene gives Vitamin A1 by:


a) ring hydroxylation
b) deamination
c) oxidative cleavage
d) dehydrogenation
e) saturation

124) Structure of Nifedipine: Metabolism occurs by:


a) demethylation
b) N-hydroxylation
c) nitrosamine reduction
d) dehydrogenation
e) decarboxylation

125) Metformin (part of structure given) is a:


a) biguanide
b) sulfonamide
c) thiazole
d) lactone
e) steroid

126)This is a:
a) thiothixene b) phenothiazine
c) imidazoline d) thiazide e) sulfonamide

127)This treats:
a) hyperuricemia
b) hyperglycemia
c) hyperkalemia
d) hyperlipidemia
e) hypercalcemia
Niacin for ttt of hyperlipidemia
DAY 2
1) Calcitonin is secreted from:
a) hypothalamus
b) anterior pituitary gland
c) posterior pituitary gland
d) thyroid gland
e) kidneys

2)This is:
a) sulfacetamide
b) acetazolamide
c) furosemide
d) hydrochlorothiazide
e) spironolactone

3)This is:
a) saturated fatty acid
b) trans fatty acid
c) organic fatty acid
d) monounsaturated fatty acid
e) Polyunsaturated fatty acid

4) Which is not used in benign prostatic hyperplasia?


a) alfuzosin
b) ibuprofen
c) finasteride
d) doxazosin
e) dutasteride

5) For which is a “shake well” auxiliary label appropriate?


a) Spiriva Handihaler (TiotropiumBromide)
b) Ventolin Diskus (Salbutamol)
c) Flovent HFA (Fluticasone)
d) Aerochamber
e) Singulair (Montelukast)

6) Which can be used in the treatment of Traveller’s Diarrhea?


a) Dukoral vaccine
b) Clarithromycin
c) Ciprofloxacin
d) Docusate Sodium
e) Senna

7) Which is used for pneumonia caused by H. Influenza?


a) amoxi/ clav
b) cephalexin
c) clarithromycin
d) levofloxacin
e) azithromycin

8) Which antibiotic works by inhibiting cell wall synthesis?


a) aminoglycosides
b) cephalosporins
c) macrolides
d) lincosamines
e) tetracyclines

9) All have the potential for abuse EXCEPT:


a) Tylenol No. 1
b) pseudoephedrine
c) dimenhydrinate
d) caffeine
e) guaifenesin

10) Which is NOT permitted in late pregnancy?


a) chlorpheniramine
b) acetaminophen
c) ASA
d) docusate sodium
e) Ranitidine
Taking aspirin during pregnancy — especially after 32 weeks — can contribute to maternal and fetal
bleeding. Aspirin and ibuprofen can also cause the premature closure of a vessel in a baby's heart, which
can lead to high blood pressure in the baby's lungs (pulmonary hypertension), and prolong labour.

11) Which is not helpful in prostate carcinoma?


a) goserelin (used in prostate cancer)
b) leuprolide
c) docetaxel
d) posaconazole
e) bevacizumab (used in colorectal & Brest & Ovaries cancer)
posaconazole is an antifungal

12) Who recommends to insurance companies and drug benefit formulary which
medications should be covered?
a) Canadian Medical Association
b) Canadian Drug and Medication Database
c) Canadian Society for Pharmaceutical Sciences
d) Canadian Pharmacists Journal

13) The second most costly expense for provincial plans is pharmaceuticals, what is the
most expensive?
a) physicians b) hospitals c) specialists d) nurses e) extended care

14) Which is beneficial in alcohol withdrawal?


a) retinoic acid
b) thiamine
c) pyridoxine
d) cholecalciferol
e) topopherol

15) Ondasetron works by centrally inhibiting which receptors?


a) serotonin
b) alpha II
c) dopamine
d) nicotinic
e) muscarinic

16) Which is not part of the immune system?


a) T lymphocyte
b) macrophage
c) plasma cell
d) basophil
e) fibroblast
17) Which can be classified as a polynucleotide catalyst?
a) mRNA
b) ribozyme
c) tRNA
d) enzyme
e) DNA polymerase

18) Mutagenesis, all EXCEPT:


a) genetic information is changed
b) always results in a mutation
c) physical deformity will appear
d) may occur spontaneously
e) may be triggered

19) What can be done to increase the contact time of ophthalmic preparations?
I increase viscosity
II add mucoadhesive polymer
III increase the number of drops instilled
a) I only b) III only c) I and II only d) II and III only e) all

20) AUC increases when drug is taken with Coca Cola. This means:
a) Coca Cola enhances drug absorption
b) Coca Cola inihibits drug absorption
c) Coca Cola enhances drug metabolism
d) Coca Cola inhibits drug metabolism
e) Coca Cola increases gastric acidity

21) In complicated partial seizures, the goal of therapy is to:


a) prolong maximum firing interval
b) shorten maximum firing interval
c) increase maximum firing capacity
d) reduce maximum firing capacity
e) inhibit brain activity

22) Structure is cyclohexane fused with:


a) amino acid
b) gamma amino butyric acid
c) alpha keto pentamine
d) glutamate
e) glycine

23) Side effect occured 2.5% of the time in control group, but only 2% of the time in the
experiment group. The relative risk reduction is:
a) 0.10% b) 0.20% c) 5% d) 10% e) 20%
RRR = [EER-CER]/EER = 2.5 – 2 / 2.5 x 100 = 20%

24) At t = 2 hours, Cp = 4.02 mg/mL. At = 4 hours, Cp = 2.01 mg/mL. What was the initial
concentration?
a) 8.04 b) 16.02 c) 25.1 d) 32.6 e) 1 x 10^99

25) An ingredient is available in 100mg/mL solution. How many mLs would you need to
make 150mL of a 5% solution?
a) 7.5 mL b) 15 mL c) 37.5 mL d) 75 mL e) 0.075 mL
5% means 5 grams in every 100 mL. So, in 150 mL, there would be 7.5 GRAMS or 7500
mg. There’s 100mg in every mL of the stock solution, so we need 75mL.

26) Rx asks for 150mg of a drug. The drug is available in 5mg/mL vials of 5 mL. How many
vials?
a) 0 b) 2 c) 4 d) 6 e) 8

27) Twin sisters going away for 6 weeks. Mefloquine 5mg/kg with a maximum dose of
250mg. They both weight 50kg. They need to take the drug once a week starting one week
before, continuing throughout the trip, and 4 weeks after. How many tablets (comes in
250mg tablets) are needed?
11 tablets for each, total = 22 tablets

28) Which is responsible for the metabolism of mercaptopuric acid?


a) CYP3A4
b) glucuronic acid
c) glutathione
d) NABQI
glutathione S tranferase is responsible for the metabolism of acetaminophen, glycine and
glutamate to mercapturic acid.

29) CH3O – R  HO – R
a) reduction
b) O‐demethylation
c) alcohol formation
d) hydrolysis
e) esterase

30) 18 yo patient with acne, tried antibiotics and topical agents, an appropriate alternative
may be:
a) clindamycin / benzoyl peroxide
b) tacrolimus
c) methotrexate
d) isotretinoin
e) hormonal therapy

31) The weight ratio of beta-carotene: vitamin A1 is:


a) 1:1 b) 1:2 c) 1:3 d) 1:4 e) 1:5

32) Epinephrine’s reactivity may be attributed to the:


a) alpha-carbon
b) N‐methylamine
c) beta hydroxyl group
d) ring aromaticity
e) 3,4 – dihydroxy substitution

33) Estrogen differs from other steroids in Ring A due to:


a) saturation
b) keto oxygen on C3
c) lipophilicity
d) esterification
e) aromaticitiy

34) Ozone toxicity is mainly the result of:


a) oxidation
b) reduction
c) free radicals
d) saponification
e) epoxide formation
35) All may cause weight gain EXCEPT:
a) insulin
b) mirtazepine
c) glimepiride
d) metformin
e) amitriptyline

36) Which is a true statement regarding the prevention of migraines?


a) acetaminophen can be taken prophylactically
b) Tyelnol No 1 can be used to relieve pain
c) sumatriptan can be taken 2 hours before acute attack
d) combination therapy helps reduce frequency of migraines
e) triggers should always be avoided

37) Nurse calls explaining that a fentanyl patch fell off one of her patients after only 24
hours. Rx says app 1 patch q72h with preset quantity and fill intervals. She asks for an
early refill to be able to continue treatment as intended. You should:
a) fill the prescription at your discretion
b) refuse to fill as your loyalty is to the law
c) request early refill authorization from the physician
d) refer the patient to the emergency room
e) tell the patient to come in and see the doctor

38) Pt experiences heart burn and acid reflux after wings and beer. Best choice:
a) antacid before meal
b) famotidine before meal
c) magnesium hydroxide antacid
d) ranitidine after meal
e) antacid after meal

39) You, pharmacy manager, are concerned your pharmacy isn’t ethnically diverse. You
may do all except:
a) get paintings or art from other cultures to promote multiculturalism
b) encourage all staff to attend seminars on multiculturalism
c) hire a diverse team to welcome customers from all ethnicities
d) learn chinese
e) another acceptable thing
40) Pt comes in telling you about a new drug in Canada. Which source do you check?
a) Compendium of Pharmaceuticals and Specialties
b) Therapeutic Choices
c) Patient Self-Care
d) health canada drug product database
e) drug bank

41) Which vitamin is found only in animal-derived foods?


a) A b) B1 c) B12 d) D e) K

42) Which is used to differentiate 1ry and 2ry


hypothyroidism?
a) thryotropin = TSH
b) cortisol
c) igG
d) Lugol’s solution
e) I131

43) Which has shown to be effective topically in the treatment of alopecia?


a) hydralazine
b) minoxidil
c) apresoline
d) diazoxide
e) nitroprusside

44) All are helpful following myocardial infarction EXCEPT:


a) clopidogrel
b) ASA
c) warfarin
d) digoxin
e) metoprolol

45) Complications of myocardial infarction include all EXCEPT:


a) anemia
b) arrhythmia
c) pericarditis
d) shock
e) heart failure
46) Digoxin with clarithromycin:
a) reduced absorption of digoxin
b) reduced absorption of clarithromycin
c) increased absorption of digoxin
d) increased absorption of clarithromycin
e) no interaction

47) Patient at risk of thrombosis. All are appropriate to watch for EXCEPT:
a) shortness of breath
b) chest pain
c) back pain
d) knee pain
e) swelling

48) 35-year-old woman who smokes often forgets to take her once daily oral
contraceptive pill Alesse (levonorgestrel / ethinyl estradiol). An appropriate alternative
would be:
a) Yasmin (drospirenone / ethinyl estradiol)
b) Tricyclen (norgestimate / ethinyl estradiol)
c) Cyestra (ciproterone acetate)
d) Depo‐Provera (medroxyprogesterone)
e) Micronor (norethisterone)

49) All are involved in the knee jerk reflex EXCEPT:


a) sensory nerves
b) motor neurons
c) quadriceps
d) patella
e) calcaneus bone or heel bone is a bone of the tarsus of the foot.

50) Side effect of donepezil involves which part of the body?


a) eyes
b) GI
c) skin

51) Most abundant protein in the human body is:


a) collagen  collagen followed by keratin, then myocin and actin
b) globulin
c) insulin
d) albumin  most abundant protein in human blood.
e) heme

52) Life span of platelets:


a) 1-2 days b) 7‐10days c) 14-28 days d) 4-6 weeks e) 3 months
LIFE SPAN OF RBC IS 120 DAYS (4 MONTHS).

53) Which age group shows the highest clearance for theophylline?
a) 1-2 weeks
b) 4-6 weeks
c) 2‐6 years
d) 18-25 years
e) 40-65 years

54) Of which home care devices must the pharmacy be knowledgeable, all EXCEPT:
a) blood pressure monitor
b) glucose monitor
c) oxygen therapy
d) nebulizer
e) diskus

55) Patient on insulin can expect all EXCEPT:


a) hypertension
b) weight gain
c) Hypoglycemia
d) pain at site of injection
e) treatment of diabetes

56) Patient wants to transfer a prescription for methylphenidate:


a) transfer it
b) refuse to transfer it
c) request from physician permission to transfer it
d) shred the prescription and call the cops
e) tell him he’ll need to get a new prescription

57) Which is considered a distinctive symptom of ascites?


a) hunched back
b) extended forearms
c) moon face
d) distended abdomen
e) buffalo hump

58) Which is used for chronic hepatitis?


a) isotretinoin
b) methotrexate
c) interferon
d) omalizumab
e) ibacitabine

59) Which helps menopausal women prevent osteoporosis but worsens hot flashes?
a) alendronate
b) tamoxifen
c) raloxifen  SERM selective estrogen receptor modulator
d) vigabatrin
e) weight bearing exercise

60) Which is false regarding gastric acid?


a) required for absorption of iron
b) required for absorption of calcium
c) incapable of eradicating H. pylori
d) effective in killing most infective agents
e) stimulated by ingestion of food

61) Why does most drug absorption occur in the small intestine?
a) weak acids require basic medium for absorption
b) small intestine has larger surface area so it absorbs more
c) residence time in the large intestine is too short
d) residence time in the stomach is too short
e) it depends on the physical properties of the drug
Theoretically, weakly acidic drugs (eg, aspirin) are more readily absorbed from an
acid medium (stomach) than are weakly basic drugs (eg, quinidine). However,
whether a drug is acidic or basic, most absorption occurs in the small intestine
because the surface area is larger and membranes are more permeable.
62) Carboxyhemoglobin is indicative of toxicity of which poison?
a) sulfur dioxide
b) carboxylic acid
c) mercury
d) carbon monoxide
e) haemoglobin

63) Respiratory acidosis is a result of:


a) increased partial pressure of CO2
b) reduced partial pressure of CO2
c) overuse of inhaled corticosteroid
d) increased ventilation
e) Reduced ventilation
Causes of respiratory acidosis include:
 Diseases of the airways (such as asthma and chronic obstructive lung disease)
 Diseases of the chest
 Diseases affecting the nerves and muscles that "signal" the lungs to inflate or
deflate
 Drugs that suppress breathing (including powerful pain medicines, such as
narcotics, and "downers," such as benzodiazepines), especially when
combined with alcohol
 Severe obesity, which restricts how much the lungs can expand

64) First pass means metabolism occurs before reaching:


a) systemic circulation
b) site of action
c) portal vein
d) liver
e) large intestine

65) Passage from the intestine to liver works by:


I hepatic vein II mesenteric vein III portal vein
a) I only b) III only c) I and II d) II and III e) I, II, and III
Enterohepatic recirculation This term refers to drugs emptied via bile into the small
intestine and then reabsorbed from the intestinal lumen into PORTAL VEIN to the
systemic circulation.
Oral drugs passage to liver GI tract  Mesenteric veins  portal veins  liver 
hepatic vein  inferior venacava  heart  systemic circulation  Renal or
hepatic elimination.
Hepatic veins are the blood vessels that drain de‐oxygenated blood from
the liver into the inferior vena cava.

66) Which can be given to prevent mouth ulcers from methotrexate use?
a) folate
b) pyridoxine
c) 5-amino salicylic acid
d) meloxicam
e) lidocaine

67) Switching ampicillin to amoxicillin is an example of:


a) generic substitution
b) therapeutic substitution
c) formulation substitution
d) class substitution
e) beneficience
Therapeutic substitution means substitution with a different medication from the same
class that is expected to have the same therapeutic effect.

68) Which vitamin is highly recommended for nursing babies?


a) A b) B c) C d) D e) E

69) Which is a blood parasite?


a) Amoeba
b) Filaria
c) Malaria
d) Loa loa -- eye
e) Ascaria – small intestine

70) Which does NOT occur in asthma?


a) airway inflammation
b) mucous secretion
c) cough
d) chest tightness
e) wheezing
71) Patient experiences cough, fever, night sweats for 4 weeks. Likely:
a) Influenza
b) common cold
c) pharyngitis
d) rabies
e) pneumonia

72) Something about plantar warts…2010


A plantar wart, verruca or myrmecia is a wart caused by the human
papillomavirus (HPV) occurring on the sole or toes of the foot.also most common
cause of Cervix epithelia cancer. GARDASIL is indicated in boys and men 9 through 26
years of age for the prevention of infection caused by HPV types 6, 11, 16, and 18 and the
following diseases associated with the HPV types included in the vaccine.
Over the counter salicylic acid
First-line therapy
Podophyllin (5%) is also used.
Cryosurgery,intralesional immunotherapy, or pulsed dye
Second-line therapy
laser therapy
Third-line therapy Bleomycin, surgical excision

73) Aspirin gets metabolized into salicylic acid by:


a) salicylase
b) esterase
c) amidase
d) acetyltransferase
e) carboxylase
Aspirin (acetylsalicylic acid) is metabolically converted to salicyclic acid by the action of
carboxylesterases.

74) Runny nose, itchy and watery eyes, and congestion are symptoms of:
a) pneumonia
b) common cold
c) allergic rhinitis
d) influenza
e) sinusitis

75) Urticaria is an allergic reaction which can be caused by all EXCEPT:


a) phenytoin
b) salicylates
c) sulfonamides
d) NSAIDs
e) hydrochlorothiazide

76) Genital chancre sores, alopecia, and some other symptom (rash maybe)
a) human papillomavirus
b) syphilis
c) gonorrhea
d) human immunodeficiency virus
e) chlamydia

77) Selective COX-II inhibitor, all EXCEPT:


a) lower risk of ulcer
b) lower risk of dyspepsia
c) lower risk of fluid retention
d) lower risk of decreasing bleeding time
e) something else true

78) Which is false regarding rhabdomyolysis?


a) can be caused by statins
b) may lead to urine discoloration
c) it causes breakdown of muscles
d) higher risk in anemic patients
e) higher risk in kidney impaired patients

79) Which used for penicillin resistant enterococci and methicillin resistant staphylococci?
a) vancomycin
b) ceftriaxone
c) norfloxacin
d) erythromycin
e) doxycycline

80) Creatine kinase requires the utilization of:


a) cAMP
b) phosphate
c) ADP
d) ATP
d) calcium
81) What is true about HIV?
a) test for tuberculosis every 3 months

82) Cardiomegaly is a side effect of: 2010


a) cisplatin c) methotrexate
b) doxorubicin d) azathioprine e) teniposide

83) Filgrastin (G-CSF) is administered after chemotherapy to prevent:


a) hemorrhage
b) recurrence
c) infection
d) emesis
e) alopecia

84) Hyperphosphatemia can be caused by:


a) liver failure
b) heart failure
c) respiratory failure
d) pancreatic failure
e) renal failure

85) Angioedema is a side effect of:


a) angiotensin converting enzyme inhibitors
b) beta adrenergic receptor antagonists
c) calcium channel blockers
d) diuretics
e) vasodilators

86) CYP450 monooxygenase contains:


a) myocin
b) myoglobin
c) heme
d) albumin
e) transferrin

87) Which is the cancer gene?


a) intron
b) oncogene
c) cytokine
d) pseudogene
e) allele

88) Dehydration can cause all EXCEPT:


a) thirst
b) dizziness
c) flushing
d) headache
e) hypertension

89) Which is used in Parkinson’s to reduce tremors?


a) entacapone
b) diazepam
c) bromopheniramine
d) propranolol
COMT inhibitors help prevent peripheral metabolism of levodopa, which increases its
availability to the brain, and have no effect if not used in conjunction with levodopa.

90) What may be added to liposomes to reduce permeability?


a) mannitol
b) cholesterol
c) glucose
d) sorbitol
e) ethanol
A liposome encapsulates a region of aqueous solution inside a hydrophobic membrane;
dissolved hydrophilic solutes cannot readily pass through the lipids. To deliver the molecules to
sites of action, the lipid bilayer can fuse with other bilayers such as the cell membrane, thus
delivering the liposome contents.

91) Clearance is with respect to:


a) Vd
b) dose
c) AUC
d) Cp
e) time
Cl = k. Vd. Directly proportional means similar in respect to changes while inversely proportional
means opposite with respect to changes. Clearance is directly proportional to Vd and inversely
proportional to time.
92) Sildenafil is contraindicated with:
a) Calcium
b) SSRIs
c) Nitrates
d) ARBs
e) Diuretics

93) Key symptom of nephrosis:


a) glucose in urine
b) protein in urine
c) ketones in urine
d) lipids in urine
e) minerals in urine

94) How do you test for long term drug adherence in diabetic patient?
a) blood glucose
b) pancreatic function
c) HbA1c levels
d) insulin levels
e) glucose tolerance

95) The gall bladder is stimulated by what in the diet?


a) bile
b) lipids
c) carbohydrates
d) cholesterol
e) glucose

96) Cardiac glycosides contain a (cardiogenic? cardio tropic?) substance plus:


a) sugar
b) amino acid
c) fatty acid
d) glycerol
e) hydrocarbon
A glycoside is a molecule in which a sugar is bound to another functional group

97) 5% salicylic acid in ointment base. Which is/are appropriate?


I dissolve salicylic acid in small amounts of water
II store in refrigerator
III final preparation will be paste-like
a) I only b) III only c) I and II d) II and III e) all

98) Class I anti-arrhythmics work on which channel?


a) calcium
b) potassium
c) chloride
d) sodium
e) HBO

99) Which is used as local anesthetic for hemorrhoids?


a) lanolin
b) witch hazel
c) zinc oxide
d) hydrocortisone
e) pramoxine
Witch hazel is used a strong anti-oxidant and astringent. It fights acne. It is often used as a
natural remedy for psoriasis, eczema, aftershave applications, ingrown nails, to prevent
sweating of the face, cracked or blistered skin, for treating insect bites, poison ivy, and as a
treatment for varicose veins and hemorrhoids.

100) Euthanasia is: ‫ﻗﺘﻞ ﺑﺪﺍﻓﻊ ﺍﻟﺸﻔﻘﻪ‬


a) Medications to enable patient‐assisted suicide
b) Administering morphine to relieve pain
c) Medications to enable abortion
d) Medications with euphoric effects
e) Administering opioids to relieve pain
Euthanasia refers to the practice of intentionally ending a life in order to relieve pain and
suffering.it is illegal in most countries.

101) All the following have interactions between the mobile and stationary phase
EXCEPT:
a) thin layer chromatography
b) paper chromatography
c) partition chromatography
d) gas chromatography
e) HPL chromatography
102) When a party negotiates terms with a union, this is called:
a) collective bargaining
b) petition
c) mediation
d) haggling

103) Advantage of incorporation as opposed to sole proprietorship:


a) less individual risk Less liability b) less individual profit

104) You can own a pharmacy in all ways EXCEPT:


a) sole proprietorship
b) partnership
c) leasehold ‫ﺍﻳﺠﺎﺭ‬
d) franchise
e) company

105) Drug procurement in hospital involves all EXCEPT:


a) drug selection
b) drug purchase
c) drug storage
Procurement of drugs is based on selected drugs and dosage forms and available financial
resources. Procedures adopted in procuring drugs include:
 Estimating quantity of each drug product required for a given period
 Finding out the prices of the different drug dosage forms required
Allocating funds for each drug dosage form depending on:
 Priority nature of the drug and dosage form
 Available finances.
Requisition for drug and dosage form is made after due consultation with prescribers.

106) All are true regarding unit dosing method for dispensing drugs in hospitals EXCEPT:
a) reduces time for nurses
b) reduces inventory requirement
c) reduces pharmacy staff requirement
d) reduces error possibility
unit dose: a method of preparing medications in which individual doses of patient
medications are prepared by the pharmacy and delivered in individual labeled
packets to the patient's unit to be administered by the nurses on an ordered
schedule. One intent of unit dose is to decrease administration error.
107) Franchises may provide pharmacies with deals regarding all EXCEPT:
a) wholesaler
b) software system provider
c) banner name
d) collective advertising
e) fixed prescription prices

108) Patient is suffering from chronic back pain and takes opioid analgesics. What is true
regarding opioid tolerance?
a) the dose will gradually increase with time
b) add gabapentin to lower opioid dose
c) it is beneficial to switch to another opioid at same dose
d) tolerance to one opioid means cross-tolerance with others

109) Cardiac output is the product of:


a) heart rate and systolic blood pressure
b) heart rate and contractility
c) heart rate and total peripheral resistance
d) heart rate and stroke volume
e) heart rate and width of artery

110) Rods are responsible for:


a) standing upright
b) color vision  cones
c) night vision
d) balance
e) rapid eye movement

111) Syncope is:


a) loss of balance
b) temporary loss of consciousness
c) breaking bone
d) adverse effects

112) IV bolus has the following pharmacokinetic model:


Cp = ae-4t + be-3t + ce-5.4t. How many compartments are involved in this model?
a) 1 b) 2 c) 3 d) 4 e) 5
IV Bolus Injection in a One Compartment Model
JANUARY 2012
1. Diazepam which is incorrect
a) Used for alcohol withdrawal
b) Cause falls in elderly
c) Forms metabolite which accumulate and cause long term sedation
d) Used long term for the treatment of insomnia
Diazepam undergoes oxidative metabolism by demethylation (CYP 2C9, 2C19, 2B6, 3A4,
and 3A5), hydroxylation (CYP 3A4 and 2C19) and glucuronidation in the liver as part of
the cytochrome P450 enzyme system. ... The main active metabolite of diazepam is
desmethyldiazepam (also known as nordazepam or nordiazepam) which has elimination
half life 36hr – 200hr.

2. Travellers diarrhea prophylaxis


f) Bismuth subsalicylate
g) Loperamide
h) Carbonate

3. Catenation?
The linkage of atoms of the same element
into longer chains.
Catenation occurs most readily in carbon,
which forms covalent bonds with other
carbon atoms to form longer chains and
structures. it is the reason for presence of
the vast number of organic compounds in
nature.

4. 4 p’s of marketing
a) place, product, price, promotion

5. Who should not use compression stockings?


f) Person with varicose veins
g) Elderly with compromised venous return
h) Person with coronary artery disease
i) Pregnant lady with pedal edema
j) Someone immobile in hospital
6. Chronic bronchitis pt had acute exacerbation with purulent discharge, which is
false?
e) Predisolone is contraindicated in the management of acute exacerbation
f) Bacteria may be the cause of exacerbation
g) Should use bronchodilators more often for acute excacerbations
h) Acute attach can last up to 48h or more

7. Someone using bimatoprost and stops, not good adherence to drug, which will
happen?
a) Conjuctivits
b) Dry eyes
c) Blindness
Used for treatment of glucoma, which if not controlled will lead to blindness. It is a
Prostaglandin F2-alpha analogue that lower IOP by increasing outflow of aqueous humor
through the uveoscleral pathway. Bimatoprost also ↑ trabecular outflow.

8. Prednisolone will affect which blood tests: Glucose/potassium

9. Which will influence a patient’s health (k type)


I. Access to health care
II. Genetics
III. Patient education

10. Metronidazole is used for all except


I. Gonorrhea
II. Candidiasis
III. Chlamydia
Used for trichomoniasis and bacterial vaginosis.

11. Which is not used in children


Ciprofloxacin  Cartilage toxicity, avoid in children and pregnancy.

12. Choose the incorrect statement about the p (probability) value?


A. The p (probability) value is the chance of a type I error.
B. If a p value is equal to/or lower than 0.05, it is unlikely that a type I error has
been made; that is, a type I error will be made 5 times or less out of 100.
C. A p value equal to/or less than 0.05 is generally considered to be statistically
significant; lower p values (e.g., p < 0.01) may be required for statistical significance
in studies with large sample sizes.
D. The p (probability) value is the chance of a type II error
the more the confedince interval (100 % - p value), the more the significance
Confidence interval (CIs) are inversely proportional to population value p-value, therefore
p-value 0.05 is expressed as a confidence interval 95%.
P = alpha means errors (results by chance) and confidence intervals are true values.

13. What does pharmaceutical care involve?


Getting information from patient to monitor the progress of the patient
Pharmaceutical Care is the responsible provision of drug therapy for the purpose of
achieving definite outcomes. The process of pharmaceutical care involves designing,
implementing and monitoring a therapeutic plan, based on the patient’s needs.
Specifically, this involves:
a) Identifying potential or actual drug-related problems.
b) Resolving actual drug-related problems.
c) Preventing potential drug-related problems

14. All used for urinary tract infection except


a) Cloxacillin
b) Nitrofurantoin
c) Trimethoprim
d) Sulphamethoxazole
e) Ciprofloxacin

15. if someone takes a drug with food and his AUC increases, this means that?
a) Vd is lower
b) Vd is higher
c) Higher clearance
d) First pass effect is decreased

16. Know the absolute risk and absolute risk reduction


absolute risk reduction, risk difference or absolute effect is the change in the risk of
an outcome of a given treatment or activity in relation to a comparison treatment or
activity. It is the inverse of the number needed to treat. In general, absolute risk
reduction is the difference between one treatment comparison group's event rate
(EER) and another comparison group’s event rate (CER)
RRR means (Big-small)/big *100
ARR means just big-small = EER-CER
NNT means 1/ARR*100
Absolute risk is drug/(drug+placebo)

17. Definition of Incidence: New case certain period


a measure of the probability of occurrence of a given medical condition in a
population within a specified period of time
Incidence proportion (also known as cumulative incidence) is the number of new
cases within a specified time period divided by the size of the population initially at
risk

18. Hospital formulary is used for?


A. To figure out ADR
B. To help physician with prescribing drugs based on cost effectiveness
C. For manufacturers

19. Schedule F drugs can be advertized if?


a) Can not be advertized
b) Can only be advertized under effect of Medscheck
c) if the ads contain only the name of the prescription drug.
Schedule F part 1:
Food and prescription drugs advertisement standards.
Have all prescription drugs that are present in NAPRA schedule 1
Canada permits only limited Direct-to-Consumer Advertising of prescription medications
(only the drug’s name). For example, TV ads for the erectile dysfunction drug Viagra don't
name its purpose.
Advertising in professional journals within the health care sector contains prescribing
information.
Drug advertisements are reviewed and pre-cleared by two independent agencies —
Advertising Standards Canada (ASC) and the Pharmaceutical Advertising Advisory Board
(PAAB) —to determine compliance with the Food and Drugs Act and the various
advertising codes.

20. A patient taking methylphenidate and changes pharmacy, in order to transfer her
prescription what should she do?
a) Get a NEW prescription
It is a controlled substance, Transfers not permitted.

21. Which ones need a Written Prescription?


a) Hydrocodone – straight narcotic only written prescriptions are accepted
b) Methamphetamine – controlled substance – verbal and written

22. Which is part of primary health care


a) Only concerned with physicians and nurses
b) Improvement of child health
c) Education for health improvement
N.B. 1ry care pharmacist, Doctor & nurse 2nd care is specialist, 3rd care is hospitalization

23. Which white blood cells are most affected by cancer chemotherapy?
f) Monocytes
g) Basophils
h) Neutrophils
i) Platelets
j) Macrophages

24. Which is not used for primary prevention for cancer?


f) Mamogram
g) Papsmear
h) Colonoscopy
i) APGAR test
j) PSA blood test
The Apgar score is a method to quickly summarize the health of newborn children
against infant mortality.
25. In a very active immunity, which is false
f) High mast cells
g) High levels of histamine
h) Low blood pressure
i) Low interleukins

26. Which causes most breakdown of platelets?


f) SLE
g) Diabetes
Immune thrombocytopenia: This type is caused by autoimmune diseases, such as lupus
and rheumatoid arthritis. The body's immune system mistakenly attacks and
destroys platelets. If the exact cause of this condition isn't known, it's called idiopathic
thrombocytopenic purpura. This type more often affects children.

27. Which is not an autoimmune disorder


f) Osteoarthritis
g) Rheumatoid
h) SLE

28. Non adherence to TCA cause all except


f) Weight loss
g) Sweating
h) Insomnia
i) Nervousness
j) Sexual dysfunction

29. Someone with a congested nose and has hyperthyroidism, which one to take for his
common cold?
e) Loratedine oral
f) Xylometazoline nasal
g) Phenylephrine nasal
h) Pseudoephedrine
Other options are sympathomimetic and they cause vasoconstriction. Desloratidine is
third generation anti histamine and have decongestant effect and it can be good option.
Because it doses not have higher sympathomimetic effects. Loratidine metabolises to
desloratadine. All sympathetic decongestant contraindicated due to their reflex
tachycardia effect in hyperthyroidism.
30. Which can be used for someone with venous thromboembolism?
f) Progestrin only oral contraception

31. Which is not a symptom of rhinitis


j) Runny nose
k) Hives
l) sneezing

32. Omeprazole when adminitstered, its actions are catalyzed by what?


a) Neutral Ph
b) Acidic ph
PPIs are prodrugs requiring proton pump activation for optimal efficacy; hence, they are
best administered 30 minutes before a meal.

33. Naltrexone given for treatment of alcohol withdrawal; it works by inhibiting


a) Serotonin
b) Dopamine
c) Norepinephrine
d) Gaba
e) Nicotine

34. Growth hormone secreted from: Anterior pituitary gland

35. Egg yolk is used as an


a. Emulsifier

36. Polysorbate 80 used as


a) hydrophilic nonionic surfactant

37. Which one shows no significant inhibition on CYP3A4?


j) Fluoxetine
k) Penicillins
l) HMGCoA reductase

38. Thalidomide is been proven to be effective in


f) Cancer treatment
g) Glaucoma
h) Multiple myeloma
39. Which is known to give Agranulocytosis: Clozapine

40. Which is not used for treatment of helicobacter pylori,2011


f) Amoxicillin
g) Clarithromycin
h) Metronidazole
i) omeprazole
j) Vancomycin

41. Which can be used for the treatment of cystic acne: Isotretinoin

42. Which one can be used as a preservative, humectant and solvent?


f) Glycerine
g) Urea
h) Ethylene glycol

43. Two parabens added together for what purpose


a. Enhance the preservative effect

44. Rate limiting step for diffusion through stratum corneum:


a. Partition co efficient

45. Angiotensin, which is right k type


I. Found abundantly in the heart
II. Works mainly on angiotensin 1 receptors

46. When there is a decrease in ADH, this means k type


I. polyuria
II. increase thirst
III. Decrease in plasma osmolority

47. Vincristine mechanism of action


a) Inhibit microtubule formation

48. Centrosome, what is it? 2010


a) Responsible for microtububle formation assembly
is an organelle that serves as the main microtubule organizing center (MTOC) of the
animal cell as well as a regulator of cell-cycle progression.
49. Which is correct about diabetes medication
a) Metformin is associated with hypoglycaemia
b) Acarbose causes flatulence and bloatations
c) Glyburide can cause Heart failure

50. Which is not a side effect of an overdose of insulin


a) Diarrhea
b) Nervousness
c) Palpitations
d) Nausea

51. The responsibilities of a Pharmacist manager or a Department Head at a hospital


pharmacy is
a) For the patients
b) For the pharmacist staff
c) For the nurses
d) For the physician

52. WHO do you report to about a medication error which occurred?


There are multiple channels (each for different class of user) for confidential/secure
reporting

53. Biggest burden on Health Care System


hospitals

54. Which is not used for acute coronary disease


Furosemide
55. Lithium therapy, u must monitor which levels
1) Blood sugar levels
2) Thyroid function tests

56. Resp for cell integrity 201: Glutathione


Glutathione (GSH) is an antioxidant in plants, animals, fungi, and some bacteria and archaea.
Glutathione is capable of preventing damage to important cellular components caused by reactive
oxygen species such as free radicals, peroxides, lipid peroxides, and heavy metals.

57. Vitamin A, all are true except?


A. To prevent night blindness
B. Its precursor is beta carotene
C. Given as a supplement to breastfeeding infants

58. To test for a pharmaceutical’s ability to withstand abrasion we use: Friabilator


Friability testing is used to test the durability of tablets during packing processes and
transit. This involves repeatedly dropping a sample of tablets over a fixed time, using a
rotating drum with a baffle. The result is inspected for broken tablets, and the percentage
of tablet mass lost through chipping.

59. Which receptors do opioids work on?


A. Mu
B. Kappa
C. M receptors
D. Alpha receptors

60. For treatment of dry skin which is best


a) o/w
b) petrolatum
c) lanolin
d) add emollient when the skin is still damp right after a bath

61. Sterile water for injection must be? Pyrogen free

62. Which can be given to a patient with renal problems to evacuate the bowel
A. PEG
B. Sodium Phosphate
C. MgOh
63. Which vitamin will cause Flushing? Vit. B3 NIACIN
A “no-flush” formulation combining inositol and niacin purports to reduce this adverse
effect. Administration of ASA 325 mg prior to taking niacin decrease flushing side effect

64. Some one admitted to the hospital and has alcohol withdrawal symptoms we give?
a) Thiamine
b) Riboflavin
c) Vitamin c
Thiamine deficiency puts patients at risk of Wernicke-Korsakoff Syndrome, which can be
fatal. Thiamine (vitamin B1) 200 mg daily recommended for patients who consume
significant alcohol amounts.

65. Nifedepine: 1,4 dihydropyridine

66. Structure of Beta Lactam....just identify that it is a lactam

67. Structure of Clofibrate is metabolized to another structure catalized by what?


Esterase
68. Structure of acetycholine and acts indirectly by
inhibiting? It is ester of acetic acid and choline, act
by inhibiting Acetylcholinesterase.

69. For An elderly patient what is not used to decrease bone fracture?
A. Biphosphonates
B. Calcium
C. Fluoride
D. Excerice for muscle endurance and strength

70. Rx 5mg prednisolone tablet and physician prescribed to take 35mg for 1st 2 days
then decrease by 5 every second day. How many days until finished?
14+12+10+8+6+4+2 = 56 tablets in 12 days

71. AUCoral/ AUCiv = ABSOLUTE BIOAVAILABILITY


Absolute bioavailability refers to amount of the drug available to the body or system. This
is measured as a ratio between the AUC after intravenous administration and
AUC oral administration. It should be a figure less than 1 since it is assumed that 100% of
the drug is available to the body after iv administration.

72. Glycosylation of protein occurs when:


a) After Transcription
b) After Translation
Post-translational Modification Including  glycosylation, phosphorylation, and
sulfatation

73. The class of RNA that carriers the genetic code


A) mRNA B) tRNA C) rRNA

74. Levigation is used


A. To dissolve solute in water
B. To dissolve insoluble powder in base
This method is also used to reduce the particle size of insoluble materials when
compounding ointments and suspensions.
Reduces the particle size by spatulating it on an ointment slab or pad with a small amount
of a liquid in which the solid is not soluble.
The solvent should be somewhat viscous such as Q. Mineral oil or glycerin.
75. For good Therapeutic gene expression what is necessary
a) Cloning of gene, transfection....
b) Gene must be incorporated into the nuclear membrane

76. Fatty acids are carried by which circulation


A. Lymphatic
B. Collateral
The lymphatic system has multiple interrelated functions:
 It is responsible for the removal of interstitial fluid from tissues
 It absorbs and transports fatty acids and fats as chyle from the digestive system
 It transports white blood cells to and from the lymph nodes into the bones
 The lymph transports antigen-presenting cells, such as dendritic cells, to the lymph
nodes where an immune response is stimulated.
Absorption
77. The rate of a drug can be used to partially identify?
A. Bioavailability
B. Clearance
C. Half life

78. When a water-soluble protein binds to the oustside of a cell membrane, and
activates the inside of the cell to become active...what is this called?
a) Hapten
b) Active transport

79. For influenza A virus, what is the enzyme name that found in virus that enables
them to be released from the host cell?
A. RNA polymerase
B. DNA polymerase
C. Neuraminidase

80. Which is used for the separation of protein based on molecular weight
a) Polymeric Gel electrophoresis
Gel electrophoresis is a method for separation and analysis of macromolecules (DNA, RNA
and proteins) and their fragments, based on their size and charge.

81. When a protein attaches to a substance which has an immunogenic response, it


called?
a) Hapten synthesis
b) Hapten-protein conjugation
Hapten: a small molecule which, when combined with a larger carrier such as a protein,
can elicit the production of antibodies which bind specifically to it (in the free or combined
state).

82. Fasting blood glucose threshold is < 7mmol (5.6 – 6.9)

83. Glycosylated haemoglobin HbA1c NOT required monthly?


It is a form of hemoglobin that is measured primarily to identify the three month
average plasma glucose concentration.
The test is limited to a three-month average because the lifespan of a red blood cell
is three months. Normal levels of glucose produce a normal amount of glycated
haemoglobin.

84. Rate of absorption can be used to partially identify?


Bioavailability
half life
clearance

85. Topoisomerase II (DNA gyrase) used to? chain formation


is an enzyme that relieves strain while double-stranded DNA is being unwound by
helicase.This causes negative supercoiling of the DNA . This process occurs in
prokaryotes (in particular, in bacteria)

86. Which calcium channel blockers cause Bradycardia (k type)


I. Nifedepine
II. Deltiazem
III. Verapamil

87. Which is responsible for calcium absorption from Distal Tubule? 2010
PTH

88. Bathochromic shift – shift to a longer wavelength 2007

89. To increase the Buffer capacity of a solution


I. Add strong acid
II. Add weak acid and its salt
III. Decrease the difference between pka and ph so that they are almost equal
90. Accumulation of blood in CHF is mainly in? LUNGS

91. Antiarrythmic (disopyramide) is used in arrhythmia acting on?


 Acts on sodium channels

92. Which drug does not have an affect on OC? Gabapentin

93. Nitrosamine group --- Nitrosamines are chemical compounds of


the chemical structure R1N(–R2)–N=O, that is a nitroso group
bonded to an amine. Most nitrosamines are carcinogenic. 2009

94. Which contraindicated in acute gout? Allopurinol

95. Patient has athletes’ foot, give: Clotrimazole

96. For first order kinetics the slope is??


a) ln (A) versus time
b) 1/A versus time
c) A versus time
The integrated first order rate law

97. All can cause ocular damage except


a) Rifampin
b) Ethambutol
c) Tobramycin
d) Amiodarone

98. Hypertrophy is
increase in the size of an organ or tissue due to the enlargement of its component
cells It is distinguished from hyperplasia, in which the cells remain approximately
the same size but increase in number. Eccentric hypertrophy is a
type of hypertrophy where the walls and chamber of a hollow organ undergo
growth in which the overall size and volume are enlarged

99. Which can be a cause of EPS?


haloperidol
100. Pseudoplastics (Shear thinning)
is a term used in rheology to describe non-Newtonian fluids which have decreased
viscosity when subjected to shear strain.
Shear thinning and shear thickening liquids. Some liquids behave differently when
stress is applied (application of force). Shear thickening liquids increase in viscosity
as stress increases. Shear thinning liquids decrease in viscosity as stress increases

101. Which does not treat severe Glaucoma? Furosemide

102. All are susceptible to venous thrombosis EXCEPT?


a) Hospitalized patient that is immobile
b) Patient with new replacement surgery
c) None of the above

103. All can be done to an ankle injury except?


a) Rest
b) Heat
c) NSAIDS
d) Compress
e) Elevate

104. In atrial fibrillation, which occurs Absence of P wave

105. Which is true about uric acid


Purine metabolism leads to uric acid formation

106. Septal defect of the Heart means? (Intervention septum) 2009


Opening or a hole in the septal wall between right and left side (could appear
between ventricle VSD and atrium ASD)
107. All of these may cause sedation except? Bupropion

108. Mercaptupouric acid metabolite is a result of which conjugation reaction


a) Glucouronide
b) Acetylation
c) Methylation
d) Glutathion
e) Hydrolysis

109. Someone who makes all the decisions in his pharmacy and doesn’t take any
advice or listens to anyone is called an
A. Autocratic
B. Democratic

110. Patient asked pharmacy A to forward her file (which contain methylphenidate
monthly) to pharmacy B because she changes her address to the other side of the
town. The pharmacist told her:
a- She should have another prescription for this medication
b- It is ok to forward her file
c- She should take an agreement from pharmacy B before forwarding her file
d- It is forbidden (because methyl phenidate is controlled I so no transfer)

111. The hormone existing during ovulation:


a- progestron
b- estrogen
c- high FSH and low LH
d- low FSH and high LH
e- high FSH and high LH

112. For contraceptive levonorgestril after intercourse, it is effective if it's used


a- Within 1 or 2 days  till 72hrs
b- After 7 days
c- Has no effect

113. A pharmacist searching on internet for a lot of (some drug) he found some one
advertising for this drug outside Canada with lowest price, when he called him, he
found that the drug has no DIN on it, what he should do:
a- Accept the lot
b- Call the Board and tell about that
c- Tell Health Canada about that
d- Become a representative for this seller
The RCMP, CBSA and Health Canada are fighting counterfeit products, so, report to
health canada if there is no DIN on the product label.

114. Interleukin II (oprelvekin):


Prevention of Thrombocytopenia due to antineoplastic agent.
N.B. neutropenia ttt by colony stimulating factors (e.g. G-CSF and GM-CSF) filgrastim
or pegfilgrastim
Neutropenia and Thrombocytopenia are due to Bone marrow depression.

115. For dry skin we should use products contain:


Emollient (e.g. petrolatum, Lanolin, fatty acids, mineral oil, glycerine)

116. All the following Drugs cause ototoxicity except:


a- Aminoglycosides
b- furosemide
C‐ Ciprofloxacin … not cause ototoxicity

117. Give for stuffy nose in patient with hypertention: Topical decongestant

118. Creamy and fishy odour discharge indicates: Bacterial vaginosis

119. Someone has thrombosis, he should use:


Clopidogril, ASA, dalteparine

120. Offlable indication means: Secondary use of the drug

121. Aniline structure is considered:


a- Weak acid
b- Weak base
c- Strong acid
d- Strong base

122. The role of sympathomimetics in ttt of glaucoma:


Increase drainage and decrease the production of aqueous humour.
123. Test for pyrogen: Limulus test.

124. Laxative for patient with renal problems:


a- Mg SO4
b- Mg citrate
c- Na2 HPO4
d- Docusate Na

125. Amorphous, which is true:


a- More soluble than crystalline
b- Has crystal lattice
c- Has distinct melting point.

126. Which statement is true regarding effervescent?


a- Must be dissolved in water before using
b- It libirates CO2 when enter stomach

127. Warts caused by: human papiloma virus

128. Which drug inhibit absorption of disacharride: Acarbose


Inhibit intestinal alpha-glucosidases resulting in delayed digestion of starch and
disaccharides and reduce post prandial glucose levels. Do not significantly inhibit
intestinal lactase.

129. Elderly has low serum creatinin due to? Low lean body mass.

130. Very high emetic anticancer: CISPLATIN


Also, Carmustine, Mechlorethamine, Streptozocin, Cyclophosphamide (>1500 mg/m2),
Hexamethylmelamine, Procarbazine

131. Side effect of leflunomide:


a- Stomatitis
b- Pneumonitis & hepatotoxicity
N.B. used in active moderate to severe rheumatoid arthritis and psoriatic arthritis. It
is a pyrimidine synthesis inhibitor

132. all are true about insulin lispro except: it can be taken orally
133. drug that cause hoarsens (dysphonia): inhaled corticosteroid
N.B: Iatrogenic (inhaled corticosteroids). Dysphonia is the medical term for disorders
of the voice

134. Enterohepatic circulation is between the liver and what?


A. Lung
B. Heart
C. Kidney
D. Small intestines

135. Enteric coated tablets consider:


a- sustained release
b-Controlled release
c-Delayed release

136. Nucleoside converted to nucleotide by: Phosphorylation by kinase


N.B. Nucleoside: sugar+base,
Nucleotide: sugar +base+phosphate.

137. Cholinergic drugs used in ttt of glaucoma: pilocarpine & Charbachol

138. Zidovudin (anti retroviral drug)


must be phosphorylated to be converted to active form

139. Which one is secreted by kidney:


Erythropiotine (secretd from kidney and helps in RBCs formation in bone marrow.)
140. In phase III hypothesis, Large number of populations to test: Safety
N.B. phase I on human healthy volunteers (small limited population)
Phase II efficacy (on large or limited population) diseasd patients.

141. Vincristine mechanism of action:


inhibit tubular formation (mitosis)=on M phase

142. Which is not a cause of prerenal failure


A. Sepsis
B. Dehydration
C. Gentamycin
D. Renal artery stenosis
There are 3 types of ARF:
Prerenal ARF Intrinsic (renal) ARF Post‐renal ARF
Occurs due to problems in organs before Occurs due to problems Occurs due to problems in
kidney as liver, heart and blood circulations. in kidney. organs after kidney like
Characterized by inadequate blood circulation ureter or bladder.
(perfusion) to the kidneys, which leaves them
unable to filter the blood properly.
Cardiac failure Drugs such as Urinary calculi
Sepsis. Blood loss aminoglycoside, Retroperitoneal fibrosis
Dehydration NSAIDs, Radioactive Benign prostatic
Vascular occlusion contrast media, enlargement
Sudden or severe drop in B.P. Lithium, digoxin, Prostate cancer
Atherosclerosis metformin, Cervical cancer
Excessive use of diuretics (water pills) nitrofurantoin, Urethral stricture/valves
amphotericin and Meatal stenosis/phimosis
sepsis.

143. Knee replacement surgery, which can be used to prevent Venous thrombosis
A. ASA
B. Clopidogrel
C. Fondaparinaux
D. Enoxaprin - always heparin for surgical prophylaxis

144. A physician asks a pharmacist about a new drug that hes presciribing to a
patient…which book will the pharmacist use for reference
A. CPS
B. TC
C. Book for Drugs in Canada
145. Which method will be used when both the drug cost and effect or outcome are
measured in dollar?
A. Cost minimization
B. Cost utility analysis
C. Cost benefit analysis

146. Two drugs with identical therapeutics and same active ingredient, which
pharmaeconomics will be used
A. Cost benefit
B. Cost utiliiy
C. Cost minimization

147. For a patient who is taking flexible Insulin dosing, in which situation will she
require to adjust her dose
A. Postprandial exercise Flexible insulin depends on carbohydrate
B. Depending on the Actual amount of food eaten intake

C. The insulin levels preprandial


D. Post prandial levels 2 hours after meal
Flexible insulin therapy is any insulin injection regimen that allows you to adjust the
amount of insulin you take at each dose.
On a flexible insulin therapy, you aim to match the insulin dose you inject to the amount
of carbohydrates in each meal you have.
For example, you may have different breakfasts on Saturday and Sunday.
On Saturday you may eat 30g of carbohydrate for breakfast whereas on Sunday you may
choose to eat 50g of carbohydrate for breakfast.
In this example, if no other activities (such as sport) are planned, then you would inject a
larger dose for Sunday’s breakfast than for Saturday’s.

148. What is the difference between Insulin Lispro Humalog, and Insulin Regular?
A. Insulin Lispro will reach its concentration peak quicker that regular insulin
B. Insulin lispro because of faster onset will cause a bigger hypoglycemic effect
C. Insulin lispro is given oral

149. In order to protect a protein when freeze drying It, which should be added
A. Humactant
B. Protectant
C. Lyoprotectant
150. All are true about ear wax except? 2011
A. Impaction of ear wax can cause hearing loss
B. Elderly will have more impacted earwax
C. Can use light mineral oil to dissolve ear wax
D. Can use triethanolamine to remove ear wax
E. Using cotton coated swabs not allowed.

151. Predisolone 5mg tablets


Rx 40mg daily for 2 weeks
30 mg daily for 2 weeks
20 mg daily for 2 weeks
Decrease tablets by 5mg every other day
The Insurance company will only cover a 30-day supply so how many tablets will u
dispense for the thirty-day supply? 2014
A. 230
B. 204
C. 196
D. 184

152. If a patient is on oral


contraceptions and forgets to take her
dose during the second weak. She is on a
Sunday-start schedual, what will u
advice her?
a. Take 2 once you remember and
then two the next day
b. Dispose the pack you have and
wait till Sunday and start a new
pack
c. Continue regulary as if nothing happened
d. Keep taking regular dose till Sunday and then start a new one

153. A nurse is taking care of a patient and she mistakenly administerd the
medication incorrectly, it is the role of the pharmacist to?
A. Report this error to someone who is in charge of the care for this patient using
the appropriate documentary procedures
B. Cover up for her
154. What is the main side effect of Varneciline?
A. Diarrhea
B. Nausea
C. Loss of appetite
Nausea (30%); may be mitigated by taking on a full stomach, increasing water intake or reducing dose.

155. What is dysgeusia? Taste disorder


Dysgeusia, also known as parageusia, is a distortion of the sense of taste.

156. Fastest relief of Heart burn?


A. Omeprazole
B. Ranitidine
C. Al/Mg OH

157. A patient has HIV and is taking medications for his HIV, He is adviced to take a
prophylactic for which one of these?
A. Pneumocystis jiroveci pneumonia (same thing as P. carinii)
B. Mycoplasma

158. Which should not be take with food


A. Methylphenidate
B. Norfloxacin
C. Cephalosporins
D. Prednisolone
Absorption decreased by antacids, calcium and iron (separate administration)

159. Someone who has Rheumatoid arthritis, which should not be given for life
long treatment?
A. Prednisolone
B. MTX
C. Sulfasalazine

160. Ud in a presctiption can be meant as: As directed

161. A cancer patient is taking certain drugs (forgot what) and her daughter comes
with a new prescription for her…what must u do when dispensing the meds
A. Councel her of the new drugs and tell her about the new side effects
B. Just dispense her the new medications, she will understand
162. Which will not trigger a cold sore
A. Fatigue
B. Spicy food
C. Emotional stress
D. UV light
E. Wind burn
CTC - Common stressors that can precipitate recurrences include emotional stress, dental
extraction, fatigue, fever, hormonal factors, hyperthermia, menstruation, physical trauma
or surgery, sun exposure (UV light) and upper respiratory infection.

163. In Ketoacidosis all may be found in EXCESS amounts in the urine except?
A. Lactic acid
B. Keton bodies
C. Acetone
D. Acetoacetate
E. Beta hydroxybutyrate

164. A patient comes to ur pharmacy and wants to waive his co payment, he claims
that other pharmacises do the same and that he never pays copayment…if you do
this for him, which ethical principle did u violate
A. It is ILLEGAL to do so
B. You violate the ethical principle of justice

165. When you take the Weight, age and Diseased state of patients, this is
comparing?
A. Statistical variable
B. Biological variable
C. Collective variable

166. Which doesn’t affect the adverse reaction of topical corticosteroid


A. Skin pigmentation
B. Concentration
C. Area applied

167. Hydroxyapatite is usually present in


A. Usually in bones it is a naturally occurring mineral form of calcium apatite
B. In the blood
C. In the skin
168. Which nerve is not present in the skin? 2008
a-pressure b-pain c-touch d- temperature e. Olfactory nerve
It is typically considered the first cranial nerve, or simply CN I.
It contains the afferent nerve fibers of the olfactory receptor neurons, transmitting
nerve impulses about odors to the central nervous system, where they are perceived by
the sense of smell.

169. A patient complains of back pain, which you are concerned with the history of
all EXCEPT?
A. Diabetes
B. Cancer
C. Trauma
D. Osteoporosis

170. A patient has osteoporosis which of these will u not advise?


A. Basketball
B. Exercise
C. Jogging
D. Weight lifting
E. Horseback riding

171. Protein C is responsible for what


A. Osmolarity
B. Clotting factor
N.B. known as autoprothrombin IIA and blood coagulation factor XI, V, is a zymogenic
(inactive) protein, the activated form of which plays an important role in regulating blood
clotting, inflammation, cell death and maintaining the permeability of blood vessel walls in
humans and other animals

172. Biphosphonates are products that resemble what: Pyrophosphate

173. This is prodrug of which one


A. 6 mercaptopurine
B. 5 flurouracil
C. Allopurinol
D. Azathioprine

174. Difference between Tadalafil and Sildenefil: Tadalafil is longer acting


175. Difference between Ipratropium and Tio: Tiotropium has long duration

176. Hyperplasia is: Increase in cell number

177. Person with parkinsons disease and also has BPH, which is contraindicated?
A. Benztropine
B. Bromoctriptine
C. Finasteride
D. Levodopa

178. Oseltamivir is CONTRAINDICATED in


A. > 65
B. < 1-year-old
C. Renal impairment
D. Hepatic Impairment
E. Someone needing prophylaxis for influenza
Oseltamivir should not be used if CrCl < 10 ml/min. Should not be used in children < 1 year

179. Which drug is first line for an adult with community acquired meningitis?
A. Ceftriaxone
B. Ceuroxime

180. The enterococci bacteria are most commonly resistant to which antibiotic
A. Cefozil
B. Penams
C. Penincillins

181. Pregnant with dental abscess, which will be recommended: Amoxi/clav

182. Surface Active Agent (surfactant) is added to contact lens solution to


A. Increase hydration
B. Increase viscosity
C. As a preservative

183. Which is correct statement according to health Canada


A. Tetanus-polio needs a booster shot routinely
B. Pneumococcal vaccine is taken annually
C. If you cut ur self with something “dirty” you will need Meningiococcal vaccine
184. Overdose on Acetaminphine for a child, wat will be the initial symptoms
experienced
A. Nausea
B. Diarrhea
C. Abdominal pain

185. Gives you the structure of an amino acid aniline and its transformation…and
asks how does this behave
A. Acidic
B. Basic
C. Amphiteric

186. Shows you a structure and after metabolism the structure loses an OH and
gains a COOH.what is this called -Dehydrogenation

187. Gives you a structure and then it is metabolized and it loses a NH3 group from
a carbon and is replaced by a C=O
This is called oxidative deamination

188. A structure is found in the retina, what does this


structure undergo?
11,12 cis-trans configurations

189. Which is incorrect about hypokalemia


A. It can be asymptomatic
B. Usually accompanied by hypomagnesia
C. Can be caused due to dehydration and vomiting

190. All can cause Left ventricular Heart failure except?


A. Hypertension
B. Ischemic cardiac disease
C. Pulmonary edema
D. Hematological disorders

191. In Microcytic Iron deficiency, you will see?


A. Increased MCV
B. Increase in serum ferittin
C. Increase in iron binding capacity
D. Decrease in serum folate

192. Which is used in a patient with CHf and pulmonary edema


A. Digoxin
B. Furosemide

193. When methyl tetrahydrofolate is produced, where will it be stored?


A. Kidney
B. Liver
C. Heart
D. Lungs

194. Lymphatic vessles, which is not true?


A. Contains valves that prevent the back flow
B. Connects into small veins
C. Is regulated by the heart rate

195. Which one does not secret into the GI?


ADRENAL GLAND (not open in duodenum)

196. Drugs that prolong QT can also cause


A. Atrial fibrillations
B. Ventricular fibrillations
C. Bradycardia
D. Hypertension

197. Organophosphate insecticide toxicity, what is the antidote?


A. Pralidoxime
B. Atropine
C. Succinylcholine

198. Which is Defficient in Alzheimers


I. Dopamine
II. Epinephrine
III. Acetylcholine

199. What is tru about both Acetylcholine and Succinylcholine (k type)


I. They have one or more tertiary nitrogen
II. Cross the blood brain barrier and enter CNS easily
III. They have high affinity for neuromuscular nicotine receptors

200. Which is caused by group a streptococcal infection (k type)


I. Hepatitis
II. Glomerulonephritis
III. Rheumatic fever

201. What are the largest expenses for a private insurance company?
Pharmaceutical drugs

202. What is the Role of the Public Health Agency, all except?
A. Travel information
B. Emergency preparedness and respone
C. Regulate pharmaceutical drugs
D. Health promotion
The role of the Public Health Agency of Canada is to:
 Promote health;
 Prevent and control chronic diseases and injuries;
 Prevent and control infectious diseases;
 Prepare for and respond to public health emergencies;
 Serve as a central point for sharing Canada's expertise with the rest of the world;
 Apply international research and development to Canada's public health programs;
 Strengthen intergovernmental collaboration on public health and facilitate national
approaches to public health policy and planning.

203. Which is not a lubricant


A. Magnesium stearate
B. Sodium lauryl sulfate
Lubricant  To ensure that tablet formation and ejection can occur with "low friction"
between the solid and the die wall. Example polyethylene glycols, stearic acid, stearic
acid salts (calcium, zinc and Q. magnesium stearate).

204. What is the name of an excipient that improves the flow of a powder in the
formulation?
A. Glidant
B. Adherent
C. Lubricant
205. The Blueprint for Pharmacy in 2008 was designed to do all except?
A. Design the pharmacy
B. Pharmacy human resources
C. Legislation, regulation and liability Information
D. communication technology Education

206. Which may be a cause for polypharmacy


A. Multiple medications
B. His complaints are very vague
C. Multiple prescribers
D. Visiting more than one pharmacy

207. Graves disease is most common hyperthyroidism, which is not a treatment


A. Ptu
B. Methimazole
C. Sodium iodide
D. Lugols soloution
E. Liothyronine

208. Antipsychotic drugs exert their action by


A. Centrally inhibiting D2 receptors
B. Centrally agonizing D2 recepetors
C. Inhibit alpha receptors

209. Alpha 1 antagonsits cause all except


A. Tachycardia
B. Hypertension
C. Arterial blood vessel dilation

210. When pain of angina is felt in the arms and jaw, this is called: Referred pain

211. Autoclave is used for


Sterilization process utilizing saturated steam

212. A Biogenetic bacterial solution is disinfected by


A. Filteration
B. Radiation
C. Sterilization
213. All are correct for Migraines except
A. Hemicranial
B. Facial pain
C. Decreased daily life activities
D. Photophobia

214. Capsaicin, all are true except


A. It is extracted from ginger root
B. It has analgesic effects
C. It can cause stinging and irritation when applied
D. It is used in osteoarthritis

215. Which has the most daily doses for an antihistamine?


A. Cetrizine
B. Meclinizine
C. Loratedine
D. Diphenhydramine – TID – QID

216. Bacterial conjunctivitis can cause all except


I. Nystagmus - rapid involuntary movements of the eyes
II. Purulent discharge
III. Stinging and pain
Signs: Normal pupillary reaction, Discharge and/or excessive tearing, Lid and conjunctival
edema, Conjunctival redness, Itching.

217. The cost of a product is 2.00$ and you sell it for a price of 2.5$ which is true?
A. Profit percent is 20%
B. Mark up of cost is 20%

218. Measurments of Carboxyhemoglobinemia is used to determine


A. CO toxicity
B. Carbon tetrachloride toxicity
C. Nitrate toxicity

219. Which is considered a Viral infection


A. Vibrio - is a genus of Gram-negative bacteria which can cause foodborne infection
B. Rubella - also known as German measles or three-day measles
C. Asperigillosis - infection by fungi of the genus Aspergillus
D. Taxoplasmosis - a parasitic disease caused by Toxoplasma gondii
220. A drug is administered into the body and a very small concentration of the
drug is excreted unchanged in the urine however When administered with antacid,
the excretion increases, this indicates that the drug is?
A. Weakly acidic drug
B. Weakly basic drug
C. Neutral drug

221. A drug (one gram) is mixed with a solution of oil and water…the final
concentration in oil is 10mg/ml and in the water, phase is 5mg/ml…what is the
partition coefficient of this drug?
A. 1
B. 2 P = solute (oil) / solute (water)
C. 3

222. The prophylactic tablet for Malaria is 250mg. An adult of 100kg is planning to
travel for 6 weeks and the prophylaxis must be administerd one week before,
during, and 4 weeks after his retun. The dose recommended for this adult is 5mg/kg
but only a maximum weekly dose of 300mg. How many tablets should he take?
A. 5
B. 10
C. 11
D. 15

223. A concentration of a drug is 4mg/ml, however the concentration of this drug


appears to be 23.45mg/ml in the Breast Milk! This means?
A. The drug is basic
B. The drug is highly protein bound
C. The drug has a very large molecular weight
in order for drugs to be excreted in breast milk they should be:
low Mol wt, Low protein binding, Low acidity & High Lipid solubility

224. For GLC, which increases its effects?


A. Extremely polar substance
B. Volatile substance

225. The plate model of chromatography is used to


Separate equilibrations of the sample between the stationary and mobile phase
226. When an IV infusion is given as an emulsion it must be (k type)
I. w/o emulsion
II. sterile
III. particles less than 5micrometers

227. What is true about eutectic mixture


a) Degrade at room temperature
b) Segragates upon storing
c) Liquefies at room temperature

228. All are true concerning emergency oral contraceptives except?


a) Levonogestrel can be given in emergency cases at a conc of 1.5mg and it is
very effective
b) Levonogestrel can be given with Estrogen
c) You have nothing to worry about as long as you take the emergency
contraceptive pills within 7 days

229. A 50-year-old women in menopause is planning to take Estraderm patch


which of these would you most commonly councel on
a) Estrogen patch will cause redness and irritation on site of application
b) Make sure to apply to non hairy open area on the arms
c) Must take for 3 weeks and then remove for one week and keep cycling it
regularly

230. Which is true for uncomplicated UTI


a) Accompanied by fever and chills
b) Pregnant women need to take prophylactic solution before delivery
c) Prophylaxis should be given to females with frequent reoccurrences

231. Which of these is responsible for an Increase in Digoxin plasma levels?


a) Phenytoin
b) Rifampin
c) Cholestyramin
d) Amiodarone
Amiodarone, clarithromycin, cyclosporine, erythromycin, itraconazole, propafenone,
quinidine, ritonavir, tetracycline and verapamil increase digoxin serum levels. (by
displacing tissue binding sites and depressing renal digoxin clearance) so plasma digoxin
must be monitored carefully.
232. A pharmacist plans a strategic plan so he developes a SWOT analysis, what is
the T stand for in this case?
A. Time
B. Threat
C. Tolerance of risk
D. Temperature

233. The Health care system in Canada is funded by


A. Primarily by federal and provincial funding and their taxation
B. Partly by federal and provincial funding as well as cash payments, and insurance
payments.
C. Completely by federal government

234. Which one undergoes metabolic polymorphism to give different analgesic


effects which are not the same in every person?
A. Fentanyl
B. Morphine
C. Codeine
D. Meperidine

235. Peroxisomes

236. Which of these drugs are not affected by any metabolic processes?
A. Gabapentin
B. Phenytoin
C. Carbamezapine
D. Valproic acid
237. A child with asthma, what is least likely to trigger an asthma attack:
A. Viral infection
B. Excercise induced
C. Cold air exposure
D. Fever

238. Whcih is the most likely to be found in pulmonary embolism


A. Fibrin
B. Platelets

239. For a normal patint with no underlying disease and good health which be the
ideal clinical Blood pressure reading?
A. 160/90
B. 150/90
C. <140/80 140/90

D. <130/80
E. <120/80

240. Infliximab works on which factors


A. IL-6
B. TNF

241. Used in motion sickness: Dimenhydrinate

242. Megaloplastic anemia is caused by


A. Decrease in iron levels
B. Decrease in Vit. B12 and Folic acid

243. Given the structure, which enzyme does it inhibit (ALLOPURINOL)

A. Xanthine oxidase
244. Symptoms of stopping antidepressant all except:
a- Insomnia
b- Sweeting
c- Tremor
d- Myalgia

245. Methamphetamin structure

246. All case V.C. except: HISTAMINE

247. The main reason (or disease) GIT:


a-GERD
b-hyperacidity
c-peptic ulcer

248. Counsling about thyroxin: Should be given in the morning to avoid insomnia

249. Angiogenesis means:


d) Formation of new blood vessels
e) Formation of new bone
f) Formation of new Red Blood Cells
Tips: Angiogenesis is the physiological process through which new blood vessels form
from pre-existing vessels. This is distinct from vasculogenesis, which is the denovo
formation of endothelial cells from mesoderm cell precursors. The first vessels in the
developing embryo form through vasculogenesis, after which angiogenesis is
responsible for most, if not all, blood vessel growth during development and in
disease.

250. Benign prostatic hyperplasia all except: polyurea


N.B. diagnosed by increase in PSA enzyme, per rectal examination.

251. Treatment of lice should be repeated after 10 days: To kill cysts and eggs
252. Ttt of scabies: For all members of the family and whole the body (by 5%
permethrine)

253. Drug cause +ve inotropic (contractility) : Dobutamine


N.B. it is B1 agonist used in ttt of heart failure, cardiogenic shock.

254. Glycerol is used as: Emulsifying agent

255. S.E of Clozapine: Agranulocytosis

256. Uric acid:


a- Metabolite of protein Purine
b- Increased in liver disease
c- Increased in using anti-cancer drugs

257. Gynecomastia is S.E. of: Spironolactone

258. Liquid – solid contact angle: Wettability


Wettability is the tendency of one fluid to spread on, or adhere to, a solid surface in
the presence of other immiscible fluids. Wettability refers to the interaction between
fluid and solid phases.
The contact angle is the angle, conventionally measured through the liquid, where a
liquid–vapor interface meets a solid surface. It quantifies the wettability of a solid
surface by a liquid via the Young equation.

259. Herpes infection transfer by: Contact lesions

260. When the drug indicated to be used 4 times daily after meals and at bed time
it means: Q. I. D PC. HS. i.e. 5 times daily

261. Mother bring a prescription for her child for otitis after 3 days:
a-fill the prscription
b- not allowed
262. Most abundant morphin receptor:
a- Mu
b- Kappa
c- Delta

263. In all age stages, patient health depends on:


a-Participation in health care centers
b-Participation in health care givers by health Canada
c-Eduction level

264. Protein denaturation cause by all except:2011


a) Add saline
b) a strong acid or base
c) a concentrated inorganic salt, an organic solvent
d) radiation or heat.
Denaturation is a process in which proteins or nucleic acids lose the quaternary structure, tertiary
structure and secondary structure which is present in their native state,

265. Counselling of clarithromycin: Shake well before use, don't refrigerate

266. Without DNA: 2008


a-fungus
b-bacteria
c-prions - so-called because they are proteinaceous, are infectious particles, smaller
than viruses, that contain no nucleic acids (neither DNA nor RNA)
d-Viroids - small "naked" infectious RNA molecules that are pathogens of higher
plants

267. Responsible for synthesis of DNA from mRNA: Reverse transcriptase

268. Anorexia nervosa all except:


1-Adapting body shape
2-loss appetite
3 -excessive excercise
N.B. anorexia nervosa is restrictive and Non purging= excessive exercise and fasting.

269. Bleaching agent in anti dandruff:


a- Salicylic acid
b- Cool tar
c-Selenium sulphide Has antifungal effect and cytotoxic effect

270. Stroke may occure with:


a- D.M.
b- Myocardial infarction & Atrial fibrillation

271. Levothyroxine S.E.


Symptoms of hyperthyroidism if overtreated; exacerbation of angina.
Rare side effects such as anxiety, diarrhea, weight loss, sweating, insomnia, and muscle
cramps.
Toxic symptoms are nervousness, palpitation, intolerance to heat.
Counselling lable of it  take at morning to avoid insomnia

272. Drug cause parkinsonism like S.E.: Haloperidol

273. Pharmacy regulation by: NAPRA


1- who regulate pharmacy practice in Canada? NAPRA
2- who regulate pharmacy members in Canada? provincial pharmacy authorities

274. QRS  Ventricular depolarization

275. T wave  Ventricular repolarization (due to K is pump out from heart cells)

276. Most blood component in inflammation:


a- Basophil
b- Neutrophil

277. The easiest type for pharmacy ownership is: 2011


a- Partnership
b- Franchise
c- Sole

278. cDNA obtained from:


a- mRNA
b- ribosoms
c- plasmid
279. drugs used to treat CHF and cause hyperkalemia and gynecomastia:
a- amiodarone (K channel blocker, antiarrhythmic drug)
b- amiloride
c- spironolactone
d- triametrine

280. prescription for prednisone, the technician made a mistake in dispensing it for
30 days. How many tablets of prednisolon 5mg each should you dispense for this
first 30 day (the correct quantity)?
Rx/ 40mg for 2 weeks
30mg for 2 weeks
20mg for 2 weeks
Then tapering 5mg every other day till finish
a- 204 tablets
b- 300 tablets
c- 310 tablets

281. Patient will take levoflxacin, what about taking grapefruit juice:
a- It is safe to use grapefruit juice with levofloxacin.
b- It is inhibited by grapefruit
Decreased absorption of fluoroquinolones with
concomitant iron, calcium, magnesium, zinc,
antacids, sucralfate. Separate doses by 2 h.

282. precursor of vit. D: 7-dehydrocholestrol

283. to find latest drug in market; CPS


Common drug review

284. activation of prodrug,


a-bioactivation
b-biotransformation

285. base not found in RNA:


a-thiamine

286. OH, CH3, NH2 is  isosters


287. antioxident is?
a-alpha-tocopherol (vit E)
b-methylparaben
c-ferrous.

288. Lymphatic vessles, which is not true?


A. Contains valves that prevent the back flow
B. Connects into small veins
C. Is regulated by the heart rate

289. QD in latinis; once daily (every day)

290. drug taken with or after food:


a) sertraline
b) amoxicillin

291. Viox (Rofecoxib) is: COX -II inhibitor (NSAID)

292. Difference between capping & lamination?


January 2011
(1) Which is acid stable:
A. Penicillin G
B. Cloxacillin, Penicillin V, Amoxicillin
C. PABA

(2) STD's Asymptomatic should be treated


A. May cause erectile dysfunction in men if untreated.
B. All asymptomatic turns to symptomatic
C. May cause pelvic infections and infertility
D. May cause colon cancer

(3) Intangible cost is:


A. Cost for care by caregivers
B. Lost money on treatment
C. Expenses for hospital admission
D. Pain and suffering
E. Unearned salaries due to disease
tangible cost are easily measured like salaries of employee which are essential to
run bussiness,
intagible cost are hard to measure but give u loss, like if employee moral is lost
or customer dissatisfaction.

(4) Humanistic outcome:


A. decrease patient visits to physician
B. Patient satisfaction

(5) SPO is:


A. structure-patient outcome
B. structure‐procedure‐outcome. (structure‐process‐outcome)
C. structure procedure objective
D. structure patient objective
1. ‘Structure’ denotes the attributes of the settings in which care occurs.
2. ‘Process’ denotes what is actually done in giving and receiving care, i.e. the
practitioner’s activities in making a diagnosis, recommending or implementing
treatment, or other interaction with the patient.
3. Outcome’ ‘measures attempt to describe the effects of care on the health status of
patients and populations.
(6) POS is:
A. Point of sale
B. Professional operating system
C. Point of speech
D. Plan open space

(7) The most expensive Burden on canada's health insurance:


A. Dental
B. Out patient medication
C. Physician fees
D. Hospitalization

(8) The Goal of Pharmaceutical care (or the definition):


A. Positive patient outcome
B. Access Pharmaceutical care
C. Effective use of pharmaceutical resources

(9) In pandemic, the least concern for a pharmacist should be:


A. Achieve as much profits as he could

(10) Paroxetine is used for all EXCEPT:


A. Generalized anxiety disorder
B. Major depression
C. Ocd
D. Bipolar

(11) All the Following SSRI need to taper dose on withdrawal, EXCEPT:
A. Sertaline
B. Fluoxatine
C. Paroxetin
D. Citaloprin

(12) OD, is a 40 Years old female, have a sulfonamide allergy, and she asks u about taking
Cox II inhib:
A. Its contraindicated with sulfonamide allergy
B. Cox II increase risk of git bleeding
(13) Metformin, cause all these side effects, EXCEPT:
A. Weight loss
B. GI distress
C. CI with hepatic and renal failure
D. Sulfa allergy
E. Lactic acidosis

(14) A drug when administered with water, its AUC=10mg, same dose administered with
Coca drink, the AUC=18mg, this means coca bev cause:
A. Increase the absorption of the drug
B. Decrease absorption rate of the drug
C. Decrease Vd
D. Decrease Plasma protein binding
E. Increase excretion

(15) A patient taking labetelol, u should monitor all EXCEPT:


A. INR
B. Bp
C. Heart rate
D. Heart Rhythm

(16) Drug X 30 ml
Methanol 30ml
[Span 60(HLB=4.7)
+Tween 40 (HLB=15.6)] 5%
Water qs 130ml

And the final Soln HLB should be 12


How many parts of Span60 and tween 40 is used:
A. 2.1 ml Span60+4.3ml Tween40 (ALLIGATION)
B. 2.5ml Span60+4.8ml Tween 40
C. 3.5ml Span60+5.5ml Tween 40
D. 4.3ml Span60+2.1ml Tween 40
E. 3.5ml Span60+3.5ml Tween 40

(16)Infliximab is Indicated to treat:


A. Osteoarthritis
B. Rheumatoid arthritis
C. Colon cancer
D. Chronic heart failure
E. Hypertension

(17) Which of the following drugs you cannot advertise by a banner on the counter:
A. A new herbal drug
B. A new supplement
C. A new OTC
D. A new anti arrhythmic prescription drug

(18) Which is An antioxidant that act on the lipid bi layer of cell membrane:
A. Vit A
B. Vit B12
C. Vit C
D. Vit E
E. Vit K

(19) Which is useful For Cell integrity:


A. Glutathione
B. Glucosamine
C. Cystiene
D. Homocystiene
E. Serine

(20) All enzymes are considered:


A. Oligopeptidess
B. Phospholipids
C. Proteins
D. Polysaccharides
E. Lipids

(21)Lecithin is considered:
A. Oligopeptidess
B. Phospholipids
C. Proteins
D. Polysaccharides

(22) K TYPE: Permethrin for the treatment of Scabies:


I) Applied to the whole body from Neck to toes
II) Applied to all the close contacts
III) Washed after 8‐12 hours

(23) K TYPE: CK is Found in high concentration in:


I) Skeletal Muscles
II) Smooth muscles
III) Heart

(24) K TYPE: Hospital Formulary Is used for:


I) Tell the physician Which Drug is more Prescribed
II) Show Physicians and pharmacist which drug is more prescribed
III) Allow manufacturers and insurance to advertise in it

(25) Alrazepam is metabolized by the enz.:


a) CYP 3A4
b) CYP 2D6
c) CYP 1A2
d) CYP 2C9
e) CYP 1A2

(26) salfasalazine is the 1st line treatment of:


a) Osteoarthritis
b) Rheumatoid arthritis or ulcerative colitis!
c) Asthma
d) Chronic heart failure

(27) Fecal occult blood is a characteristic of all the following, EXCEPT:


a) Peptic ulcer
b) Hemorrhoids
c) Colorectal cancer
d) Prostate cancer
e) Ulcerative colitis

(28) AS, a 28 years old patient, with allergy to salicylate, is complaining about blood in
stool, he first observed this 24 hrs ago, what is your advice:
a) See a physician
(29) NK is a 40 year old Male, he is healthy and just had a medical check up, where his
glucose level was 8mmol/L , he also has a Brother who is recently diagnosed with DM type
II, The drug of choice for NK is:
a) Sulfonyl urea
b) Meglitinide
c) Metformin
d) Insulin
e) Exercise & diet

(30) K TYPE: For elderly having trouble sleeping, u should advise them to:
I) Have a nap at day time
II) Regular exercise
III) Have a regular schedule for sleeping and waking times

(31) Bed sores is also known as:


a) Dropsy
b) Claudication
c) Phelibitis
d) Pressure atrophy

(32) All Cause phototoxicity EXCEPT:


a) Penicillin
b) Tetracycline
c) Trimethoprim
d) Sulfonamide

(34) Dysphagia means:


a) Difficulty swallowing
b) Loss of appetite
c) Thirst
d) Impaired phagocytosis

(35) Stoke's Law determines:


a) Rate of absorbtion
b) Degree of ionization
c) Free Energy
d) Sedimintation rate
(36) oral liquid is preferred in elderly due to, Dysphagia

(37) Diffusion is affected by all EXCEPT:


a) Directly proportion to the surface area of solid
b) Depend on the concentration difference between solid and liquid layers
c) The diffusion coeff.
d) The length of stagnant layer
e) Temperatutre of the liquid

(38) A thermodynamic term that means Randomness


a) Entropy ‐ degree of disorder or randomness in the system
b) Exotropy
c) Anisotropy
d) Heat Capacity
e) Temperature

(39) Methanol 15%


Benzyl chloride 5%
Benzyl alc. Act as:
a) Solvent
b) Preservative
c) Suspending agent
d) Humectant
e) Cosolvent

(40) Which is a suitable auxillary label for" clarithromycin" suspension


a) Shake well before use
b) Keep refrigerated
c) Don’t shake before use

(41) Which of the following should be shaked before used:


a) Fluticazone (HFA inhaler)
b) Memicazone (Diskus)
c) Salbutamol (Handihauler)
d) salmetrol (tubuhaler)
e) budesonide (Nebulizer)

(42)Drug X is given as infusion, which of the following will reach Css first:
a) T half= 1 hr, and 100% eliminated by Liver
b) T half= 2 hr, and 50 % eliminated by Liver
c) T half = 4 hrs and 100% eliminated by kidney
d) Thalf = 12 hrs and 50% eliminated by liver and kidney
e) T half = 24 hrs and 50% by liver and kidney

(43) Htn affect all Organs except:


a) Brain
b) Eye
c) Kidney
d) Lung

The body structures that chronic high blood pressure affects most include:
 Blood vessels
 Heart
 Brain
 Kidneys
 Eyes.

(44) The most causing organism for typical Community acquired pneumonia (CAP) :
a) M. catarahlis
b) S. Pyogens
c) Chlamydia
E. coli
d) H. influenza

(45) A physician giving amoxicillin before dental procedure, for the prophylaxis of:
a) Pharingitis
b) Sinisitus
c) Endocarditis
d) Pretonitis

(46) Cisplatin Act by


a) Attach to DNA complementary strand
b) Platin form covalent bond with dna
c) Decrease transcription
d) Decrease toleomerase enzyme
(47) Treatment of severe Acute asthma, all used except:
a) IV corticosteroid
b) Inhaler SABA
C & D are no more used
c) IV Epinephrine
d) Inhaler nedocromil

(48) Which is not effective in CAS Coronary (acute coronary syndrome, like angina)
a) ASA
b) Feurosemide
c) Alteplase
d) Bblocker
e) Nitrates

(49) ASA is effective as one dose/day as anti platelet, Because:


a) Its is converted to salicylate which has longer half life
b) ASA has other effects on body
c) ASA t half is 20 hr
d) ASA is eliminated in 20 hr
e) ASA gives a prolonged inhibition of Platelet (irreversible platelet inhibition)

(50) Lung thromboebolism is mainly due to:


a) Fibrin
b) Platelet
c) C protein
d) Other components

(51) With B blocker treatment, its important to know History of all except:
a) Heart Disease
b) Asthma
c) Allergy

(52) The difference between LMWT heparin and Heparin is:


a) LMWT can be given for ambulotary patient at home
b) LMWT is better in ttt of DVT
If you had to chose one….choose A
The Kinetics of LMWH are more predictable than those of UFH and their T1/2 is longer. These
factors make weight‐adjusted fixed‐dose sc dosing of LMWH an excellent alternative to adjusted
dose IV UFH in the initial treatment of Venous Thromboembolism
(53) Which is used as skeletal muscle relaxant: Diazepam

(54) Health Canada covers all except: Out patient diagnostic tests

(55) Osmotic Pumb, which is true:


a) Release is zero order
b) Release is drug dependant
c) Release depent on the erosion of the material
d) No Dose dumping if there's a defect in membrane

(56) Mitochondria, which is true? ATP production

(57) Limping and pain because of circulatory problem is known as:


a) Phelibitis
b) Myalagia
c) Myopathy
d) Thrombophelibitis
e) Cludication

(58) All are 1st generation antipsychotics, EXCEPT:


a) Chloropromazine
b) Haloperidol
c) Thioridazone
d) Droperidol
e) Clozapine

(59) Agranulocytosis is a major Side effect of: Clozapine

(60) Nitrofurantoin for the treatment of UTI is given in dose of:


a) 1 day
b) 2 days
c) 7 days
d) 14 days
e) Single dose

(61) Which is Phase II metabolism:


a) O- Demethylation
b) Hydrolysis
c) Desulfuration
d) Deamination
e) Glucuronidation

(62) This structure is found in:


a) Antihistamines
b) Anticholinergics
c) TCA
d) Thiazide diuretics

(63)K TYPE: Hard Lens is ccc by:


I) high water content
II) highly rigid(reduced fragilability)
III) Longer life

(64) Acetaminophen Metabolism reactive phase I metabolite is done by:


Glutathione s‐transferase This is phase II NOT I

(65) Total slaes= 1,500,000 $


Margin= 450,000 $, Cost of saled goods= 1,050,000, Net profit= 120,000
Net profit margin is:
a) 1 %
b) 2 %
c) 3 %
d) 5%
e) 8%

(66) Drug X is basic, By alkalinization of urine:


a) Increase reabsorption
b) Increase execretion
c) Decrease execretion

(67) In a 2-compartment system, K12 Means:


a) The rate constant of drug transfer from compartment 1 to 2
b) The rate constant of drug transfer from compartment 2 to 1
c) The half life of drug is 12 hrs
d) The elimination rate is 12 mg/hr
(68) K of a drug = 0.35 1/hr, its elimination is first order, what is the time to reach Css:
a) 0.7 hrs
b) 2.15 hrs
c) 9 hrs ???
d) 24 hrs

(69) HSV occurs mostly in:


a) Lungs
b) Eyes
c) Mouth
d) Bones
e) Connective tissue

(70) All cause denaturation of protein EXCEPT:


a) Increasing ph more than 11
b) Adding organic solvent
c) Adding saline
d) Freezing
e) Increasing temperature

(71) Isotonic soln is:


a) 0.9% Nacl
b) 0.09 % Nacl
c) 0.9 g /L NaCl
d) 0.09 mcg/L Nacl

(72) What is the effect of stomach acid on protein:


a) Emulsification
b) Hydrolysis
c) Denaturation
d) Oxidation
e) Denaturation

(73) K TYPE
Test for parentrals include
I) Pyrogen free
II) Sterile
III) Presevative efficacy
(74) All of the following should be sterile, EXCEPT:
a) Water for injection
b) Water for irrigation
c) Opthalmic soln
d) Nasal Soln

(75) Plasmid is Charcterised by All of the following Except:


a) In bacteria, it carries the same genetic material as the DNA
b) A circular Chromosome
c) Undergoes replication

(76) Antibodies are ccc by all EXCEPT:


a) They are proteins in nature
b) A ccc of Vertebrates
c) Each Ab Fits to a certain Ag
d) Human has a large number of circuilatin AB against all Ags

(77) Cystic Fibrosis is treated by:


a) Pancreatic enzymes
b) High dose Antibodies
c) Prtyolitic Enzymes

(78) Homeostasis is: 2009


a) The ability of body to maintain Intact living condition
b) The ability of body to heal itself The tendency to maintain a stable, relatively constant internal
environment is called homeostasis. The body maintains
c) The ability of body to stop bleeding homeostasis for many factors in addition to temperature. For
instance, the concentration of various ions in your blood must
d) The ability of body to balance electrolytes be kept steady, along with pH and the concentration of glucose.

(79) Which is a type of analysis that depend on the destruction of analyte:


a) Spectroflurimetry
b) Mass spectroscopy (ionization – fragmentation)
c) Hplc
d) Gas chromatography

(80) Gel Electrophoresis seperation Depend on


a) Isoelectric point
b) Mwt
Two‐dimensional gel electrophoresis: The two dimensions that proteins are separated into using
this technique can be isoelectric point, protein complex mass in native state, and protein mass.
(81) Which of the following contain a prophyrin ring: Cytochrome oxidase

(82) Drug used for the prophylaxis of Traveller's Diarrhea: Ciprofloxacin

(83) Drug Combination used To treat MRSA:


a) Vancomycin + Folic acid
b) Erythromycin + Doxycyclin MRSA : Vancomycin + clindamycin , linezolid
C. defficile: Metronidazole , Vancomycin
c) Metronidazole+ Ampicillin
d) Vancomycin + Metronidazole Wrong answer .. answer is E
e) Vancomycin + clindamycin

(84) The first line treatment for Pain in Osteoarthritis is:


a) Morphine
b) Ibuprofen
c) Acetaminofen

(85) SOAP stands for:


a) Subjective, Objective, Assessment, Plan
b) Services, Opportunity, Assessment, Plan
c) Subjective, Outcomes, Assessment, Procedure

(86) A manufacturer Made a mistake, A whole lot of drug x containers contain another
drug with different shape and size, Pharmacist Role include all the following EXCEPT:
a) Put a note on the counter to warn patients
b) Ask patients to check if there is any change in their medication appearance
c) Call all the patients that their records show they are taking this medication
d) Ask any patient that is taking the wrong drug to contact his physician
e) Give the manufacturer the records of the patients taking this drug

(87) A medication organizer with four compartment, Breakfast, lunch, dinner and
bedtime, for an old person
The drugs he's taking is
Furosemide 20mcg bid
Digoxin 30mg once daily
Glyburide 10 mg tid
What should be a concern for the pharmacist when he see's it:
a) 2 tabs of furesomide in the breakfast container
b) 1 tablet of digoxin at the lunch container
c) A tablet of glyburide in each container
(88) All the following group undergoes acetylation except:
a) NH2
b) NHNH2
c) SONH2
d) OH
e) CONH2

(89) Net profit = 90,000, What would be the income to Make a profit of 15%
a) 150,000
b) 90,000
c) 250,000
d) 600,000
e) 800,000

(90) Li toxicity Increase in case of:


e) Renal failure
f) Increase sodium
g) Old age
h) Enalapril ‐‐‐‐decrease Na/H2O retention so increase sodium excretion
therefore inc Li

(91) What is not a risk factor for hypothyrodism:


a) Male gender
b) Normal T4 and T3 but elevated TSH
c) History of autoimmune disease
d) Treatment of Hyperthyrodism

(92)A written Will of a patient is considered:


a) Patients orders
b) Care givers Orders
c) Specific directives

(93)K TYPE
AS is a patient taking Ramipril, and about to start Prazocin, What should you monitor:
a) Hypotension
b) Cough
c) Faniting is a complication of A (hypotension )
(94) Patient on nicotine patch for smoking cessation, and it is really helping him, but he
complains about bad dreams and difficulty sleeping, what is your Advice:
a) Change to another dosage form
b) Advice him to remove the patch before sleeping

(95) Status Epilipticus is treated by,


a) IV phenytoin
b) IV Phenobarbital
c) IV lorazepam
d) Oral Ethosuximide
e) Oral valproic acid

(96) Oral Route in a prescription is written as:


a) P.O
b) O.R.
c) V.O.
d) O

(97) OS means
a) Left ear
b) Right ear
c) Left eye
d) Right eye

(98) A prescription for codeine Give 60, I tab/day 30 days x1


Is this prescription accepted?
a) No, because refills are not allowed This writing is refill which is not allowed ..
Only part fills are allowed
b) No, because part fill are not allowed
c) Yes, because the total amount is clear
d) Yes, because the amount is easily identifiable

(99) Estrogen Differ in ring number one because it is


a) Heterocyclic ring
b) Prophyrin ring
c) Piperazine ring
d) Aromatic ring
(100) Which doesn't undergo oxidation
a) Carboxylic acid
b) Alchol
c) Amide

(101) Canada's health act includes all the following except:


a) Universality
b) Portability
c) Public Admission
d) Accessability
e) Speciality

(102) Alkalosis is caused by all the following EXCEPT:


is the result of a process reducing hydrogen ion concentration of arterial blood
plasma (alkalemia)
Respiratory alkalosis is caused by hyperventilation, resulting in a loss of carbon dioxide.
Metabolic alkalosis can be caused by repeated vomiting, resulting in a loss of hydrochloric acid
within the stomach content. Severe dehydration, and the consumption of alkali are other causes

(103) All is 1/day except:


a) Enalapril
b) Ramipril
c) Lisinopril
d) Captopril (tid or bid)

(104) Vit E After Oxidation in lipid bilayer, return to its reduced form by the action of
a) Vit A
b) Vit B12
c) Vit C
d) Vit K
e) Vit D

(105) Megaloplastic anemia: Is due to folic acid or vit b12 def

(106) Mutagensis. All true EXCEPT:


a) Change in DNA nucleotides
b) Involve endoplasmic lipoxygenase enzymes
c) Affect mostly the dividng cells
(107) Unmutated Enzyme Called:
a) Wild
b) Silent
c) Recessive
d) Major

(108) Amino acid that undergoes dimerization:


a) Serine
b) Cystiene
c) Therionine
d) Glycine- maybe
DIMERIZATION Any chemical reaction in which
two monomers react to form a dimer.

(109) Aneurism is:


Abnormal prolongedVasodilation of a blood vessel

(110) Atherosclerosis is due to: Homocystiene

(111) The Major Cause of stroke is


a) Hyperthyroidism
b) Congestive heart failure
c) Atrial fibrillation
d) Renal failure

(112) Digoxin Is useful in case of: Atrial fibrillation

(113) Female 56 at risk osteoporosis


a) Ensure enough VitD intake
b) Ensure enough Ca intake
c) Start Estrogen therapy

(114) Cold sores are ccc by all the following EXCEPT:


a) Latent in neurons for long time
b) Increase incidence infants
c) Caused by HSV
d) Not contagious
(115) The HPLC analysis depend on all the following EXCEPT:
a) Length of column
b) Rate of adding mobile phase
c) Type of column
d) Type of detector

(116) Which is not taken with Li: Fe Fumarate

(117) Ag+E+Ag*↔AgE+Ag*E, Competitive, on increasing Ag, which is true:


a) Ag* will decrease
b) E will increase
c) AgE will decrease
d) Ag*E will decrease

(118) Non complicated Diaper rash is treated with:


a) Coal tar
b) Nystatin
c) Salicylate
d) ZnO cream

(119) Dextromethorphan with Venlafaxine


a) Increase serotonin effect
b) Increase anticholinergic effect
c) Decrease dextromethorphan effect
d) Decrease venlafexine effect
DXM (Dextromethorphan, Robotussin): This can be a very DANGEROUS combination! Do not
take DXM and SSRIs together! Combining DXM with an SSRI can lead to too much serotonin in
your brain, which can cause the potentially FATAL serotonin syndrome

(120) Dextromethorphan is cc by:


a) Act on kappa receptor
b) Not abusive
c) Less constipation than codeine
d) Has antitussive effect

(121) Sensory Receptor differ from pain receptors:


a) Less effect on them by morphine analgesic dose
b) Unmylinated
c) Response is more delayed
d) End in ventral spine

(122) K TYPE: Nasal Route ccc by


I) higher surface area than small intestine
II) Good Blood supply
III) mucocilliary execretion

(123) Buccal Route is ccc by:


a) No membrane barrier
b) Less metabolic activity
c) For high dose drugs

(124) Amiodarone patient asking about Grape fruit


a) No grapefruit juice with amiodarone
b) Limit to 250 ml / day
c) Take grape fruit juice freely
d) Take only frozen concentrate Grabe fruit juice

(125) Autoclave
a) Use dry heat
b) Used for aq. Preparation

(126) IV TPN, on addition of electrolyte, a hazy ppt. is formed, which is due to:
a) The formation of Na Phosphate
b) The formation of Ca phosphate

(127) Wheezing, cough, reversible obstruction of airway:


a) Emphysema
b) Chronic bronchitis
c) Asthma

(128) Cause generalized Edema all EXCEPT:


a-Cirrhosis
b-Long term corticosteroid
c-Renal failure
d-Chronic heart failure
e‐Hyperthyrodism
(129) All are triggers of migrane except:
a) Chemicals as MSG
b) Chocolate
c) Lights
d) Sounds
e) Long term use of Acetaminophen

(130) Which is approved for the preventing of osteoporosis:


a) Tamoxifene
b) Finasteride
c) Testosterone
d) Raloxifene

(131) Topical treatment of hair loss: Minoxidil

(132) Spironolactone: Decrease sodium, with no potassium loss

(133) Ear wax all except?


a) Accumulation may lead to hearing loss
b) Increase with age
c) Q-tipps is not allowed
d) Triethanolamine drops can be used
e) Mineral oil used to remove it

(134) Elixirs are:


a) Suspension of drugs
b) Glycosylated drug derivative
c) Sweet flavored liquid
d) Hydroalcholic liquid

(135) Ointment are all except:


a) May be hydrophobic or hydrophilic
b) A semisolid
c) May break during storage

(136) Previously authorized drug, means:


a) Only if approved by insurance
b) Only if prescribed by the doctor before
c) Only if taken by patient before
(137) Toxic shock syndrome causing microorganism: Staphylococcus aureus

(138) Flavin to be reduced, need how many H atoms:


a) 1
b) 2 (FAD  FADH2)
c) 3
d) 4
e) 5
In the medi ring and the right ring you have N= you will make them N‐H and break
the bond So you need 2H for this reduced form

(139) K TYPE: Patient asking about H2 R blockers, and antacid for the treatment of his
GERD, you tell him:
a) Antacid is faster than H2R blockers
b) Antacid work by neutralizing acid
c) H2 R blocker prevent acid secretion

(140) GERD side effects are all except:


a) Dysphagia
b) Weight gain
c) Nausea
d) Heart burn

(150) Merchandise organizing in the self selection area is Visual selling

(151) All are found in self care Book, EXCEPT:


a) Treatment of non complicated diaper rash
b) Immunization schedule

(152) Easiest type of pharmacy is:


a) Franchise
b) Ownership
c) partnership

(153) Major metab. Of Codiene:


a) Hydromorphone
b) Morphine
c) Oxymorphone
(154) What cause Withdrawal in morphine dependant patient:
a) Meperidine
b) Codiene
c) Nalbulone

(155) K TYPE: Which is taken with high amount of water?


a) Allopurinol
b) Cholestyramine All of them
c) etidronate

(156) Colon bacteria has a profile of: 10% Aerobic + 90% anaerobic

(157) Pyuria and bacteruria is found in:


a) Uremia
b) Nephritic syndrome
c) UTI * PYURIA=pus in urine
d) Acute necrotic syndrome

(158) All are transmitted, EXCEPT:


a) Gonorrhea
b) Hepatitis C
c) Chicken pox
d) Botulism

(159) Mast cell can be differentiated from all the following, EXCEPT:
a) Erythrocytes
b) Neutrophils
c) Eiosinophils
d) Basophils

(160) The Right Passage of drug from Lung to brain is: From lung to:
a) Right atrium, Right ventricle, aorta, brain
b) left atrium, left ventricle, aorta, brain
c) Right atrium, left ventricle, aorta, brain
d) left atrium, right atrium, aorta, brain
The systemic loop begins when the oxygen‐rich blood coming from the lungs enters the upper left
chamber of the heart, the left atrium. As the chamber fills, it presses open the mitral valve and the
blood flows down into the left ventricle. When the ventricles contract during a heartbeat, the
blood on the left side is forced into the aorta. This largest artery of the body is an inch wide. The
blood leaving the aorta brings oxygen to all the body’s cells through the network of ever smaller
arteries and capillaries

(161) Which is the LEAST cause for GI bleeding, with ASA:


a) Taking with food or milk
b) H pylori
c) Peptic ulcer
d) History of ulcerative colitis

(162) Which has the LEAST risk of GI bleeding:


a) Naproxen
b) Ibuprofen
c) Indomethacin
d) ASA
e) Cox II

(163) Anti pyretic effect of ASA is due to: Decrease of pg in hypothalamus

(164) the presence of alternating residues of β-(1,4) linked N-acetylglucosamine and N-


acetylmuramic acid is found in:
a) Plasma membrane
b) Cell wall
c) Cytoplasm
d) Mitochondria

(165) Fishy Odour is a ccc of:


a) Candidiasis (creamy cottage cheese like discharge)
b) Trichomoniasis (frothy wet discharge)
c) Bacterial vaginalis (creamy fishy odour)
d) Vaginitis

(166) Nystagmus is ccc by all EXCEPT:


a) Oscillation
b) Repetitive
c) Involuntary
d) Eye purges(exopthalmic)
(167) Follicules of SAA surface active agent (surfactant)called:
Micelles when increased it makes aggregation

(168) What is not used in soft gelatin capsules with high amount:
a) SAA
b) Glycerin
c) Hydrocarbons

(169) H Pylori treatment include all the following EXCEPT:


a) Omeprazole
b) Clarithromycin
c) Amoxicillin
d) Bismuth subsalicylate
e) Ciprofloxacin

(170) Nasal Solution Must be isotonic:


a) Prevent Hemolysis
b) Prevent irritation

(171) KTYPE: CFC is Stopped because


a) It contributes to Ozone depletion
b) Long term cause toxicity
c) Concern about flammability

(172) Drug is given IV, its Auc= 400mg, if given oral with the same dose, its AUC= 200mg,
then the absolute bioavailability is:
a) 50%
b) 100%
c) 2%
d) 15%
e) 25%

(173) Drug that decrease uric acid synthesis:


a) Pencillamine
b) Sulfapyarzine
c) Allopurinol
(174) Aldosterone is secreted from:
a) Adrenal medulla
b) Adrenal cortex

(175) ACTH is secreted by:


a) Hypothalamus
b) Anterior pituitary

(176) A liquid droplet in other liquid, immiscible with each other:


a) Suspension
b) Emulsion

(177) Methanol toxicity is due to: Formic acid

(178) Aminoglycoside is indicated for all the following except:


a) DM infected ulcers
b) Gm –ve bacteria
c) S. pneumonia
d) UTI

(179) Eye drops: PG f2 analoug cause eye lid pigmentation (latanoprost)

(180) Which doesn't increase Flu complications:


a) Osteoarthritis
b) DM
c) Asthma

(181) Phenytoin in blood bind to:


a) Albumin
b) Alpha acidic glycoprotein
Phenytoin is highly bound (90%) to plasma proteins, primarily albumin.

(182) Which is not a side effect of Flu:


a) Fever
b) Fatigue
c) Loss of Apetite
d) Headache
e) Nasal congestion
(183) Water used for reconstitution of an additive for a large volume parenteral should be:
a. sterile water
b. distilled water
c. Bacteriostatic Water for Injection (United States Pharmacopeial standard)
d. Water for Injection (United States Pharmacopeial standard)
e. Sterile Water for Injection (United States Pharmacopeial standard)

(184) Approximately 50% of dicloxacillin is excreted unchanged in the urine. If the


normal dosage schedule for dicloxacillin is 125 mg q6h, a patient with renal function
20% of normal should receive:
a. 25 mg q6h
b. 31.25mgq6h
c. 62.5 mg q6h
d. 75 mgq6h
e. 125 mgql2h
you have 50% excreated via kideny and 50% is non renal, The 50% of non‐renal is
Ok, so 125/2=62.5 can be taken safely, the other half 62.5 is excreted renally, and
we have 20% normal kidney, so 62.5*0.2= 12.5 ,, this is the ability of the kidney to
eliminated the drug, so sum 62.5+12.5=75 mg

(185) Hypothyrodism is related to the following (k type)


I-Grave's disease
II‐I 131 treatment
III‐Hashimoto syndrome
After radioactive Iodine, pt will be hypo for life

(186) Sumatriptan is used in treatment of migraine due to its effect as


f) Dopamine antagonist
g) 5HT1b/1d agonist
h) Acetylcholine antagonist
i) 5HT1b/1d antagonist
j) Dopamine agonist

(187) The initial side effect of felodipine is : Headache

(188)Emphysma is:permenant enlargement of alveoli


(189) Which clotting factor found normally in blood?2009
A-Thrombin
B‐Prothrombin
C-Heparin
D-Warfarin
E-Factors 2, 7, 9, and 10.

(190) identify non essential amino acid (PVT TIM HALL),2009


a.tryptophan
b. methionine
c. proline
d. phenylalanie
e. histidine

(191) Amino acid precursor for nitric oxide:


A) methionine
B) arginine
C) aspartate
D) phenylalanine
E) dopamine

(192) Which of the following is an endogenous pain producer?2010


A) Bradykinin
B) Pyrogen
C) prostaglandin E2
D) Thromboxane
E) Ecosonide
PG is pain producer but bradykinin is more pain producer

(193) CYP 450& Haemoglobin contain:


a-Myosin
b-Myoglobin
c‐Heme
d-Albumin
e-Transferrin

(194) all can be given to pregnant woman except


a- acetaminophen
d‐ Ramipril
e- Methyldopa

(195) all are invovlved in ttt of ascites except


a‐ levothyroxin
b- Diet Salt restriction
f- Diuretics eg: Spironolactone
g- paracentesis

(196) What is the mechanism of pioglotazone


a‐ Inhibit PPAR (Peroxisome Proliferator Activated Receptor gama

(197) Fluoxetine structure contains: (do not require


dose tapering)
a) Primary amine
b) Secondary amine
c) Tertiary amine
d) Thiophene

(198) All of the following used in treatment of breast cancer Except


f) Anastrozole - oral non-steroidal aromatase inhibitor
g) Letrozole - oral non-steroidal aromatase inhibitor
h) Tamoxifen - used for Albright syndrome also , prodrug
i) Flutamide ‐ synthetic,non-steroidal,pure antiandrogen used primarily to treat
prostate cancer
j) Exemestane - aromatase inhibitors

(199) Flumazenil is an antidote for:


d) Benzodiazepines
e) Barbiturates

(200) Tranformation (metabolism) of prodrug to drug is:


• Bioactivation (Prodrug to active drug)
• Bio transformation

(201) all are involved in senses except: 2008


a‐ sciatic nerve
b-trigemminal nerve - responsible for sensation in the face and motor functions
such as biting and chewing
c-olfactory nerve - is the sense of smell & contain chemoreceptor

(202) sildenafil is contraindicated with nitrates due to:


a‐ rebound hypotension
b-hypertension

(203) acetyl salicylic acid caused all except:


a. Upset stomach and heartburn
b. bleeding from the stomach/intestine
c. difficulty hearing, ringing in the ears
d. Reye’s syndrome in children with chickenpox and flu
e. metabolic alkalosis

(204) all are not chronic carrier except: Hepatitis C

(205) Which drug is used in prophylaxis of TB


a‐ISONIAZIDE  common cause of granuloma is Mycobacterium

(206) all open in the duodenum except: 2010


a‐ parotoid gland‐ opening into the vestibule of the mouth

(207) the drug of choice in petite mal epilepsy: 2010 ETHUSUXIMIDE

(208) Mechanism of Action of Zafirlukast


oral leukotriene receptor antagonist (LTRA) or is a selective and competitive receptor
antagonist of leukotriene D4 and E4 (LTD4 and LTE4)
(209) Competitive antagonist definition? a receptor antagonist that binds to a receptor
but does not activate the receptor

(210) Side effects of phenytoin include? 2010


I-Gingival hyperplasia, ataxia, nystagmus
II-Steven Johnsons syndrome I
II-Hirsutism and acne
A-I only B-III only C-I and II only D-II and III only E-I, II, III
(211) Pernicious anaemia is due to deficiency of:
e‐ Vitamin A
f‐ Vitamin C
g‐ Folic acid
h‐ Vitamin E
i- Cyanocobalamin B12

(212) Mechanism of action of orlistat:


Orlistat is a reversible inhibitor of gastrointestinal lipases. It exerts its therapeutic
activity in the lumen of the stomach and small intestine by forming a covalent bond
with the active serine residue site of gastric and pancreatic lipases.

(213) MELENA: (BLOOD IN THE STOOL) refers to the black "tarry" feces that are
associated with upper gastrointestinal bleeding. The black color is caused by the
hemoglobin in the blood being altered by digestive chemicals and intestinal bacteria.

(214) Which doesn't contain flexor muscles? Tongue

(215) insulin out side refrigerator it's half life: 28 days

(216) causative organism of acne: 2010


Propioni bacterium acne OR Corynebacterium

(217) The main screening test for colorectal cancers is


f) Blood tests
g) Ultrasound scans
h) PPD test
i) Pap smear
j) Fecal occult blood not for prostate cancer

(218) enzymes are: any of various proteins, as pepsin, originating from living cells
and capable of producing certain chemical changes in organic substances by
catalytic action, as in digestion.

(219) side effects of gentamycin include? Ototoxicity & Nephrotoxicity

(220) For a clinical trial if the P value is said to be 0.001 this means that?
That the probability of null hypothesis to be right is high

You might also like